You are on page 1of 286

T. Bzduek, J. Imrika, J. Zvodn a kol.

Zbierka

FX

Vytvoril FKS Fyziklny Korepondenn Seminr


www.fks.sk

Vydal Trojsten

Tom Bzduek, Jakub Imrika, Jakub Zvodn, 2009


ISBN 978-80-970297-0-8

Predslov a [poakovanie]
Kdesi tamsi, podelo sa mal toidielko krdlat. Stali sa veci, dovolm si
nazva, bizarn. Ach ty mj semafrik venezelen (pocta ako vyinut od
motoristu), preo netla hodiny smerom dopredu? Komu sa nelen, toho
sekne v kroch. Ale s na tom udia aj horie. Naprklad, ke maj napsa
k veci vod ku vzorku.
Tom Rybr, vod do vzorku FKS

Britsk fyzik Stephen Hawking v predslove svojej najznmejej knihy


A Brief History of Time uvdza, ako mu niekto povedal, e kad rovnica,
ktor do svojej knihy pojme, zni jej predaj na polovicu. Keby toto bola
cel pravda, kniha, ktor prve drte v rukch (ako i kad in odborn
fyziklna literatra) by vlastne mala by vopred odsden na nespech. Avak ma jeden mj priate (myslm, e to bol Tom Kulich) upozornil, e
kad Feynmanov vrok a kad vtipn poznmka pod iarou predaj knihy,
naopak, zdvojnsob. Tch tu itate njde naozaj nemlo, o mi dva ndej,
e Zbierka FX vzbud v cieovej skupine ud nemal zujem.
Tieto riadky by ste zrejme netali, keby Vs nezaujmalo, o njdete vo
vntri. Najprv vak nieo o tom, o knike predchdzalo. Prbeh, ktor sa
zavil jej vznikom, sa zaal saou s rovnakm nzvom FX (taj [f:ks]).
T vznikla r. 2001 z iniciatvy Mata Meda a Daniela Nagaja ako prestna kategria korepondennho seminra FKS (www.fks.sk), uren pre
vtedajch nadencov. Po dlhej odmlke ju v roku 2005 jeden zo spomnanch nadencov znovu prebral k ivotu, odkedy funguje nepretrite a s oraz
vym potom rieiteov. Po ukonen strednej koly sa niektor z nich stvaj nadenmi organiztormi, m sa zskava oraz profesionlnej rz.
Dnes, po tyroch ronkoch sae, sa meme pochvli nielen kvalitnmi
vzorovmi rieeniami, ale aj plejdou spechov, ktor nai rieitelia zskali
na medzinrodnej rovni. Z Medzinrodnej fyziklnej olympidy v Singapre (2006), Irne (2007), Vietname (2008) a Mexiku (2009) si doniesli tyri
zlat, dve strieborn a p bronzovch medail!
Zbierka FX me pri prvom otvoren v loveku vyvola strach. Tu by
som rd povedal tri veci. Po prv, neprepadajte panike!
Po druh, hoci sa v knike asto pouva netrivilna matematika, ako
napr. derivcie a integrly, pokia ide o fyziklne porozumenie textu, plne
sta vedie, o tieto opercie znamenaj (a teda nie je nutn vedie ich
i

naozaj pota). Najm kvalitatvne tj., e derivcia funkcie miera i


rchlos jej rastu , integrl fukncie obsah plochy pod jej grafom i aksi
jej kumulatvny set , nezakod vak i kvantitvne pomocou deninch
vzahov
+

( + ) ()

() = lim
,
()d = lim
( +
)

0

=0

(pokraujte v neprepadan a premyslite si, e denin vzahy dvaj presne


to, o chceme). Pokia Vs zaujma aj exaktn rieenie lohy, po dotudovan
matematickho apartu sa k nemu mete vrti veci sa Vm bud zda
jasnejie. Tu treba zdrazni, e hoci maj derivcie odstraujci nzov a hoci
sa uia a vo tvrtom ronku gymnzi, ich potanie nie je vbec zloit.
Poriadne sa daj naui u za pr hodn z mnohej dostupnej literatry.
Po tretie, shlasm, e latku sme nasadili poriadne vysoko. Tak to vak
poda ns m by! Ak sa chce lovek naui riei problmy, mus zapja
mozgov zvity a s a na hranicu svojich monost. Nesmie zaspa na vavrnoch rieenm tuctov typovch prkladov. Fyzika rozhodne nie je o pouvan
namemorovanch vzorcov, ale o zdravom (niekedy priam sedliackom) uvaovan a o tom, ako svoje mylienky matematicky zapsa. O tomto by vedel
vea bsni americk fyzik R. Feynman, ktor je pre ns vekm vzorom
(jeho dielo Feynmans Lectures on Physics, ktor r. 2001 vylo v etine rozhodne odporame kadmu zujemcovi o fyziku). Aby sme Vm tudovanie
Zbierky FX uahili, uvdzame spolu so zadaniami aj grack znzornenie
obtianosti na kle od 1 do 5 hviezdiiek.
Slovenskm itateom ponkame uebnicu fyziky, ak tu ete nebola.
Ako ju ohodnotia a i sa aj im uke ako prospen, to uke a as. Radi
by sme vak na tomto mieste vyslovili niekoko poakovan. Za nann
podporu akujeme Jednote slovenskch matematikov a fyzikov, predsedovi
slovenskej Fyziklnej olympidy Ivovi povi a najm Agentre na podporu
vskumu a vvoja, ktor podporila projekt zdruenia Trojsten na vzdelvanie talentov. Obrovsk vaka patr Eugenovi Hrukovi, Samuelovi Hapkovi
a predovetkm Petrovi Perenimu za odstrnenie mnostva tylistickch,
typograckch i matematickch chb. akujeme tie naim rieiteom za
mnoh alternatvne spsoby rieenia problmov. Napokon patr vaka aj
nmu priateovi Daliborovi Blaekovi, ktor bol saou FX naden a vydanie tejto kniky vemi aktvne podporoval.
V prpade akchkovek otzok ns nevhajte kontaktova na fx@fks.sk.
za Kolektv Autorov
Tom Bzduek, 11. janura 2010

ii

Physics is like sex. Sure, it may give some practical results, but thats not
why we do it.
Richard P. Feynman

Zoznam [loh]
A

Mechanika I.

FX A1 Lano, str. 2
Horolezec Tom zliezol dokonale hladk horu tvaru kuea s vrcholovm
uhlom 2, ale zabudol si na nej navleen kruhov sluku lana hmotnosti .
Akou silou je napnan tto sluka?

FX A2 pirla, str. 4
Fajo naiel vo vesmre dlh a vemi ak valec s polomerom . Na jeho
plti bol uviazan pagt s vonou dkou , s malm kamienkom pripevnenm na druhom konci. Ako sprvny fyzik, Fajo natiahol pagt tak, aby sa
uviazanm koncom prve dotkal valca, a kamienku udelil rchlos kolm
na pagt tak, aby sa zaal namotva v rovine kolmej na os valca. Za ak
as sa pagt plne namot na valec?

FX A3 Momentky, str. 7
Vypotajte bez pouitia integrlov moment zotrvanosti
(a) obdnikovej dosky vzhadom na uhloprieku,
(b) kocky vzhadom na os prechdzajcu stredmi dvoch protiahlch stien,
(c) pravidelnho tvorstenu vzhadom na os prechdzajcej vrcholom a stredom protiahlej steny.

FX A4 Most, str. 15
Katka si medzi polikou a stolom postavila most. Most je tvoren ahkmi
tyami vone spojenmi kbmi, tak ako na obrzku. Dky ikmch a vodorovnch ty s v pomere 5:6. V bode E je zavesen zvaie s hmotnosou .
Ktor z ty me Katka nahradi ohybnmi vzbami, t.j. naprklad pevnm
pagtom rovnakej dky? Akou silou je namhan ty BD?
iv

FX A5 Krok, str. 20

Mme horizontlny upevnen valec s polomerom . Na om je navleen


prstenec s polomerom (pochopitene > ) a hmotnosou . Vypotajte
peridu malch kmitov tohto prstenca (v jeho rovine), ak po valci nepremykuje.

FX A6 Kladky, str. 24
Tinka nala v pivnici obrovsk kopu lana a kladiek, preto sa rozhodla
postavi si kladkostroj. Ba dokonca dva! Jeden kraj ako druh, ako to vidno
na obrzkoch. Ak bud zrchlenia jednotlivch telies po uvonen kladiek?
Hmotnosti vetkch kladiek i lana s zanedbaten, poet kladiek na pravom
obrzku povaujte za nekonene vek.

FX A7 Motorov pla, str. 32

Predstavte si reaz na motorovej ple. Jej dka je , hmotnos a plne


bez trenia sa napnut pohybuje rchlosou okolo dvoch valekov s polomerom . Predpokladajme, e valeky sa neotaj. Reaz nijakm spsobom
nepohame ani nespomaujeme.
v

(a) Za ak as klesne jej rchos pohybu na polovin?


(b) Kvalitatvne popte (a zdvodnite), o sa stane s reazou, ak by valeky
(a zvyok motorovej ply) zrazu zmizli.

FX A8 Rusk sud, str. 36


Fajo zhal na trhu sud, do ktorho bude dva jablk na plenku. Jeden
obchodnk mu ponkal vemi zvltny sud. Mal tvar valca, jeho podstavy boli
z pevnho kovu. Pl je vak vyroben z prunej gumovej ltky, do ktorej s
votkan tenk ohybn a neroztiahnuten vlkna tak, ako na obrzku. Fajo
sa hne zaal zaujma, o sa stane, ak do sudu natlakujeme vzduch. Ktor
z troch tvarov na obrzku zaujme sud v zvislosti od uhla ?

Optika

FX B1 Zrkadl, str. 40

Polopriepustn zrkadlo s priepustnosou (kde 0 1) je tak


zrkadlo, na ktor ak (z ubovonej strany) zasvietime l s intenzitou , prepust l s intenzitou a odraz l s intenzitou (1 ).
(a) Filip zobral zrkadlo s priepustnosou 1 a hne za postavil rovnoben
zrkadlo s priepustnosou 2 . Koko svetla prepust tto dvojica zrkadiel,
ak na u spredu zasvietime l s intenzitou ? o ak zasvietime zozadu?
(b) Vlado sa nenechal zahanbi. Vytiahol vetky polopriepustn zrkadl, o
naiel v pivnici, a tie ich postavil pekne za seba. Jeho zrkadl maj
priepustnosti 1 , 2 , . . ., , v tomto porad. Ako sa bude sprva tto
sstava?

vi

FX B2 Svetlomety, str. 43
Bzduo m stl tvaru tvorca a chyst sa ho osvetli zdrojom so svietivm
vkonom , a to dokonca takm, o iari do vetkch smerov rovnako. Tmto
zdrojom sa bli k rohu stola troma rznymi spsobmi (vi obrzok). Pri
kadom z nich vypotajte hodnotu, ku ktorej sa bli svietiv vkon dopadajci
na stl, ke sa zdroj bli k rohu stola. (Vetky hrany na obrzku okrem pok
s rovnako dlh a uhly medzi nimi prav, tvary na obrzku s teda kocky.
pky naznauj smer pohybu zdroja. Stl je vyznaen edo.)

FX B3 Minca, str. 47
Ujo z prkladov o minci na dne bazna sa pozer na mincu na dne bazna.
Vid ju v hbke a chce vedie, v akej hbke sa nachdza naozaj. Stala sa
mu vak vek galiba nevid ju toti kolmo zhora, ako to u v takchto
prkladoch bva, ale pozer sa na u pod uhlom (vzhadom na normlu
hladiny). Pomte ujovi zrta, v akej hbke sa nachdza minca naozaj!

FX B4 Vlkno, str. 51

Mao si minule kpil optick vlkno dky a konene si svoj pota


na intrku pripojil priamo ku serveru FKS. Jadro optickho vlkna je dlh
valec polomeru vyroben zo skla s rznymi prmesami tak, aby sa rchlos
renia svetla v om zvyovala linerne so vzdialenosou od jeho osi. Index
lomu v strede je 1 , na kraji 2 . V strede jednej jeho podstavy je bodov zdroj
svetla, ktor vyle krtky sveteln impulz. Ak dku trvania bude ma impulz
prijat na druhom konci optickho vlkna? Rdov zanedbania s prpustn,
typick hodnoty velin s = 1 km, = 20 m, 1 = 1,445, 2 = 1,44.

Elektrick pole

FX C1 Guky, str. 60
Peo rozostavil do medzihviezdneho priestoru 2009 vakovakch kovovch
guliek. Guky nabil nbojmi , 2, 3, . . ., 2008 a 2009, priom je
kladn. Dokte, e aspo jedna z guliek m na celom svojom povrchu kladn
hustotu nboja!

vii

FX C2 Stavebnica, str. 63

Marcelka dostala na narodeniny sadu bodovch nbojov vekosti a k nim


vakovak vzby. Hne sa pustila do velijakch experimentov a m na vs
tieto otzky.
a) Dva nboje xujeme v priestore a tret nechme vone pohybova po nimi
danej priamke. Ak je jeho perida kmitov v rovnovnej polohe?
b) tyri nboje xujeme vo vrcholoch tvorca, piaty nechme vone pohybova v nimi danej rovine. Njdite jeho peridu kmitov v strede tvorca.
c) Vymyslite kongurciu xovanch nbojov v priestore tak, aby vznikla
stabiln rovnovna poloha pre al nboj a vypotajte jeho peridu
kmitov v nejakom smere.

FX C3 Rebrk, str. 66

Juro si na chate postavil rebrk z rezistorov s odporom . A to nie hocijak, dokonca dvojit a nekonen, tak ako na obrzku.
(a) Vypotajte odpor medzi bodmi a .
(b) Vypotajte odpor medzi bodmi a , ak s body a vodivo spojen
(vodiom s nulovm odporom).
(c) Vypotajte odpor medzi bodmi a .

FX C4 Balvan, str. 73
Kubk naiel v Tatrch vek balvan. Naiel si na om vek hladk plochu
a na u si nakreslil mal tvorec so stranou . Ke medzi body
a pustil prd , medzi bodmi a nameral naptie . Ak je mern
elektrick odpor balvanu?

viii

FX C5 krupina, str. 77

Mme vodiv izolovan guov krupinu nabit nbojom . Peo m bodov nboj rovnakej vekosti a chce si ho niekam odloi. Njdite vetky jeho
mon polohy v priestore tak, e na nepsob iadna sila.

FX C6 iara, str. 83
Ke bola Marcelka na prednke, Janka jej zo stavebnice s nbojmi zobrala dva rovnak kladn nboje o vekosti a priblila ich do vzdialenosti
od seba. Potom si nakreslila siloiaru, ktor vychdzala z jednho z tchto nbojov pod uhlom ku spojnici oboch nbojov. Do akej najmenej vzdialenosti
od roviny symetrie nbojov sa tto siloiara dostane?

Gravitcia

FX D1 Ploch Zem, str. 96

Je veobecne znme, e Zem je gua s polomerom . Kedysi si vak udia


mysleli, e Zem je nekonen homognna plata s hrbkou . Zistite, ak
by musela by tto hrbka , aby bolo na plochej Zemi rovnak gravitan
zrchlenie, ako je teraz. Predpokladajte, e hustota plochej Zeme by bola
rovnak, ako je priemern hustota Zeme teraz.

FX D2 Asteroid, str. 103


Azag sa hral so svojm novm alekohadom, ke zrazu spozoroval pohybujci sa asteroid. Azag zistil, e tento asteroid sa prve nachdza vo vzdialenosti od Slnka, jeho okamit rchlos je , a smer jeho rchlosti zviera
uhol so spojnicou asteroid-Slnko. Ak je jeho perida obehu okolo Slnka?

FX D3 Polgule, str. 108


V rmci ekonomickch opatren pred oslavou jubilejnho roku 3000 sa byrokracia rozhodla skrti vdavky tm, e odstrnia polovicu Zeme. Vkonn
ata najprv rozrezala Zem na dve polgule a potom jednu z nich vymazala
(prkazom delete, samozrejme).
(a) Akou silou sa tieto dve polgule priahovali?
(b) Po vymazan druhej polgule, ak bolo gravitan zrchlenie v mieste,
kde sa kedysi nachdzal stred Zeme?

ix

FX D4 Hmota, str. 113


Marin si kpil asteroid s objemom z plastickej hmoty o hustote , a
v priestore si vyznail bod . Ako m tento asteroid vytvarova a kam ho
m umiestni, aby bolo v bode o najvie gravitan pole? Ak by bola
vekos tohto gravitanho poa, ak by parametre a asteroidu boli zhodn
s tmito parametrami Zeme?

Mechanika II.

FX E1 Gua, str. 120


Vla Amlka si svoju kritov guu poloila na vek stl pokryt obrusom. Ke mala slvnostn nladu, vekolepo strhla obrus zo stola (vodorovnm pohybom). Ak bude rchlos gule po tom, ako na stole prestane premykova? Mete predpoklada, e toto sa stane ete predtm, ako spadne
zo stola, a e gua pri sthan obrusu neposkakuje. Polomer gule je , hmotnos , koecient trenia o obrus 1 a o stl 2 .

FX E2 Kolieska, str. 130


Marcel sa rd hr s kolieskami. Minule vzal dve homognne kolieska
s hmotnosou a polomerom a pripojil ich na opan konce spolonej
ahkej osky dky 2 tak, aby sa mohli ota nezvisle od seba. Potom toto
udo poloil na vodorovn podloku a kolieska roztoil tak, aby sa oska otala uhlovou rchlosou okolo zvislej osi prechdzajcej jej stredom. Ak
je celkov kinetick energia tohto uda, ak kolieska nepremykuj?

FX E3 Struna, str. 139


Tina sa u hra na gitare. Okrem jej podmanivho zvuku ju vak zaujala
aj skutonos, e ke si gitaru nalad v teple domova a potom s ou vyjde von
do chladnej zasneenej noci, gitara u nelad. Aby ste Tine vysvetlili, ako je
to mon, tak
(a) odvote vlnov rovnicu pre strunu;
(b) njdite zvislos zkladnej frekvencie struny v zvislosti od jej predenia ;
(c) zistite, ako sa zmen zkladn frekvencia struny, ak s gitarou zjdeme
do treskcej zimy s teplotou o menou.
x

Struna m dku = 0,8 m (jej as na kobylke zanedbvame), priemer =


0,6 mm, je zhotoven z ocele s hustotou = 8000 kg/m3 , modulom prunosti
= 220 GPa a tepelnou rozanosou = 11,106 K1 a na zaiatku hr
s frekvenciou 0 .

FX E4 ab, str. 145

Baa si na zhrade postavila dlh ab na odvod vody. Jeho rka je a


sklon . Ke sa raz rozpralo, v abe tiekla voda prdom o vke (meranej
kolmo na dno abu). Ak bol prietok vody abom, ak jej prdenie povaujeme
za laminrne? Mete predpoklada .

FX E5 Rutherford, str. 150

Pn Rutherford znova vytiahol zo pajze zlat fliu a delo astc. Vystrelil asticu rchlosou smerom na jadro zlata, a ona sa mu odchlila
o uhol (od pvodnho smeru letu). Zistite, o koko pn Rutherford netral toto jadro zlata! (t.j. ako aleko od pvodnej priamky letu sa toto jadro
nachdzalo)

FX E6 Pinkaka, str. 158


Jano chce porazi Jura v squashi a tak poctivo trnuje. Minule si naprklad zohnal loptiku s hmotnosou a katuu tvaru kvdra s hmotnosou
. Potom kopol do katule tak, aby sa mkala po zemi rchlosou
smerom kolmo na stenu, a do jej drhy poloil vo vzdialenosti od steny
nehybn loptiku. Vypotajte do akej najmenej vzdialenosti od steny sa
krabica dostane. Trenie katule aj loptiky o zem povaujte za nulov, vetky
zrky za dokonale prun a predpokladajte, e krabica sa neota (cel pohyb loptiky sa deje na jednej priamke kolmej na stenu). Odpove sta do

prvho rdu v
.

FX E7 Via ferrata, str. 164

Mme oceov lano dky = 20 m, prierezu = 1 cm2 , s medzou pevnosti = 1 GPa, hustotou = 8000 kg.m3 a modulom prunosti =
220 GPa. M sli na zaistenej ceste v horch (t.j. na via ferrate) pre odvneho feratistu na prelez ponad roklinu. Aby mohol by takto most uznan
bezpenm, tak naptie v lanch nesmie prekroi 10-tinu medze pevnosti
lana, ak je feratista s hmotnosou = 80 kg v strede lana. Urte, ak najiriu roklinu vieme pomocou tohoto lana prekona za dodrania predpisov!
xi

Straty hmotnosti feratistu v dsledku fyziologickch prejavov strachu mete


zanedba.

FX E8 James Bond, str. 172


James Bond sa chyst na aliu akciu, kde sa mus vyplha na strechu
vysokho domu. Zaobstaral si kotvu o hmotnosti = 2 kg, ktor doke
vystreli rchlosou 0 = 25 m/s. Kpil si tie horolezeck lano, ktorho jeden
meter vi =100 gramov. Do akej najvej vky doke vystreli kotvu
(s upevnenm lanom, samozrejme)? Odpor vzduchu neuvaujte.

FX E9 Pruinky, str. 178

Samo naiel rovnakch teliesok s hmotnosou , + 1 pruiniek s tuhosou , a jednu priamku. Pruinky teda pospjal za seba na priamku a
medzi kad dve nasledujce upevnil jedno teliesko. Zaiatok prvej pruinky
a koniec poslednej pevne zaxoval, ale telieska nechal vone pohybova po
danej priamke. Na obrzku vidte nkres situcie pre = 2:

(a) Urte peridu vetkch harmonickch pohybov, ktor me sstava vykonva, pre = 2.
(b) Urte peridu vetkch harmonickch pohybov, ktor me sstava vykonva, pre = 3.
(c) Kvalitatvne popte, o sa bude dia pre vie hodnoty .

Termodynamika

FX F1 Puding, str. 188


Lenka navarila dva hrnce pudingu:
Vanilkov puding m tepeln kapacitu
teplotu 1 , okoldov m kapacitu 2
vo forme makroskopickej prce z nich
Potajte rchlo, km nevychladne!)

xii

jeden vanilkov a jeden okoldov.


1 (nie mern, t.j. u v J.K1 ) a
a teplotu 2 . Koko najviac energie
vie vyai? (Puding pri tom nezje.

FX F2 Kompost, str. 194


Judita sa rozhodla vyui teplo kompostoviska vo svojej zhradke na dobroinn ely. S tm zmerom si postavila dva vratn stroje periodicky pracujce poda dejov a . Pracovnou ltkou je vzduch, pretoe je
lacn. Ak je pomer innost tchto dejov?

Teria relativity a kozmolgia

FX G1 Supernova, str. 198


Kdesi aleko vybuchla supernova a obrovskou rchlosou vyvrhla materil na vetky strany. Peo tieto jej pozostatky u ist as pozoruje so svojm alekohadom. Pozn vzdialenos supernovy , zmeral aj uhlov rchlos
rozpnania jej oblky (vzaovania pozostatkov od hviezdy) , a tak si jednoduchm vpotom ( = ) vypotal, e sa oblka rozpna rchlosou 4/3
krt vou, ako je rchlos svetla. Akou rchlosou sa oblka rozpna naozaj?
(Oblka supernovy sa rozpna rovnomerne.)

FX G2 Vesmr, str. 204


Peo vyniesol do vesmru svoju obben druicu. Druica sa dlho tlala
hlbokm vesmrom a dostala sa a do vzdialenosti od Zeme, ke jej zaalo
by smutno a zaala vysiela rdiov signl s frekvenciou . Kubo si chcel
naladi rdio na Peovu druicu, a tak sa zaal zama nad nasledujcimi
problmami.
(a) Ak frekvenciu signlu druice nameria Kubo, ak sa druica pohybuje
(hoc i vekou) rchlosou smerom od Zeme?
(b) Ak frekvenciu signlu druice nameria Kubo, ak druica stoj, ale vesmr sa rovnomerne rozpna tak, e vzdialenos medzi druicou a Zemou
xiii

sa v ase vysielania zvuje rchlosou ? Rovnomern rozpnanie vesmru v tejto lohe znamen, e objektvna vzdialenos medzi kadmi
dvoma bodmi sa zvuje kontantnou rchlosou, priom v kadom momente je rchlos vzaovania sa dvojice bodov priamo mern vzdialenosti tchto bodov.

FX G3 Zbavn fotny, str. 213


Halucinka nala na povale zopr fotnov vo vemi dobrej nlade. Jej obben elektrn je u niekoko dn smutn a nehybne stoj na mieste, rozhodla
sa ho teda obveseli: poslala k nemu usmievav fotn s vlnovou dkou .
S poteenm zistila, e po tomto stretnut sa elektrn predsa len zaal pohybova. Vyslan fotn sa vak pri stretnut odchlil od pvodnho smeru o uhol
a zmenil svoju vlnov dku. Ako zvis nov vlnov dka fotnu a kinetick energia elektrnu od uhla ?
Do rozbehnutho elektrnu po ase narazil druh fotn. Po tejto zrke
zostal elektrn znova smutne st a fotn sa odchlil do smeru, v ktorom sa
pvodne pohyboval prv fotn. Ak je vlnov dka odlietajceho fotnu?
Nezabudnite, e rchlos elektrnu medzi zrkami me by relativistick.

FX G4 Enterprise, str. 222


Cez vianon przdniny sa Marcel kadorone vracia z interntu domov. Kee doma nem internet, vinu asu trvi hranm potaovch
hier a pozeranm serilov. Naposledy naprklad dal za jeden de toko dielov
Star Trek-u, e mal sen, v ktorom bol na palube vesmrnej lode Enterprise.
T sa prve nachdzala v asti vesmru, odkia bolo vidno hviezdy na oblohe rozmiestnen rovnomerne, t.j. tak, e poet hviezd na priestorov uhol
d/d = bol pribline kontantn.
(a) Ak rozdelenie hviezd by na lodi pozoroval, keby sa vzhadom na predol sstavu pohyboval dopredu rchlosou ?
Predstavte si, e by sme namiesto hviezd mali na oblohe monochromatick
zdroje svetla s frekvenciou s celkovm vkonom .
(b) Ako sa zmen frekvencia svetla v pohybujcej sa sstave v zvislosti od
smeru, ktorm sa Marcel pozer?
(c) o vieme na zklade ast () a () poveda o zmene hustoty iarivho
vkonu = d/d hviezdneho pozadia?
xiv

Matematick lohy

FX H1 Romantika, str. 232


Bea nedvno tala Malho Princa a rozhodla sa, e aj jej sa vemi pia
zpady slnka. Aby jej vak nezovedneli a aby si ich patrine vychutnala,
odsahovala sa na severn pl. Tam je toti zpad slnka len raz za rok, a
navye trv sakramentsky dlho. Ozaj, ako dlho?

FX H2 Starcraft, str. 236


Ferko a Tom s zarat do svojej obbenej potaovej hry, klikaj ostoes, stavaj armdy, schyuje sa k vekej bitke. Tom posiela do boja jednotiek, Ferko rovnakch jednotiek. Njdite asov priebeh potu jednotiek
na oboch stranch, as trvania bitky i konen stav.
Mete predpoklada, e obaja hri s sksen jednotky po sebe strieaj fotnovmi delami optimlnou stratgiou, a (km sa nevykynoia), je
ich vemi vea. Jedna jednotka stlym strieanm zabije druh (nebrniacu
sa) za as .

FX H3 Brka, str. 241


Mat prve sedel na lke a potal fyziku, ke sa zrazu prihnala vek
brka. Poas brky do zeme nhodne udieraj blesky, priom na kilometer
tvorcov za cel brku priemerne udrie bleskov. Ak je priemern hodnota
vzdialenosti Mata a miesta, kde udrel k nemu najbli blesk?

FX H4 Mesiac, str. 246


Kokonsobne silnejie iari Mesiac v splne ne v prvej tvrti? Uvaujte
Lambertovsk model rozptylu svetla na mesanom povrchu.

FX H5 Elektrika, str. 253


Ak vonku pr, schodky na nstup do elektriky s pinav od blata. Ke
sa vak otvraj a zatvraj dvere, as tchto schodkov pri svojom pohybe
poumvaj kefami pripevnenmi na ich spodku. Vypotajte tvar a plochu
tchto poumvanch ast.1 Vimnite si tie horn upevnenie druhej dosky
1 Pri rien uvaujte naprklad elektriky Tatra T3, kde sa jedno krdlo dver sklad
z dvoch rovnakch zvislch obdnikovch dosiek navzjom spojench kbom na zvislej
hrane. Von hrana jednej z nich je upevnen na zvislej otavej tyi, hrana druhej m
pohyb obmedzen v rovine prechdzajcej touto tyou.

xv

dver. V novch modeloch elektriiek tu nie je xovan jej horn roh v rovine
zatvorench dver, ale bod na upevnenom vstupku, pomerne aleko od samotnch dver. uduj sa svete, vodiaca lita pre tento vstupok je tie rovn.
Ak polohu (vzhadom na druh dosku dver) me ma vstupok vo vodiacej
lite, aby tto mohla ma tvar seky?

FX H6 Skmavka, str. 260


Evka sa na biolgii rada hr so skmavkami, minule naprklad do skmavky tvaru valca s polomerom 1 cm naliala vodu a pozorovala, ak tvar
bude ma jej povrch. Njdite kontaktn uhol medzi povrchom vody a sklom,
tie njdite celkov prevenie hladiny vody na kraji a v strede skmavky,
a nakreslite tvar jej povrchu. Povrchov energia rozhrania voda-vzduch je
70 mJ.m2 , energia rozhrania voda-sklo 40 mJ.m2 a energia rozhrania sklovzduch 100 mJ.m2 . seln vsledky plne staia.

xvi

A
Mechanika I.

Iste ste si u vimli e, hmotnos je vlastne veliina charakterizujca nechu k pohybu. Naprklad, ke zjete jeden cel drevorubaov kltik (kilo rezov, chlieb a baranie rohy), vaa chu k hocijakmu aktvnemu pohybu prudko
klesne. lovek je vlastne vcelku fyziklne zaloen tvor.
Tom Rybr, poznmka pod iarou vo vzorku FKS

FX A1 Lano
Horolezec Tom zliezol dokonale hladk horu tvaru kuea s vrcholovm
uhlom 2, ale zabudol si na nej navleen kruhov sluku lana hmotnosti .
Akou silou je napnan tto sluka?
Zdroj: lohu vymyslel Kubus

l Vzork Jakub l
Hne na vod poviem, e t historka s tpkom, o zliezol horu, nm
nijako nedeterminuje polohu lana, ktor mme skma. Intuitvne vetci
tume, e lano sa bude prirodzene rado nachdza v horizontlnej rovine.
Inak by zrejme malo viu potencilnu energiu v tiaovom poli Zeme (aj
ke ma vbec nenapad, ako to jednoducho ukza; inak ani zloito ma to
nenapad :-)).
Pome k veci: mme lano vo vodorovnej rovine. Prklad vykazuje symetriu voi osi kuea. To vyuijeme toti meme skma ubovon ksok

lana o dke a hmotnosti 2


, ktor obta kue / tg pod jeho
vrcholom a uhol, ktor mu prislcha je . Vi obrzok zhora, kde je do
bodu projektovan rotan os naej hory.

Na tomto obrzku aj pekne vidme naptie v lane , ktor sa kad


ksok sna natiahnu. Nakoko je situcia rotane symetrick, tak je jasn,
2

FX A1

Lano

e na kad jeden ksok lana musia psobi z oboch strn rovnak naptia.
Vborne, potom ich vslednica, vi obrzok, smeruje do rotanej osi a m
vekos 2 sin 2 , o sa vak pre mal (a ja si mem zvoli ubovone
mal!) rovn . To je sila z druhho obrzka, ktor ukazuje situciu
zboku. Ete vyjadrm =
2 . Kee sa n sledovan ksok lana nehbe,
tak musia by sily v rovnovhe.

To nutne plat aj pre zloky sl kolm na normlu kuea v mieste tudovanho lana (ie kolm na lebo to je normlov sila, kee trenie je
nulov), z oho dostanem podmienku cos = sin , o mi po vyjadren
a d

cos = sin ,
2

a teda

.
2 tg

K tomuto vsledku sa d dopracova troku trikovo aj inm zaujmavm


a astokrt vemi efektvnym spsobom cez virtulne prce. Metda virtulnych prc vychdza z toho, e teleso je v rovnovnej polohe vtedy, ke
malm posunutm ni na svojej potencilnej energii nestrat ani nezska (do
prvho rdu).
Sksme teda uvaova prun lano napnan silou . Pozrime sa teraz,
o sa stane s lanom, ke ho posunieme o ksok, ozname , nadol (nahor). Potom sa polomer sluky zv (zmen) o tg a teda energia lana
v dsledku jeho pruenia stpne (klesne) o pribline 2 tg . Zrove ale
klesne (stpne) jeho potencilna energia v tiaovom poli, a to o . M
plati

2 tg = 0

=
,
2 tg
o je n predol vsledok. Tu by bolo fajn ete urobi diskusiu o tom, i to
meme urobi pre absoltne tuh lano. Odpove sa mi hada nechce. Ani
netreba, lebo vo svete absoltne neprun lan neexistuj ie diskusia by
nemala vek hodnotu. A pre mal natiahnutia meme vdy aproximova,
e naptie lana sa vemi nezmen (sta naahova o innitenzimlne mlo),
o n postup odobruje. Hotovo.
3

FX A2 pirla
Fajo naiel vo vesmre dlh a vemi ak valec s polomerom . Na jeho
plti bol uviazan pagt s vonou dkou , s malm kamienkom pripevnenm na druhom konci. Ako sprvny fyzik, Fajo natiahol pagt tak, aby sa
uviazanm koncom prve dotkal valca, a kamienku udelil rchlos kolm
na pagt tak, aby sa zaal namotva v rovine kolmej na os valca. Za ak
as sa pagt plne namot na valec?
Zdroj: lohu vymyslel Bzduo

l Vzork Bzduo l
Mnoh lohy sa vemi zjednoduia, ak njdeme nejak zkon zachovania.
Zd sa to neuveriten, ale n kamienok sa bude cel as pohybova poiatonou rchlosou. Jedin sila, ktor na psob, je ah v pagte, a ten je
kolm na smer pohybu. Nekon preto iadnu prcu a energia kamienka sa
nemen. Kee sme vo vesmre, aleko od gravitanch pol a odporovch
prostred, jedinou relevantnou formou energie je kinetick energia. Navye
valec, o ktor je kamienok pripevnen, je vemi ak, take nebude odobera skoro iadnu energiu. (Rozmyslite si!) Rchlos kamienka sa preto meni
nebude.
Mme poznatok, s ktorm sa daj robi mov kroky. Obzrime si situciu
na obrzku v ase, ke dka nenamotanho povrzku je a sledujme, ako
sa zmen situcia o as d.

Filip Kubina rieil tto lohu nasledovne: Vimnime si, e za tento as


sa pagt oto o uhol d a o ten ist uhol sa oto aj bod, v ktorom sa
pagt dotka valca. pagt sa skrti o t svoju as, ktor sa namot na
valec, preto
d = d.
4

pirla
FX A2
V kadom okamihu akoby sa pagt otal okolo bodu, kde sa dotka valca,
rchlosou . Mono preto poveda, e uhlov rchlos kamea okolo tohto
bodu2 je = , kde je okamit dka nenamotanej asti pagtu. Z pohybu
kamea komo potom vyta
d = d =

d.

Ak toto dosadme to vzahu pre skrtenie vonej asti pagtu, dostaneme

d = d.

Rovnicu rozseparujeme a zintegrujeme:

d =

d.

To vedie rchlou cestou k vsledku


2
= ,
2

ie

2
.
2

In prstup k rieeniu zvolil a k sprvnemu vsledku sa dopracoval Jan


Hermann. Pozrime sa na situciu v rovine, kde sa namotva pagt. Ak si
ju navye natome tak, e na zaiatku sa pagt dotkal valca v bode A
= [, 0] a kamienok bol v bode B = [, ], kamienok sa zane pohybova
proti smeru hodinovch ruiiek. Von as pagtu bude ma v zvislosti
od uhlu namotania dku = . Polohu kamienka bud urova dva
vektory: poloha bodu, kde sa pagt dotka valca vzhadom na stred valca
a poloha kamienka vzhadom na tento styn bod. Poloha kamienka v naej
sradnicovej sstave je stom tchto vektorov, pouitm oho sa mono
dopracova k rovniciam
= cos ( ) sin ,
= sin + ( ) cos .
Ke sa uhol zmen o d, obe sradnice sa mliko posun a z Pytagorovej
vety mono poveda, e prejden drha je
( )
( )2
2

d
d
d = (d)2 + (d)2 =
+
d.
d
d
2 Ten

sa s asom sce men, ale to nie je iadnou prekkou.

pirla
FX A2
Na zaiatku bolo lano navinut o nulov uhol, na konci pohybu bude navinut
o uhol . Prejden drha je preto jednoducho

d
d

)2

(
+

d
d

)2
d.

Jedin, o ostva, je porta tie karedo vyzerajce derivcie, umocni


ich na druh a odmocni. lovek by si pomyslel, e to bude nespotaten,
ale mete sa presvedi, e po vetkch tchto pravch dostanete pekne
vyzerajci a zintegrovaten vzah

( ) d =

2
.
2

Vieme, e tto drhu prejde kamienok kontantnou rchlosou , take sa


namot za as
2

.
= =

2
o viac sa d ete doda? :-)

FX A3 Momentky
Vypotajte bez pouitia integrlov moment zotrvanosti
(a) obdnikovej dosky vzhadom na uhloprieku,
(b) kocky vzhadom na os prechdzajcu stredmi dvoch protiahlch stien,
(c) pravidelnho tvorstenu vzhadom na os prechdzajcej vrcholom a stredom protiahlej steny.
Zdroj: lohu vymyslel Kubus

l Vzork Bzduo & Kubus l


Najprv si spomeme, o je moment zotrvanosti poda nejakej osi. Ak
mme len jedin hmotn bod, momentom zotrvanosti nazvame veliinu
= 2 , kde je hmotnos nho hmotnho bodu a je jeho vzdialenos od danej osi. Ak mme tchto hmotnch bodov viac, celkov moment
zotrvanosti bude stom prspevkov od vetkch bodov. No a ak je tch
hmotnch bodov cel teleso, pripeme husac krk a suma sa zmen na integrl. Take moment zotrvanosti veobecnho telesa je

=
2 d =
2 d
(1)

Vidme ho tam, hada! Naskyt sa otzka, ako sa integrlu vo vpote


mono zbavi. Skr, ne si tento mal zzrak ukeme, cvine si spotame
as (a) aj pomocou integrlov.
as (a)
Take najprv to sksime karedo pointegrova, vyjdeme priamo z dencie
(1). Kee doska je skoro plon tvar, hrbku meme zanedba a n
integrl sa zmen na dvojrozmern:

=
2 d,
(2)

kde je plon hustota dosky, rovn tie /, ak si ozname


jej hmotnos
a rozmery krt . Dka uhloprieky je potom = 2 + 2 Zvome
si sradnicov systm s osami a rovnobenmi so stranami obdnika a
poiatkom v jeho strede a ozname si ete uhol, ktor zviera uhloprieka so
stranou dky ako .
7

Momentky

FX A3

Potom vzdialenos bodu so sradnicami [, ] od uhloprieky bude =


cos sin .3 Avak sin = / a cos = /. Teraz u meme
spokojne integrova:

+/2

/2
+/2

/2

+/2

( cos sin )2 d d

/2
+/2

/2

2
2 2

+ 2 2 2

d d

Vimnime si, e posledn len v ztvorke sa zo symetrie vyhub, pretoe


integrujeme od mnus dao po plus dao. Preto ho hne teraz vynechme
z potania. Teraz je na ubovli, poda ktorej premennej budeme integrova
ako poda prvej. My si zvolme !4
[

]
2 =+/2
3 2
2
+

d
3 2
2 =/2
/2
(
)

3
+/2 2 3
2
+

d
=
/2
122
2
[ 2 3
]=+/2

3 3

+
=

122
3 2 =/2

+/2

2 2
6 2 + 2

(3)

3 Na pravom z dvojice obrzkov je vzdialenos rozdielom dok znzornench preruovanch pok. Tie predstavuj kolmice na uhloprieku spusten z bodov [, 0] a [0, ].
4 Ide o zvolaciu vetu, preto vykrink. Podobne ako vo vete Vyberm si teba, Pikachu! ,

Momentky

FX A3

Celkom pekn vsledok. Hne mme pocit, e by sa k nemu mohlo da


prs aj nejako jednoduchie. Radi by sme vm teraz predstavili jeden netradin postup na hadanie momentu zotrvanosti niektorch telies. Spomeme si na dve veci:
Prax nm hovor,5 e moment zotrvanosti homognnych telies uritho tvaru sa d vdy vyjadri ako
= (1 , 2 , . . .),

(4)

kde je celkov hmotnos telesa a (1 , 2 , . . .) je funkcia od rznych dkovch parametrov telesa (napr. dky hrn kvdra) majca
rozmer meter tvorcov. To vak znamen, e funkcia mus spa
podmienku.
(1 , 2 , . . .) = 2 (1 , 2 , . . .).

(5)

Steinerova veta nm vrav, e ak moment zotrvanosti telesa o hmotnosti okolo nejakej osi prechdzajcej aiskom je , tak okolo inej
rovnobenej osi ktor je vo vzdialenosti od aiska bude ma to ist
teleso moment zotrvanosti
= + 2 .

Ako by sme to mohli vyui? Rozdeme si pvodn obdnik na 4 menie s polovinmi dkami strn. Moment zotrvanosti je zrejme stom
momentov od vetkch ast. Vidme, e dva z nich maj nulov vzdialenos
aiska od osi. alie dva maj aisko od osi vzdialen o
=

sin =
=
.
2
2
2 2 + 2

(6)

5 A nielen prax, ale aj zdrav rozum. Spomeme si na denciu (1). Keby sme zvili
hustotu telesa -nsobne a zachovali rozmery, tak sa kad prspevok do stu zv nsobne aj cel hmotnos sa zv -nsobne. aa, . Analogicky si vieme odvodi,
e keby sme vetky rozmery telesa -nsobne zvili, vsledn moment zotrvanosti sa
zv 2 -nsobne. To u ale znamen, e MZ mus ma tvar (4).

Momentky

FX A3

Ni nm nebrni zapsa rovnos:


=
(
(
(
))
(
)
)

2
(, ) = 2

+2
+
4
2 2
4
2 2
4

(7)

Spomeme si na rovnicu (5) a dosame aj za zo (6). Pre jednoduchos


ozname = (, ). Tak dostvame rovnicu a nsledne upravujeme
(
)
(
)

2 2
= 2
+
+2
4 4
4 4
4 4 (2 + 2 )
2 2
1
1
= +
4
8 2 + 2
2 2
1
(8)
=
6 2 + 2
Spomeme si, e = a dostvame znova ten ist vsledok.6
as (b)
Tu sa u nejdeme zdrova integrlmi. Pri kocke existuje dokonca viacero
ikovnch spsobov na njdenie momentu zotrvanosti,7 no my ostame pri
prve objavenom triku.
Ozname hmotnos kocky , jej dku hrany a zanime pota. Moment zotrvanosti kocky bude = () = . Rozdeme si kocku
na 8 malch kocoiek s polovinou dkou hrany. Tie bud ma osminov
hmotnos a polovin dku hrany. Ich moment zotrvanosti by teda bol
6 Akoby
7 Ned

inak! Pn Steiner bol zaiste vemi mdry!


mi vak nespomen tento: (pozri aliu stranu)

10

Momentky

FX A3

=
=
8 (/2)
8 4 . Ku vetkmu sa ich aisk nachdzaj vo vzdia
2
lenosti = 4 od osi pvodnej osi, ako to pekne vidno na nasledujcom
obrzku:

Tak meme hne zapsa rovnicu a riei:


(
)

2
= 8
+
8 4
8 8
1
1
= + 2
4
8
1
= = 2
6

(9)

Jednoduch, nie? Tak isto by sme mohli spota aj moment zotrvanosti


kocky okolo telesovej uhloprieky alebo okolo osi spjajcej stredy dvoch
protiahlch hrn. Skste si to.
Keby sme nau kocku zlisovali, vzdialenosti jednotlivch
bodov od osi sa nezmenia. Vzniknut tvorec m teda rovnak MZ. Pre vetky rovinn teles (napr. na dolnom z dvojice obrzkov) vak plat = + , o mono odvodi
z Pytagorovej vety

= (2 + 2 ) d =
2 d + 2 d = +
V prpade nho tvorca si ponechme pvodn os a zvolme si ete dve osi spjajce stredy jeho protiahlch strn.
Ale okolo kadej z tchto dvoch os m tvorec rovnak MZ
ako seka okolo osi kolmej na jej stred. A to je u znma
tabukov hodnota. Dosta sa k MZ kocky je potom u len
vec nsobenia dvomi.

11

Momentky

FX A3

as (c)
. . .tvorsten . . . ve to neme by o ni aie. Rozdelme si tvorsten
na menie tvorsteny a . . . ale poka! Ve to sa predsa ned! no, ned.
Ale predsa len sa sksime trochu s tou geometriou pohra. Majme tvorsten o hmotnosti a strane . Z jeho rohov teraz vyreeme 4 tvorsteny
s polovinou dkou hrany. o nm to ostalo?
Vemi rchlo sa presvedme, e tento ohryzok m 8 stien v tvare rovnostrannch trojuholnkov s dkou strany /2.8 Osemsten!

Ak z vrcholov tohoto osemstenu vyreeme 6 osemstenkov s polovinou


dkou hrany, o nm ostane? Na pravom z dvojice obrzkov vidno, e to
nebude jedno teleso, ale hne niekoko. Po chvli bdania zistme, e sme
dostali 8 tvorstenkov. A uduj sa svete, vetky maj t sprvnu orientciu.9
Nieo nm naepkva, e by sme mohli zabi dve muchy jednou ranou, tak
uvaujme nielen moment zotrvanosti tvorstenu = , ale aj osemstenu
(vzhadom na os prechdzajcu stredmi dvoch protiahlch stien) = ,
kde , resp. je hmotnos tvorstenu, resp. osemstenu a , s neznme
funkcie od dky ich hrany , spajce funkcionlnu rovnicu
(1 ) = 21 ()

resp.

(2 ) = 22 ().

Sasne tie ozname () = a () = , kde = /2 je dka hrany


ohryzku vzniknuvieho pri prvom ohrzan .
Dos oznaovania, zapme konene rovnice. Na avom obrzku vidme, e
pvodn tvorsten mono nahradi jednm osemstenom polovinej a tyrmi
tvorstenmi osminovej hmotnosti, vetko s polovinou dkou hrany. Spomedzi vetkch sa aisk troch tvorstenov nachdzaj vysunut od pvodnej
8 Po

jednom ostalo na kadej stene a po jednom vzniklo pri kadom reze.


s hore nohami , ale to nm nepreka. Dleit je, aby ich osi,vzhadom na
ktor potame MZ, boli rovnoben.
9 tyri

12

Momentky

FX A3

osi o 3/6, teda v psmenkch


=

+4
2

8 4

( )2
3
+ 3
.
8
6

Teraz o vidme na pravom obrzku: Z osemstenu vznikne 6 osemstenkov


s polovinou dkou hrany. Hne zo
symetrie vidno, e ich aisk s od osi
rovnako vzdialen, konkrtne o 3/12.10 S polovinmi rozmermi musia
ma osminov hmotnos. Aby sa nm hmotnos nestrcala, vieme si spota,
e kad jeden tvorsten tvor 1/32 hmotnosti osemstenu, teda m hmotnos
/64.
Ak si ete overme, e dva z tchto 8 tvorstenkov
maj aisko na p
vodnej osi a ostatnch 6 vo vzdialenosti 3/12, meme zapsa druh
rovnicu

(
( )2
)2
(
)

3
+8
.
= 6
+
+ 6
2
16 4
16
12
64 16
64
12
Mme dve rovnice o dvoch neznmych. Ak sa zbavme vade prtomnej
hmotnosti a dme dokopy zlomky s rovnakmi psmenkami, dostvame
ich v krajom tvare:
7
1
1
13
1
5 2
= + 2
a
=
+

8
2
32
32
128
512
Sce sa to nezd, ale tto sstava m celkom sympatick rieenie
1 2
1 2

a
=
.
20
40
Ale to je u nielen rieenie naej lohy! Ako bonus sme spotali aj moment zotrvanosti osemstena. Vsledky teda s:
=

1
2
20
1
1
2 =
2 ,
=
40
10
=

(10)
(11)

kde sme u oboch telies vyjadrili vsledok pomocou ich hmotnosti a ich dky
hrany. (Pripomeme, e u osemstena = /2.)
Nabdame itateov, aby si tmto spsobom sksili vypota in zaujmav momenty zotrvanosti a pokochali sa scenriou bez akhokovek integrovania.11 Kto sa dotal a sem, tie by mohol povaova za zaujmav
10 Premyslite

si!
s integrovanm me by asto vemi zaujmav najm pre milovnkov adrenalnovch portov.
11 Scenria

13

Momentky

FX A3

zistenie, e moment zotrvanosti kocky, tvorstena i osemstena je rovnak


vzhadom na vetky osi prechdzajce aiskom. Kov slovo: sfrick zotrvank.

14

FX A4 Most
Katka si medzi polikou a stolom postavila most. Most je tvoren ahkmi
tyami vone spojenmi kbmi, tak ako na obrzku. Dky ikmch a vodorovnch ty s v pomere 5:6. V bode E je zavesen zvaie s hmotnosou .
Ktor z ty me Katka nahradi ohybnmi vzbami, t.j. naprklad pevnm
pagtom rovnakej dky? Akou silou je namhan ty BD?

Zdroj: Feynmanove prednky z fyziky

l Vzork Filip l
Skr, ne sa pustme do rieenia prkladu, zamyslime sa nad ohybnmi
vzbami. Ty sa od ohybnej vzby naprklad nejakho pagtiku li
len v tom, e m stly tvar. Ohybn vzba sa me ubovone kri (teda
skracova), no neme sa naahova. Zjavne teda ni nepokazme, ak tye,
ktor s naahovan, nahradme pagtikmi. Tye, ktor s stlan, vak
nahradi nememe.12
Po tomto zamyslen sa pustime na prklad. Ukeme si tu tri rzne postupy. Jeden je priamo cez sily, druh a tret cez energie.13
Sily:
V prpade, e sa rozhodneme pota sily, tak si sta napsa vea rovnc
o vea neznmych a snai sa vyrta silu . To je dos neprjemn. Naastie meme vyui matice.14 To sa ahko hovor, ale aie sa to aplikuje
12 Mohli by sme ich vak nahradi naprklad telesom, ktor je nestlaiten, ale pri
rozahovan sa rozpadne. Tak vea z drevench kociek je peknm prkladom takhoto
uda odolnho len stlaeniu.
13 Metdou tzv. virtulnych posunut a virtulnych prc.
14 Rieenie cez sily nie je matematicky jednoduch. Ba dokonca nie je vbec elegantn!
Ak neviete, o s to matice, prejdite priamo na elegantn rieenie cez energie, ktor sa
nachdza o pr strn alej.

15

FX A4

Most

v praxi. o to za rovnice v nej pouvame? Jednoduch rovnice zohadujce rovnovhu sl. Pre kad kb mus plati, e vsledn sila na psobiaca
je nulov in by sa dan kb pohyboval so zrchlenm, o je v statickej
situcii oividne neprpustn. Zapeme si teda vetkch 6 rovnc pre horizontlne zloky sl psobiace na body a (prvch 6 riadkov v matici)
a nsledne aj vertiklne zloky sl pre body a tak, e prv stpec zodpoved koecientom pri sile , druh koecientom sily , tret , at.
Preo pouvame prve tieto rovnice? Preo nie. S pekn, je ich dostaton
poet a hlavne s linerne nezvisl15 .
Treba si vak uvedomi pr detailov. Ak sily psobia na koncoch mosta?
Prve tu je dleit, e bod je na koliesku. Hoci tam psob nejak (zatia
neznma) vertiklna sila, horizontlna zloka mus by nulov, in by sa
koliesko hbalo. Na most psob tiaov sila smerom dole a teda vieme, e
aby bola celkov vslednica sl psobica na most nulov, horizontlna zloka
mus by nulov aj v bode . Je celkom oividn, e set vertiklnych
zloiek je rovn tiai. Ke u mme premyslen rovnice, tak si dajme pozor
na znamienka. Zvolme si smer sradnicovch os a peme. Kee vslednica
sl psobiacich na tye mus by nulov, ty psob na oboch koncoch opane
orientovanou silou.
A u nm ni nebrni pusti sa do psania. Toto je teda naa matica, ktor
chceme upravi. Pre prehadnos sme ju vynsobili piatimi (nsobenm sa
ni nepokaz).

15 O

3
5
0
3 0
3
0 5 3
0
0
0
0
0
0
0
0
0
4 0 4
0
0
4
0
0
0
0
0
0
0
0
0

0
0
0
0
5
0
0
0
0
3
5
0
5 3 0
3
0
0 5 3
0
0
0
0
0
0
0
0
0
4
0
0
0 4 0 4
0
0
0
4
0
0
0
0

0
0 0 0
0
0 0 0
0
0 0 0
5
0 0 0
0
3 5 0
5 3 0 3
0
0 0 0
0
0 0 0
0
0 0 0
0
4 0 0
0 4 0 4

om sa d presvedi pravami naej matice alebo zamyslenm sa.

16

0
0
0
0
0
0
0
0
0
5
0

FX A4
pravou tejto matice

1
0

=
0
0

0
0

Most
na redukovan stupovit tvar16 dostvame maticu

0 0 0 0 0 0 0 0 0 0 5/12
1 0 0 0 0 0 0 0 0 0
1/4

0 1 0 0 0 0 0 0 0 0
5/12

0 0 1 0 0 0 0 0 0 0
1/2

0 0 0 1 0 0 0 0 0 0 5/12

0 0 0 0 1 0 0 0 0 0
3/4

0 0 0 0 0 1 0 0 0 0
5/12

0 0 0 0 0 0 1 0 0 0
1

0 0 0 0 0 0 0 1 0 0
5/6

0 0 0 0 0 0 0 0 1 0
1/2
0 0 0 0 0 0 0 0 0 1 10/12

Skvel, mme to, o sme chceli! Sila =


2 . Po tokej radosti si
vak prezradme aj mal problm takhoto rieenia. asov zloitos tohto
potania je (2 ), ie pri vekom moste by sme sa naozaj nartali.17 Na
druhej strane je vak vekm poteenm, e po vyrieen matice dostaneme
darek zdarma18 zoznam ty, ktor meme nahradi pagtikom. S to
prve tie koecienty, ktor vyli kladn, o znamen, e ahaj kb k sebe.
S to tye , , , , , .
Energie a momenty sl:
Tu stailo ukza (a zdvodni), ktor palice s nahraditen a nsledne
vypota silu psobiacu na ty BD.
Pri zdvodovan vyuijeme princp virtulnych prc. Hoci mme dokonale tuh paliky, predstavme si, e nie s spojen pevnm kbom, ale dria
ich mal trpaslci.19 Teraz si predstavme, e trpaslk rozpa jednu ruku, o
je vemi podobn situcii, ako keby sa palika predila o d.20 o sa stane
s energiou sstavy? Principilne s mon tri vsledky. Potencilna energia
sstavy sa zv, zmen alebo nezmen. Ak sa potencilna energia zv,
trpaslk mus vykona prcu, a preto meme poveda, e palica je stlan.
Naopak, ak sa potencilna energia zmen, trpaslk mus tejto zmene zabraova, palica je naahovan. To, e by sa potencilna energia nezmenila, sa
nm nestane, ale znamenalo by to, e tu nepsob iadna sila a teda iadnu
ty ani nepotrebujeme. Ak jednoduch.
16 Tak

sa vol t vec, na ktor sa matica pri rieen sstavy rovnc upravuje vdy. :-)
je to prca pre pota. Mne sa nechcelo maticu s jedenstimi riadkami ani napsa na papier!
18 Ani nemusia by Vianoce. :-)
19 Niektor maj tri i dokonca tyri ruky!
20 Ako to u v rozprvkach bva, trpaslci s mal. Neskr vs nauia, e trpaslk je tak
epsilonov.
17 Pravdupovediac,

17

FX A4

Most

Pozrime sa teraz na tye , a To s tie na spodnej strane


mosta. Ak by sa predila ubovon z nich, most by sa prehol do tvaru
V. To znamen, e potencilna energia zvaia klesne.21 Naopak, pri preden ty , , , sa most prehne naopak. Zostvaj u len
uhloprieky. Ale aj tu sa sta len trochu zamyslie. Sta si len predstavi,
ako sa pomenia uhly, ke sa jedna z nich natiahne a i sa most prehne do
tvaru V alebo opane. vahami dospejeme k rovnakm tyiam, ako ke sme
potali psobiace sily.
Ke u sme hotov s prvou asou lohy, pome vyrta silu, ktor stla
ty . S vyuitm rovnosti momentov sl to ide vemi rchlo. Vieme, e
set sl psobiacich od podloky v bodoch a mus kompenzova tiaov
silu telieska a sasne ich moment sl vzhadom na mus by nulov. S
to toti jedin vonkajie sily psobiace na most, ktor vzhadom na teliesko
spsobuj nejak momenty.
= +
=
12 = 6

=
=
2
3
Zamyslime sa teraz nad asou mosta a zvome si v om bod .
Vsledn moment vonkajch sl psobiacich na trojuholnk vzhadom na
tento bod (ako aj na ubovon in) mus by nulov. Vonkajie sily psobiace v maj moment nulov, zostvaj nm teda iba sily psobiace
v bodoch a . Sila v smeruje kolmo hore22 a u sme ju vyrtali. Jedin
vonkajia sila psobiaca v bode je od tye , teda smeruje vodorovne.
no, je presne t, ktor chceme vyrta. Ozname ju . Vka trojuholnka
je 4. Take dostvame rovnicu:
6 4 = 0
=

.
2

Energie a geometria:
Op si na pomoc pozveme trpaslkov, ale alej budeme postupova vlune geometricky. Predstavme si, o by sa stalo, ke sa zv dka
21 Pozor! Uvedomme si, e pri nehmotnom moste je hmotn zvaie jedin vec, o ns
zaujma. Avak vo veobecnom prpade by to a tak jednoduch nebolo a museli by sme
poctivo vyrta zmenu celkovej potencilnej energie celej sstavy.
22 Bod je na pohyblivom koliesku a preto ubovon horizontlna sila bude hba
s kolieskom, a do polohy, v ktorej bude nulov.

18

FX A4

Most

o d. Tm sa nm zmen aj uhol , ozname to ako + d. Plat:


3 + d/2
5
3 + d/2
sin() cos(d/2) + cos() sin(d/2) =
5
3
4
3 + d/2
1 + d/2 =
5
5
5
d
d =
.
4
sin( + d/2) =

Most sa nm v bode prehne a vznikne trojuholnk s vkou d.


ahko si vimneme, e uhol pri vrchole je d. Ak ozname uhly pri
ako d a pri ako d, tak vieme, e d = 2d.23 Set uhlov m by
a preto
d
d
d =
=
.
3
12
Odtia u pre zmenu vky zvaia mme
d
d

= 6d
d
=
.
2

A mme vsledok ak trpaslk rozpa o d, potencilna energia sstavy


klesne o d/2. To znamen, e na drhe d musel vykona zporn prcu
takejto vekosti a teda ty je stlan silou /2.

23 Toto plat iba pre mal uhly alfa a beta, ke je vka d trojuholnka vemi
mal.

19

FX A5 Krok
Mme horizontlny upevnen valec s polomerom . Na om je navleen
prstenec s polomerom (pochopitene > ) a hmotnosou . Vypotajte
peridu malch kmitov tohto prstenca (v jeho rovine), ak po valci nepremykuje.
Zdroj: KMaL

l Vzork Kubus l
Energetick prstup:
Pohyb obrue na valci (ktor vbec nie je tak jednoduch, ako by sa na
obyajn kolsanie mohlo zda) si meme rozdeli na dva ovea jednoduchie
a zkladnejie pohyby: Pohyb jej stredu (aiska) po krunici s polomerom
, ako to naznauje nasledujca dvojica obrzkov, a otanie obrue
okolo svojho aiska. Zaveme si sradnicu ako uhol, ktor zviera spojnica
stredu obrue, stredu valca a dotykovho bodu obrue so zvislicou.

A o ak uhol, ozname ho , sa otoila obru ako tak?24 Obru i valec


sa dotkali nejakm sekom ich povrchov a ich dky musia by na oboch
telesch rovnak.25 Spota dku krunicovho oblku je hraka. S pomocou
obrzku dostvame rovnos
( ) =

= ( ) .

(12)

mono fakt, e uhol je odlin od , ukza nasledovne. Predstavte si,


e ke bola obru vo svojej najniej polohe, nakreslili sme na jej dotykov bod s valcom
mal erven bodku. Ke sa obru trochu preval, dotka sa valca inm bodom. Keby
platilo = , dotkali by sa stle v tej iste ervenej bodke, o sa d dosiahnu len
mkanm obrue po povrchu valca nie jej prevaovanm.
25 Predstavte si, e na ich povrchoch s miniatrne ozuben kolieska. Obe sa posunuli
o rovnak poet rovnako vekch zbkov. To sa rovn rovnakej dke.
24 Jednoducho

20

FX A5

Krok

Teraz nm u ni nebrni napsa si rovnicu zkona zachovania energie a


upravova:26

= kin + rot + pot


1
1
=
2 + 2 +
2
2
1
1
2
=
[( ) ]
+ ( 2 ) 2 + ( ) (1 cos )
2
2
1
2 2
( ) + ( )2 ,
(13)
2
priom v poslednej prave sme vyuili (12) a s vedomm, e sa pozerme
iba na mal vchylky , tie aproximciu cos 1 2 /2.27 Rovnicu (13)
meme pokojne zderivova poda asu:
( )2

d( 2 )
d

( )2 (2 )
+

1
d(2 )
( )
2
d

= 0

( ) ()

= 0

Po vykrten nadbytonch lenov28 a troche usporadvania dostvame


diferencilnu rovnicu pre harmonick kmity:

(14)
2( )
Rieenm tejto rovnice je funkcia tvaru cos() + cos(), kde

=
.
2( )
26 Pouvame tu tandardn bodkov notciu pre asov derivciu premennch:

d/d,
d/d

= d2 /d2 .
27 Skste si to odvodi z Taylorovho rozvoja kosnusu alebo binomickho rozvoja odmocniny po aplikcii Pytagorovej vety.
28 V nvale krtania si vimnime, e av strana je nsobkom
aa, nehybn krok
je tie rieenm.

21

FX A5

Krok

Ns zaujma perida kmitov. T je

2
2( )
=
= 2
.

Silov prstup:
lohu meme vyriei podobnmi vahami, ak budeme pouva sily namiesto energi. Kee sa nm krok to, budeme pouva radej momenty
sl. Vemi uiton bod, okolo ktorho meme tieto momenty pota, je
stred valca. Napme si druh vetu impulzov:
=

(15)

Podobne ako v minulom prpade, moment hybnosti si meme rozdeli


do dvoch ast: Pohybu aiska okolo stredu a otania krku okolo aiska,

teda = ( ) + ,29 kde = ( ) a = .


Ak je moment sily psobiaci na krok? Jedin sila, ktor spsobuje
nenulov moment sily, je tia s psobiskom v aisku. Jej moment je
= ( ) sin ,

(16)

priom pre mal meme znovu napsa sin . Dosame teda vetko
do rovnice (15), spomeme si na (12) z predolho postupu a upravujme:
( )

= ( ) +

( )

= ( )2 + ( 2 )

= (2 )

(17)

Tto pohybov rovnica nm dva peridu kmitov krku

2
= 2
.

Juj! Silov a energetick prstup dvaj rzne vsledky. Rovnice (14) a


(17) nie s rovnak ale oni predsa musia by, ve popisuj ten ist krok!
Vsledok mus by ten ist bez ohadu na to, ak prstup pouijeme!
29 So znamienkom plus, lebo v sradniciach, ako sme si ich zvolili my, s v rovnakom
smere.

22

FX A5

Krok

Silov prstup revisited:


Samozrejme, e nm to vylo inak. Zabudli sme na trenie. Trenie nememe len tak zanedba, pretoe je vemi dleit, priam potrebn. Bez neho
by krok dokonale premykoval.30 A kee momenty potame vzhadom
na stred valca, moment sily spsoben trenm je nenulov. Tomuto by sme sa
mohli vyhn potanm momentov vzhadom na dotykov bod, ale sstava
s nm spojen nie je inercilna a museli by sme uvies nejak sily navye.
V skutonosti sme zamlali ete jednu silu, a to kolm reakn silu od valca,
ale jej moment je naozaj nulov a kee je kolm na pohyb aiska, nekon
ani iadnu prcu. Ozname si teda treciu silu ako a napme si impulzov
vety ete raz.
= sin +
= ( ) sin ,

(18)
(19)

kde rovnica (18) popisuje pohyb obrue v smere rovnobenom s povrchom.


Vimnime si znamienka . Ak sa dohodneme, e m opan smer ako
kolm komponent gravitcie, potom m rovnak moment, pretoe psob na
opanej strane od bodu otania.31
Pomocou starch znmych vec upravme av strany rovnc (18) a (19)
nasledovne:
d
( ) = ( )
(20)
d
(
)
d
( ( ) + ) = ( )2 + 2

(21)
d

Rovnica (18), resp. (19) prejde pomocou (20), resp. (21) na tvar
( ) = +
(22)
)

( )2 + 2
(23)
= ( ) .

Teraz ku rovnici (23) pritame rovnicu (22) vynsoben , aby sme eliminovali . Potom smelo upravujeme. Po nevekej prci prdeme k rovnici

.
=
2( )
(

To sa zhoduje s rovnicou (14) zskanou z energetickho prstupu. Vetko


je teda v poriadku, nemme tu iadny spor. Skuton perida kmitov je

2( )
= 2
.

30 Vimnime si, e trenie potrebujeme zohadni v silovom prstupe, i ke nekon iadnu


prcu. Keby vak bolo menie, mohlo by sa sta, e krok by zaal premykova a energia
by sa zaala strca!
31 Mohli sme si to zvoli aj naopak, potom by nm vylo zporn.

23

FX A6 Kladky
Tinka nala v pivnici obrovsk kopu lana a kladiek, preto sa rozhodla
postavi si kladkostroj. Ba dokonca dva! Jeden kraj ako druh, ako to vidno
na obrzkoch. Ak bud zrchlenia jednotlivch telies po uvonen kladiek?
Hmotnosti vetkch kladiek i lana s zanedbaten, poet kladiek na pravom
obrzku povaujte za nekonene vek.

Zdroj: BAUPC & Archv FKS

l Vzork Bzduo l
aa. Kladky. A aby ich nebolo mlo, tak hne nekonene vea. Pre
zaiatok si vak dajme przemnejie ciele Zhrnieme si, ako t fyzika naozaj
vyzer, ke je kladiek mlo (naprklad jedna):
Lano vone natiahnut medzi kladkami, resp. medzi kladkou a zvam
je po celej svojej dke napnan rovnakou ahovou silou.
Preo? Uvaujme ksok lana malej dky. Na hornom konci je ahan
ahovou silou nahor, na dolnom silou + d nadol. Rozdiel tchto
sl spolu s tiaovou silou udelia ksku pagtu s hmotnosou d zrchlenie .32 Z Newtonovho zkona mme d = d( ) d.
Vrmci lana sa teda sce ahov sila men, ale rdovo len o tia lana.
Kladkostroje dvhaj ak bremen, take psobia rdovo vmi silami, ne je tia lana. Mal zmeny mono zanedba.
32 Rdovo! V relnych kladkostrojoch dosahuj lan a zvaia zrchlenia od 0 po niekoko mlo .

24

Kladky

FX A6

Set sl psobiacich na nehmotn kladku je 0 (nula) a to aj v prpade


visiacej (nie pevne uchytenej) kladky.
Dvod je podobn a platnos rovnako (ne)presn: Za bench okolnost
je tia kladky vemi mal v porovnan s psobiacimi silami. Nerovnovha tchto sl by poda = /0 viedla k obrovskm zrchleniam.
Tie by vak mali by rozumne mal .33 V limite 0 0 mus nutne
0.
Lano prehoden cez nehmotn kladku je napnan rovnakou silou na
oboch stranch kladky.
Rozdiel vo vekosti tchto sl by viedol k uhlovmu zrchleniu
kladky = /0 (tu je polomer kladky). Mus vak nadobda
rozumne mal hodnoty ( /) a preto pre 0 0 mus aj 0.
V praxi vinou, naozaj, .34

Zbrane nabit, vojsk rozostaven. Ideme bojova! :-)


as (a)
Ozname si jednu z ahovch sl ako . Nech je to napr. t, ktor psob
na prav teleso nahor. Kee s kladky nehmotn, tak ah v lane mus by
33 pecilne

pre upevnen kladku nulov! Vtedy tvrdenie plat automaticky.


je to len tak mvanie rukami vo vetre. Mj kamart Bus raz povedal
Kubusovi toto: Kubk, vimol si si niekedy e kad fyziklny dkaz je tak nedveryhodn, e sa vdy uvdza aspo s jednm konkrtnym prikladom pre nejak plne trivilny
pecilny prpad? Dokonca v prpade, e dokazovan tvrdenie neplat sa pre istotu uved
dva plne rozlin dkazy. Preto si skste zrta tento plne trivilny pecilny prpad:
Cez otav kladku je prehoden lano, na ktorho koncoch s zavesen zvaia s hmotnosami 1 , 2 . Moment zotrvanosti kladky je 0 , jej polomer . Vypotajte, o sa
len d. Z troch rovnc (Newtonov zkon pre obe teles + momentov veta pre kladku)
s troma neznmymi (ahov sily 1,2 v lanch + zrchlenie sstavy ) urme vetko
mon, pecilne aj
(1 2 ) 0
1 2 =
,
(1 + 2 ) 2 + 0
o ide naozaj do nuly v limite 0 0 a o dva nenulov hodnotu jej uhlovho zrchlenia
34 Samozrejme,

(1 2 )
(1 2 )
=
.
0
(1 + 2 ) 2 + 0

25

Kladky

FX A6

rovnak na oboch stranch kadej jednej. Tak dostvame nasledujci obrzok


(vetky pky s rovnako vek):

Skr, ne zaneme zapisova pohybov rovnice, si vak vimnime sily


psobiace na prav kladku. Nahor ju ah sila , nadol 2 . Avak, ako sme
si povedali vyie, set sl psobiacich na kladku je rovn nule. Preto nutne
= 0. Ni netreba riei, obe zvaia bud pada vonm pdom.
Vne? Neupadli sme slepo do nesprvnych dadaistickch vah? Nie,
neupadli! Ak uvaujeme, e teles s rdovo aie ako kladky, tak je to
naozaj tak. Finta je v tom, e teles vlastne vbec nie s uchyten. Ak
si predstavme hmotnos avho telesa sstreden v kladke nad nm, tak
tto kladka a druh teleso sa snaia pada vonm pdom. Brni im v tom
nieo? Nebrni! Vimnite si, e posunutm pravej kladky nadol o sa uvon
pagtu (dole odbudne 2 a hore pribudne ), ktor sa me vyui na
klesnutie zva. Hmotnos kladky je zanedbaten, take pohybu telies sa
podd. Takto to bude, a km sa doln dve kladky nezrazia a prav kladka
u nebude mc dalej klesa. Nie je v tom iadny spor To len zdravmu
sedliackemu rozumu sa nieo nepozdva. Zrejme je to tm, e kolabujce
kladkostroje sa zmerne nestavaj.35
as (b)
Ne sa pustme do samotnho rieenia, urobme jednu astrlnu vahu:
Predstavte si, e mte doma na povale (tj. v gravitanom poli ) kladkostroj
(hocijako komplikovan). Teoreticky alebo experimentlne urte zrchlenia
vetkch telies a ahov sily vo vetkch lanch. Vzpt sa presuniete na
in plantu36 s gravitanm poom a postavte tam rovnak kladkostroj
s rovnakmi zvaiami. Ak bud nov zrchlenia a nov ahov sily?
35 A

ako ukazuje tento prklad, postavi sa daj.


hovorme o astrlnej vahe.,

36 Preto

26

Kladky

FX A6

Sksenos ukazuje, e vetky sily a zrchlenia na kladkch s priamo


mern tiaovmu zrchleniu.37 Nov ahov sily a zrchlenia teda sta prensobi bezrozmernm a to sa bude dia. Predsa len som vak trochu
zavdzal, ke som povedal, e vaha je astrlna. V skutonosti je celkom
przemn a to op doslovne, pretoe ju mono realizova aj v zrchujcom
vahu. Tu zrejme sta spravi zmenu .
Vrme sa ku kladkostroju v zadan. Na avej strane najhornejej kladky
je zvaie . Na pravej strane je... ale ve to je ten ist kladkostroj! no,
m o jednu kladku menej, ale to je nekonene mal (= nijak) zmena.38 Ak
vetky kladky okrem prvej zavriem do jednej vekej krabice, vyrobil som si
spomnan vah, v ktorom namiesto pociujem + , kde je zrchlenie
prvho telesa nadol.
Dva rovnak kladkostroje. Jeden zavesen v gravitanom poli cti ,
druh pad v gravitanom poli, cti + . Vieme, e podiel ahovch sl
v prslunch lanch kladkostrojov bude / + . Ale ten vieme uri! Vezmime si naprklad ahov sily v lanch psobiace na prv a druh zvaie.
ahko prdeme k nasledujcemu obrzku:

Tad. Mus plati:

=
/2
+
= /2

(24)

Prv teleso sa teda pohybuje nahor so zrchlenm /2. Zrchlenia ostatnch telies dopotame analogicky. Ak sa pohrme s geometrickmi radmi,
37 Argumentov je viacero, uvediem jeden rozmerov: Ak mme na potanie k dispozcii
vea hmotnost a jedno , zrchlenie mono vyrta len ako krt bezrozmern faktor
(tj. nejak podiel hmotnost). Pre sily zasa = , o je linerne zvisl na ako
dsledok linernej zvislosti pre vetky zrchlenia.
38 Matematici by ma po pretan tejto vety mohli zavesi na hk. Pri pouit slova
nekoneno vo fyzike si v skutonosti treba predstavi relnu situciu so veobecnmi
hodnotami a potom spravi limitu.

27

Kladky

FX A6
zrchlenie -tho zvaia v smere nadol dostaneme ako
=

2 3
.
2

Alternatvne rieenie asti (b)


Ukeme si trik, vaka ktormu pre ns u vina prkladov s vea
kladkami nikdy nebude problm. U sme spomenuli, e vetky ahov sily
v lanch s priamo mern pociovanmu zrchleniu . Polome si bsnick otzku: Mono hocikde v kladkostroji kladku so zvaiami nahradi
jednm zvam? Aby bolo jasn, o tm chcel bsnik poveda, chcel tm
poveda toto:

Kee som zl bsnik, odpove vm prezradm hne: D. Uvaujme


najprv situciu, kde je kladka upevnen k stropu (tj. pociuje ). Lano,
ktor ju na strope dr, bude pri tom napnane nejakou silu , ktor sa
mus da zapsa ako . Pome njs, omu je rovn .
Ak ozname zrchlenie zva a naptie v dolnom lane ako , dostvame sstavu rovnc
1 = 1
2 = 2
s rieenm
=

1 2

1 + 2
21 2
.
1 + 2

(25)

Z rovnovhy sl alej = 2 , take dostvame


=

41 2
.
1 + 2

To vak ete nie je asn. asn je a skutonos, e keby tto kladka


bola zavesen vo vahu alebo by padala na nejakej inej kladke, take cel
28

Kladky

FX A6

sstava by pociovala nejak in , zmenu mono znova urobi, pretoe sila


nm vyla priamo mern .39 Meme teda zakresli nau spokojnos!

Vrme sa k nekonene vea kladkm v zadan. Kadej z nich mono


prisdi nejak hmotnos . Vetky kladky s vak identick, mus teda
plati
=

4
,
+

o je po roznsoben kvadratick rovnica s dvoma rieeniami


= 3

alebo

= 0.

Nekonene vea kladiek si mono predstavi ako jednu kladku, ktor m


naavo zvaie a napravo zvaie . Z rovnice (25) dostvame pre zrchlenie najvyieho zvaia

= ,
2

resp.

= .

Rados vak strieda zmtenie. Mme dva vsleky. Mimochodom, ak sa


dobre zahladme na rovnicu (24) z predolho postupu, tak aj ona m skutone rieenie = , pokia = 0. Aby to bolo vidno, sta roznsobi
na

( ) = .
2
Toto rieenie je celkom zmyslupln. Zrchlenia jednotlivch telies v om
vyzeraj nasledovne:
39 Inak

to ani nemohlo vyjs!

29

Kladky

FX A6

Dve rieenia. V skutonosti sa me dia len jedno z nich. Problm je


v naom nedslednom pouvan slova nekoneno. Kad fyziklny problm,
treba zadenova pre konen veliiny a a vzpt spravme limitu s nekonenom. Pod konenou situciou si predstavme tak, ktor obsahuje
kladiek, priom posledn je zakonen nejako takto:

Poda zadania 0 = , ale budeme pracova so veobecnm 0 , aby sme


pochopili, kde je problm. Ak pouijeme nau ntu s , meme kladky
zospodu postupne nahradzova zvaiami. Doln kladku ahko nahradme
hmotnosou
40
1 =
+ 0
a pre vyie kladky plat
+1 =

4
,
+

kde oznauje poradie kladky potajc zdola. Pekn rekurentn vzah.


Ns zaujmaj jeho konvergentn vlastnosti. Pre lepiu prehadnos zaveme
= /, teda hmotnos potan v jednotkch . Rekurentn vzah
ahko upravme na
4
+1 = 4
,
1 +
ie ide o hyperbolu. Zakreslme si jej as v kladnch slach gracky:
30

Kladky

FX A6

Vidme, e pre vetky 0 > 0 konverguje 3. Jedin prpad,


kedy rad konverguje k nule je pre 0 = 0, tj pre von pagt. Aj ten vak
m v skutonosti nepatrn kladn hmotnos,40 take sprvnym rieenm je
skutone

= .
2

40 O

hmotnosti nekonene vea nehmotnch kladiek ani nehovoriac...

31

FX A7 Motorov pla
Predstavte si reaz na motorovej ple. Jej dka je , hmotnos a plne
bez trenia sa napnut pohybuje rchlosou okolo dvoch valekov s polomerom . Predpokladajme, e valeky sa neotaj. Reaz nijakm spsobom
nepohame ani nespomaujeme.
(a) Za ak as klesne jej rchos pohybu na polovin?
(b) Kvalitatvne popte (a zdvodnite), o sa stane s reazou, ak by valeky
(a zvyok motorovej ply) zrazu zmizli.

Zdroj: lohu vymyslel Kubus

l Vzork Bzduo & Kubus l


as (a)
Ozname si = / dkov hustotu reaze a pozrime sa na to, preo
by vbec mala zaa spomaova. Zamerajme sa iba na kus reaze, ktor
sa momentlne nachdza v blzkosti jednho z valekov. Tento kus sa ota
okolo valeka a vracia sa nasp, no a pri tom men svoju hybnos. Zmenu
jeho hybnosti vypotame neformlne: za as d zmen kus dky d rchlos
z + na , preto
d
2 d
=
= 2 2
d
d
To znamen, e na to, aby si reaz udrala svoju rchlos, musme ju
stle aha silou vekosti 2 2 . To ist plat o druhej polovici reaze, take
na udranie stavu s rchlosou potrebujeme vkon
= = 4 3
Predtm ne by sme sa do dadaistickch vsledkov ponorili prihlboko,
vs musm upozorni, e je to zjavne blbos. Tento vsledok sa urite nepi
zkonu zachovania energie ani momentu hybnosti a argument sa d rovnako
aplikova na toiace sa koleso. Kde udlali soudruzi z NDR chybu?
Sksme sa zamyslie nad tm, o vlastne psob na reaz nejakou silou.
Uvaujme nejak ksok reaze, ktor sa prve obta okolo valca. Ten evidentne men svoju rchlos, preto na neho mus psobi nejak sila. Trenie
neuvaujeme, preto jedin, o na ksok reaze me psobi, s susediace
32

Motorov pla

FX A7

ksky reaze.41 Inmi slovami, v reazi je nejak pnutie. Ete by sme nemali
zabudn, e valce mu psobi na reaz na povrchu nejakou tlakovou silou
od podloky. No dobre, ale ako to bude spomaova pohyb reaze?
Valce sa nehbu, reaz je teda akmsi spsobom napnut a nem monos zmeni svoj tvar. To znamen, e vetky je asti musia ma v kadom
okamihu rovnak vekos rchlosti, a teda tie rovnak vekos spomalenia.
Teraz nasleduje jedna diferencilna vaha. Ak reaz dostatone zazoomujeme, kad jej ksok bude prakticky rovn. Aby sme dosiahli nejak (obvodov) spomalenie , musia na koncoch ksku dky d psobi sily, vekosou
sa liace o
d = d = d.
To ete neznie ako problm, ale ak sa posunieme o d alej, sila (vzjomn psobenie ah v reazi) mus by znova o d vie. Po kskoch
vak meme prejs dokola po celej reazi. Tak zrazu zisujeme, e tam, kde
posledn ksok psobil na prv silou , prv ksok psob na posledn silou
+ = + . To je ale proti zkonu akcie a reakcie. Tieto sily musia
by rovnak, teda jedin mon hodnota zrchlenia mus by nula. Tie
zisujeme, e ah je pozd celej reaze kontantn.
Vsledok? no. Z nioho ni nm spadol z neba. Argumentcia je vak
jednoduch: Cel reaz mus ma rovnak vekos zrchlenia. To znamen
kontantn gradient ahovej sily v reazi. T vak mus by po prejden jednej otoky rovnak ako na zaiatku a teda gradient ahovej sily je nulov. Na
kad ksok reaze psobia na koncoch rovnako vek sily takmer opanho
smeru, ktor mu ma jedine dostrediv inky na valec v kolmo smere.
aa, reaz nebude spomaova.
as (b)
Intucia nm naepkva, e po zmiznut valcov bud reaz deformova
ahov sily. Ve ni in na reaz nepsob.42 Ak vek bude ahov sila
v reazi? Argument pouit v asti (a) predpokladal nemeniaci sa tvar reaze
a rovnak zrchlenie pozd celej reaze. Ten vak teraz nemono poui.
Ve prve zmena tvaru je to, o ns zaujma. Nevieme posdi, i je pnutie
v reazi vade rovnak, nieto ete ak je vek. Stojme pod zaujmavm
problmom. Mme vea spojench kskov reaze, ktor sa ria znmymi
smermi. Akmi vekmi silami bud na seba psobi susedn ksky?
Je bezpochyby obrovsk problm riei takto poloen problm. Pome
sa radej vrti do asti (a), kde boli valce ete namieste a uvaujme nad
tm, akmi silami psobili na reaz. Valce psobili na kad ksok reaze
41 Tie

je nm jasn, e n ksok reaze psob reakciou na susedn ksky reaze.


osvieenie pamte: Odstrediv sila existuje len v neinercilnych (rotujcich) vzanch sstavch. My rozprvame o inercilnej. Preto v nasledujcich vahch nem o
hada!
42 Pre

33

Motorov pla

FX A7

nejakou malou reaktvnou silou. Z inho uhla pohadu vieme poveda, e oba
valce psobia na reaz rovnako vekmi silami opanho smeru, ozname ich
vekos .

Na obrzku je zobrazen kus reaze, ktor sa pohybuje rchlosou . Sily


znzoruj pnutie psobiace na koncoch kusu reaze. Vslednica tchto
sl zmen za as d ksoku reaze o dke d a hmotnosti d rchlos
z + na . Z Newtonovho zkona tak dostvame:
2 =

d
= 2 2 ,
d

priom sme u odvodnili, e pnutie mus by po celej dke reaze rovnak.


Sila vak me by ubovon v zvislosti na tom, ako situciu postavme.
Rovnako tak si meme zvoli aj = 0, o je zrejme fyziklne najbliie
k situcii bez valcov. To je prpad, kedy je reaz u natiahnut medzi valcami
na sprvny tvar, ale ete nie je napnut. Vidme, e v tom prpade je v reazi
pnutie
= 2 .
Ale poka! Tu nm z neba pad al vsledok. Ak si predstavme, e
valce nenapnaj reaz, tak akoby tam vbec neboli. Ako sa bude reaz pohybova za tchto podmienok? Uvaujme ksok reaze idci po krunici s polomerom , ktormu prislcha stredov uhol d.
Na jeho koncoch psobia vyie uren sily , ktor zvieraj uhol 180
d. ahko sa mono presvedi, e ich vslednica je d = d. Tto sila
je kolm na smer pohybu a spsobuje dostrediv zrchlenie
=

d
d

2
2
=
=
=
= .
d
d

aa, dostrediv sila psobiaca na ksok s polomerom krivosti nm


vyla tak, e nti tento ksok pohybova sa po krunici s polomerom .
Nem teda snahu zjs do valca, ale samovone sa pohybuje po jeho povrchu.
Inmi slovami,reaz nemus by nijako vytlan z valca nepsobia na seba
nijakou silou. Prekvapiv zistenie: Reaz nebude meni svoj tvar.
Vsledok? Vyzer to tak. Znova sme sa k nemu dostali len vemi nenpadne. Ak uvaujeme, e valce napnaj reaz nulovou silou, reaz sa sama
napne prve natoko, aby nemenila svoj tvar. Dostali sme tak bezrozporn
34

Motorov pla

FX A7

rieenie, ako sa bude reaz pohybova. Rieenie, ktor vyhovuje vetkm zkonom mechaniky. Tak me existova jedin to sprvne.
Poznmka: Vimnite si, e polomer nebol v rieen nijako pecick.
Hral iba lohu poiatonho zakrivenia. Ak by sme pomocou vea valekov
nechali reaz pohybova sa v hocijakom terickom tvare, po zmiznut valekov
by na kad ksok psobila prve tak sila, aby sa v danom mieste reaz
ostala pohybova s danm zakrivenm. Jej tvar by sa nemenil.

35

FX A8 Rusk sud
Fajo zhal na trhu sud, do ktorho bude dva jablk na plenku. Jeden
obchodnk mu ponkal vemi zvltny sud. Mal tvar valca, jeho podstavy boli
z pevnho kovu. Pl je vak vyroben z prunej gumovej ltky, do ktorej s
votkan tenk ohybn a neroztiahnuten vlkna tak, ako na obrzku. Fajo
sa hne zaal zaujma, o sa stane, ak do sudu natlakujeme vzduch. Ktor
z troch tvarov na obrzku zaujme sud v zvislosti od uhla ?

Zdroj: Kvant

l Vzork Kubus l
Napriek tomu, e vyzer hrzostrane, bol tento prklad celkom ahk.
Stailo si poriadne premyslie geometriu celej situcie a nakresli si psobiace
sily.
Predstavme si Fajov sud v poiatonej polohe v tvare valca. Ozname si
jeho polomer ako , jeho vku ako a vekosti uhloprieok jednotlivch
kosotvorekovch oiek medzi vlknami v jeho plti ako a , vetko tak,
ako na obrzku.

Ako je u naznaen aj na obrzku, pl sudu bude ma rozmery 2


krt . Na rku (pozd obvodu podstavy) v om teda nartame = 2

36

Rusk sud

FX A8

oiek a na vku = oiek. Navye, pre neskorie pouitie z obrzku


vytame, e tg = .
Teraz do sudu natlakujeme vzduch, ozname si rozdiel tlakov vntri a
vonku sudu ako , a pozrieme sa, ak sily bud psobi na jeho pl (ktor
je ete stle v tvare valca).
Na kad podstavu valca psob tlakov sila vekosti 2 . Takouto silou
je teda rozahovan pl v zvislom smere inmi slovami, ak by sme ho hocikde rozstrihli vodorovnm rezom, potrebovali by sme obe asti dra silou
2 . Kee v kadej vke valca njdeme veda seba kosotvorekovch
1
oiek, kad z nich mus nies
-tinu tejto sily. Inmi slovami, ak by sme
spomnan vodorovn rez viedli cez uzly, kde sa vlkna pretnaj, prerezali
by sme rovnocennch uzlov, preto je kad z tchto uzlov rozahovan
vertiklnou silou vekosti
=

2
2

=
=
.

2/
2

Len o nieo zloitejie je njs silu, ktorou je pl napnan vo vodorovnom smere. Ozname si tto silu ako a zhora sa pozrime na vsek pla
zaberajci obvodov uhol 1. Na tento vsek psob zvyok pla dvoma
silami vekosti , zvierajcimi uhol . Ich set bude ma vekos pribline
(v prvom rde od ) a bude smerova dovntra sudu.

Kee rozmery nho vseku s pribline (na rku) krt (na


vku), presne v opanom smere psob tlakov sila vekosti . Ak by
bol sud v pokoji, tieto dve sily by museli by v rovnovhe a platilo by teda
= ,

teda

= .

Teraz meme zopakova vahu s rezanm sudu. Ak by sme jeho pl


rozrezali vertiklne, potrebovali by sme ho prida silou . Kee by sme
tak rozrezali uzlov, kad z nich je rozahovan horizontlnou silou vekosti
=

=
= .

/
37

Rusk sud

FX A8

Ak je teda sud v pokoji, kad kosotvorekov oko je vertiklne rozahovan silou a horizontlne silou . Lene lank, z ktorch s tieto
kosotvoreky poskladan, vedia prena silu iba vo svojom smere. Preto
sa ahko sa dopracujeme k tomu, e mus plati tg = . (Bu pozriemvidm z obrzka, alebo z pr jednoduchch rovnc.)

My vak vieme aj to, e = tg . Pouijc tieto dva poznatky a vyie


odvoden vzahy pre a dostvame
tg =
z oho

1
tg = ,
2
2

/2

1
=
=
=
,

2
2 tg
(
teda

= arctg

35 .

Pre takto uhol bude valcov sud v rovnovnom stave.


Pre vie uhly bude plati
tg >

=
,
2 tg

inmi slovami, sila bude privek (resp. sila primal) na to, aby boli
lank v rovnovhe, ok v plti sa teda bud naahova do rky a sud bude
tunej. Naopak, pre menie bude rovnovhu poruova privek sila ,
ok sa bud naahova do rky a sud bude thlej. Koec.

38

B
Optika

If I could explain it to the average person, I wouldnt have been worth the
Nobel Prize.
Richard P. Feynman

39

FX B1 Zrkadl
Polopriepustn zrkadlo s priepustnosou (kde 0 1) je tak
zrkadlo, na ktor ak (z ubovonej strany) zasvietime l s intenzitou , prepust l s intenzitou a odraz l s intenzitou (1 ).
(a) Filip zobral zrkadlo s priepustnosou 1 a hne za postavil rovnoben
zrkadlo s priepustnosou 2 . Koko svetla prepust tto dvojica zrkadiel,
ak na u spredu zasvietime l s intenzitou ? o ak zasvietime zozadu?
(b) Vlado sa nenechal zahanbi. Vytiahol vetky polopriepustn zrkadl, o
naiel v pivnici, a tie ich postavil pekne za seba. Jeho zrkadl maj
priepustnosti 1 , 2 , . . ., , v tomto porad. Ako sa bude sprva tto
sstava?

Zdroj: lohu vymyslel Kubus

l Vzork Kubus l
Tento prklad nebol ak, vetko potrebn bolo napsan v zadan. Bez
alch prieahov ho teda vyrieme. Pozrime sa najprv na Filipovu situciu
s dvoma zrkadlami s priepustnosami 1 a 2 .
Ak na tto dvojicu zrkadiel zasvietime l s intenzitou , l s intenzitou
(1 1 ) sa odraz nasp od prvho zrkadla a l s intenzitou 1 ceze
prejde. Tento l alej dopad na druh zrkadlo, od ktorho sa odraz l
s intenzitou 1 (12 ), priom zvynch 1 2 prejde na druh stranu. Ale
pozor! Tu prbeh nekon. L odrazen od druhho zrkadla znova dopadne
na prv, kde sa as znova odraz a znova prde k druhmu. Ksok prejde
alej, ale ksok prejde znova k prvmu a nasp. Stle slab a slab l sa
bude odra medzi zrkadlami, a pri kadom odraze z neho ksok prenikne
von.

40

FX B1

Zrkadl

Ako u bolo spomenut, prv z lov, o prejde na druh stranu, bude


ma intenzitu 1 2 (musel toti prejs cez obe zrkadl). al z nich bude
ma intenzitu 1 2 (1 1 )(1 2 ) (musel prejs cez prv zrkadlo, odrazi
sa od druhho a prvho, a prejs cez druh), al 1 2 (1 1 )2 (1 2 )2
(tento sa od oboch zrkadiel zvntra odrazil dvakrt), a tak alej. Celkov
intenzita la, ktor prejde na druh stranu, bude teda
= 1 2 + 1 2 (1 1 )(1 2 ) + 1 2 (1 1 )2 (1 2 )2 + . . .
(
)
= 1 2 1 + (1 1 )(1 2 ) + ((1 1 )(1 2 ))2 + . . .
1
1 2
= 1 2
=
.
1 (1 1 )(1 2 )
1 + 2 1 2
Posledn riadok sme dostali stanm nekonenho geometrickho radu v tvare
1 + + 2 + 3 + . . .
Na stavan nekonenho radu samozrejme nie je ni zl, ukme si vak
aj in, trochu veobecnej postup rieenia. Ke na zrkadl spredu zasvietime
l , bud medzi nimi a velikde naokolo svieti velijak le. Ns vak
nemus zaujma kad z nich, len celkov intenzita lov, ktor id kadm
relevantnm smerom. Ozname si ako na obrzku intenzitu lov od prvho
k druhmu zrkadlu ako , od druhho k prvmu ako , od druhho smerom
doprava ako a od prvho nasp ako .

Vieme, e kad z lov (teda sborov lov s danou celkovou intenzitou)


, a sa pri dopade na patrin zrkadlo rozdel presne poda priepustnosti
tohto zrkadla. Kee sa na zrkadlch iadne alie svetlo nevyrba, vieme
presne vyjadri, koko svetla odchdza ktorm smerom z kadho zrkadla.
Dostvame rovnice
= 1 + (1 1 )
= 2
= (1 2 )
= (1 1 ) + 1 .
Rieenm tejto jednoduchej sstavy rovnc dostaneme rovnak rieenie ako
prvm postupom.
41

FX B1

Zrkadl

o sa stane, ak na zrkadl zasvietime z druhej strany? Presne to, o


by sa stalo, keby sme vymenili 1 a 2 . A teda, zhodou okolnost, vbec
ni: vimnite si, e vzah pre intenzitu je plne symetrick vzhadom na
vmenu 1 a 2 .
Zostva u len vyriei otzku, o sa stane, ke za seba postavme zrkadiel viac. Napoved nm k tomu predol odstavec. Kee dvojica zrkadiel
1 2
s priepustnosami 1 a 2 prepa z oboch strn presne 1 +
svetla,
2 1 2
sprva sa nerozlitene (o do prepania svetla) od zrkadla s priepustnos1 2
ou 12 = 1 +
. Ak teda za ne postavme zrkadlo s priepustnosou 3 ,
2 1 2
12 3
vetky tri sa bud sprva ako zrkadlo s priepustnosou 123 = 12 +
.
3 12 3
Postupnm pridvanm zrkadiel po jednom by sme takto mohli vypota
priepustnos sstavy zloenej z ubovonho potu zrkadiel.
Mohli. Neexistuje vak nejak kraj vzah pre vsledn priepustnos?
Alebo aspo nejak vzah, a nie len algoritmus na jej vpoet?43 Ak si
rozpeme vzahy pre 123 a 1234 pomocou jednotlivch :
1 2 3
1 2 + 1 3 + 2 3 21 2 3
1 2 3 4
,
=
1 2 3 + 1 2 4 + 1 3 4 + 2 3 4 31 2 3 4

123 =
1234

nie je ak uhdnu vzah pre veobecnch zrkadiel, a nie je ani ovea


aie dokza ho matematickou indukciou.
Ukme si preto troku in nadhad do problmu. Vimnime si, e vzah
pre vsledn priepustnos dvojice zrkadiel vieme prepsa ako
12 =
ie

1
1
1

1
2

1
1=
12

=(

1
1

1 +

1
(

1
2

)
,
1 +1

) (
)
1
1
1 +
1 .
1
2

Inmi slovami, kvantita 1 1 (o je vlastne 1


, teda pomer odrazenho
a prepustenho svetla) sa pri skladan zrkadiel stava. Po krtkom zamyslen
si overme, e takto stavanie mus fungova aj pre sstavu viacerch zrkadiel, celkov priepustnos Vladovho systmu zrkadiel s priepustnosami
1 , 2 , . . . , bude spa vzah
)
(

1
1
1
).
1=
1 , a teda =
( 1

1 + =1 1
=1
43 Vtavej z vs sa mono ptaj, ak je v tom v skutonosti rozdiel: nie je aj normlny
vzorec i vzah vlastne len postup na vpoet?

42

FX B2 Svetlomety
Bzduo m stl tvaru tvorca a chyst sa ho osvetli zdrojom so svietivm
vkonom , a to dokonca takm, o iari do vetkch smerov rovnako. Tmto
zdrojom sa bli k rohu stola troma rznymi spsobmi (vi obrzok). Pri
kadom z nich vypotajte hodnotu, ku ktorej sa bli svietiv vkon dopadajci
na stl, ke sa zdroj bli k rohu stola. (Vetky hrany na obrzku okrem pok
s rovnako dlh a uhly medzi nimi prav, tvary na obrzku s teda kocky.
pky naznauj smer pohybu zdroja. Stl je vyznaen edo.)

Zdroj: BAUPC

l Vzork Bzduo l
vodom hne upozornm, e tto loha bola naozaj vemi ahk a vzork
je tak dlh len kvli tomu, aby bol prstupn vetkm rieiteom. loha sa
dala riei viacermi spsobmi, v tomto vzorku ukem najzaujmavej a
najtrikovej z nich. Skr, ne sa pustme do samotnho rieenia, sksme sa
zamyslie nad tm, preo by mal by sveteln vkon dopadajci na povrch
stola v jednotlivch prpadoch rzny.
Ke sa budeme limitne pribliova k rohu stola, meme si postupne meni mierku obrzku tak, e vzdialenos bodu od stola bude vyzera rovnak,
ale stl sa bude zvova. Je zrejm, e v limite bude povrch stole prakticky
zabera cel tvrrovinu. Tie je zrejm, e na miesto stola bezprostredne
pod bodovm zdrojom svetla bude dopada znan as svetelnho vkonu.
Situcie s teda rzne v tom, e v prvom prpade sa nachdzame nad stolom,
v druhom prpade nad rohom a v treom prpade sme niekde mimo. Komu
tieto slov nestaia, ponkam dvojrozmern analgiu, kde svetlo dopad na
povrch polpriamky.

43

Svetlomety

FX B2

Vidno, e v dvojrozmernom svete by v prvom prpade na priamku dopadlo 38 celkovho vkonu, v druhom 14 a v treom len 18 celkovho svetelnho
vkonu. Pritom sme sa blili vdy k tomu istmu koncu nejakej seky, ale
z rznych smerov. Vrme sa vak sp k stolu tvaru tvorca a vimnime si
nasledujce pekn polohy.

Vimnime si najprv polohu (1) na obrzku. Otzka znie, ak as dopad


na tvorec, ak je zdroj svetla umiestnen v strede jemu prislcahjcej kocky.
Zo symetrie mus na kad stenu kocky dopada rovnak podiel svetelnho
vkonu. Kocka m 6 stien, take v tomto prpade dopad na povrch stola 16
celkovho svetelnho vkonu.
Teraz sa pozrime sa polohu (2). Vidme, e sa m voi vekmu tvorcu
rovnako, ako bod uprostred kocky voi jednej tvrtine tohto tvorca: z bodu
(2) dopadne na vek tvorec rovnak podiel svetla ako zo stredu kocky
na tvrtinu jednej steny. Ale takchto plch je na povrchu kocky 24, preto
1
z bodu (2) dopadne na povrch nho tvorca 24
celkovho svietivho vkonu.
To nm zatia vedie sta. Mimochodom, je zrejm, e ke podstavu rozdelme po naznaenej uhloprieke, tak na obe asti podstavy bude v oboch
prpadoch dopada rovnak podiel svetelnho vkonu, ie pre (1) by to bola
1
1
12 a pre (2) 48 .
Teraz si predstavme, e sa po zakreslenej pke v tom istom obrzku pohybuje k vrcholu tvorca. Ak as svetelnho vkonu dopadne na jeho povrch
teraz? Konene sme sa teda dostali k samotnej lohe v zadan. Konkrtne
k prvej situcii. Zo symetrie mus na doln, predn a prav stranu kocky
dopada rovnak sveteln vkon a rovnako takisto mus rovnak sveteln
vkon dopada aj na horn, av a zadn stenu kocky. Lene tieto vkony
u mme portan, pretoe ide o situciu (2) z obrzka vybrat vrchol a
prslun steny s sce in, ale maj sa k sebe rovnako. Mete si overi e
vhodnmi symetriami meme situciu pretransformova na t v obrzku.
Navye, celkov sveteln vkon, ktor dopad na jednotliv steny kocky je
zrejme rovn . Ozname podiel svetelnho vkonu, ktor dopadne v prvej
situcii v zadan na povrch tvorca ako . Mus plati rovnos
3+3

1
24

ie

= 1,
44

7
24 .

Svetlomety

FX B2

Vidme, e to vbec nebolo namhav. Druh situciu zo zadania op


vyrieime hravo. K rohu stola sa blime priamo zhora. Ak budeme limitne
blzko k rohu, tvorec bude prakticky zabera cel stvrrovinu. My si meme
dokresli tri alie rovnak tvrroviny, priom na kad dopadne rovnak
as svetelnho vkonu. Rovnak tyri tvrroviny si meme dokresli aj
nad n zdroj. Kee sa tiahnu neobmedzene aleko, kad44 l mus na
niektor z nich dopadn. Dovedna mme 8 rovnocennch plch. Na kad
teda mus dopada rovnak podiel svetelnho vkonu, ie
= 18 .
Zostva nm posledn, najkomplikovanejia monos. Ale aj t hravo
zvldneme. Cestou vak budeme musie spota sveteln vkon dopadajci
na povrch tvorca v ete jednej situcii. V nasledujcom obrzku je vyznaen n tvorec a tyri body X,Y,Z,W, z ktorch sa blime k rohu tohto
tvorca. Prslun podiely svetelnho vkonu, v prpade, e sa z danch bodov limitne pribliujeme k vrcholu tvorca ozname ako , , , .

7
U vieme = 24
a = 18 . K ureniu zvynch vkonov pouijeme
usporiadanie na nasledujcom obrzku.

Budeme sa limitne bli z bodu N do bodu C. Celkov vkon, ktor


dopad n steny kvdra ABLKEFMN mus by . Ale vkony dopadajce
44 Okrem

dokonale vodorovnch, tch je vak nekonene mlo. :-)

45

Svetlomety

FX B2
na jednotliv tvoreky u skoro vetky vieme. Tu s:45


1
=
=
=
=
=
=
,

24


7
=
=
,

24
a nakoniec


=
=

s nae neznme. Dostvame rovnos


6

1
24

+2+2

7
24

= 1,

1
o je linerna rovnica s rieenm = 12
. U nm len sta uri a vyhrali
sme. Na to vezmeme tyri susedn kocky ako na nasledujcom obrzku:

Ke budeme limitne blzko k povrchu, doln tyri tvorce sa bud javi


ako cel rovina a dopadne na ne polovica celkovho svietivho vkonu. To
znamen, e + 2 + = 12 . Ak dosadme, za a , ktor u poznme,
1
dostvame = 24
. Vlastne to vbec nebolo ak, len bolo treba poksi sa
njs nejak ntu.

45 Vimnime si, e rovnos vyjadruje fakt o tom, e predn, zadn, horn a av tvorce
stien na obrzku s voi bodu C orientovan rovnako.

46

FX B3 Minca
Ujo z prkladov o minci na dne bazna sa pozer na mincu na dne bazna.
Vid ju v hbke a chce vedie, v akej hbke sa nachdza naozaj. Stala sa
mu vak vek galiba nevid ju toti kolmo zhora, ako to u v takchto
prkladoch bva, ale pozer sa na u pod uhlom (vzhadom na normlu
hladiny). Pomte ujovi zrta, v akej hbke sa nachdza minca naozaj!
Zdroj: tandardn uebnice fyziky

l Vzork Kubus l

Na to, aby sme vedeli vypota tento prklad, musme si najprv uvedomi, ako nae videnie vlastne funguje. o znamen, ak vidme na nejakom
mieste mincu?
Tto minca vysiela nejak (odrazen) svetlo, ktor dopad do naich o.
Nae oi musia toto svetlo v prvom rade sprvne zaostri, aby sa na sietnici
vytvral ostr obraz. Takisto sa vnemy z kadho oka musia zosynchronizova,46 pridvajc vylepen 3D obraz sveta okolo ns.47
Toto svetlo normlne prichdza smerom z miesta, kde je minca. Ke je
vak minca na dne bazna, svetlo sa na hladine lme a prichdza z inho
smeru. Navye, ich vzjomn uhol sa tie me zmeni. Preto ujo vid mincu
v inej vzdialenosti a v inom smere, ako naozaj je. Pri pohade oovkou by
sa mu mohla javi zmenen/zven.
Nech je zdanliv vzdialenos mince od hladiny a skuton. je
uhol, ktor zviera ujov pohad s normlou hladiny, je zase uhol ktor
zvieraj le z mince s normlou hladiny ete vo vode. Pozrime sa na dva
le vychdzajce z mince, ktor sa separuj vo vodorovnom smere o mal
uhol. Naprklad tak dva le, z ktorch jeden dopadne do ujovho avho a
jeden do pravho oka.
46 To

je u prca centrlnej nervovej sstavy.


potom do toho vstupuje alie podvedom spracovanie obrazu a nae
sksenosti (napr. odhad vzdialenosti pomocou vekosti a uhlovho priemeru predmetu).
47 Samozrejme,

Toto spracvanie m vak tie svoje chyby. S poda vs


ikm iary na pravom obrzku rovnoben? Overte si to
pravtkom!

47

FX B3

Minca

Tieto le sa pri vchode z vody zalomia. Vimnime si vak, e drha


kadho la sa bude nachdza v istej zvislej rovine.48 Zvisl roviny lov
sa pretnaj v zvislej priamke, ktor zrejme obsahuje mincu. Ak si teraz
predime asti lov, ktor vchdzaj do ujovch o, akoby sa nelmali na
hladine (t.j. zdanliv chod lov z ujovho pohadu), zo symetrie sa musia
niekde pretn. Obe leia vo svojej rovine, take sa zrejme mu pretn iba
niekde na spomnanej priamke, t.j. zvislo nad mincou.
Inak povedan to znamen, e vodorovn vzdialenos od miesta, kde l
prechdza vodnou hladinou od skutonej polohy mince je rovnak, ako od
zdanlivej polohy mince. V rei psmeniek na obrzku
= .
Pouijc tento poznatok vidme, e mus plati

= tg

a podobne

= tg .

Vydelenm tchto dvoch rovnc a aplikovanm Snellovho zkona lomu


48 Presne t rovina, ktor si kreslme na hodinch fyziky, ke sa preber lom svetla.
Tto rovina je uren bodom na hladine a dvoma z neho vychdzajcimi polpriamkami.
Dobre sa d cel nasledujca argumentcia predstavi pri pohade zhora.

48

FX B3

Minca

sin = sin dostvame:

tg
=

tg

(26)

sin cos
sin
=
=

cos sin
cos

2
sin sin2

cos
sin

2
sin2
=
cos

1 sin2
sin

(27)

Toto je teda skuton hbka mince.


Poka poka. Neskonme hne takto za rna.49 o by to bolo za hanbu,
dvojstranov vzork! V naom postupe sme uvaovali dva le, ktor sa
vzaovali od seba v horizontlnom smere. Pozrime sa na dva le, ktor sa
vzauj vo vertiklnom smere.
Nech tieto dva le zvieraj malik uhol d. Pri hladine bude ich odstup
(kolm vzdialenos) , pretn hladinu v bodoch vzdialench od seba a ich
vzjomn vzdialenos resp. uhol sa zmenia na resp. d:

Vyjadrime si teraz vzdialenos . Po troche hrania sa s geometriou a


pravouhlmi trojuholnkmi dostvame = / cos (vi. zazoomovan obrzok), ale kee d je mal, = d(/ cos ) (vi. oblk krunice so
49 Za

rna, za rosy, dobre sa kos. . .

49

FX B3

Minca

stredom v minci). Podobnm postupom zistme, e = / cos a tie


= d(/ cos ). Odstrnenm , a z rovnc dostvame:

d cos

cos

d cos
cos

d cos2
=

d cos2

(28)

Stle mme na pamti, e d a d s mal, priam ich meme posla


do nuly. aa, derivcia! :-) Vzah medzi a nm poskytne Snellov zkon. Sta spravi diferencil na oboch stranch rovnice sin = sin . Tak
dostvame
cos d = cos d.
To meme dosadi do rovnice (28). V pr pravch sa u len zbavme
uhlu , op pomocou Snellovho zkona, postupne dostvajc:

cos cos2
cos3
=
= 3
2

cos cos
cos

3
(
)3
2
1 sin2

1 sin2

=
=

cos
cos

= 2

(
)3
2 sin2
cos

(29)

A mme problm. Nae vsledky nm nesedia. Kde sa stala chyba?50


Nestala sa. Ono to takto naozaj vychdza. Le sa inak rozbiehaj v horizontlnom a inak vo vertiklnom smere, a nedaj sa presne zaostri. Pre lepiu predstavu, nieo podobn by nastvalo, keby sme vzali obyajn oovku,
v jednom smere ju natiahli, a chceli vypota jej ohniskov vzdialenos.
Prv vsledok je pravdepodobne bliie k realite, lebo odstup o nm
podva lepiu predstavu o hbke sveta ako zaostrenie jednotlivch o. Ke
ujo stoj aleko od bazna, mal rozdiely v uhloch, pod ktormi prichdzaj
le do jednotlivch o sa javia nevrazn popri vekom horizontlnom rozostupe o. Ak si vak doma hodte do umvadla mincu a budete sa na u
pozera pod vekm uhlom s okom tesne nad hladinou, zistte, e na u neviete poriadne zaostri. Mete si tento efekt sksi zachyti aj fotogracky,
chce to vak dostatone kvalitn techniku.
Poznmka na zver: Ani jeden z vsledkov nie je asne jednoduch,
ale meme si ich aspo ako-tak overi na okrajovch podmienkach. Naprklad pre = 0 dostvame = , o nm potvrd ujo z prkladov o minci na
dne bazna. Tie pre = 1, ke nenastva iadny lom, dostvame oakvan
= .
50 Vimnite

si tie, e tentokrt sa zdanliv poloha ani nenachdza nad skutonou!

50

FX B4 Vlkno
Mao si minule kpil optick vlkno dky a konene si svoj pota
na intrku pripojil priamo ku serveru FKS. Jadro optickho vlkna je dlh
valec polomeru vyroben zo skla s rznymi prmesami tak, aby sa rchlos
renia svetla v om zvyovala linerne so vzdialenosou od jeho osi. Index
lomu v strede je 1 , na kraji 2 . V strede jednej jeho podstavy je bodov zdroj
svetla, ktor vyle krtky sveteln impulz. Ak dku trvania bude ma impulz
prijat na druhom konci optickho vlkna? Rdov zanedbania s prpustn,
typick hodnoty velin s = 1 km, = 20 m, 1 = 1,445, 2 = 1,44.
Zdroj: IPhO

l Vzork Bzduo l
Pokia by som sveteln l pustil presne v smere osi vlkna, as za ktor
prde na opan koniec by bol /0 , kde 0 = /1 (rovnako ako v celom
alom rieen) oznauje rchlos svetla v strede vlkna. Absoltna presnos
sa vak ned dosiahnu51 a nezabrnime skutonosti, e svetlo vojde do
vlkna s istm rozptylom. Le, ktor vchdzaj dovntra pod nenulovm
uhlom, bud na dosiahnutie opanho konca vlkna potrebova viac-i-menej
odlin as. Pointou tejto lohy je uri, o koko najviac sa me nejak l
omeka alebo predbieha v porovnan s priamo idcim lom.
Vopred podotknem prekvapiv vsledok, e vetky odchlen le prejd
vlkno za krat as. e preo? Ako uvidme, odchlen le sa bud pohybova po cikcakovitej drhe, ktor je dlhia ne cesta piama. Avak vinu asu
bud prechdza opticky redm prostredm (tj. s menm indexom lomu) a
teda vou rchlosou!
Hor sa do rieenia! Na zaiatok pr uvaovanch predpokladov:
Optick vlkno je rovn: Vzhadom na to, e aj poskrcan optick
vlkno m polomer krivosti rdovo v ako je jeho prieny rozmer,
vsledky by sa lili a na vzdialenej platnej slici.
Svetlo, ktor dosiahne povrch optickho vlkna sa pohlt: Ak
sa l odraz raz, zrejme sa odraz ete mnohokrt. Ak sa pri kadom
odraze strat o i len mal podiel svetla, pri obrovskom pote odrazov
l strat prakticky vetku intenzitu.
51 i

u kvli Parkinsonovi alebo kvli Heisenbergovi... :-)

51

FX B4

Vlkno

Ozname uhol, pod ktorm l vojde stredom podstavy do vlkna ako


.52 Svetlo sa postupne dostva do miest s oraz menm indexom lomu,
kvli omu sa bude zakrivova jeho smer do oraz plytieho uhlu. Mohlo by
sa sn sta, e v istej vzdialenosti sa ane pohybova rovnobene s osou
valca? Uvidme. Pome sa pozrie, o nm daj rovnice:

Mechanizmom zakrivovania la nie je ni in, iba Snellov zkon lomu.


Optick vlkno si budeme predstavova ako obrovsk mnostvo vrstiev s hrbkou d, ktor sa navzjom lia indexom lomu o mal hodnoty d. Ke bude
svetlo prechdza z jednej vrstvy do druhej, nastane lom poda vzahu
cos
+1
=
.
cos +1

Pokia svetlo prechdza vrstvami, pre kad prechod plat analogick


vzah. Vynsobenm vetkch rovnc53 dostaneme
cos 1 cos 2
cos 1

cos 2 cos 3
cos
cos 1
cos

=
=

2 3

1 2
1

.
1

Take ak chceme zisti, pod akm uhlom sa ri svetlo v nejakej vzdialenejej vrstve, sta ju porovna priamo s prvou, kde svetlo vstupovalo pod
uhlom , a nemusme sa zaujma o vrstvy medzi nimi.
Tera u presne vieme, o ak le sa budeme v rieen zaujma. Najv
uhol, ktor ns ete zaujma, je tak, pre ktor sa svetlo bude pohybova
rovnobene s osou vlkna na povrchu vlkna, tzn.
cos

2
1
4, 77 .
=

52 Mysl sa uhol na vntornej strane valca, tzn. u je zahrnut lom svetla, ktor na
podstave nastal.
53 Znie to hrozne odpudivo, ale perfektne to z rovnc odstrni vetko nezaujmav.

52

FX B4

Vlkno

Otzka je, ako sa bude l pohybova po dosiahnut rovnobenosti s osou


vlkna. Na prv pohad by sa zdalo,54 e pjde u navdy rovno. Naozaj to
vyzer ako samozrejmos. Skutonos vak ukazuje, e l sa rovno pohybova nebude Okamite sa zane sta sp k osi vlkna. Kee ide o naozaj
oemetn vec, budeme sa jej venova v poznmke 1 na konci rieenia. Teraz
to budeme bra ako fakt.
Teraz by sa patrilo uri as, za ktor vychlen l prejde jeden oblk.
Je zrejm, e oblk sa bude len mlo odliova od oblku krunice. Nhoda
je, e pre nami zvolen zvilos indexu lomu vntri vlkna bude drha la
presne oblk krunice. To znie ako drsn nta! Najprv si vak tento fakt
dokeme.

Ozname polomer krivosti v nejakom konkrtnom mieste ako .55 V tomto


mieste mme tie tenk vrstviku hrbky d, do ktorej svetlo vstupuje pod
uhlom rchlosou a vychdza z nej pod o nieo menm uhlom d a
o nieo vou rchlosou + d. Zkon lomu dva
cos

=
.
cos ( d)
+ d
Roznsobenm, pouitm stovo vzorca pre kosnus56 a aproximciou
pre limitne mal uhly sin d = d a cos d = 1 dostaneme
cos d = sin d.
Poda zadania sa rchlos men so vzdialenosou od osi vlkna linerne,
tj. poda vzahu = 0 + . Dosadenm = 0, resp. = mono uri
54 A nielen na prv. Mne sa to zd plne zakadm, ke sa nad tmto problmom
zamyslm.
55 Polomer krivosti funkcie je dobre denovan vec a nielen nejak hdzanie rukami.
V poznmke 3 na konci rieenia njdete fyziklny argument, preo by to tak malo by.
56 cos ( ) = cos cos + sin sin

53

FX B4

Vlkno

kontanty:
0

1
(
)
1
1

.
2
1

My do naej rovnice potrebujeme diferencil rchlosti, pre ktor zrejme


plat d = d, m sa naa rovnica transformuje na
cos d = sin d.
Zrove vak z jednochej geometrie (pre krunicov oblk resp. pravouhl
trojuholnk) vyplvaj rovnice:
}
d = d/sin
= d = sin d
d = d
Dosadenm do naej rovnice dostvame po malej prave
=

1
0

=
= kontanta,
cos
cos

kde sme si pravu pomocou rchlosti v strede 0 a vstupnho uhla mohli


dovoli vaka zkonu lomu. Ako vidme, polomer krivosti la nezvis na
vzdialenosti od stredu vlkna a bude teda cel as kontantn. Tad L
sa pohybuje po krunicovom oblku! Ozname dku jednho oblka ako .
Z obrzka vidno, e = 2 sin .

Prejdime konene k samotnmu asu. sek d krunicovho oblka prejde


l za as
d
d cos
d cos
d =
=
=

cos
0 cos
54

FX B4

Vlkno

a zrejme prv polovicu oblka prejde l za rovnak as, ako druh. Cel
oblk preto prejde l za as

2 cos d
.
=
0
0 cos
To vyzer ako dos kared integrl.57 Preto sa nebudeme otravova analytickm rieenm, ale spravme priblenie pre mal (a tie my mme!).
Vtedy mono psa
)
(
2
2 cos
1+
+ (4 ) d
=
0
2
0
(
)
3
2 cos

=
+
+ (5 ) .
0
6
Pre as, za ktor svetlo prde na koniec vlkna, dostvame

=
=

(
)

2 cos
3
+
+ (5 )
2 sin
0
6
(
)(
)
2
3
4
5
1 2 + ( ) + 6 + ( )
,
3
0
6 + (5 )

kde ( ) oznauje funkciu, ktor rastie rchlosou nanajv , tj.


( )
=0
0

< : lim

a v naich vahch je teda nekonene mal.


Ke sme sa dostali a sem, nemali by sme sa zastavova kvli nejakmu
. Vyuime, e pre oividne plat ( ) = (1 ) a e pre mal plat
57 D sa riei univerzlnou trigonometrickou substitciou = tg (/2). Pre diferencil
plat
2d
.
= 2 arctg

d =
1 + 2

alej sa vyuva

1
2

=
1 + 2
1 + 2 1 + 2
1
2
1 2
cos = cos2 2 sin2 2 =

=
.
1 + 2
1 + 2
1 + 2
2 d
d
na jednoduch 1
Takto naprklad prevedieme cos
2.

sin

2 sin

cos

= 2

55

FX B4

Vlkno

1
1+ 1 . Potom upravujeme alej, priom sta sledova koecienty pri
jednotlivch mocninch :
)(
)(
)
(

2
2
2
4
4
4
=
1
+ ( )
1+
+ ( )
1+
+ ( )
0
2
6
6
(
)

2
2
2
=
1
+
+
+ (4 )
0
2
6
6
(
)
2

1
0
6

Vidme, e l s = prde na koniec vlkna ako prv. Priamo idci


l predbehne o
2

=
.
60
Ak vyuijeme, e 0 = /1 , cos = 2 /1 a zavedieme = 1 2 ,
mono vsledok upravi na elegantn tvar

5, 5 109 s
3

Poznmka 1: Preo svetlo po dosiahnut rovnobenosti nebude pokraova rovno, ale zane sa vraca sp k osi vlkna? V praxi sa meme
vyhovori na mikroskopick nehomogenity prostredia, ktor vdy existuj a
ktor urite l aspo trochu vychlia z rovnho smeru. Akkovek mal
uktucia smerom k osi vak l urite vrti sp. Preto sa d poveda, e
l idci paralelne s osou vlkna sa nachdza v akejsi labilnej polohe.
Skutonos je vak ete zloitejia. L by sa zaal vraca k osi aj keby
sme mali dokonale homognnu ltku (idelny krytl). Sprvne treba argumentova Fermatovm princpom extremlneho asu. Ten vrav, e svetlo sa
pohybuje z A do B po takej drhe, ktor je spomedzi vetkch nekonene
podobnch drh najextrmnejia (najdlhia alebo najkratia) v tom zmysle,
e na nej svetlo strvi najmenej asu. Bohuia, v tejto chvli nm nezostva
inak, ne kri plecami. o to znamen nekonene podobn drha? Ako toto
tvrdenie zoveobecuje vlnov a geometrick optiku? A ako z toho vyplva
pohyb la v naej lohe? To vetko s urite zaujmav otzky. Odpovede
sa dozviete v treom ronku na matfyze na varianom pote.
Poznmka 2: Kad slun funkcia je na dostatone malom seku dostatone linerna. Preto naprklad aj rchlos svetla vo vzduchu v zvislosti
na vke mono povaova za linernu funkciu. To znamen, e aj svetlo
56

FX B4

Vlkno

v atmosfre sa v skutonosti pohybuje po krunicovch oblkoch. Ich polomer je vak niekoko desiatok tisc kilometrov. Je to sce mal zakrivenie, ale
vaka takmuto lmaniu svetla vidme slnko na oblohe o ksok vyie, ne
v skutonosti je. Tento jav je najvraznej ke je slnko nzko nad obzorom.
Ke by jeho doln as nemalo by vidno, stle je v nejakej vke nad obzorom a Slnko sa jav sploten. Inak povedan, vaka tomuto javu trv de
o nieo dlhie, ne by trval bez atmosfry.
Poznmka 3: Plat ete silnejie tvrdenie: Kadej hladkej funkc sa d
v kadom pripsa tzv. oskulan krunica, kt. je na dostatone malom seku
zhodn s danou funkciou. Fyziklny argument je takto: Predstavte si, e
vezmete drt a vytvarujete ho do tvaru danej funkcie. Potom cez funkciu
prevleiete korlku a nechte ju pohybova sa po nej kontantnou rchlosou. Mal obyvatelia korlky (termiti?) by poas pohybu po funkcii nutne
pociovali odstrediv silu. T je meraten. Zo vzahu = 2 / potom
vyplva jednoznanos (dokonca meratenos) polomeru krivosti .
Existuje aj vzah pre vpoet polomeru krivost funkcie v bude , no
nie je nijako pvabn. Skste si odvodi, e
(
)3/2
1 + ()2
() =
.
()

Poznmka 4: Predstavte si, e by ste optickm vlknom vysielali naprklad morzeovku v podobe svetelnch impulzov. Aby boli signly na opanom
konci rozliten, museli by by vysielan bodky resp. iarky oddelen dostatone dlhou medzerou. Vzniknut rozptyl svetla teda obmedzuje mnostvo
informcie, ktor me prejs vlknom za dan as. V praxi sa samozrejme
nepouva morzeovka, ale pri podobnch technikch s obmedzenia analogick. Ak uvime, e impulz zodpovedajci jednmu bitu informcie mus
trva rdovo aspo as (inak by sa susedn bity zlievali kvli rozptylu),
dostvame teoretick maximum pre prenosov rchlos v rdoch stoviek megabitov za sekundu.

57

placeholder

58

C
Elektrick pole

Vopred sa ospravedlujeme, ak tu itate naraz aj na pojmy dverne


znme a spsoben strata asu ho vyvedie z emocionlnej rovnovhy; vytrhnutie, dkladn rozuvanie a nsledn spopolnenie prslunej pase zvyajne
vrti v krtkom ase hladinu adrenalnu do hranc normy. Svoje pohdanie
autorom meme da najavo aj tm, e sa textu niekoko dn (prpadne u
nikdy) nedotkneme.
Marin Fecko: Diferencilna geometria a Lieove grupy pre fyzikov,
poznmka pod iarou

59

FX C1 Guky
Peo rozostavil do medzihviezdneho priestoru 2009 vakovakch kovovch
guliek. Guky nabil nbojmi , 2, 3, . . ., 2008 a 2009, priom je
kladn. Dokte, e aspo jedna z guliek m na celom svojom povrchu kladn
hustotu nboja!
Zdroj: Kvant

l Vzork Bzduo l
loha nie je ak, len sa jej treba chopi zo sprvnej strany. A prve
to chopenie sa tu me by problematick. Samotn rieenie bude ahko pochopiten a vystame si s plne elementrnymi vedomosami o elektrickom
poli.
V rieen sa budeme opiera o niekoko tvrden o vodioch. Kee tie nemusia by kadmu znme, venujem ich odvodneniu pr odstavcov. Sksen
borci sa nemusia necha zdriava a mu ich preskoi. Jedinou nronejou
vecou v rieen bude Gaussov zkon,58 o ktor sa oprieme pri dokazovan poslednho z tvrden. Ako sa vak uke, v rieen si vystame aj bez tohoto
takto zskanho silnho tvrdenia.
Niekoko tvrden o vodioch59 :
I. Vntri vodia je nulov elektrick pole Vo vodii s prtomn
von elektrny. Ak by vo vodii existovalo makroskopick elektrick pole,
a t by ich printila prdi
na elektrny by psobila elektrick sila =
na in miesto. Tento pohyb vak nebude existova navdy, bude utlmen
kvli elektrickmu odporu prostredia. Po vloen vodia do elektrickho poa
sa preto elektrny prakticky okamite preskupia tak, aby na ne nepsobila
iadna elektrick sila. Vntri vodia je preto = 0 .
II. Elektrick pole na povrchu vodia je kolm na tento povrch
Argumentcia je podobn ako v predolom prpade. Aj na povrchu vodia
s toti von elektrny. Keby elektrick pole nebolo kolm, elektrick sila
psobiaca na elektrny by mala zloku rovnoben s povrchom.
=
58 Ako sa d odvodi z Coulombovho zkona njdete vo vzorku lohy FX D1 Ploch
zem, pozri poznmku 2 na jeho konci.
59 Samozrejme, myslia sa vodie, ktor nie s napojen na zdroj naptia a neprechdza
nimi elektrick prd.

60

Guky

FX C1

Elektrny by teda mohli vone prdi a preskupova sa. Tento pohyb vak
mus po ase usta. To znamen, e elektrny sa preskupili tak, aby na
povrchu vodia bolo kolm na povrch.
III. Povrch vodia je ekvipotencilov plocha Ide o priamy dsledok II. tvrdenia. Ak by sme vzali mal pomocn nboj a pohybovali s nm
je kolm na
po povrchu vodia, nevykonme iadnu prcu, pretoe =
. iadna prca znamen iadny potencilov rozdiel. Podobne z I. tvrdenia
vyplva, e dokonca vntri celho vodia je rovnak elektrick potencil.
IV. Vntri vodia je nulov hustota elektrickho nboja Z tvrdenia I. vyplva, e ak si vntri vodia zvolm plne hocijak uzavret plochu, tak
tok elektrickho poa cez u bude nulov. Poda Gaussovho zkona
= /. Ak m by splnen pre plne vetky uzavret plochy,
plat d
tak potom nutne = 0 v celom objeme vodia. Z toho vyplva, e nboj
sa pri vloen vodia do elektrickho poa bude sstreova len na povrchu
vodia.60 Dleitm pojmom sa teda stva plon hustota nboja .
V. Elektrick pole bezprostredne nad povrchom vodia m vekos
= /0 Skmajme pole nad bodom A na povrchu vodia. Obkolesme
ho malou gaussovskou plochou tvaru valca, ako na obrzku pod odstavcom.
Kee vntri vodia je pole nulov a nad vodiom kolm na povrch, jedin
miesto, kde pole teie zakreslenou plochou je horn podstava. Tam mono
navye povaova za kontantn, pretoe zakreslen plocha je rdovo menia ne rozmery vodia. Z Gaussovho zkona mme

= =
d
,
0
odkia vidno hadan vzah. To samozrejme plat len pre vemi mal vzdialenosti!

60 Elektrny nemu vodi opusti, na to im treba doda ist energiu. To sa d realizova


viacermi spsobmi, naprklad osvetlenm vodia (tzv. fotoelektrick jav ) alebo teplom
(tzv. termoemisia).

61

Guky

FX C1

Samotn rieenie
V tvrden V. sme dokzali, e pre elektrick pole tesne nad povrchom
vodia plat = /0 . Ak > 0, tak siloiary z kovu vychdzaj, ak < 0,
siloiary smeruj dovntra.61 Ak sa sname dokza, e na povrchu aspo
jednej guky je vade > 0, je to ekvivalentn dokazovaniu tvrdenia, e
aspo do jednej guky nevchdza iadna siloiara. Budeme teda skma siloiary elektrickho poa. Siloiary smeruj z miesta s vym potencilom na
miesta s nim potencilom a mu by zakonen len na nejakom nboji62
alebo pokrauj a do nekonena.
Kee celkov nboj guliek je 2 + 3 . . . + 2009 = 1005 >
0, na vemi vekch vzdialenostiach63 mono pozorova elektrick siloiary
smerujce od guliek do nekonena, ale nie opanm smerom! To znamen,
e kad siloiara, ktor smeruje do nejakej guky, mus zana na nejakej
inej guke. In zaiatky tu toti neexistuj.
Poda III. tvrdenia kadej guke zodpoved nejak potencil. Skmajme
nhodne vybran guku. Pravdepodobne netrafm na tak, do ktorej vstupuje nejak siloiara. Tto siloiara mus poda predolho odstavca nutne
zana na nejakej inej guke. Aby to bolo mon, tto in guka mus ma
nutne v potencil, ne skman guka.
Tto vahu mem spravi pre kad nhodne vybran guku okrem jednej. Mus toti existova guka s najvym potencilom. Aby do nej mohla
vstupova nejak siloiara, musela by tto odniekia prichdza. Z inej guky
to vak by neme (iadna guka nem ete vy potencil a in nboje,
ne s tie na gukch, v lohe nevystupuj) a z nekonena prichdza tie
neme (pretoe celkov nboj na gukch je kladn). To je vetko. Dokzali
sme, e existuje guka, ktor m na celom svojom povrchu kladn hustotu
nboja . Ide o guku, ktor ma najvy potencil.
Na zver dodajme, e potencil nezvis len od nboja na guke, ale aj
od jej umiestnenia v priestore medzi ostatnmi gukami. Hadan guka
teda nemus by t s najvm kladnm nbojom. To nm vak nepreka.
lohou bolo dokza existenciu takej guky, nie zisti, ktor guka to bude.
A to sme splnili.

61 Naa analza je dokonca zbytone presn. plne si tu vystame zo zkladokolskm


elektrick siloiary smeruj z + do .
62 Ak sa ptate preo, odpoveou je (zase raz) Gaussov zkon. Poda neho si meme
predstavi, e elektrick pole teie. Prameom tohoto teenia s kladn elektrick nboje,
v zpornch nbojoch sa tento tok kon. V oblastiach bez nbojov me elektrick pole
len preteka, omu zodpoved pokraujca siloiara.
63 Tak vekch, e sa nm vetky guky zlej do jednho bodu.

62

FX C2 Stavebnica
Marcelka dostala na narodeniny sadu bodovch nbojov vekosti a k nim
vakovak vzby. Hne sa pustila do velijakch experimentov a m na vs
tieto otzky.
a) Dva nboje xujeme v priestore a tret nechme vone pohybova po nimi
danej priamke. Ak je jeho perida kmitov v rovnovnej polohe?
b) tyri nboje xujeme vo vrcholoch tvorca, piaty nechme vone pohybova v nimi danej rovine. Njdite jeho peridu kmitov v strede tvorca.
c) Vymyslite kongurciu xovanch nbojov v priestore tak, aby vznikla
stabiln rovnovna poloha pre al nboj a vypotajte jeho peridu
kmitov v nejakom smere.
Zdroj: lohu vymyslel Kubus

l Vzork Kubus l
as (a)
Zanime pekne od zaiatku, najjednoduchou asou a. Polome si xovan nboje na os do polh a . Je zrejm, e jedin rovnovna
poloha pre tret nboj xovan na osi je presne v strede medzi nimi, teda
v bode = 0. Ak sila na bude psobi, ak ho vychlime do inej polohy ?
Z Coulombovho zkona to bude
(
)
2
1
1
=

.
40 ( + )2
( )2
Rozmyslite si preo sed znamienko sily (t.j. kladn smer sily je rovnak ako
kladn smer polohy ). Toto sce nie je linerna zvislos sily od polohy, ak
by sme pre harmonicky kmitav pohyb oakvali, avak pre mal vchylky
, ke meme leny 2 , 3 , . . . zanedba, bude zvislos pribline linerna:
=

2 ( )2 ( + )2
2 4
2

.
40 ( )2 ( + )2
40 4
0 3

Uhlov frekvenciu kmitov zistme pomocou analgie s hmotnm bodom na


pruinke, alebo si rovnicu upravme do tandardnho tvaru
=
= 03 .
2

63

FX C2

Stavebnica

Naprklad dosadenm veobecnho rieenia v tvare = sin() +


cos()
zistme, e harmonick kmity s uhlovou frekvenciou kmitania = 2 /(0 3 )
bud rieenm tejto rovnice. Perida kmitov bude teda

2
0 3
2
1 =
= 2
=
0 .

as (b)
V dvojrozmernom prpade postupujeme podobne; nech s xovan nboje v polohch [1 , 2 ], potom bude -ov zloka sily psobiaca na
von nboj v polohe [, ]:
=

1 ,2

2
1
1

,
2
2
40 (( 1 ) + ( 2 ) ) ( 1 )2 + ( 2 )2

kde posledn zlomok len vyber -ov zloku. Pouili sme pri tom pekn
zpis inpirovan rieenm Jana Hermanna, kde stavame cez vetky tyri
nboje, pouvajc indexy na rozlenie znamienok pochdzajcich z -ovej a
-psilonovej sradnice. alej u len zanedbvame vetky leny vyieho rdu
a tie pouijeme prvordov aproximciu (1 + ) = 1 + . (Pre odvodenie
tohto vzorca si skste si av stranu zderivova.)
=

1 ,2

2
( 1 )
40 (( 1 )2 + ( 2 )2 )3/2

1 ,2

2
( 1 )
40 (22 1 2 2 2)3/2

1 ,2

2 ( 1 )
(1 1
40 (22 )3/2

3 1
2

3 1
2 )

2
( 1 23 )
4 20 3

2
.
=
4 20 3
V smere osi bude teda stredn nboj kmita s peridou


4 20 3
2
2 = 2
=
4 20 = 25/4 1 .
2

Kvli symetrii to bude rovnako aj pre vchylku v smere . Vimnime si tie,


e sila nezvis od vchylky a podobne ani nezvis od (do prvho
64

FX C2

Stavebnica

rdu). Preto bud mal kmity v osi a nezvisl, no a kee ich perida
je rovnak, mete si rozmyslie, e nboj bude v strede tvorca harmonicky
kmita v ubovonom smere a to s rovnakou peridou 2 .64
as (c)
Ako to teda bude v priestore? Ak si naprklad sksite porta silu psobiacu na nboj blzko stredu kocky, zistte, e nenulov leny sa objavuj
a v treom rde iadne harmonick kmity sa teda nekonaj. Ba o viac,
poloha v strede kocky nie je pre von nboj stabiln. A ak by ste nhodou
mali nervy hra sa s almi kongurciami, zistte, e iadna nem stabiln
rovnovne polohy.
A dvod? Predstavme si, e v nejakej kongurcii nbojov v priestore
by vznikla stabiln rovnovna poloha pre al nboj. Stabiln poloha znamen, e pre dostatone mal vchylky sa nboj z nej vychlen vrti nasp.
Ak by sme si teda nakreslili dostatone mal guov krupinu (sfru) okolo
tejto polohy, sila psobiaca na n nboj by musela psobi smerom dovntra. Ak je > 0, to by znamenalo, e elektrick pole na sfre smeruje
do jej vntra, a teda jeho plon integrl po nej je zporn. Lene Gaussov
zkon nm potom hovor, e celkov nboj vntri tejto sfry je zporn to
vak nie je mon, kee vntri je len przdny priestor a nanajv nejak
z xovanch nbojov > 0. (Ak < 0, argument je podobn.)
Inmi slovami, v neexistuje elektrostaticky vytvoren stabiln rovnovna poloha. Dalo by sa ete namieta, ako je to mon, e sme v predolch
dvoch astiach dve tak polohy nali. Je to mon len kvli tomu, e sme
von nboj viazali do danej priamky alebo roviny. Ak by sme naprklad nboj zo situcie so tvorcom nexovali v jeho rovine, poloha v strede tvorca
by bola labiln v smere kolmom na tto rovinu. Nboj by stle vedel kmita
v rovine tvorca, ale pri ubovonom vychlen z tejto roviny by uiel do
tretieho rozmeru. . .

64 Me

dokonca kri po hocijakej malej elipse v rovine tvorca!

65

FX C3 Rebrk
Juro si na chate postavil rebrk z rezistorov s odporom . A to nie hocijak, dokonca dvojit a nekonen, tak ako na obrzku.
(a) Vypotajte odpor medzi bodmi a .
(b) Vypotajte odpor medzi bodmi a , ak s body a vodivo spojen
(vodiom s nulovm odporom).
(c) Vypotajte odpor medzi bodmi a .

Zdroj: lohu vymyslel Mat

l Vzork Bzduo l
Aby sme mali jednotn terminolgiu oznaovania uzlov (bodov) schmy,
dohodnime sa, e body napravo od 65 budeme oznaova 1 , 2 , . . . , body
za B budeme oznaova 1 , 2 , . . . a za C to bud body 1 , 2 , . . . Teraz
sa meme s chuou pusti do prce.
Zaneme teda pekne po porad. Najprv urme odpor medzi bodmi a
. Situcia je symetrick poda osi urenej uzlami a naepkva nm, e
potencil v bode sa mus rovnako vemi li od potencilov v bodoch a
, teda mus by strednou hodnotou potencilov v tchto bodoch. Ale to ist
plat pre plne vetky body . Dospeli sme teda k zveru e vo vetkch
bodoch bude rovnak potencil.
Rovnos potencilov nm umouje z obvodu krtn"66 vetky odpory
spjajce akkovek dva body , , to znamen cel jeden riadok odporov.
Ptate sa preo? Pn Ohm nm zanechal zkon pre teenie prdu, poda
ktorho = /, kde = je rozdiel potencilov v danch bodoch
65 Bod si mono predstavi ako bod , ale nebudeme to tak robi kvli kompatibilite
0
so zadanm.
66 Poznmka z Prruky mladch fyzikov: Pri konench sieach pouvaj najm experimentlni fyzici termn odpoji".

66

FX C3

Rebrk

a je odpor, ktorm s spojen a ktorm potan prd pretek. A vidme


to. iadne naptie, iadny prd. No a rovnako vek iadny prd bude
tiec medzi danmi bodmi a aj vtedy, ke odpory medzi nimi
vynechm. Prdy v ostatnch vetvch a potencily v jednotlivch bodov sa
tm neovplyvnia a odpor schmy medzi bodmi a sa nezmen. Dostvame
tm nasledovn schmu:

N pvodn dvojit rebrk sa, pokia skmame len odpor medzi bodmi
a , bude sprva rovnako ako tento nov elegantn jednoduch rebrk.
Vimnite si, e je tvoren nekonenm mnostvom pravidelne sa opakujcich
dielcov67 znzornench na avom z nasledujcej dvojice obrzkov. Ozname
hadan odpor medzi bodmi a ako . Zrejme pokia k zaiatku rebrka pridm ete jeden dielec a budem skma odpor medzi novmi koncovmi bodmi a , tak bude tak ist ako predol, pretoe ide o rovnak
nekonen schmu vzhadom na (topologicky) tie ist body ako pred tm.68
To znamen, e odpor medzi a bude znova .

V pravom obrzku sme cel schmu medzi bodmi a nahradili hadam odporom . Ten ist odpor oakvame aj medzi bodmi a , o
nm pouitm vzahov pre paralelne a sriovo zapojen rezistory umouje
67 Aksi

LEGO skladaka, ale z odporov.


sieti je teraz sce zapojen jedna skladaka navye. Otzka je, i to m nejak
zmysel, ak predpokladme, e ich je nekonene vea. Pre konen schmy by bol nov
odpor naozaj in, ale s rastcim potom prieok sa rozdiel zmenuje a konverguje k nule.
Premyslite si preo.
68 V

67

FX C3

Rebrk

vyjadri pomocou seba samho :


=

2 (2 + )
.
2 + (2 + )

Trochu zvltne, e sme vyjadrili pomocou , ale v tom prve


spova n trik, pretoe sme tm zostavili rovnicu, kde je jedinou neznmou a z ktorej sme schopn ho uri. Jednoduchmi pravami dostvame
kvadratick rovnicu
2

+ 2 42 = 0
s dvoma rieeniami, sprvnym je nezporn z nich,69

= ( 5 1).
Ak je odpor medzi bodmi a , pokia s body a vodivo spojen?
V prvom rade si treba uvedomi, e odpor medzi a je rovnak ako
medzi bodmi a , pretoe bezodporovm spojenm sme z bodov a
spravili prakticky jeden bod a mme zruku, e v oboch bude vdy rovnak
potencil.
Sksme rovnako ako v predolej asti lohy sksi nieo poodpja alebo
pospja. K tomu musme skma potencily v jednotlivch bodoch. Avak
symetria, tzn. fakt e v bodoch a je rovnak potencil, nm naepkva,
ktor body s pre ns zaujmav. Konkrtnejie, symetria nm vrav, e
rovnak potencil je oakvaten v dvojici bodov a pre kad .
Tieto body nie s spojen iadnym odporom, take krta nebudeme.
Spravme presn opak. V bodoch a je rovnak potencil, preto medzi
nimi nebude tiec prd, ani ak ich spojme vodiom s nulovm odporom.
Prdy prechdzajce jednotlivmi asami siete bud stle rovnak. Vznikne
nm teda nieo takto:

69 Druh

rieenie sce vyhovuje tej istej rovnici, no nem fyziklny zmysel.

68

FX C3

Rebrk

Schmu ete prekreslme. Ak v nej zaznaen vodiv spojenie skrtime


na nulu, uvidme, o o vlastne ide. Body a sa pre kad stan jednm
bodom, nech ho naalej oznaujeme ako . Vimnime si, e kad body
a s spojen paralelne dvoma odpormi , teda ich mono nahradi
odporom /2. Rovnako tak s dvoma paralelnmi odpormi spojen aj
kad dva body a +1 , preto aj toto spojenie nahradme odporom
/2. Tak dostvam nov jednoduch rebrk na nasledujcom obrzku:

Avak podobn problm sme u rieili v prvej asti lohy. Akurt naa
skladaka je tento raz o troiku in. Ak ozname hadan odpor ako ,
tak musme ten ist odpor dosta aj ak nadpojme jednu skladaku. Schematick znzornenie skladanie je teraz takto:

Ak budeme postupova rovnako ako v prvej asti, dostaneme pre neznmy odpor vyjadrenie
(
)
3
2
2 +
=
,
2 +
ktor analogickm spsobom vedie na kvadratick rovnicu
2
4
+ 6 32 = 0

so sprvnym nezpornm rieenm

=
69

21 3
.
4

FX C3

Rebrk

Zostva nm posledn as lohy; njs odpor medzi a . Budeme ho


oznaova . Ak sa pozrieme na zadan schmu, nevieme njs iadnu
symetriu, ktor by nm pomohla situciu prekresli nejakm krtanm alebo
spjanm na nieo jednoduchie. Nevieme toti nijakm spsobom njs dva
body s rovnakm potencilom. Ani LEGO skladaku nevieme poui. Chce
to nieo nov.
V elektrickch obvodoch plat nieo, o mono jednm slovom nazva
superpozcia a ktor vystihuje nasledovn fakt: Pokia na svorky , a
,70 tak sa
naej siete prilome elektrick potencily (1 , 1 , 1 ) =

1
v celej sieti ustlia prdy vyhovujce Ohmovmu a Kirchhoovm zkonom a

z jednotlivch svoriek bud vyteka prdy postupne (1 , 1 , 1 ) = 1 .71


, tak
Ak by sme vzali nejak in trojicu potencilov (2 , 2 , 2 ) =

2
rieenm bude nejak (vo veobecnosti in, ale nie nutne) trojica vytekajcich

prdov (2 , 2 , 2 ) = 2 . Princp superpozcie vrav, e ak na svorky

privediem linernu kombinciu potencilov


= 1

1
2 2 , tak zo svoriek

bude vyteka rovnak linerna kombincia prdov = 1 1 + 2 2 .


Sksme troku nahliadnu do problmu, preo by to tak malo by.72
Tvrdenie nejdeme odvodzova Iba ukeme, e takto rieenie naozaj vy, tak okrem toho,
hovuje rovniciam. Ak na svorky privediem potencil

e zo svoriek bude vyteka prd 1 , vo vetkch uzloch obvodu sa ustlia


nejak potencily a vo vetkch vetvch nejak prdy. Ak by boli jednotliv
? Na prv
potencily a prdy, keby sme na svorky priviedli potencil 1

1
pohad ak otzka, ale pome overi tip, e vetky potencily a prdy
bud 1 nsobn. Ak platil Kirchhoov zkon o prdoch v kadom uzle pred
tm, mus plati aj teraz: set prdov vtekajcich do kadho uzla bola
nula, a ke sa vetky prdy z1 -nsobia, tak to bude stle nula. A Ohmov
zkon o naptiach bude plati naalej tie, pretoe v rovniciach pre kad
sluku vystupuj len leny typu ", resp. typu ". Ak ich rovnos platila
pred tm, mus plati aj teraz, ke s -ka aj -ka 1 -nsobn.
, tak mi z nich tiekli
alej viem, e ak som na svorky pripojil naptia

prdy 2 , priom na vetkch uzloch sa zase spravili nejak konkrtne potencily a tiekli medzi nimi nejak konkrtne prdy. o ak prilom na svorky
+
? Naa hypotza je, e potencil v jednotlivch bodoch
potencil

1
2
bude stom potencilov v oboch situciach a prdy prechdzajce kadou
70 Vektorov notciu zavdzame len kvli jej strunosti. Pod vektorom potencilu, resp.
elektrickho prdu si predstavte len trojicu premennch. Nejde tu o nijak magick priestory a arovn pky:-)
71 Z Kirchhoovho zkona o prdoch priamo vyplva, e jeden alebo dva z tchto prdov
s poda tejto dencie zporn, pretoe niektorou svorkou mus do obvodu elektrick prd
aj vchdza. Nikde v obvode sa neme hromadi elektrick nboj.
72 Pokia sa vm nasledujci odsek bude zda po prvom pretan nron alebo nebodaj
nepochopiten, skste si najprv v dkaze nahradi za 1 konkrtne slo 2. Potom si dkaz
prebehnite aj so veobecnm parametrom.

70

FX C3

Rebrk

vetvou bud stami prdov v oboch situciach. Znova mete sksi jednoducho nahliadnu, e Ohmov aj oba Kirchhoove zkony bud naalej
plati v celej schme.73
Ke sme si princp superpozcie poriadne odvodnili, sksme ho aplikova
na n problm. Situciu, ke by sme merali odpor , si predstavme tak,
e sme chceli necha medzi bodmi a prechdza prd a zisovali
sme, ak napov rozdiel 1 je na to potrebn (prv obrzok). Pritom
samozrejme vieme, e to bude 1 = . Podobne, ke sme merali odpor
, zisovali sme, ak napov rozdiel je potrebn vytvori, aby obvodom
prechdzal rovnako vek prd a zistili sme, e to je nejak 2 (druh
obrzok), priom zase vieme, e 2 = . V treom prpade potrebujeme
zska situciu, kde z bodu nevytek iaden prd. Je prve jedna a je
zobrazen v nasledujcej schme.74

Zo schmy (a z Ohmovho zkona) vidno, e hadan odpor mus


spa rovnicu:
2
2

= 22 + 1 /2
= 2 + /2
= + /4.

Ak dosadme za u spotan odpory a , dostvame


(
)

5 + 21
=
1 .
4
Napokon ete mal odozva k vaim rieeniam. Niektor ste pri potan
poslednej asti lohy vyuvali trik, ktormu sme dali pracovn nzov ierna
73 In pekn dkaz mono njs naprklad v Prruke mladch fyzikov. Okrem neho
sa v kapitole o superpozcii dotate o vlnch na vode a Schrdingerovej make, ale aj
o tom, ktor krajiny maj najvhodnejiu geograck polohu z hadiska medzinrodnho
obchodu alebo o tom, ku ktorej pokladni v Tescu sa treba postavi, aby sme akali pokia
mono o najkrat as.
74 Sprvne vzat, je to aj hocijak nsobok tejto situcie. Vetky s charakteristick
tm, e pomer namieania situci (a) a (b) je 1 : 2.

71

FX C3

Rebrk

skrinka. Povedali ste si, e schmu medzi bodmi , a si mono predstavi


tak, akoby bol kad bod spojen s kadm prve jednm odporom (zapojenie do trojuholnka) alebo e kad z tchto bodov je spojen s akmsi
uzlom uprostred prve jednm odporom (zapojenie do hviezdy). Potom ste
si povedali, e zo symetrie s dva z tchto odporov rovnak, m vm ostali v nhradnej schme len dva neznme parametre, ktorch hodnoty sa u
dali uri z ast (a) a (b) lohy. Potom ste u ahko dortali odpor v asti
(c). Tento trik naozaj funguje, ale vbec nejde o trivilny fakt. Existuje
dokonca sostikovanejie tvrdenie, ktor vrav, e ak mm iernu skrinku
s svorkami, mem si namiesto jej skutonho obsahu predstavi nhradn
schmu, kde je kad svorka spojen s kadou prve jednm odporom. Pokia
ste tento trik pouvali a nleite ste jeho sprvnos neodvodnili, strhol som
vm bod. Kto m zujem, me si pozrie korektn Kubusov dkaz tohto
tvrdenia na www.fks.sk/fx/zbierka/eos.pdf.
Aby som vs ete troku util, poviem vm smutn sprvu. Nhradn
schma ku iernej skrinke nie je a tak dokonale uiton, pretoe sa
pri nej nartate ako draci pri urovan jednotlivch odporov, ale lohy sa
pomocou nej daj porta prve vtedy, ke sa daj vyrta aj cez vo vzorku pouit ntu so superpozciou. Toko k vm. Zostva len popria vea
dobrch npadov v alej srii. Dohovoril som. Howgh!

72

FX C4 Balvan
Kubk naiel v Tatrch vek balvan. Naiel si na om vek hladk plochu
a na u si nakreslil mal tvorec so stranou . Ke medzi body
a pustil prd , medzi bodmi a nameral naptie . Ak je mern
elektrick odpor balvanu?
Zdroj: tandardn uebnice fyziky

l Vzork Jakub l
Prv krok je ujasnenie si podmienok, zvan tie idealizcia problmu. N
kame budeme povaova za nekonene vek polpriestor, dokonale homognny (s mernm elektrickm odporom vade rovnakm) a vzduch v druhom polpriestore za dokonale nevodiv a nepolarizovaten (najlepie nech
to je rovno vkuum). To by bola t ahia as za nami. . .
alej vyuijeme siln kaliber princp superpozcie. Vaka nemu nemusme skma priloenie oboch kontaktov sasne, ale meme si posvieti na
situciu s jednm priloenm kontaktom, potom osobitne s druhm kontaktom (o s vaka naej idealizcii rovnocenn situcie) a potom vsledn
rieenie je superpozciou (stom, resp. zloenm) iastonch vsledkov. Tu
by som poznamenal, e princp superpozcie je osi absoltne zkladn a
Maxwellove rovnice (popisujce kompletne cel elektromagnetizmus) tento
princp samozrejme umouj. V naom prklade ho zneuijeme nasledovne:

SITUCIA 1: v mieste mm priloen kontakt s potencilom 0 (meranm voi Zemi dostatone aleko od , inak povedan, nulov hladinu
potencilu som umiestnil do nekonena) takm, e mi ceze do kamea teie
prve prd o vekosti . Potencil v mieste oznam 1 , v 2 a napokon
potencil bude 3 .
73

FX C4

Balvan

SITUCIA 2: v mieste mm priloen kontakt a chcem, aby mi tadia


z kamea tiekol prve prd o vekosti . Teraz si ale spomeniem, e tak osi
som u predsa niekedy videl, len v trochu odlinom ate a troku posunut
(o vzhadom na rozahlos nekonenho rozhrania balvan-vzduch neme na
vsledku ni meni). Je to presne rovnak situcia ako v 1-tke, len s opanm
smerom prdu. Zrejme teda ke vetky potencily z asti 1 dostan mnusko
a budem ich op mera voi nule v nekonene (referenn hladinu som
zachoval), tak to bude pracova tak ako budem ja pska! Pre ns zaujmav
body s sce zrkadlovo pretoen, ale to pri naej symetrickej lohe neme
hra iadnu rolu (zrkadlovo pretoen situcia 1 je stle 1 lebo tam mm
iba jeden vznan bod, ktorm do balvanu pam prd). Take mem
smelo poveda, e potencil v bude 0 , v 1 , v 3 a v 2 .
SUPERPOZCIA SITUCI 1 A 2 (stam potencil a prd v kadom
mieste priestoru): v bode mi kontaktom teie prd do kamea, v mi
teie prd z kamea do kontaktu. Potencil (op meriam voi nekonenu,
kde je aj po superpozcii nulov potencil) v je 0 1 , v je 1
0 , v je 2 3 a v je 3 2 . Vimnime si, e tto superpozcia
jednoduchch situci 1 a 2 je presne n zadan problm, ktor mme riei
(lebo balvanom naozaj pretek prd medzi bodmi a , o je jedin
podmienka, ktor musme splni, aby sme dostali ekvivalentn situciu).
Ostva nm teda uri 2 a 3 v zvislosti od , a potom naptie zo
zadania je rovn absoltnej hodnote rozdielu potencilov v bodoch a ,
ie = 2 2 3 . Oakvame teda, e cez 2 a 3 dostaneme zvislos
od , a , z ktorej vyjadrme pomocou znmych velin , a .
Rieme teraz situciu 1. Mme bod , ktorm do balvanu pumpujeme
prd , mern elektrick odpor balvana je a v nekonene je nulov potencil. Vidno, e situcia je rotane symetrick, preto meme bez vch
okolkov vyhlsi, e 1 = 2 . A to nie je vetko. Ak sa lepie prizrieme,
zbadme, e situcia je dokonca v rmci kamennho polpriestoru sfricky
symetrick to znamen, e potencil na ubovonej polsfre (povrch polgule) so stredom v bode je kontantn.
Teraz urobme krok, ktor sa vymyk predstavivosti bench indivdu,
hoci je zrejm, e m zdrav jadro konkrtne, ten n balvan meme
chpa ako vodi (drt) medzi bodom a nekonenom , so zvujcim
sa prierezom = 22 , kde zna vzdialenos od bodu . To meme
preto, lebo prd naozaj teie vdy len smerom von z polgule so stredom
v . Sksime uri odpor takhoto vodia: Urite poznte vzah pre odpor
vodia kontantnho prierezu , dky a s mernm elektrickm odporom
v podobe = . Tak my si n polguovit vodi meme rozkskova
na tenuk krupinky hrbky a prierezu = 22 , ktor s zapojen
sriovo za sebou. Meme to urobi prve preto, lebo na povrchu kadej
takej krupinky bude kontantn potencil.
74

FX C4

Balvan

Vsledn odpor nho vodia od poiatku do bodu vo vzdialenosti


bude potom suma vetkch odporov krupiniek a po t krupinku, ktorej
polomer je prve .

Nu a ke prejdeme od krupiniek malilinkej hrbky k innitezimlnej


hrbke, tak sa nm suma zmen na integrl, z sa stane diferencil d a
dostvame vzah
[
]

=
() =

+
.
2
2
2
2
20
0
0
Toto je vak troku neprjemn, lebo sme dostali v menovateli nulu (hovorme, e integrl diverguje do plus nekonena). Sksime sa s tm nejako
vyrovna. Najprv odstrnime ideologick problm (skutone by bolo neprjemn, keby nm n odpor vyiel nekonen, lebo by sme nm nemohli
pretlai iaden prd) tm, e kontakt nikdy nebude presne bodov. Kad
rozumn kontakt m nejak rozmer, ozname ho , a ten mem pri istej
dvke predstavivosti povaova za vemi dobre vodiv a polguovit. Potom
by sme pri urovan odporu vo vzdialenosti od bodu A dostali ako vsle

dok integrlu () = [ 2
] = 2
+ 2
, o u je konen. Praktick
problm vyrieime ete jednoduchie, lebo my potrebujeme uri len

2 2 3

= 2(( 2) ())
2
d
= 2

22

[
]2

= 2
2

( 2 1)

=
,
2
=

o na rozmere kontaktu i jeho bodovosti vbec nezvis. Odtia dostvam


aj vsledok

2
=
.
( 2 1)
75

FX C4

Balvan

Ete dodm, e pre odpor nevemi extravagantnch


vodiov sa skutone

pouva vzorec na vpoet odporu = d


,
ktor
m
vak obmedzen

platnos pre vodie, v ktorch je smer prdu viac-menej rovnoben s dkou


(t.j. so smerom, v ktorom uvaujeme d). Tto poiadavka je splnen plne
len pre vemi symetrick prpady: naprklad pre vodi kontantnho prierezu
alebo pre ubovon vsek gule (napr. aj polguu) v takom prpade toti
prd teie kolmo na asti sfr a to je presne smer, v ktorom hovorme o dke
(d).
Ete jedna poznmka o napt potrebnom na pustenie prdu do zeme.


] = 2
. ie o vekosti
Pre situciu 1 mus plati 0 = () = [ 2
potrebnho naptia rozhoduje okrem mernho elektrickho odporu kamea
a poadovanho prdu aj vekos kontaktu (pri polguovom kontakte
naprklad jeho polomer ). Presne rovnako to plat aj pre situciu 2. Take
naptie zdroja potrebn na vyvolanie prdu spomnanho v zadan (za
predpokladu rovnako vekch kontaktov o polomere ) je
(
)
1
1
zdroj = 2 0 1 = 2 0 (0 ()) =

.

Alternatvne rieenie zadanej lohy vyuva taktie superpozciu na
tak dve situcie ako sme mali aj my, avak situciu nepopisuje pomocou
potencilu, ale pomocou elektrickej intenzity . Tto je poda Ohmovho
zkona (zapsanho v diferencilnom tvare) priamo mern plonej hustote prdu = 1 . Kee prd poznme a n problm je sfricky symetrick, tak vieme, e kadou sfrou mus tiec prve prd o vekosti
a smerom kolmm na u. V rei forml (nie tch motorovch...) zapeme


( ) = 2
, kde zna polohov vektor miesta A, kam pame prd

3

; a vektor uruje miesto, v ktorom zisujem plon hustotu prdu . Spolu


s Ohmovm vzahom tak ale fakticky poznme elektrick intenzitu v celej

oblasti. Vieme aj to, e = d (integrl po drhe z bodu C do D). Ak


si rozumne zavedieme sradnicov sstavu, tak vyjde celkom slun integrl,
ktor sa d spota pomocou substitcie. Vsledok je, ako inak, rovnak.

76

FX C5 krupina
Mme vodiv izolovan guov krupinu nabit nbojom . Peo m bodov nboj rovnakej vekosti a chce si ho niekam odloi. Njdite vetky jeho
mon polohy v priestore tak, e na nepsob iadna sila.
Zdroj: lohu vymyslel Bzduo

l Vzork Kubus l
Zamyslime sa najprv, o to vlastne znamen, e guov krupina je vodiv. Znamen to, e nboj sa po nej me vone pohybova. Ke teda niekde
do priestoru umiestnime bodov nboj aj s jeho prspevkom k elektrickmu pou, nboj na krupine to bude cti a zane sa nejako hba. Bude
sa premiestova po krupine a dovtedy, km ho k tomu elektrick sily neprestan nti (nti hba sa po krupine mu ho tlai von zo krupiny,
lebo tomu zabrauj vntorn sily v materili krupiny). Elektrick pole na
krupine bude teda vade kolm na jej povrch.
Toto je celkom uiton informcia, ale zrta rozmiestnenie nboja na
krupine len z nej by bolo dos zloit. Pozrime sa na jej in formulciu.
Potencil na celej krupine mus by rovnak, pretoe v opanom prpade by
nboj z miest s vym potencilom tiekol na miesta s nim potencilom
(a km by sa to nevyrovnalo). Toto je u trochu krajia podmienka, stle
z nej vak nevieme zrta rozmiestnenie nboja na krupine. Pome si ho
teda istm spsobom tipn.
Mono poznte metdu zrkadlenia pri potan interakcie nboja a vodivej roviny, sksme sa ou inpirova. Keby sme vodiv krupinu nahradili
nejakm bodovm nbojom, ktor by spsoboval kontantn elektrick potencil akurt v mieste pvodnej krupiny, a keby t kontantn hodnota
bola rovnak ako v prpade pvodnej krupiny, vedeli by sme ju nm potom
nahradi a postupova podobne ako v prklade s vodivou rovinou. Umiestnime teda n nboj do poiatku sradnicovej sstavy a uvaujme druh
nboj v bode (, 0). Elektrick potencil v bode (, ) bude potom
=

40 2 + 2
40 ( )2 + 2

1
kde pre potencil bodovho nboja pouvame tandardn vzah = 4
,
0
kde je vzdialenos od nboja, priom v nekonene polome potencil nulov. Sksme njs vetky body, kde m potencil nejak pevn hodnotu

77

krupina
FX C5
0 . Vlastne, ak nahliadneme do nasledovnch vpotov, zistme, e ovea
jednoduchie sa nm bud hada body, kde bude ma potencil hodnotu 0.
(S nenulovm by boli rovnice dos kared.) Upravujeme a dostaneme

=0
40 2 + 2
40 ( )2 + 2

( )2 + 2 = 2 + 2

2 ( )2 + 2 2 2 2 2 2 = 0
2 (2 2 ) 22 + 2 2 + 2 (2 2 ) = 0

(2 2 )(2 2 2
2 +
2

4 2
(2 2 )2 )

+ 2 (2 2 ) =

2 2
2 2 )

+ 2 =

4 2
2 2

2 2

2 2 2
(2 2 )2 .

Aha. Mnoina vetkch bodov (v rovine ), v ktorch bude potencil nu2

75
lov, bude akurt krunica so stredom v [ 2
2 , 0] a polomerom 2 2 .
Ak by sme si to cel napsali v priestorovch sradniciach, alebo ak si uvedomme symetriu celej situcie poda osi , zistme e v troch rozmeroch je
tto mnoina guov krupina (sfra) s rovnakm stredom a polomerom. Ak
teda potrebujeme ma nulov potencil na krupine s polomerom a stredom vo vzdialenosti od nho nboja , pre parametre a pridanho
nboja mus plati
2
=
2 2

= .
2

Jednoduchm rieenm tchto rovnc dostaneme


=

2
,

priom z si vyberieme , pretoe znamienko nboja mus by opan ako


nboja . Toto meme vidie naprklad z druhej rovnice v predchdzajcich
pravch, jej umocnenm na druh sa alej tto informcia stratila. Cel
situcia bude vyzera naprklad ako na tomto obrzku:
75 Geometricky je to mnoina bodov s rovnakm podielom vzdialenost od dvoch danch
bodov Rieenm je znma Apolniova krunica.

78

krupina

FX C5

Zatia ni zloit. Priam a vemi pekn, vimnime si najm to, e nboj


2
bude vo vzdialenosti od stredu guovej krupiny, na priamke spjajcej
stred krupiny a n nboj .76 Ak bol vonku, bude vntri a naopak.
m bliie bude ku krupine nboj , tm bliie k nej bude .
Ak m teda naa guov krupina prve nulov potencil (o mimochodom znamen, e sa sprva akoby bola uzemnen), meme poui zrkadliaci trik podobne ako pri vodivej rovine. (Ak ste takto prklad nevideli,
skste si potom spota silu, ktorou psob nekonen vodiv rovina na nboj veda nej.)
Take trik: predstavme si dve situcie. V jednej mme nboj a vodiv
guov krupina s nulovm potencilom (nech je nboj vonku zo krupiny
ak je vntri, vieme argumentova podobne). V druhej mme nboj a
druh nboj vo vypotanej polohe vzdialenej tak, aby v mieste guovej
krupiny bol nulov potencil. V oboch situcich mme v oblasti priestoru
mimo naej guovej krupiny presne rovnak tzv. okrajov podmienky: potencil je nulov v nekonene a na krupine, okrem toho je vade przdny
priestor okrem bodovho nboja . Z terie elektrostatickch rovnc (Poissonovch, Laplaceovch rovnc) vyplva, e takto situcia me ma nanajv
jedno rieenie pre hodnotu potencilu, a teda aj hodnotu eletrickho poa.
Inmi slovami, v oblasti mimo gule je nm jedno, i je nulov potencil na
jej okraji spsoben vodivou krupinou alebo nbojom , vetky elektrostaticky meraten veliiny bud rovnak. Okrem inho bude rovnak aj sila
psobiaca na n nboj :

76 Ak poznte sfrick (krunicov) inverziu z geometrie, polohu nboja njdeme


presne zobrazenm polohy nboja sfrickou inverziou.

79

krupina

FX C5

F. Viem, e toto vysvetlenie nie je ani nhodou pln alebo korektn,


ale bez sostikovanejej terie elektrostatiky (o Poissonovej rovnici pre potencil a podobne) sa ned poriadnejie formulova, a bez pokroilej terie
diferencilnych rovnc sa asi ned rozumne dokza. Ale sn sa to intuitvne
pochopi d, a je to celkom uiton trik.77 (Najm v prpade u spomnanej
vodivej roviny namiesto vodivej gule.)
Kadopdne, ak by mala naa vodiv krupina nulov potencil, vieme
silu psobiacu na nboj ahko vypota krupinu jednoducho nahradme nbojkom a silu zrtame pomocou Coulombovho zkona. Lene o
ke je jej potencil nenulov? Alebo, inak povedan, o ak mus ma in celkov nboj, aby mala nulov potencil? Ni jednoduchie. Ak by krupina
musela ma celkov nboj na to, aby mala nulov potencil, sta, ke
na pvodn krupinu rovnomerne rozmiestnime nboj , a je to. Alebo
naopak, sta, ke na krupinu s nulovm nbojom rovnomerne rozmiestnime nboj , a mme nau krupinu s nbojom .78 ()
U len treba zisti, ak nboj mus ma krupina, aby mala nulov potencil. Znova nateraz predpokladajme e je vonku. Spomeme si, e ak sa
pozerme len na priestor okolo krupiny s nulovm potencilom, meme si
ju nahradi vypotanm nbojom . (Samozrejme, nboj mus by tam,
kde sme s nm rtali pri potan polohy a vekosti a , nememe prina
nov nboje ani inak meni situciu meme iba mera elektrick pole a
podobne.) Ak teda obalme nau krupinu do myslenej uzavretej plochy a
spotame celkov tok elektrickho poa touto plochou, dostaneme rovnak
vsledok, ako keby sme to spravili pre nboj . No ale z Gaussovho zkona
bude tento tok presne dnu
, kde dnu je celkov nboj vntri plochy. Take
0
v oboch situciach je celkov nboj vntri plochy rovnak, a teda celkov
nboj na krupine s nulovm potencilom je presne .
77 Zujemcom odporam si pozrie si nieo o medde zrkadlenia v literatre, naprklad
vo Feynmanovch prednkach z fyziky, 2. diel, 6. kapitola. Alebo na internete, v anglitine
"method of images".
78 Uvedomme si, e krupina m vade rovnak potencil, i u m hocijak celkov
nboj. Ak na u teda pridme nejak nboj navye, tento sa rozmiestni homognne po
celej krupine.

80

krupina
FX C5
A mme to. Poda vahy () sa naa vodiv guov krupina bude sprva
ako superpozcia vypotanho nhradnho nboja a rovnako vekej guovej krupiny rovnomerne nabitej nbojom = . Kee je z nej
vonku, tto rovnomerne nabit krupina sa bude sprva ako bodov nboj
rovnakej vekosti.79 Nhradn nboj bude vntri krupiny, ie rovnakm
smerom od ako stred krupiny (sily sa preto bud stava). Celkov sila
psobiaca na je teda
1
1 ( )
+
2
40
40
2
2

1 2 (1 +
1
=
+
(
)
2
40 2
40
2
2
1
2

Ak m by = 0, potom

2
2

=
(1 +
2
40 2 (1 2 )2
40 2

0 = (1 +

)(1

)
2 2
2 )

Ozname =
a dostaneme rovnicu piateho stupa pre . Naastie sa d
napsa ako sin kvadratickej a kubickej rovnice,

0 = (1 + )(1 2 )2 = 5 + 4 23 22 + 1
= (2 + 1)(3 1).
Jej jedin kladn relny kore men ako 1 (stle
uvaujeme vonku zo

51
krupiny a teda < ) je prve kore = 2 prvho, kvadratickho

5+1
delitea. V tom prpade =
= 2 .
Pozor, ke je nboj vntri krupiny, nememe poui presne t ist
vahu. Toti jednoznanos rieenia a teda rovnakos situcie pre vodiv
krupinu a pre nhradn nboj je teraz zabezpeen iba vntri krupiny.
S naou gaussovskou plochou sa teda nememe hra mimo krupiny, a hranie
sa vntri nm na urenie nboja krupiny prli nepome (rozmyslite si).
Nboj vak jednoducho zistme inou vahou. Ke je na krupine nulov potencil a mimo nej je len przdny priestor, mus by nulov potencil
aj vade okolo nej. (Spomete si naprklad na jednoznanos rieenia mimo
nej, alebo si rozmyslite ako by mohol vznikn nenulov potencil len tak
v przdnom priestore.) Take Gaussova plocha okolo krupiny nameria nulov celkov nboj (teraz sa vbec nerozprvame o nahradzovan nbojom
79 Podobne ako sa rovnomerne nabit gua sprva ako bodov nboj s rovnakm celkovm nbojom.

81

krupina
FX C5
, len o pvodnej situcii, take nm ni nebrni hra sa s gaussovskmi
plochami aj mimo krupiny), ie celkov nboj krupiny mus by rovn
, aby vyrovnal nboj vntri.80
Teraz sa znova meme pusti do vpotov poda vahy (). Mme teda
vntri krupiny a vodiv krupinu sme nahradili nbojom a rovnomerne
nabitou krupinou s nbojom = () = 2. Ale vlastne
tto rovnomerne nabit krupina na nboj nebude psobi iadnou silou, lebo tento je v jej vntri!81 Take sila na nboj bude pochdza len
od nhradnho nboja,
1
=
.
40 2
Toto nikdy nie je nula, kee > 0.
Okrem toho, e sme zabudli na pecilny prpad = 0 vtedy vbec
nie je denovan a cel doterajie rieenie nefunguje ale vtedy je nboj
v trede krupiny a zo symetrie je jasn, e na nebude psobi iadna
sila. A tie sme zabudli na pecilny prpad = 0, ke na zjavne nebude
psobi iadna sila v ubovonej polohe.
Na nboj teda nebude psobi iadna sila ak bude
sm nulovej vekosti,

ak bude v strede krupiny, alebo vo vzdialenosti 5+1

od jej stredu.
2

80 Mono u tute, e na nm nakoniec vbec nebude zlea, ale chcel som ukza
aj tto vahu.
81 Spomete si naprklad na analogick fakt, e gravitan pole vntri homognnej guovej krupiny je nulov.82

82

FX C6 iara
Ke bola Marcelka na prednke, Janka jej zo stavebnice s nbojmi zobrala dva rovnak kladn nboje o vekosti a priblila ich do vzdialenosti
od seba. Potom si nakreslila siloiaru, ktor vychdzala z jednho z tchto nbojov pod uhlom ku spojnici oboch nbojov. Do akej najmenej vzdialenosti
od roviny symetrie nbojov sa tto siloiara dostane?
Zdroj: Kvant

l Vzork Bzduo l
Nu, pitci. Tento vzork zanem vemi stroho. Tto loha je naozaj
ak. Napriek tomu, naozaj m analytick rieenie. Hdam bude pre vs
ponauenm, e niekedy si treba zvoli naozaj bizarn sradnice, aby sa rovnice dali jednoducho vyriei. Mnoh ste istotne oksili, e prca v kartzskej
sradnicovej sstave je slep alebo prinajmenom vemi tnist ulika. Je
lepie vrti sa sp na zaiatok a vymyslie nejak in cestu.
A teraz u pekne a po porad. Alfou a omegou nho rieenia bude Gaussov zkon, bez ktorho by sa sce loha riei dala, avak jednoduchie by
bolo postavi z hracch kariet Koloseum. Pre poriadok a iste z pedagogickch dvodov si dovolm mal vsuvku, ktor mu znalci s istm svedomm
preskoi.
Gaussov zkon
Zaneme krtkou reklamou:
Gaussov zkon je plne super!
Aby ste mi ho kpili, mal by som ho dobre predva, a uveden npis
vyzer urite krajie ako rovnica, ktor sa za nm skrva:

1
d =
d.
0

Gaussov zkon je pre elektrostatick situcie ekvivalentn Coulombovmu


zkonu, len je zapsan vo vemi odlinom tvare. Nebudem ho tu z Coulombovho zkona odvodzova, len si poriadne vysvetlme, o jeho rovnica ukrva.
Zaneme vznamom jednotlivch psmen:
predstavuje nejak (ubovon) myslen uzavret oblas priestoru
(napr. zemiak, vntro balna), predstavuje hranicu tejto oblasti
(napr. zemiakov upa, povrch balna).
83

FX C6

iara

d predstavuje diferencilny ksok hranice myslenej oblasti, d je tejto


plke prislchajci vektor. M t vlastnos, e je kolm na povrch
d
hranice , smeruje von z oblasti a pre jeho vekos plat d
d = d.
je vektor elektrickej intenzity.
je hustota elektrickho nboja.
Ostatn psmenk, ako naprklad permitivitu vkua 0 , by sme mohli
v dave rozozna.
Najprv si premyslime integrl na pravej strane rovnice. Integrujeme hustotu nboja cez nejak oblas. Vsledok bude analogick, ako ke zintegrujeme hustotu hmoty cez nejak objem, tzn. dostaneme celkov nboj, ktor
je obklopen hranicou oblasti . Mnostvo tohto nboja je ete predelen
bulharskou kontantou 0 .
av strana je o nieo komplikovanejia. Je tam integrl akhosi skalrneho sinu cez aksi plochu . . . jednoducho vec, ktor si zasli nasledujci
obrzok. Je na om zakreslen diferencilna plka d, ktor je tak mal, e
ju mono prakticky povaova za ploch, a ktor tvor drobn ksok hranice
:

Tto plka je tie dos mal na to, aby sa elektrick intenzita na jej
povrchu prakticky nemenila. Ak si elektrick intenzitu v tomto mieste rozlome do smeru kolmho na povrch a do smeru rovnobenho s povrchom
, je jasn, e ku skalrnemu sinu d prispieva prve .
Aby sme sa s pochopenm dostali o ksok alej, zmenme na krtky as
ter naej pozornosti. Sksime si predstavi in, nasledovn situciu. Cel
priestor je vyplnen vodou, ktor vo svojom objeme nejako prdi. Znova
si predstavme v priestore nejak xovan oblas ohranin uzavretou
84

FX C6

iara

plochou . Ide len o myslen oblas priestoru, o znamen, e prdiaca


voda o nej v princpe nevie a velijako ou me prechdza.
Takejto kvapaline mono priradi rchlostn pole (, , , ), ktor hovor, ak je vektor rchlosti v jednotlivch bodoch kvapaliny v danom mieste
a ase. Vezmime si nejak konkrtny as a pome skma, ako by sa dal
interpretova integrl

d
d.

Analogickm postupom sa dostaneme k rieeniu otzky na diferencilnej


plke d. Uvaujme, o koko sa na takejto plke posunie voda za vemi
mal as d. Voda sa posunie v smere vektora rchlosti, ako to ukazuje
nasledujci obrzok:

Vieme si predstavi, e v prpade vemi malej oblasti d by bol objem


kvapaliny, ktor vytiekla cez tto plku, aksi rovnobenosten so zkladou
d a s vkou d = d. Objem vyteenej kvapaliny by bol d = d d
a objemov prietok vody touto plkou by bol
d
= d,
d
priom v prpade, e by voda prdila dnu, bolo by tam ete zporn znamienko. To sa d formlne d zapsa presne ako
d =

d = d
d.
Teraz porozmajme nad tm, omu sa bude rovna integrl cez cel
hranicu oblasti . Zrejme dostaneme

d = ,

85

FX C6

iara

kde Q je celkov vtok vody zvntra tejto oblasti. Ak predpokladme, e


voda je nestlaiten a vo vntri sa nm nevytvraj nijak zhadn bublinky, tak hustota vody je vade v priestore kontantn a zrejme rovnak
mnostvo vody, ak z naej oblasti niekde vyteie, do nej mus v nejakej
inej asti aj vteka. To znamen, e celkov je rovn nule a plat

d = 0.

Tto rovnica sa dos podob na Gaussov zkon s nulovou hustotou nboja. Aby sme pochopili, e svislos teenia vody s Gaussovm zkonom je
omnoho silnejia, rozmajme alej. Predstavme si, e mme vntri oblasti
zdroje vody. Nejak rozprvkov vodovodn kohtiky, v ktorch sa voda
vytvra z nioho. Kad takto kohtik bude ma nejak objemov prietok
,83 ktor me by kladn alebo zporn poda toho, i vodu do systmu
dodva, alebo ju vyciciava. V takom prpade by sa n Gaussov zkon pre
tecu vodu zmenil na

d =
.

No ale poka, tto rovnica m presne rovnak tvar ako Gaussov zkon!
Ve suma je tie len akoby integrl . A takejto rovnici u celkom jednoducho rozumieme. Hovor len o tom, koko vody vytek z nejakej uzavretej
plochy, ke s v nej nejak zdroje a nejak odberae vody. Sksme njs
nejak analogick pochopenie elektrickch velin v Gaussovom zkone. Napme si ho ete raz:

1
d =
d.
0

o takto predstavi si, e aj elektrick pole teie?84 Smer a aksi rchlos


teenia je vyjadren elektrickou intenzitou . Nboje uzavret oblasou sa
sprvaj ako vodovodn kohtiky s prietokom /0 . Integrl na avej strane
rovnice teda hovor o tom, koko elektrickho poa vytek cez hranicu , a
prav stran hovor o zdrojoch tohoto teenia, ktormi s elektrick nboje.
Predstava, e elektrick pole teie, sa zd na prv pohad dos chyln.
Avak tento pojem sa naozaj zauval. Ke si vezmem hocijak (nie nutne
uzavret) plochu , tak integrl

=
d

83 Je

priam rozprvkov zzrak, e aj prietok vody, aj elektrick nboj sa zvykn znai


rovnakm psmenom. V naich gaussovskch vahch maj a npadne podobn reprezentciu.
84 Dokonca teie ako nestlaiten kvapalina! Pri alom tdiu navye zistte, e elektromagnetick pole sa ri priestorom rchlosou svetla, m vlastn energiu, hybnos a
dokonca aj vlastn moment hybnosti. Myslte si stle, e pole je len matematick predstava, alebo ho u povaujete za aksi formu hmoty?

86

FX C6

iara
sa nazva tok elektrickho poa (resp. tok intenzity elektrickho poa )
plochou . No a podobn toky zohraj jednu z kovch loh pri rieen
problmu s naou siloiarou.
Po dlhej odboke samotn rieenie
o vieme poveda o mieste, kde sa siloiara najviac pribli k rovine
symetrie? Naprklad to, e elektrick intenzita v tom mieste bude rovnoben
s touto rovinou, ako to zobrazuje prav z nasledujcej dvojice obrzkov.

Ak si zavedieme parametre , , a tak ako na obrzku, tto podmienku


si vieme napsa ako
1
1
cos =
cos ,
2
40
40 2
odkia dostvame
2 cos = 2 cos .
Z geometrie trojuholnka, konkrtne zo snusovej vety, dostvame alie rovnice:

=
=
.
sin
sin
sin ( + )
Tieto tri rovnice nevyzeraj nijako zle, ale treba si uvedomi, e je to prve
vaka dobre zvolenm parametrom.
Vimnite si, e zatia mme tri rovnice o tyroch neznmych. Vidme,
e nm do systmu zatia nijako nevoiel uhol . Zrejme mnoina rieen
takejto sstavy rovnc vedie k njdeniu mnoiny vetkch bodov, v ktorch
87

FX C6

iara

sa nejak siloiara najviac pribli k rovine symetrie. Otzka teraz znie: ak


dodaton podmienka plat pre pecilnu siloiaru, ktor vychdza z nboja
pod uhlom ?
Teraz nadchdza ten kov okamih, kedy po dobrom marketingovom
ahu konene pouijeme Gaussov zkon. Vezmime si povrch, ktor vznikne
rotciou naej siloiary okolo spojnice nbojov,85 a k tomuto povrchu sprava
nalepme rovinn plochu. Napravo od nboja nm takto vznikne uzavret
oblas tvaru akhosi stlaenho kuea, ktorho pl tvor povrch sledujci
siloiary a ktorho podstavu tvor kruh prechdzajci aj hadanm bodom
bodom najbliieho priblenia naej siloiary ku rovine symetrie nbojov.
Cel situcia je znzornen v reze na nasledujcom obrzku naavo.

Pome si pre tto oblas (a plochu dan jej povrchom) zapsa Gaussov
zkon. plne najprv musme uri, koko nboja je uzavretho vo vntri.
Aby sa nm to podarilo, nememe si nboj predstavova ako bodov, ale
ako miniatrnu guku, ktor je usdlen v ostrom rohu naej plochy. Ak
as tohto nboja je v jej vntri?
Ak by bol naprklad nboj guky rovnomerne rozloen po jej povrchu,

s povrchovou hustotou = 4
uzavret vntri
2 , dostaneme pre nboj
naej plochy


d
= d =
(2 sin )
=
{z } |sin
cos |
{z}

=
cos

2 d = 22 (1 cos ) =

(1 cos ) ,
2

85 Ide zrejme o body vetkch siloiar, ktor vychdzaj pod uhlom v rzne natoench
rovinch.

88

FX C6

iara

kde sme si povrch guky rozloili na mnostvo prstencov s vrcholovm


uhlom , ktor vytnali na osi interval rky d a pre ktorch povrch sme
vyuili vzah d = d, kde je obvod prstenca a je jeho skuton
rka. Nakreslite si k tomu obrzok!
Mete si rozmyslie, e rovnak vsledok by sme dostali aj pre homognne nabit guku, alebo vlastne hocijak rozloenie nboja, ktor by
spsobovalo rovnak (vonkajie) elektrick pole ako bodov nboj.86
Ak sme teda nali celkov nboj vntri naej gaussovskej plochy,
vieme, e tok elektrickho poa touto plochou mus by rovn /0 . Navye, a presne pre toto sme si zvolili plochu prve takto, vetko pole vytek
len kruhovou podstavou. V ostatnch miestach je toti gaussovsk plocha tvoren siloiarami a preto tadia iadne pole nevytek.87 Tok kruhom
budeme oznaova , a teda plat
=

(1 cos ) .
20

Avak, pre elektrick pole plat princp superpozcie, preto mus by tento
tok rovn stu tokov spsobench jednotlivmi bodovmi nbojmi. Na urenie toku cez n kruh v prtomnosti iba jednho z nbojov meme poui
podobn postup ako doteraz, ke sme tam mali nboje dva. Tentoraz bud
dokonca siloiary rovn, take pouit gaussovsk plochy bud pekn kueovit ale to ns vlastne vbec netrpi, v konenom dsledku zvis len
na uhle, pod ktorm vychdzaj z nboja. Ak sa zapozerme do druhho
z ostatnej dvojice obrzkov, prdeme takto k rovnosti

(1 cos )
20
1 cos
1 cos

(1 cos )
(1 cos )
20
20
1 cos 1 + cos

cos cos .

alia rovnica a op vyzer pekne, ak sa zape pomocou uhlov a


. Urite preto u nejdeme lozofova a v takto zvolench sradniciach88
privedieme problm ku astnmu koncu.
86 Alebo, ak vm takto postup nevonia, mete si nboj necha bodov, ale gaussovskej
ploche odstrihn rotek. Potom by bol celkov nboj vntri plochy nulov, ale museli by
sme vypota tok elektrickho poa cez plku, ktor vznikla odstrihnutm. Ak si tto
strihaciu plku zvolme sprvne, vbec to nie je zloit, a dopracujeme sa k rovnakmu
vsledku.
87 Skalrny sin d je tam rovn nule.
88 Hadan bod leiaci na naej siloiare v rovine je tmito dvoma uhlami jednoznane
uren, preto si mem dovoli termn sradnice

89

FX C6

iara

Zostva problm, ako z tchto rovnc rieenie nejako vybi. Vezmime si


naprklad nau rovnicu z Coulombovho zkona a rovnicu zo snusovej vety
2 cos = 2 cos

=
.
sin
sin

Ich vhodnm ponsobenm sa zbavme dok a , a dostaneme


sin2 cos = sin2 cos .
Odtia postupnmi pravami
(1 cos2 ) cos = (1 cos2 ) cos
cos3 cos3 = cos cos
)
(cos cos ) cos2 + cos cos + cos2 = cos cos
(

cos2 + cos cos + cos2 = 1.


V poslednej prave sme vyuili fakt, e cos = cos . To by toti znamenalo,
e siloiara sa dostane a k rovine symetrie, o zrejme plat iba pre pecilny
prpad = 0.
Podarilo sa nm zni stupe rovnice na 2, take je u pre ns vrazne
jednoduchia. Ak si za niektor z uhlov dosadm z rovnice zskanej pre tok
, dostanem po prave kvadratick rovnice
cos2 + cos (1 cos ) + (cos2 2 cos )/3 = 0

resp.

cos + cos (cos 1) + (cos 2 cos )/3 = 0.


2

Ich rieenm89 dostvame

(3 cos ) (1 + cos ) /3
cos =
2

1 cos + (3 cos ) (1 + cos ) /3


cos =
.
2
cos 1 +

resp.

Tm je kus karedej roboty vyrieen. Ke poznme kosnusy uhlov


a , vieme si vyjadri aj snusy. Vo vsledku ich teda vyuijem ako vhodn
89 Vylime

pri tom menie z rieen, pretoe ved na zporn hodnoty kosnusov.

90

FX C6

iara

substitciu. Pre hadan dku plat jednoducho

cos
2

=
sin cos
2 sin

=
sin cos
2 sin ( + )
(
)
2 sin cos

1
.
=
2
sin ( + )

To je u vsledok v asi najkomprimovanejom tvare. Za uhly dosdza


u nebudeme. Ozaj by to nevyzeralo pekne.
Rieenie poda Dalimila Maza,
ktor napriek tomu, ako som uvdzal tento vzork, lohu v kartzskych
sradniciach vyrieil. Pri ukazovan jeho postupu budem pouva niektor
v predolom texte nadobudnut poznatky.
Zaveme si sradnicov sstavu s osou zhodnou so spojnicou nbojov a
s osou v rovine ich symetrie. Vezmime si vetky siloiary, ktor vychdzaj
z jednho z nbojov pod uhlom . Tvoria rotane symetrick priestorov
teleso, ktor najprv vyzer ako kue, no potom sa rovnako ako siloiara sto
sp. Vezmime si tak ist plochu vychdzajcu aj z druhho z nbojov.
Priestor medzi tmito dvoma plochami obsahuje na zklade u popsanch vah a vpotov nboj vntri = 2
2 (1 cos ) = (1 cos ).
Napokon uzavrime tento priestor valcom s rotanou osou na spojnici nbojov. Polomer valca nech je , a v rovine nech siloiary z nbojov pretn
valec v bodoch [, ]. Polomer valca nevolme nutne tak, aby jeho pl
prechdzal hadanm bodom, ale plne veobecne.

91

FX C6

iara

Takto sme si vytvorili uzavret plochu, ktor na oboch koncoch sleduje


smer siloiar (a teda tok poa cez u je nulov) a uprostred, na intervale od
po , m tvar valca s polomerom . Plom valca mus poda Gaussovho
zkona preteka tok = 0 (1 cos ).
My vak vieme, e na povrchu pla je kolm zloka intenzity elektrickho poa rovn

(
, ) =

40 (

+ (
+ /2)

)3/2 + (

+ (
/2)

)3/2 ,

rovnos pre jej tok cez pl valca nm teda udva podmienku, ktor musia
spa sradnice a :

d =
(1 cos ) .

Pre strunos ozname (1 cos ) = . Ak si napeme d = 2, d


a
pri integrovan zvolme substitciu
2 = sinh ,90 dostaneme po spotan
integrlu rovnicu
+ /2
/2
=
+
.
2
2
2
+ ( + /2)
+ ( /2)2
Takto sme ikovnou ntou dostali rovnicu siloiary. Vetky dvojice bodov [, ], ktor spaj tto rovnicu, leia na siloiare vychdzajcej pod
konkrtnym uhlom . Teraz sksime njs na tejto konkrtnej siloiare bod,
ktor je najbliie k rovine symetrie.
Spravme si diferencil obidvoch strn u zskanej rovnice. Na pravej
strane sce zskame kared vraz,91 ale na avej strane bude nula, a v najb90 Hyperbolick

funkcie s denovan ako


1
2
1
sinh =
2

cosh =

(
)
+

(
)
,

priom si mete overi rovnos cosh2 sinh2 = 1 a e navzjom je jedna derivcia


druhej. V naom integrli potom dostaneme

sinh
sinh
d
+ =
= tanh + =
+ .
cosh
cosh2
sinh2 + 1
91 Nejdem ho tu vypisova, skste si to spota na papieri. Nezabudnite, e diferencil
treba robi zo vetkch premennch, take naprklad d(2 + 2 ) = 2 d + 2 d.

92

FX C6

iara

liom bode siloiary k rovine symetrie navye d = 0.92 Ak zskan vzah


ete vydelm d, zjednodu sa na
0=

+ /2
( 2

3/2
/2)2 )

+ ( +

/2
( 2

+ ( /2)2 )

3/2

Ha, rovnica pre mnoinu bodov v rovine, ktor s na svojej siloiare


jej najblim bodom k rovine symetrie. Mm teda dve rovnice o dvoch
neznmych, ns zaujma hodnota . Avak, ako odtiato vyjadri ? iada si
to mazan substitcie93 a vea trpezlivosti. Pre jednoduchos ozname /2
ako . Pokia vezmeme substitciu
=

2 + 2 + 2
2

a dosadme do naich dvoch rovnc, daj sa tieto upravi na tvar

2 =
0

+
+1
1
+

+
.
3/2
3/2
( + 1)
( 1)

Z druhej z tchto rovnc ahko prdeme k vzahom


(
+1=

(
1+

)2/3 ,

1=

(
1+

)2/3

)2/3

Ke zskan vzahy dosadme do prvej zo sstavy rovnc a poriadne upravme, dostvame ju v tvare

(
)2/3 (
)

1/3
2/3
2 = 1
( + ) ( + ) ( )
.
+
Je rozumn hada rieenie rovnice v tvare () = (). Po tomto
dosaden a alej prave dostvame rovnicu do tvaru
(
)3/2

2/3
2/3
2 = (1 + ) (1 )
.
92 Ak celkov diferencil nie je v kamart, skste sa na to pozrie alternatvne. Vraz
na pravej strane rovnice je pozd siloiary kontantn (lebo ten na avej strane je) a
v najbliom bode ide siloiara v smere . Take zderivovanm toho vrazu v najbliom
bode poda musme dosta nulu.
93 od mazanho rieitea

93

FX C6

iara

Pokia obe strany rovnice umocnme na druh a roznsobme ztvorku


na pravej strane, meme postupne upravova:
42

4/3

(1 + ) 3 (1 + )
2/3

+3 (1 + )

(
)
4 2 1

(
)
4 1 2

4/3

(1 )

2/3

2/3

3 (1 + )

2/3

(1 )

(1 )
2/3

(1 )
(1 )
(
)
2/3
(
)2/3
3 1 2
2

(1 + )

2/3

Posledn rovnicu mono umocni na 3/2 a potom vydeli 2 . Tak dostvame


(
(
)3/2
)
4 1 2
= 3 3 1 2 ,
o je to ist ako

2 +

16 (1 2 )
1 = 0,
27
2

teda pekn kvadratick rovnica s jedinm kladnm, a teda hadanm rieenm

3
3
2
4 (1 )
4 (1 2 )
=
+1
.
2
2
27

27

Teraz sta vrti vetky substitcie a dostvame rieenie

2 )3
2 )3
1
(1

(1

cos
)
1
1
(1

(1

cos
)
.
=
+
27
(1 cos )2
4
27
(1 cos )2
Nu, je na vs aby ste sa rozhodli, ktor rieenie je elegantnejie a ktor
vsledok je v krajom tvare... a vbec, aby ste si overili, e oba vsledky s
naozaj rovnak!94

94 o

je, zasa raz, loha pre mazan matematick softvr.

94

D
Gravitcia

To show this diagram properly, I would really need a four dimensional


screen. However, because of government cuts, we could manage to provide
only a two dimensional screen.
Stephen Hawking

95

FX D1 Ploch Zem
Je veobecne znme, e Zem je gua s polomerom . Kedysi si vak udia
mysleli, e Zem je nekonen homognna plata s hrbkou . Zistite, ak
by musela by tto hrbka , aby bolo na plochej Zemi rovnak gravitan
zrchlenie, ako je teraz. Predpokladajte, e hustota plochej Zeme by bola
rovnak, ako je priemern hustota Zeme teraz.
Zdroj: lohu vymyslel Kubus

l Vzork Bzduo l

Km sa dme do prce, zopakujmesi dva znme fakty o gravitanom poli.


Hmotn bod s hmotnosou d vytvra v kadom bode gravitan pole
dan Newtonovm vzahom
d =

d
,
2

kde je jednotkov vektor smerujci od hmotnho bodu do miesta,


v ktorom hadme gravitan pole.
Gravitan pole od viacerch hmotnch bodov zskame jednoducho ako
set gravitanch pol od jednotlivch hmotnch bodov.
Ak sa zmierime so znmym vzahom pre gravitan pole gule a potom
integrovanm zskame vzah pre gravitan pole rovnej dosky, loha je vyrieen. To naozaj urobme, ale vzpt si ukeme, ako sa dala loha vyriei
plne bez integrovania.95 Avak dokaj asu, ako hus hlasu! Aby sme docenili krsu spomnanho triku, najprv si ukeme pracn rieenie s pouitm
integrlov.
Predstavme si nekonen dosku s hrbkou d a s hustotou . Pome
zisti, ak je gravitan pole vo vke nad ou. Zaveme si oznaenia
vzdialenost a uhlov, ako na nasledovnom obrzku:
95 Aby som nezavdzal, jeden integrl v rieen zapeme, ale v skutonosti integrova
nebudeme. Posta nm znalos, e integrl vyjadruje aksi set.

96

FX D1

Ploch Zem

Zo symetrie je jasn, e hadan bude smerova kolmo na rovinu. Rovnako vzdialen asti roviny pritom prispievaj rovnako vemi. Preto si ju
rozdelme na prstence s polomerom a rkou d. Z geometrie vidno, e ak
si ako sradnicu zvolme uhol , tak
=

cos

= tg

d =
d.
cos2
Hmotnos sstreden v jednom prostenci je
d2 = 2 d d
a ku gravitanmu zrchleniu vo zvolenom bode prispieva
d2 rovina

d2
cos
2
d
= 2 d 2 cos

2
cos2 tg d
= 2 d
cos
2
=

cos2

= 2 d sin d.

Integrciou cez od 0 po /2 dostaneme celkov pole od nekonenej


tenkej roviny
/2
drovina = 2 d
sin d
0

2 d.
97

FX D1

Ploch Zem

Vidme, e pole od nekonenej roviny nezvis od vzdialenosti . Je rovnak v celej polrovine. Preto ak vezmeme mnostvo takchto tenkch rovn
a poukladme ich do jednej vrstvy s vekou hrbkou , vsledn pole bude
jednoducho
rovina = 2.

Tento vsledok sta porovna s gravitanm poom gule, o ktorom je


znme,96 e
gua

2
4
.
3

=
=

Porovnanm zskanch vsledkov dostaneme, e gravitan zrchlenia


bud rovnak, ak
=

2
.
3

Trik Gaussov zkon


Teraz si ukeme, ako mono zapsa gravitan zkon v inom tvare,
ktor je uiton v niektorch pecickch situcich.97 Uvaujme ubovon uzavret plochu v trojrozmerom priestore (naprklad plocha ohraniujca zemiak, ale v naom prpade ide o nehmotn, myslen plochu). Nech sa
niekde v jej vntri nachdza hmotn bod s hmotnosou d. Pome skma
nm vytvoren gravitan pole na naej myslenej plne hocijakej uzavretej
ploche.

96 Ke sa nad tm zamyslte, vbec nie je jasn, preo by pole od homognnej gule


malo by rovnak, ako od hmotnho bodu rovnakej hmotnosti umiestnenho v strede tejto
gule. Vezmime si naprklad bod v blzkosti povrchu gule: Vidno, e niektor asti gule s
vrazne bliie, ne stred gule. In zas prli aleko. alie dokonca psobia gravitanou
silou plne zlm smerom! Napriek vetkmu, ono to tak naozaj je! Ide vak o netrivilny
matematick vsledok.
97 Najm v takch, ktor s nejakm spsobom symetrick.

98

FX D1

Ploch Zem

Ak sa budeme pozera z hmotnho bodu smermi vntri malho priestorovho uhla s vekosou d, vymedzme na povrchu plku s vekosou d.
Z ahkej geometrie98 nahliadneme, e
d = 2 d/ cos ,
kde je uhol, ktor zviera spojnica plky s hmotnm bodom a kolmica na
plku. Malej plke ete priradme normlov vektor, ozname ho d
d. Ten
je kolm na nau plku a jeho vekos sa rovn vekosti plky.
Ak si ete spomenieme na vzah pre gravitan pole, meme sa pusti
do boja. Vimnime si zporne vzat skalrny sin
d d

d d cos
d 2 d
=
cos
2 cos
= d d.
=

), tzn. cez cel


Tento teraz zintegrujeme cez cel povrch (vetky plky d
priestorov uhol. Pre kad ksok povrch je vak aj d kontantn, preto
98 Priestorov uhol je denovan ako plocha, ktor vybranmi smermi vymedzme na povrchu jednotkovej gule.

99

FX D1

Ploch Zem

meme upravova

d d

d d

= d d
=

= 4 d,
kde sme vyuili fakt, e cel priestorov uhol99 je 4.
Ete nekonme! Predstavte si vntri naej plochy viacej () hmotnch
bodov a sami si rozmyslite nasledovn tvrdenia:
Gravitan pole na povrchu mono vyjadri ako vektorov set jednotlivch pol, tj.
= d 1 + . . . + d .
Skalrny sin zvis od zloiek vektorov linerne, preto
d = d 1 d + . . . + d d
d.
Integrl cez cel povrch mono rozdeli na set integrlov, v ktorch
u vystupuje len jedna hmotnos a ktor s identick s integrlom o pr
riadkov vyie. Preto plat

d = 4,
kde M je set hmotnost vetkch hmotnch bodov vntri myslenej
plochy.
Tie si sami premyslite, preo hmotn body mimo myslenej plochy do integrlu neprispievaj.100
Sp k rieeniu
Dokeme si, e gravitan pole gule je identick s gravitanm poom
hmotnho bodu. Zo symetrie je jasn, e hadan gravitan pole smeruje
do stredu gule a zvis len od vzdialenosti (nie od smeru). Obklopme teda
nau guu myslenou guovou plochou s polomerom > gua . Vidme, e
99 Tj.

povrch jednotkovej gule.


Nulov priestorov uhol. Teleso sce vidme v istch smeroch, ale v kadom
jednom smere vidme dva povrchy opane orientovan! Ich prspevky do integrlu sa
navzjom vyruia.
100 Hint:

100

FX D1

Ploch Zem

vade na tejto ploche je rovnako vek a navye rovnoben s d (tj.


kolm na povrch). Preto mono psa

= gua = 4
d
gua 42
gua

= 4

=
.
2

Vidme, e polomer gule do vsledku nevstupuje.101


Teraz gravitan pole nekonenej roviny (dosky). Zo symetrie je jasn,
e smeruje kolmo do roviny. Zvislos od vzdialenosti jasn nie je. Preto
si ikovne zvolme tak gaussovsk plochu, aby sa nm integrl dobre
potal.

Tto uzavret plocha pozostva z dvoch podstv toho istho, no inak


ubovonho tvaru, ktor sa nachdzaj v rovnakej vzdialenosti na opanch stranch dosky. Plochu kadej z nich ozname . Plocha sa uzatvra
cez pl spusten kolmo na rovinu.
omu je rovn integrl v tomto prpade? Vimnime si, e na kadom
ksku pla je skalrny sin d rovn nule, teda pl do hodnoty
integrlu vbec neprispieva. Sta teda zrta integrl cez podstavy. Tam je
vak kontantn a navye kolm na povrch. A my u vieme, e to predsa
nie je iadne integrovanie! Meme hne psa

d = 2rovina = 4
2rovina

rovina

2 .

101 Znova

pripomnam, e tento vsledok nie je samozrejmosou, a e za vame zvilosti gravitanej interakcie od 1/ 2 .

101

FX D1

Ploch Zem

Op raz dostvame, e takto pole nezvis na vzdialenosti od roviny.


Ak jednoduch! Takto rchlo sa daj pomocou Gaussovho zkona odvodi
vzahy pre gua a rovina . Dopracova sa k vsledku je teraz u trivilna
zleitos.
Poznmka 1: Skste si ako cvienie pomocou Gaussovho zkona dokza, e gravitan pole vntri homognnej krupiny je nulov (in dkaz
njdete v poznmke pod iarou vo vzorom rieen lohy FX D3 Polgule).
Tie si mete dokza, e vntri homognnej gule kles gravitan pole
linerne z hodnoty na povrchu na nulov hodnotu v jej strede.
Poznmka 2: Bohuia, iadnu praktick lohu na vyuitie Gaussovho
zkona pre gravitan pole nenjdeme. Gauss ho vak v skutonosti vbec
neformuloval pre gravitan, ale pre elektrick pole. Naozaj, ve sta spravi
zmeny , a 1/40 a dostvame

d = .
0
aa, konene vidme, preo je v Coulombovom zkone hlpy faktor 1/4.
Aby iadny hlpy faktor nevystupoval v rovniciach vyej fyziky.
Aplikcie tohto tzv. Gaussovho zkona s alekosiahle. Ide o jednu zo tyroch Maxwellovch rovnc, ktor podvaj plny popis elektromagnetickch
pol a ktor v sebe kduj aj pecilnu teriu relativity!

102

FX D2 Asteroid
Azag sa hral so svojm novm alekohadom, ke zrazu spozoroval pohybujci sa asteroid. Azag zistil, e tento asteroid sa prve nachdza vo vzdialenosti od Slnka, jeho okamit rchlos je , a smer jeho rchlosti zviera
uhol so spojnicou asteroid-Slnko. Ak je jeho perida obehu okolo Slnka?
Zdroj: lohu vymyslel Bzduo

l Vzork Jakub l
Najprv si sformulujem lohu tak, aby som ju bol schopn vyriei. To je
zklad :-). Ako prv zabudneme na vetky teles okrem Slnka a asteroidu.
alej sa uspokojme s nerelativistickm rieenm. To by ete stle mohlo by
ak, take sa uskromnme s popisom Slnka a asteroidu pomocou modelu
hmotnho bodu. Ako jedin interakciu medzi Slnkom a asteroidom budem
uvaova newtonovsk gravitciu (ie iaden tlak slnenho iarenia, je mi
to). Celkovo sme situciu maximlne sprehadnili za cenu toho, e ignorujeme krsnu otvoren hviezdokopu Plejdy (pred svitanm je ju krsne
vidie) a okrem toho aj plejdu zaujmavch javov spojench s vplyvom
okolitch plant (precesia perihlia a in poruchy drhy), relativistickmi
efektami (tie precesia perihlia; pozorovan najlepie na drhe Merkru),
rotciou telies (precesia zemskej osi, nutcia), deformovatenosou telies (slapov javy: prliv/odliv, spomaovanie rotcie Zeme) alebo slnenm vetrom
(plachtenie kozmickej plachetnice). Vybrali sme si vak tak, aby jav (pohyb
po trajektrii), ktor chceme tudova, ostal vcelku nedotknut ustanovenmi zjednueniami.
lohu vemi efektvne vyrieime, ak si spomenieme na Keplerove zkony
a zkon zachovania energie (mechanickej, pravdae). Mali by sme si teda
pripomen prv dva Keplerove zkony v ich sasnom znen - tie sa nm
bud hodi. Platia pre sstavu pozostvajcu z dvoch hmotnch bodov,
ktor na seba psobia prostrednctvom potencilu tvaru () = kde sa
uvauje > 0.102
Kad z telies sa pohybuje po elipse/parabole/hyperbole103 , v ktorej
ohnisku (resp. v jednom z ohnsk) sa nachdza aisko sstavy.
102 Napr. pre gravitciu poda Newtona plat = , kde je gravitan kontanta
a , s hmotnosti oboch telies.
103 Patologick prpady, ke sa teles pohybuj iba po priamke k sebe alebo od seba sa
daj chpa ako limitn prpad tchto slunch kueloseiek.

103

FX D2

Asteroid

Pre kad z telies plat zkon zachovania momentu hybnosti vzhadom


na aisko sstavy. Konkrtne pre zloku momentu hybnosti kolm na
rovinu pohybu telies104 dostaneme znmu formulciu o kontantnosti
plochy opsanej sprievodiom telesa za jednotku asu.
Kee n asteroid je zrejme omnoho ah ako Slnko105 , tak aisko sstavy je vemi blzko bodu, do ktorho sme skoncentrovali hmotu Slnka. Inak
je potanie bohatie o jeden koecient, avak neposkytuje bohat ivot...
To sa hovor. My sa o tom presvedme! Nech teda hmotnos je hmotnos
asteroidu a je hmotnos Slnka. Rieme problm z aiskovej sstavy. T
je vaka horeuvedenm predpokladom inercilna.106 Kvli prehadnosti zavediem kontantu =
+ . Potom ak je vzdialenos telies od seba, tak
vzdialenos asteroidu od aiska sstavy je = . Vzdialenos Slnka od aiska sstavy je = (1 ). itate by si mal premyslie, e v aiskovej
sstave platia aj vzahy, ktor vyta z nasledujceho obrzka.

V obrzku je jednotkov vektor v smere pohybu asteroidu, je jednotkov vektor v smere od slnka k asteroidu, je okamit relatvna (vzjomn)
rchlos telies a uhol je v zadan spomnan uhol, ktor zviera vektor rchlosti asteroidu (rchlos vzhadom na aisko sstavy ) so sprievodiom.
Vektor je rchlos Slnka vzhadom na . Tu si treba premyslie aj to, e
uhlov rchlos pohybu oboch telies okolo ich spolonho aiska je jedna a
t ist .
Druh Keplerov zkon nm d rovnicu 2 = kont. Nae tvrdenie podporme obrzkom

104 Kad

by si mal dkladne premyslie, preo sa pohyb d umiestni do 1 roviny!


to tak bva, ale ktohovie, oho sa ete doijeme...
106 ie je v rovnomernom priamoiarom pohybe alebo v pokoji vzhadom na vetky
ostatn inercilne vzan sstavy.
105 Zvyajne

104

FX D2

Asteroid

z ktorho vidme, e plocha opsan sprievodiom asteroidu (ktor zana


v aisku) za as je rovn 12 ( +) sin .107 Kee asov interval
meme voli ubovone mal, tak aj a bud prslune mal. Potom
meme plnm prvom poui aproximciu sin . Ke sa obmedzme na leny, ktor zvisia od iba v prvej mocnine108 , tak nm ostane
plocha 21 2 . Teda rchlos opisovania109 plochy sprievodiom asteroidu
2
0 2

je lim

= 12 2 = 12 2 2 . Poda prvho obrzka vidme, e rchlos

opisovania plochy sprievodiom asteroidu je 12 = 12 2 sin . Porovnanm vzahov dostvame = sin . Hne teda vidme, e podmienka
ozn.
(I) 2 = kon + t = sin = nm zaru kontantnos opisovania
na prijatenej rovni. :-)
Zkon zachovania (mechanickej) energie nm d rovnicu (II)
1

1
2 + 2
,
2
2

1

= 2
,
2

1 2 2

,
= 2
2 sin

mech =

kde =
e

je takzvan redukovan hmotnos. Pritom zo zadania vieme,


mech =

1 2

.
2

Prv Keplerov zkon nm d obraz toho, ako situcia vyzer. Ak teles


obiehaj po uzavretch krivkch (elipsch), tak pohyb vyzer zhruba ako na
obrzku.

107 Ak zanedbme oblkovitos trajektrie, o vak regulrne meme, lebo ns zaujma


0.
108 Vyie leny ns nezaujmaj, lebo plochu opsan sprievodiom dostanem tak, e
plochu, ktor ope za as predelm tmto asom a urobm limitn prechod k 0.
109 To vzniklo spontnne! :-)

105

FX D2

Asteroid

Tento obrzok nm hne aj dva nvod, ako zisti vznan charakteristiky


pohybu. Toti, hadajme tak body trajektrie asteroidu, v ktorch je sprievodi kolm na rchlos (a teda aj dotynicu a plat = 2 sin = 1
= ) - pohadom na mon kueloseky zistme, e takto celkom urite
zskame daje o vrcholoch110 drhy. Uke sa, e in body tto vlastnos
nemaj. Dosadenm za z rovnice (I) do (II) dostanem kvadratick rovnicu
pre
2

mech =

,
2
2

ktorej diskriminant je (dosadm za a mech a upravm)


= ( )2 + 2 2 mech
= ( )2 2 2 sin2 + 2 2 4 sin2
= ( 2 sin )2 + 2 2 sin (1 sin ) > 0.


Rieenie rovnice je 1,2 = 2
pre = 0.
Zrejme ns interesuj iba kladn rieenia.

D sa priamo ukza111 , e ak mech < 0, tak obe rieenia s kladn.


Svis to s tm, e teles nemaj dos energie na to, aby sa od seba
mohli vzdiali ubovone aleko112 . Jedin so slunch kueloseiek
s takouto vlastnosou je elipsa a t m 2 vrcholy.
sin
Pre mech = 0 je rieen rovnica linerna a jej rieenie je = 2
.
Trajektria je parabola. V limite pre sa teles prestan hba.
Pohyb nie je periodick.
2 2

Pre mech > 0 sa d ukza, e kladn rieenie je prve jedno. Trajektria je hyperbola. V limite pre sa teles neprestan hba.
Pohyb nie je periodick.
Rieenie
Zrejme ak mech 0, tak pohyb nie je periodick, o formlne meme
zapsa = . Ak mech < 0, tak trajektria je elipsa, u ktorej poznm
vzdialenosti 1 , 2 oboch vrcholov od jednho z ohnsk. Potom vekos hlav2
2
nej polosi elipsy asteroidu je = 1 +
a excentricita je = 1
. Ve2
2

2
2
kos vedajej polosi je = . Teraz u viem ahko vyjadri peridu
110 Vrchol

kueloseky nazvam bod, v ktorom m krivka najvaiu krivos.


Odporam prekona odpor, sadn si za stl a presvedi sa o tom runestrune. A kto s tm bude ma problm, nech sa ozve! (Aj tak sa nikdy nikto neozve...)
112 Na to by museli ma celkov energiu nezporn, lebo ak by sa aj v limite nekonenej
relatvnej (vzjomnej) vzdialenosti nepohybovali, tak by museli ma potencilnu energia
pre a t je nulov.
111 taj:

106

FX D2

Asteroid

obehu, lebo plocha elipsy asteroidu je = a rchlos opisovania plochy


sprievodiom asteroidu je 12 2 = 12 2 sin = kont, teda
=

= = 2( + )
1 2

sin
2
(
2

2( + ) 2

2 2

)3/2

)3/2
.

Zrejme naozaj ak , tak meme vraz + nahradi hmotnosou


Slnka . Vimnite si, e vsledok nezvis od uhla .
Ete jeden mal detail sme si nechali na zver. Toti ak = 0 = ,
tak sa pohyb realizuje po priamke. Ak v naom priblen hmotnch bodov
ignorujeme monos prpadnej zrky, tak by sme mali riei diferencilnu
rovnicu
d2

= 2 =
= ,
d

d2

=
2
d

d(0)
= .
s podmienkami
(0) = ,

d
Tto rovnica by poda takej naej fyziklnej intucie mala ma rieenie a
po okamih prvej zrky (t.j. situciu, ke = 0) zhodn s rieenm keplerovskm v limite pre 0. Neskr je to u celkom obskurn zleitos,
kee rovnica m v bode = 0 singularitu. Sn nem vemi zmysel hovori o tom, o by bolo, keby sa nae hmotn body predsalen nezrazili a
preleteli cez seba. Fyziklne je ovea zaujmavejie uvaova pohyb eliptick
a zvi limitu, ke 0 (resp. 0). Toti pohyb presne po priamke m
praktick nulov pravdepodobnos. Tm sme hotov!
ie rovnicu

107

FX D3 Polgule
V rmci ekonomickch opatren pred oslavou jubilejnho roku 3000 sa byrokracia rozhodla skrti vdavky tm, e odstrnia polovicu Zeme. Vkonn
ata najprv rozrezala Zem na dve polgule a potom jednu z nich vymazala
(prkazom delete, samozrejme).
(a) Akou silou sa tieto dve polgule priahovali?
(b) Po vymazan druhej polgule, ak bolo gravitan zrchlenie v mieste,
kde sa kedysi nachdzal stred Zeme?
Zdroj: KMaL

l Vzork Kubus l
as (a)
Prvm kovm osvietenm je, e ak sa odhodlme integrova tto silu
cez mal objemov elementy jednej polgule, nemusme pri tom pota iba
s gravitanm poom druhej polgule. Meme pokojne uvaova gravitan
pole celej Zeme. Ve ono tam aj naozaj je. Prspevky psobenia uvaovanej
polgule na sam seba sa nm vyruia.113
Takisto si spomeme, e homognna guov krupina psob na svoje
troby celkovo nulovou gravitanou silou.114 Ete predpokladajme, e Zem je
homognna, tj. m vade rovnak hustotu , jej hmotnos je a polomer .
113 Dkaz
114 Tu

je jednoduch. Ide len o princp akcie a reakcie.


je dkaz komplikovanej:

Predstavme si homognnu krupinku, hociktor jej vntorn bod a z neho dva nprotivn smery. Z jednoduchej
geometrie sa d ukza, e ak si okolo nich urobme innitezimlne priestorov uhly d, tak nimi vymedzen plocha
bude priamo mern tvorcu vzdialensti krupinky v danom
smere, kontantu mernosti ozname . Kada z tchto plok vytvra vo zvolenom bod gravitan intenzitu
=

2
= 2 = 2 = = kont
2

Kad dva protiahl ksky vytvraj vo zvolenom bode


rovnk intenzitu ale opanho smeru, dovedna teda nulov. A kee kad ksok povrchu
m svoj nprotivok, set vetkch prspevkov do ostva nula. Takto vahu mono
spravi pre kad bod vntri krupinky, take v celom jej vntri je nulov gravitan pole.

108

Polgule

FX D3

Potom vekos sily psobiacej na ksok Zeme d vo vzdialenosti od stredu


bude

3 d
d = 3
= 3 d.
(30)

Vmajc si symetriu celej situcie je nm jasn, e vetky zloky sily


rovnoben s rovinou rezu sa vyruia. Sta nm teda stava iba tie kolm.
Z podobnosti trojuholnkov mme / = /, kde je kolm vzdialenos
elementu d od roviny rezu. Potom

d
3

d = 3
d.
=
3

polgua
polgua

d =

(31)

Predtm, ne sa bezhlavo vrhneme do integrovania, zamyslime sa, o


vlastne znamen d. Toto je vlastne kolm vzdialenos aiska naej
polgule od roviny rezu vynsoben jej hmotnosou! (aa, dencia aiska!)
Vediac, e vzdialenos aiska polgule od roviny rezu je 38 meme priamo
psa
3 2
3
=
1,1 . 1025
3 8 2
16 2
2 2 4
=
.
3
=

alebo

(32)

Ak nevieme, kde je aisko polgule alebo chceme silou-mocou integrova,


rovnicu (31) si meme prepsa(zoskupi d-k, ktor maj rovnak , do
tenukch valcov s polomerom 2 2 a hrbkou d):


= 3
d =
(2 2 ) d

3 0
polgua

43 3 2
=
( 3 ) d
3
0
[ 2 2
]

4
4
2 2

=
3
2
4 0
=

2 2 4
.
3

Aby toho nebolo dos, ukme si ete jeden pekn prstup, tentoraz energetick (resp. cez virtulnu prcu). Predstavme si, e tieto dve polgule posunieme od seba o mal vzdialenos . Sila, ktorou sa bud tieto polgule
priahova, sa vemi nezmen, preto meme napsa, e sme vykonali prcu
1 = .
109

Polgule

FX D3

Teraz prde nta f. Na to, aby sme polgule od seba odsunuli, teda dosiahli ten ist efekt, ako pri odsunut o vzdialenos , meme urobi aj
toto. Vykopeme vetku zeminu v okol roviny rezu tak, aby medzi polguami
vznikla medzera rky , a tto zeminu porozotierame po povrchu Zeme tak,
aby obe asti znova vyzerali ako polgule. Vimnime si, e i ke tto zeminu
nebudeme po celom povrchu rozotiera rovnomerne, hrbka pridanej vrstvy
bude v kadom mieste ovea menia ne .
Ak prcu sme vykonali tentoraz? Valec zeminy o vke a polomere
sme vyaili na povrch Zeme. Poda rovnice (30) pre gravitan silu vntri
Zeme potrebujeme na vytiahnutie kusu zeminy o hmotnosti d zo vzdialenosti od stredu Zeme prcu

d =
d

() d =

d
d
3

d 2
( 2 ).
=
23

Zeminy vo vrstvike hrubej d, irokej , vo vzdialenosti od stredu Zeme


je presne 2 d. Celkov prca na vytiahnutie vetkej zeminy bude teda

2 2
( 2 ) d
23
0

4
2 = 2 2
(2 3 ) d
3
0
2 2 4
.
2 =
3
2 =

(33)

Kee toto natieranie m rovnak efekt ako odsunutie polg, muselo


ns st rovnako vea energie. A ha, poloenm 1 = 2 zskavame star
znmy vzah (32) pre praliv silu medzi polguami .
as (b)
Ako v predolom prklade, aj tu meme slepo integrova. Naprklad si
zavedieme sfrick sradnice , a tieto si meme predstavi tak, e
ak sme Zem rezali pozd rovnku, potom 0, 2) je zemepisn dka,
0, /2 zemepisn rka a jednoducho vzdialenos od stredu. Potom
objem malho ksku Zeme popsanho tmito sradnicami bude
d = ( d) ( cos d) d = 2 cos d d d.
110

Polgule

FX D3

Ete si vimnime symetriu celej situcie sta, ak budeme pota zloky


gravitanho zrchlenia kolm na rovinu rezu, pretoe ostatn sa staj
do nuly:

= =
d =
sin d
(34)

d
d
=
sin 2 =
sin

2 /2
2 cos
=
sin
d d d
(35)
2
0
0
0
/2
=
2 sin cos d d

alebo

=
3
3

= 0 7,36 m.s2 .
= 4 3 =
2
4
4
3

(36)

To by vak nebolo FX, keby sme si neukzali ete jeden postup. Najprv
si vimnime, e kad polkrupinka s hrbkou d prispieva ku rovnako,
pretoe je mern jej hmotnosti, ie ploche, ie 2 , ale je takisto mern
1/2 z gravitanho zkona, ako je zrejm z rovnice (35).115 Preto sa meme
pozrie len na jednu polkrupinku (naprklad t s polomerom ) a potom
115 Pozor, v tomto tdiu silne zvdza poveda si, e kee polkrupinky kadho polomeru prispievaj rovnako, nafkneme ich vetky na polomer a sta nm rta gravitan
pole jednej polkrupiny o polomere a hmotnosti /2. To vak nie je pravda, pretoe
pri ekvivalentnom nafukovan , kde sa nemen hrbka d krupinky sa nm nutne men
jej objem a teda i hmotnos!

111

Polgule

FX D3

vsledn sila bude tokokrt via, koko polkrupiniek sme mali teda
/d krt.
Kad kus d tejto polkrupinky psob na dao o hmotnosti v strede
Zeme silou kolmou na povrch polkrupinky. Vekos tejto sily je priamo
mern vekosti plochy tohoto kusu krupinky, pretoe hmotnos je mern
ploche:
d d
d
d =
=
d
2
2
Poda predchdzajcej vahy bude z tohto smeru sila od vetkch krupiniek /d krt via ako sila od tejto krupinky:
d d

=
d
(37)
2
d

Teraz by sme to chceli integrova cez povrch najvej krupinky. Zastavme sa vak na chvu a zistme, e cel tento vraz a prli npadne
pripomna tlakov silu! Ak si predstavme cel polguu v ndobe s vodou,

kde je vade tlak =


, nae d bude ma vekos i smer presne rovnak
ako tlakov sila psobiaca na plku d. Take celkov sila bude rovn
celkovej tlakovej sile psobiacej na pl polgule.116
Pozorn itate si vimne, e zatia sme sa ete nikam nedostali, iba sme
si problm pretransformovali na in, ekvivalentn. Ete pozornej itate si
vak vimne, e na spotanie tlakovej sily na pl polgule nepotrebujeme
ni integrova. Na nau polguu ponoren vo vode toti psobia dve sily:
Tlakov sila na pl a tlakov sila na podstavu. Kee vade naokolo je
rovnak tlak , polgua sa nem preo hba a tieto dve sily musia ma
rovnak vekos. Ha! Ale tlakov silu na podstavu vieme spota avou
zadnou, je to predsa
d =
d

podst. = podst. = (2 )
2
= podst. =
= .

Vekos tiaovho zrchlenia u vypotame jednoducho ako


=

= ,

(38)

(39)

dostvajc rovnak vsledok ako (36).117


116 Pod

plom sa mysl len zakriven as povrchu, t.j. bez roviny rezu.


vahy v poslednom rieen meme matematicky zapsa aj takto (poda konvencie
normlov vektor smeruje von z uzavretej plochy):

d
=

pl

polgua
podstava
117

112

FX D4 Hmota
Marin si kpil asteroid s objemom z plastickej hmoty o hustote , a
v priestore si vyznail bod . Ako m tento asteroid vytvarova a kam ho
m umiestni, aby bolo v bode o najvie gravitan pole? Ak by bola
vekos tohto gravitanho poa, ak by parametre a asteroidu boli zhodn
s tmito parametrami Zeme?
Zdroj: BAUPC

l Vzork Bzduo l
lohu si rozdelme na dve asti, ktorm sa poriadne povenujeme. Najprv
musme njs tvar telesa, v om nie je nijak ak matematika. Ke sa
vysporiadme s nm, zistme vekos gravitanho zrchlenia v bode , o
nm d priestor si z chuti zacvii integrovanie.
Vlome si bod do poiatku sradnicovej sstavy a poadujme, aby
vsledn pole v tomto bode bolo rovnoben s osou .118 Aby sme boli
Posledn integrl je jednoducho

podstava

d = .

Z Gaussovej rovnice sa d odvodi rovnos

d d =

polgua

) d
(

(40)

0 d = 0

Skste si ako cvienie odvodi Archimmedov zkon! Pre zujemcov ete dodatok o odvoden rovnosti (40). Uvaujme ubovon nenulov vektor a upravujme vraz

(1)
=
)) d
) d =
) d
( + (
(
(
d
=

(2)
) d.
) d =
(
(
=

) = (
) + (
) a v prave (2)
Kde v prave (1) sme vyuili identitu (
skutonos, e je kontantn vektor. Kee rovnos plat pre ubovon vektor , zrejme
aj integrly na zaiatku a na konci musia by rovnak.
118 Premyslite si, e symetria situcie vyaduje aby hadan tvar asteroidu bol symetrick
okolo osi .

113

FX D4

Hmota

dohodnut plne jednoznane, tak nech pole v smeruje doprava a teda aj


asteroid sa bude nachdza napravo od osi .
Predstavme si, e v rukch drme nejak asteroid. Ak by sa dala jeho
hmota nejakm spsobom popresva tak, aby sa v zvilo pole, tak by
sme ju presunuli. Ak budeme v ruke dra sprvny asteroid, akkovek presun hmoty bude pole v danom bode len zmenova. Znamen to, e povrch
asteroidu mus by tvoren bodmi, ktor prispievaj k -ovej zloke gravitanho zrchlenia v rovnakou mierou. Inak by sme predsa mohli odlepi
kus hmoty odniekia, kde prispieva menej, a prida ju niekam, kde prispieva
viac.
Ak si vezmeme rez telesa rovinou , tak hmotnos d v nejakom mieste
[, , 0], vytvra v bode pole vekosti
d =

d,
+ 2

kde je gravitan kontanta. Pre ns je dleit -ov zloka tohto gravitanho zrchlenia,

d =
d =
3 d.
(2 + 2 ) 2
2 + 2
Vieme, e hustota hmoty je kontantn, preto aby rovnako vek kus
hmoty na ubovonom mieste povrchu prispieval k pou v bode rovnako,
mus pre cel povrch asteroidu plati

(2 + 2 ) 2

= ,

kde je aksi kontanta, o ktorej zatia ni bliie nevieme. Ke si tento


vraz vhodne upravme, dostaneme rovnicu pre povrch asteroidu v tomto
reze.

2
= 3 2
Nov kontanta je rovn 3 . Pripomnam, e asteroid je rotane symetrick okolo osi x. Ak cheme zska predstavu, ako ten asteroid vyzer,
meme vyui dky mazan softvr:
2

114

FX D4

Hmota

Teraz je ten sprvny as na zvrhl integrovanie. Ak siln pole vytvor


takto asteroid s objemom na svojom povrchu?119
Rozrieme najprv problm normovacej kontanty, tj. ak mus by v naom vraze pre , aby objem asteroidu bol naozaj ? Rozsekajme si pozd
osi x asteroid na nespoet diskov s hrbkou d. Kad z nich bude ma
objem
d = 2 d.
Celkov objem urme integrovanm. Doln medza integrlu je zrejme
3
nula.120 Ak polome = 0, dostaneme aj horn medzu = 4 . Ak to
vetko spojme dokopy, postupne dostvame
[
]= 43
34
34
2
5
3
1 3
2
2
=
d =
( 3 ) d = 3
,
5
3
0
0
=0
po dosaden a prave dostaneme
=

4 9
4 ,
15

(
ie

15
4

) 49

119 itateovi odporam vzia si do rk ceruzku a papier a vetky integrly si sksi


predstavi na vlastnom obrzku.
120 Zaviedli sme to ete pri odvdzan rovnice povrchu asteroidu

115

FX D4

Hmota

Konene sa meme pozrie na vytvoren gravitan pole. Aby sme mali


vsledok s m porovna, spotajme si vopred, ak je tiaov zrchlenie Zeme
vyjadren pomocou jej hustoty a jej objemu . Zem vytvra gravitan
pole ako hmotn bod,121 preto na jej povrchu bude
=

2 .

3
Ak si vezmeme = , kde objem je 43
, dostvame
(
) ( )2
(
)1
1
4
4 3
4
16 2 3
3
=
=

= 3
.
3
3
3
9

Teraz sa pozrime na gravitan pole nho asteroidu. Op si asteroid


rozsekme na mrne ksky. Konkrtne znova na tenuk disky s polomerom
, ktor ale ete bud sam poskladan z akchsi prstencov so stredom na osi
x. Ozname prspevok jednho konkrtneho disku vo vzdialenosti a hrbky
d od poiatku sradnicovej sstavy ako d. Ten zskame integrovanm prspevkov od spomnanch prstencov. Ozname preto ete prspevok jednho
konkrtneho prstenca s polomerom , hrbkou d ako d2 . K vslednmu
pou v bode bud prispieva len zloky d2 v smere osi , kolm zloky
sa zo symetrie vyruia. Ke si ete uvedomme, e hmotnos d takhoto
prstenca bude rovn d = 2 d d,122 dostaneme
d2 =

2 d d

2 + 2
2 + 2

Pre spomnan disk dostvame


=()
d =
d2
=0

2
3 2

2 d

=
0

[
= 2 d

d
(2

+ 2 ) 2

]= 32 2

1
1

+ 2 ) 2 =0
(
)
(
)
1
1
1 2
= 2 d
1
= 2 1 3 d.
3

(2

121 Aby tento fakt platil plne presne, musela by by Zem homognnou guou, alebo by
sa jej hustota v zvislosti od vzdialenosti od stredu musela meni vo vetkch smeroch
rovnomerne, o je do rozumnej miery splnen.
122 Predstavte si ho ako slik tvaru kvdra so stranami d, d a 2, ktor sme zatoili
do kruhu a konce spojili. Ak je dos tenk (d dos mal), pri tejto opercii sa objem
prli nezmen.

116

FX D4

Hmota

Vsledn pole v bode dostaneme integrovanm d od 0 po max =


3
4 .123 Postupne dostvame
max
d
=
0

)
1 2
1 3 d

0
[
]= 34
5
3
= 2 3
5
=0
(
)
3
3 ( 3 ) 53 1
= 2 4
4
2
5
3
4
=
4 .
5

= 2

Ak si spomenieme, e kontantu sme si vyjadrili pomocou objemu ,


meme ju za nahradi a po pr pravch dosta tvar
(
=

1
3

48 2
25

) 13

Konene sme doli k rieeniu. Sta u len porovna gravitan zrchlenie


vytvoren Zemou na jej povrchu a pole vytvoren nam asteroidom v bode
. Pre podiel ich vekost dostaneme

3 27
=
1,026.

25
Vidno, e to nie je vek rozdiel a e gua je vcelku optimlny tvar. Vetky
integrly sa dali samozrejme pota aj v sfrickch sradniciach, vsledky
s vak tie ist, tak sa s tm nebudeme trpi. Dfam tie, e vetky hladn
otzky, ktor bolo treba zodpoveda, s zodpovedan a zvyn si zodpovie
kad aj sm.

123 Rovnos

sme odvodili pri potan objemu asteroidu z podmienky = 0.

117

placeholder

118

E
Mechanika II.

Does a one-legged duck swim in circles?


R. Morgan

119

FX E1 Gua
Vla Amlka si svoju kritov guu poloila na vek stl pokryt obrusom. Ke mala slvnostn nladu, vekolepo strhla obrus zo stola (vodorovnm pohybom). Ak bude rchlos gule po tom, ako na stole prestane premykova? Mete predpoklada, e toto sa stane ete predtm, ako spadne
zo stola, a e gua pri sthan obrusu neposkakuje. Polomer gule je , hmotnos , koecient trenia o obrus 1 a o stl 2 .
Zdroj: lohu vymyslel Bzduo

l Vzork Jakub l
V nasledovnom texte budem nau kritov guu povaova za dokonale
homognne teleso. Moment zotrvanosti homognnej gule vzhadom na ubovon os prechdzajcu jej stredom (ktor je tie aiskom) je = 25 2 ,
kde a s veliiny znme zo zadania. Popisova ju budem pomocou modelu tuhho telesa. Sstava spojen so stolom bude pre ma inercilna.
Poas trhnutia, ak je dostatone prudk, bude gua zrejme premykova.
Ak je vak trhnutie iba tak relatvne pomal potiahnutie, tak sa me sta,
e gua premykova nebude. Nepremykovaniu sa d tie pomc zvenm
koecientu trenia 1 . Kee bez konkrtnych dajov nevieme poveda, ktor
z monost sa bude realizova, tak by sme mali rozobra vetky monosti.
Avak tch je mnoho, lebo zadanie iaden konkrtny priebeh zrchlenia obrusu nepredpisuje. Take obrus meme najprv trhn a alej u len pomaly
zrchova. A cel to nemus prebieha len v jednom smere meme najprv
trhn jednm smerom a potom ete aj smerom kolmm na, naprklad.
Podrobn analza velijakch prpadov by bola znane komplikovan a oemetn, a ako ukeme, aj zbyton. My sa poksime vyriei vetky prpady
sasne. Zvldneme to pre ubovon trhnutie v rovine obrusu (s ubovonm priebehom vektora zrchlenia obrusu) a ete aj pre ubovon zkon
trenia. itateovi predkladme dve rzne rieenia:
A. Rieenie na zklade dynamiky tuhho telesa inpirovan Michalom Spiiakom.
B. Formlna analza situcie z pohadu zkona zachovania momentu hybnosti.
Vstraha! V alom texte sa to bude hemi vektormi. Budeme ich znai
tunm psmom (naprklad vektor rchlosti o vekosti ), a budeme s nimi
120

FX E1

Gua

operova najm pomocou vektorovch, resp. skalrnych sinov. Odporam


itateovi pripomen si zkladn vlastnosti tchto vektorovch operci.
Taktie sa tu vyskytn derivcie a integrly vektorovch velin, ktor s
zaveden vemi prirodzenm spsobom, ha:
)
(
()
d
d () d () d ()
=
,
,
,
d
d
d
d

(
()d =

()d,

()d,

)
()d .

A. Rieenie na zklade dynamiky tuhho telesa inpirovan Michalom Spiiakom.


lohu budeme riei v inercilnej vzanej sstave spojenej so stolom.
Na guu psobia poas celho pohybu (a pokm neprestane premykova) prve 3 sily: tiaov sila , normlov sila od podloky a trecia sila
od podloky . Kee guka poda zadania nenadskakuje, tak v kadom
okamihu mus plati, e zvisl zloka vslednice psobiacich sl ju nulov
(teda tiaov sila je vdy kompenzovan normlovou silou)124 . Vodorovn
zloka vslednice je potom samotn trecia sila a teda zrchlenie aiska guky
()
je () =
(tu treba upozorni na to, e trecia sila je vo veobecnosti
funkcia asu m vyrieime sasne premykovanie i nepremykovanie
me meni vekos i smer, vdy vak le v rovine stola).

alej zisujme, o sa bude dia s otanm guliky. To je celkom jednoduch, lebo v naom jednoduchom prpade (gua je tzv. sfrick zotrvank, ktor je charakteristick tm, e m moment zotrvanosti vzhadom
na kad os prechdzajcu aiskom rovnak) plat pre moment sily ()
124 primne povedan, plat to tak skoro vdy. Ke gulika schdza z obrusu, tak zrejme
troku men svoju vku. alej sa uke, e tento prstup rieenia je vak vo svojej podstate
voi zvislm silm plne necitliv. Postaujce je, ak trecia sila je vodorovn.

121

FX E1

Gua

a moment hybnosti () potan vzhadom na aisko tuhho telesa plne


veobecne (dkaz mono njs v B. asti)

() =

d
d
() = (()) = ().
d
d

V tomto vzahu () zna celkov moment sl psobiacich na dan teleso


uren vzhadom na stred (aisko) guliky v ase ; () je moment hybnosti
telesa potan tie vzhadom na stred guliky; je moment zotrvanosti
() (kde
telesa vzhadom na os prechdzajcu aiskom; vektor () = ()
() je jednotkov vektor) je vektor uhlovej rchlosti otania, priom jeho
vekos () je to, o sa bene pod pojmom uhlov rchlos mysl, a jeho
smer dan vektorom () uruje os otania; () je prv asov derivcia
(), ie vektor uhlovho zrchlenia a v prpade, e smer m toton so
smerom (), tak jeho vekos je nm bene znme uhlov zrchlenie.
V prpade zobrazenom na obrzku vyie je vektor a s nm aj
kolm na obrzok (to je dan tm, e trecia sila le v rovine obrzka).
Celkov psobiaci moment vzhadom na aisko guky je dan len momentom trecej sily (ostatn sily leia na priamke prechdzajcej aiskom). Plat
() = 0 (), kde vektor 0 je zvisl vektor o dke z aiska guliky do jej dotykovho bodu (v kadom ase rovnak). Tento moment jej
()
teda udeuje uhlov zrchlenie () =
.

Je zrejm, e guka v nejakom konenom ase (oznam ho ) prestane


premykova. alej sa bude pohybova rovnomerne priamoiaro. Trenie na
u potom u nebude psobi. Ke sa to stane, tak dotykov bod bude voi
stolu nehybn, o zapeme rovnicou

( ) = ( ) + ( ) = ( ) + ( ) 0 = 0 ,

ktor vyjadruje, e rchlos dotykovho bodu vzhadom na stl napsan ako


vektorov set rchlosti aiska vzhadom na stl a obvodovej rchlosti dotykovho bodu okolo osi prechdzajcej aiskom (to si u kad na prstoch
over, e to je presne ten druh len) je nulov. Pritom ale vieme, e

() d =

122

() d

FX E1

Gua

a tie vieme (posledn prava vyuva fakt, e vektory () a 0 s na seba


v kadom ase kolm)
( ) = ( ) 0
{
}

=
()d 0
0

=
0
2

0 ()
0

)
d

() d.

Vnmavej itate u iste vid, o prde alej. Toti, vieme e


( ) = ( ) + ( )
(
)
1
2
=
+
() d

0

7
() d = 0 .
=
5 0
To ale znamen, e v ase, ke guka prestane premykova, je celkov impulz

trecej sily 0 ()d a s nm aj rchlosti ( ), ( ) a nutne aj uhlov


rchlos otania ( ) nulov. Guka zastane.
Slovne sa to d okomentova tak, e trecia sila v kadom okamihu spsobuje zrchlenie taiska = v smere psobenia trecej sily a sasne

zrchlenie obvodovej rchlosti dotykovho bodu gule = 5


2 v rovnakom smere. Pomer tchto zrchlen je v kadom ase kontantn (konkrtne 25 ). Fakt, e gua v ase nepremykuje, je ekvivalentn s rovnosou
( ) = ( ). Potom je u jednoduch nahliadnu, e ak
2
() = ()
pre vetky [0, ],
5
(0) = (0) = 0
a zrove
( ) = ( ),
potom nutne
( ) = 0 .

123

FX E1

Gua

Ete lepiu predstavu si lovek me urobi pomocou ilustranho obrzku, ktor zachytva krivku rchlosti aiska a obvodovej rchlosti dotykovho bodu. Je tam vyznaen stav v ase 0 . Pri pohade na tento obrzok
by u kadmu malo by jasn, e ak poadujeme rovnos ( ) = ( )
v nejakom ase , tak to nutne v naej situcii znamen ( ) = 0 .
Rezultt: Guka ostane naveky nehybn za predpokladu, e sa
sprva ako tuh teleso a trecia sila (inak ubovon) le vdy v rovine stola.
B. Formlna analza situcie z pohadu zkona zachovania momentu hybnosti.
V tejto asti sa na situciu pozerm z inercilnej vzanej sstavy spojenej so stolom. Jej poiatok umiestnim do ubovonho bodu na povrchu
stola, ktor oznam .
Zanem zahka prpravou apartu. Bude to zaha dkladn odvodenia
znmych vzorcov otavej mechaniky tuhho telesa. Nerobme to zbytone
toti, tie znme vzorce platia zvyajne iba v pecilnych prpadoch.
Oboznmme sa najprv s modelom tuhho telesa. Tuh teleso je absoltne
nedoformovaten, so stlymi vzjomnmi vzdialenosami vetkch jeho bodov. Samotn teleso si predstavm ako sstavu (predstav si mnoho) hmotnch bodov s hmotnosami nachdzajcimi sa v mieste danom polohovm
vektorom (). Potom vektor do aiska viem uri ako ven priemer
() =

=1 ()

=1

()
.

=1

alej si denujem polohov vektor -tej astice s poiatkom v aisku ako


() = () (). Je jasn, e vaka nedeformovatenosti tuhho telesa
sa jeho dka nebude meni. Tie je ahko nahliadnu, e

=1

() =

[ () ()] =

=1

=1

124

() () = 0 .

FX E1

Gua

Ak mme teleso, ktor sa ota okolo osi prechdzajcej aiskom, tak


rchlos -tej astice bude (d sa ahko preveri)
() = () + ().
Teraz spotame moment hybnosti tuhho telesa vzhadom na ubovon
bod:
() =

=1

() () =

=1

() ()

=1

() [ () + ()]

=1

]
() () +

=1

() [ ()]

=1

= () () +

[ () + ()] [ ()]

=1

= () () + ()

]}
()

=1

() [ ()]

=1

= () () + () ( 0 )
+

()].
() [

=1

Nu, prv len je u v konenom tvare, druh je nulov. Ostva tret,


s ktorm vybabreme jednoducho, lebo mme teleso, ktor je symetrick voi
osi prechdzajcej jeho stredom (a to dokonca ubovonej). Toti ten len
pre teleso symetrick voi osi urenej vektorom nadobudne tvar
{

}
2

()]
[

=1

}
[ ()]

= .

=1

Preo to tak je? Majme teleso symetrick okolo osi danej jednotkovm
vektorom , ktorho sasou v nejakom ase je bod o hmotnosti . Jeho
je
polohov vektor z aiska si rozlome ako = + , kde = ( )
vektor v smere osi a je kolm na. Vaka symetrii vieme, e v danom
125

FX E1

Gua

ase je urite sasou telesa aj bod naproti nmu bodu (vnimkou by bol
bod, ktor le na osi, o je trivilny prpad) s polohovm vektorom z aiska

= = ( ) = 2( )
a rovnakou hmotnosou .

Pome spota, ako vyjde ten tret len (oznam ho ) pre tieto dva
hmotn body nho telesa:
=

()]
() [

=1

] + [
]}
= { [
] + [2( )
] [
[2( )
]]}
= { [
] [2( )
] [
]}
= { [
] [
]} = 2 { [
]} .
= 2 {[ ( )
Na itatea nechvam dkaz (sta preveri, e vekos, smer a orientcia
vslednch vektorov je zhodn; resp. preveri analyticky, e jednotliv zloky
sa rovnaj), e sa to rovn
2
= 2[( )2 ] = 2 .

Potom ale pre symetrick teleso plat skutone to, o sme tvrdili vyie.
Vsledn moment hybnosti tuhho telesa symetrickho voi osi, okolo ktorej
sa to, teda je
() = () () + .
126

FX E1

Gua

alej pome zderivova moment hybnosti poda asu:


[
]
[
]
d
d
d
() =
() =
() ()
d
d =1
d =1
]
[

d ()
d ()
() + ()
=
d
d
=1
=

[ () () + () ()]

=1

[00 + () ()]

=1

() = ().

=1

Dfam, e vsledok nikoho neprekvapil. Hoci to z rovnc vidno, napem


ete po udsky, e ten celkov moment () psobiacich sl nm vyjde
vzhadom na rovnak bod, vzhadom na ktor sme mali uren moment
hybnosti (). Tie treba poveda, e celkov moment sily psobiaci na tuh
teleso je determinovan iba silami psobiacimi na teleso zvonku. Ak by to
tak nebolo, tak by sa tuh teleso mohlo samo od seba roztoi, o by bolo
prinajmenom udn.
Spojme posledn dva vzahy dokopy. Zamerajme sa nejak xovan as
(a preto zvislos velin od neho nebudem v tomto odseku explicitne psa).
Uvaujme pritom moment hybnosti a celkov moment sl uren vzhadom
na aisko. Potom = 0 a v rovnici pre bude prv len nulov. Pre
teleso, ktor m moment zotrvanosti vzhadom na aisko rovnak okolo
ubovone natoenej osi, je potom kontanta (to je prpad naej gule).
Dostvame vzorec, ktor sa nm ziiel v asti A.
=
o vak v prpade, e mme iba teleso symetrick okolo aktulnej osi ota. Nu, ak by sme vedeli zarui, e sa nm os
nia?125 Potom plat =
otania nebude meni, tak potom by to sna mohlo fungova, no nie? No
no! Predpokladajme, e mme celkov moment sily v smere osi otania126 . Potom vektor zmeny momentu hybnosti m tie smer . Vaka
125 Ideme

odvodi rovnicu pre tie najbenejie stredokolsk prpady!


prpade xovanej osi vznikn v chytoch sily, ktor zabezpeia, e celkov moment
sl na teleso bude v poadovanom smere (o pre teleso nesymetrick voi osi otania
nemus by smer osi otania).
126 V

127

FX E1

Gua

tomu zotrv aj moment hybnosti v tomto smere. S nm ostane v tomto


smere, samozrejme, aj os otania a teda aj zmena vektoru uhlovej rchlosti
je v tomto smere. Sumrne meme kontatova, e rovnica = plat
aj v tomto prpade.
Teraz konene priiel as, ke sa od terie vrtime k nmu prachom
zapadnutmu prkladu. Toti, poas celho deja je moment sl psobiacich
na guu vzhadom na bod (pripomnam, e je to bod na povrchu stola a
situciu skmame z inercilnej vzanej sstavy) nulov! Potom ale moment
hybnosti gule mus by kontantn (jeho asov zmena je nulov) a konkrtne
nulov (lebo pred trhnutm je nulov). Preverme to podrobnejie: obrusom
mem trha ubovone rchlo, akmkovek smerom ak sa mi spodok gule
bude dra v rovine stola, tak trecia sila bude lea v nej a jej moment sily
vzhadom na bude cel as nulov. Zrove to implikuje, e vka aiska
gule je nemenn a teda nutne vertiklne zloky vetkch psobiacich sl sa
vzjomne kompenzuj. To je ale ekvivalentn s tm, e cel as je tiaov sila
rovnako vek ako normlov. Kee leia na jednej priamke (ich psobisk
s presne nad sebou, posunut v smere ich psobenia), tak ich momenty sa
akurt ruia.
Predpokladajme, e guka sa po trhnut a premykovan na stole dopracuje do stavu, ke sa jej aisko hbe rchlosou a ona nepremykuje. Aby
sme sa neprepracovali, tak si vyberme bod (mem si zvoli predsa ubovon bod povrchu stola) tak, aby leal na priamke danej polohou guky
v ubovonom ase, ke u dopremykovala, a smerom . alej si sstavu
natom tak, aby sa mi guka hbala po -ovej osi. Os nech je zvisl
orientovan nahor a os bude potom kolm na os leiaca v rovine stola
s orientciou takou, aby sstava bola pravotoiv. Vi obrzok niie. Potom
plat = (, 0, 0), = (0, , 0) =
2 127 a () = ( (), 0, ).

Dotykov bod guky (v kadom okamihu in) dan polohovm vektorom


z aiska = (0, 0, ) mus by nehybn, o zapeme rovnicou
+=0
127 Kde

= .

2 = (0, 1, 0) je len prepychov pomenovanie pre jednotkov vektor v smere osi

128

FX E1

Gua

A n tromf, moment hybnosti:


() = () () +
=
2 +
2
(
)
2
= +
2 = 0 ,
z oho = 0.
Odpove: Nech trhme ako chceme, pokia guka nezane poskakova,
tak po istom ase na stole zastane. To by bolo vetko. Vsledok je plne
exaktn, avak len pre tuh teleso a aproximciu nulovej hrbky obrusu128 .
Op meme kontatova, e trecia sila me by plne ubovon
nemus repektova vzah = , ktor je sce vemi jednoduch, ale zato
znane neserizny!129
PS1: Ak by naa guka nebola tuh teleso, ale deformovaten teleso, tak
to radej nerieim. To je u skuton fyzika. Toto je FX. :-)
PS2: Vimnite si drobn rozdiel v podmienkach platnosti v jednotlivch
astiach! V asti A. sme poadovali, aby trecia sila bola v kadom ase vodorovn. V B. sme iadali, aby guka nenadskakovala a zrove sme potrebovali
nulov hrbku obrusu.

128 o sme ticho predpokladali cel as v B. asti. Inak by trecia sila na stole alebo
podloke mala nenulov moment sily.
129 Pre informciu: o osi slunej a presnej vzah = () pouvaj technici; tu

je koecient trenia pre dvojicu stynch materilov nejak (zvyajne jednoduch) funkcia
rchlosti .

129

FX E2 Kolieska
Marcel sa rd hr s kolieskami. Minule vzal dve homognne kolieska
s hmotnosou a polomerom a pripojil ich na opan konce spolonej
ahkej osky dky 2 tak, aby sa mohli ota nezvisle od seba. Potom toto
udo poloil na vodorovn podloku a kolieska roztoil tak, aby sa oska otala uhlovou rchlosou okolo zvislej osi prechdzajcej jej stredom. Ak
je celkov kinetick energia tohto uda, ak kolieska nepremykuj?
Zdroj: KMaL

l Vzork Jakub l
Najprv si lohu skonkretizujeme, aby bola primerane ahk/ak:
Kolieska budem povaova za dokonale tuh teleso v podobe nekonene
tenkho homognneho disku (keby to bol homognny valec, tak by sa
to dalo riei presne rovnako, len by sa zmenil tenzor zotrvanosti).
Spjajca oska je dokonale tuh a nehmotn (jej kinetick energia je
nulov).
Podloku povaujem za inercilnu vzan sstavu a v tejto sstave
budem pota kinetick energiu toho zadanho uda.130
Taktiku boja zvolme priamoiaru . Dsledne odvodme formuly pre
kinetick energiu tuhho telesa a nsledne ich aplikujeme na dan lohu.
Varovanie! V tomto rieen sa bud vyskytova vo vekom pote vektorov
veliiny, psan tune. Operova s nimi budeme adekvtne ich povahe. Hojne
sa bude pouva skalrny a vektorov sin. Ako cvienie si skste dokza
platnos identity (ako roznsobi tvorec dvojlena)
+ )2 (
+ ) (
+ ) = + 2(
) + 2 + 2(
) + 2.
(
Na zaiatok rchle opakovanie o tuhom telese. Tuh teleso s celkovou
hmotnosou si predstavm ako sstavu (predstav si mnoho, vea) hmotnch bodov s hmotnosami nachdzajcich sa v miestach danch polohovmi vektormi (). Potom vektor do aiska viem uri ako ven priemer

=1 ()
=1 ()
() =
=
.

=1
130 Niekto vemi prefkan by toti mohol rta kinetick energiu tej naej hraky v sstave rotujcej s uhlovou rchlosou okolo nehybnej zvislej osi prechdzajcej stredom
2
osky koliesok. Potom by mahom ruky povedal, e = 2 12 2 , kde = 2 je mo
ment zotrvanosti plnho homognneho disku okolo jeho rotanej osi a =
je uhlov
rchlos otania koliesok okolo svojej rotanej osi.

130

FX E2

Kolieska

alej denujem polohov vektor -tej astice s poiatkom vo vybranom bode


danom vektorom 0 () ako () = () 0 (). Je jasn, e ak je bod 0 ()
xovan v telese, vaka nedeformovatenosti tuhho telesa sa dka vektora
() nebude meni. Tie je ahko nahliadnu, e

() =

=1

[ () 0 ()]

=1

() 0 () = [ () 0 ()] .

=1

Zamyslime sa nad tm, ak najveobecnej pohyb me tuh teleso vykonva! Kee tuh teleso m v kadom okamihu vzjomn vzdialenosti
vetkch jeho bodov rovnak, tak iadame, aby zobrazenie telesa v ase 1
na teleso v ase 2 bolo zhodn.131 To nm vak nesta, lebo iadne relne teleso neviem zozrkadli.132 Preto iadam ete aj, aby to bolo priame
zhodn zobrazenie. Nu a priame izometrie v priestore s len posunutie a
otoenie okolo osi a ich kombincia. Dopracovali sme sa k tomu, e najdivokejie rdeo, ktorho je tuh teleso schopn, pozostva z translanho
pohybu spojenho s rotciou okolo nejakej okamitej osi.
Ako popsa pohyb tuhho telesa? Translciu viem popsa polohovm
vektorom 0 (), ktorm sledujem ubovone vybran bod 0 nami sledovanho tuhho telesa. Znalos polohy bodu 0 mi dovouje mylienkovo zaxova
tento bod a nsledne uvaova rotciu telesa upevnenho v bode 0. Smer osi
rotcie133 popem jednotkovm vektorom () a rchlos rotcie okolo tejto
osi nie je ni in ako uhlov rchlos (). Vektoro mem cel informciu
(). Pri takomto popise plat,
o rotcii zapsa pomocou vektora () = ()
e rchlos -tej astice viem vyjadri rovnicou
() = 0 () + () (),
kde druh len vyjadruje obvodov rchlos otania -teho bodu okolo bodu
0; d sa o tom ahko presvedi (samozrejme, iba ak si itate spomenie na
vlastnosti vektorovho sinu).134
131 Inmi

slovami, zobrazenie m by izometria.


stredovo zosmerni.
133 Orientciu volme tak, aby prsty pravej ruky ukazovali smer otania ak palec m
smer a orientciu vektora .
134 Tu si treba uvedomi, e rzni pozorovatelia povauj za os rotcie rzne navzjom
rovnoben osi. Napr. pre rovnomerne sa kotajce koleso by nehybn pozorovate povedal, e koleso sa v kadom okamihu ota okolo osi prechdzajcej dotykovm bodom,
kdeto kamart idci na bicykli zarovno s kolesom by povedal, e koleso rotuje okolo osi
prechdzajcej stredom kolesa. Kadopdne, n popis je v poriadku nech bod 0 le kde132 Alebo

131

FX E2

Kolieska

Teraz spotame kinetick energiu tuhho telesa v nejakom ase (zvislos velin na ase u alej nebudem psa)

1
1
2 =
( 0 + )2
2 =1
2 =1

1
1
20 +
[ 0 ( )] +
( )2
2
2
=1
=1
{
[
]}

1
1
)2
(
= 20 + 0

+ 2
2
2
=1
=1

1
1
20 + 0 { [ ( 0 )]} + 2
2
2
2 =1

1
1
) 2 ,
20 + 0 { ( () 0 ())} + 0 (
2
2

(vi obr. niie) je vzdialenos kde veliina = sin =

2
) =
teho bodu od osi rotcie prechdzajcej bodom 0; 0 (
=1 =

2
) je moment zotrvanosti telesa vzhadom na bod 0 pre
=1 (
otanie okolo osi prechdzajcej bodom 0 v smere . Vidno, e stredn
len meme ahko znulova. Sta, ak vyberieme za bod 0 tak bod, ktor
stoj,135 alebo ak zoberieme za bod 0 aisko. Nm sa bude hodi vyjadrenie
pomocou aiska ako referennho bodu. Vektor bude v nasledovnom texte
chpan vlune ako vektor idci z aiska tuhho telesa do jeho -teho bodu.
Potom plat
0 =

= 0

=1

1
1
) 2 .
2 + (
2
2

Naou alou snahou bude vyetri moment zotrvanosti vzhadom na


). Vchadzame z rovnice
aisko okolo ubovonej osi (
) =
(

2 ,

=1

kde je vzdialenos -teho bodu od osi otania prechdzajcej aiskom.


Pomeme si obrzkom,
kovek. Toti, ak by neleal na samotnej osi otania (nech na nej le bod ), tak jeho
rchlos bude 0 () = () + () [ 0 () ()], ie () = 0 () + () () =
() + () [ 0 () + () ()] = () + () [ () ()]. o je presne to, o
chceme.
135 Vzhadom na sstavu, v ktorej potam kinetick energiu mjho tuhho telesa.

132

FX E2

Kolieska

z ktorho hne vidme, e


)2 ,
2 = 2 (
lebo je vekos priemetu vektora do smeru jednotkovho vektora
.136 Inak je to ist Pytagova veta.
Uvaujme nejak pravouhl pravotoiv sradnicov sstavu s poiatkom v aisku, pevne spojen s telesom. Vyjadrime pomocou nej vektor
( , , ) a tie jednotkov vektor smeru osi otania = ( , , ).
Potom mem psa
) =
(

=1

2 =

]
[
) 2
2 (

=1

[
]
(2 + 2 + 2 ) ( + + )2 .

=1

Teraz roznsobme druh ztvorku a prv ranovane prensobm jednotkou


v tvare 1 = 2 = 2 + 2 + 2 :
) =
(

[
(2 + 2 + 2 )(2 + 2 + 2 )

=1

(2 2 + 2 2 + 2 2 + 2 + 2 + 2 )
136 Teda

)
je samotn priemet vektora do smeru jednotkovho vektora
vektor (

133

FX E2

Kolieska

[
2 (2 + 2 ) + 2 (2 + 2 ) + 2 (2 + 2 )

=1

2 2 2
= 2

(2 + 2 ) + 2

=1

(2 + 2 ) + 2

=1

2
= 2 +

=1
2

(2 + 2 )

=1

=1

=1

+ 2 + 2 + 2 + 2 ,

o sa d prehadne zapsa pomocou maticovho

(
)
) =
(

zpisu

priom matica s J-kami sa nazva tenzor zotrvanosti telesa. Pre jeho komponenty platia vzahy

= =1 (2 + 2 )
= = =1

= = =1
= =1 (2 + 2 )

= =1 (2 + 2 )
= = =1 .
Lep pohad na diagonlne komponenty nm odhal, e sa jedn o momenty
zotrvanosti telesa okolo jednotlivch os.137 Nediagonlne leny s pre pekne
symetrick a homognne teles nulov.
Teraz prichdza tak mal fakultatvna odboka. Bude sa tka Steinerovej vety a je tu uveden len kvli tematickej spriaznenosti. Na vpoet
) a chceme
prkladu ju nebudeme potrebova. Uvaujme, e poznme (
) pre nejak in bod . Oznam vektor z aiska do bodu
uri (

ako = . Potom vektor z bodu do -teho bodu telesa bude .


Rozpsan na drobn plat
) =
(

( )] =
[

=1

) (
)]
[(

=1

[
]
)2 2(
) (
) + (
)2
(

=1

) = , o mus
to ani by neme, lebo ve pre = (1, 0, 0) dostaneme (
by prve moment zotrvanosti telesa vzhadom na -ov os.
137 Inak

134

FX E2
=

)2 2
(

=1

Kolieska

]}

) +
(

=1

)2
(

=1

) 0 + (
)2 ,
= (
je vzdialenos bodu od
o je skutone znma Steinerova veta, lebo
osi rotcie prechdzajcej aiskom. (Pretoe je jednotkov vektor v smere
tejto osi.) Vimnite si, e stredn len v predposlednom riadku vypadol kvli
sume .
Konene priiel as riei n prklad. Zo zadania sa d vyrozumie, e
aisko celho uda sa vzhadom na podloku nehbe. Avak nae udo nie
je tuh teleso, kee jednotliv koles sa mu ota vaka bombastickm vlastnostiam nehmotnej a dokonale tuhej osky plne nezvisle na sebe.
Zrejme atomistick problm bude kinetick energia jednho kolesa a je jasn,
e kinetick energia druhho kolesa bude rovnak ako toho prvho. Pozrieme
sa na problm z inercilnej vzanej sstavy spojenej s podlokou. Ako sradnicov sstavu si zvolm pravouhl pravotoiv bzu tak, e v nejakom
ase 0 problm vyzer tak ako je vyobrazen na obrzku.

Budem pota kinetick energiu prednho kolesa v tomto xovanom ase


0 .138 Toto koleso kon tri zkladn pohyby: jeho aisko sa pohybuje rchlosou =
2 , ota sa uhlovou rchlosou
3 (okolo zvislej osi) a
uhlovou rchlosou
1 (okolo -ovej osi). Podmienka je, e koleso nepremykuje na stole, o je ekvivalentn tomu, e rchlos bodu je nulov,
resp.
(0 ) = (0 ) + (
3 ) (
3 ) + (
1 ) (
3 ) = 0 ,
138 Oividne

nezvis explicitne od asu, ale iba od .

135

FX E2

Kolieska

kee vektor z aiska do dotykovho bodu je =


3 . Druh len
je nulov a teda priamo dostvame

2 +
2 = 0 ,

teda

Potom ubovon bod kolesa uren vektorom z aiska sa pohybuje


v ase 0 rchlosou
(0 ) = (0 ) + (
3 ) (0 ) + (
1 ) (0 )
= (0 ) + (
1
3 ) (0 ),
o znamen, e vektor uhlovej rchlosti otania kolesa v ase 0 je
(
)

=
1
3 =
1 3

( )
2

+ 1
=
1
3 .

( )2 + 1
( )2 + 1

Pretan
do udskej rei to znamen, e koleso rotuje uhlovou rchlosou
2
= (
) + 1 okolo osi danej jednotkovm vektorom139

= ( , , ) =


( )2

+1

1
, 0, ( )
2

.
+1

Potrebujem ete uri jednotliv komponenty tenzora zotrvanosti. To by


som mal urobi v nejakej pravouhlej bze pevne spojenej s telesom. V tej
istej bze mm potom uri komponenty jednotkovho vektora osi rotcie
v ase 0 . Nu, mem vyui bzu, ktor je v zakreslen v obrzku vyie.
V ase 0 ju toti naozaj mem chpa ako pevne spojen s kolesom.140
Kee koleso povaujem za rovinn,141 tak zjavne
=

= 0

=1

= 0.

=1

139 Tto os je znzornen v obrzku Spja stred osky (ktor mem chpa ako nehmotn sas sledovanho prednho kolesa) a bod . Oba s v ase 0 nehybn, take
leia na jeho okamitej osi rotcie.
140 Fakt, e bza pevne spojen s kolesom je rotujca a pohybujca sa bza neinercilnej
vzanej sstavy ns netrpi, lebo skmame iba stav vo xovanom ase 0 , v ktorom
potrebujem v danej bze vyjadri komponenty jednotkovho vektora , o u mm dokonca
hotov.
1
141 Pre homognny valec s hrbkou by sme dostali komponenty
2
= 2 ; =
1
= 14 2 + 12
2 ; = = = 0.

136

FX E2

Kolieska

Vaka symetrii kolesa vzhadom na -ov os (rovnako dobre by poslila aj


-ov) dostaneme

=
= 0.
=1

Napokon diagonlne leny bud pre homognny disk


=

(2 + 2 ) =

=1

1
2 ,
2

o je moment zotrvanosti plnho disku vzhadom na jeho stred pre otanie


okolo rotanej osi, resp.
=
=

(2 + 2 ) =

=1

=1

(2 + 2 ) =

=1

=1

2 =

1
2 ,
4

2 =

1
2 ,
4

o s momenty zotrvanosti plnho disku vzhadom na jeho stred pre otanie okolo osi leiacej v rovine disku.142
( )2
Pre viu preadnos rovnc si ozname
+ 1 ako . Potom kine
tick energia jednho kolesa je
1
1
)2
2 + (
2
2
)
1
1( 2
2
= ()2 +
+ 2 ()
2
2[
]
(
)2
(
)2
1
1

1
1
1
= 2 2 +
2 +
2 2 2
2
2

4
(
)
1
1 1
1
= 2 2 +
2 + 2 2
2
2 2
4
(
)
3 2 1 2
= 2
+ .
4
8

Dostvame rieenie (kinetick energia celej sstavy ie oboch kolies)


(
)
3 2 1 2
2
=
+ .
2
4
by by jasn, e plat = 2 = 2 , lebo sumy

2
s zhodn a suma
=1 je nulov.
142 Malo

137

=1

2 a

=1

FX E2

Kolieska

Alternatvne rieenie vyuije vyie dokzan skutonos, e kinetick


energiu tuhho telesa (ale to sa d dokza aj pre ubovon sstavu hmotnch bodov) mem uri ako set translanej kinetickej energie aiska
( 21 2 ) a rotanej kinetickej energie. Nu, a t rotcia je, ako sme u spomnali, rotcia okolo -ovej osi uhlovou rchlosou a sasn rotcia okolo
-ovej osi uhlovou rchlosou . Ozname si vektorom rchlos -teho
bodu telesa vzhadom na aisko. Meme si vimn, e rotcia okolo osi
) a (
) . Rotspsob, e body kolesa maj nenulov zloky rchlost (
cia okolo osi vak vaka nulovej hrbke kolesa d bodom kolesa rchlos
) . Take rchlosti jednotlivch
len v smere osi , ie spsob nenulov (
bodov kolesa za tieto dve rotcie s na seba kolm. To nm umouje
rotan energiu vemi jednoducho rozdeli na dva leny, z ktorch prv len
mus by kinetick energia rotcie okolo osi a druh len kinetick energia
rotcie okolo osi :

1
2
2 =1

]
[
1
)2 + (
)2 + (
)2
(
2 =1

]
[
1
1
)2
)2 +
)2 + (
(
(
2 =1
2 =1

1
1
2 + 2 .
2
2

Tm je hlavn potia tejto lohy prekonan bez vyjadrovania vektoru okamitej osi rotcie, vekosti uhlovej rchlosti a momentu zotrvanosti vzhadom
na os okamitej rotcie. Dopotanie spova len na uren , , a
a dosaden do rovnice
= 2 + 2 + 2 .

138

FX E3 Struna
Tina sa u hra na gitare. Okrem jej podmanivho zvuku ju vak zaujala
aj skutonos, e ke si gitaru nalad v teple domova a potom s ou vyjde von
do chladnej zasneenej noci, gitara u nelad. Aby ste Tine vysvetlili, ako je
to mon, tak
(a) odvote vlnov rovnicu pre strunu;
(b) njdite zvislos zkladnej frekvencie struny v zvislosti od jej predenia ;
(c) zistite, ako sa zmen zkladn frekvencia struny, ak s gitarou zjdeme
do treskcej zimy s teplotou o menou.
Struna m dku = 0,8 m (jej as na kobylke zanedbvame), priemer =
0,6 mm, je zhotoven z ocele s hustotou = 8000 kg/m3 , modulom prunosti
= 220 GPa a tepelnou rozanosou = 11,106 K1 a na zaiatku hr
s frekvenciou 0 .
Zdroj: lohu vymyslel Bzduo

l Vzork Bzduo l
as (a)
Odvodenie vlnovej rovnice struny urobme formlne, aby sa dalo ahko
zopakova pri odvoden vlnovej rovnice v mnohch inch situcich. Budem
pritom zdrazova vetky zanedbania, ktorch sa dopame.
Ozname vchylku struny v jednotlivch miestach ako .143 To bude
bude zvisie od polohy i asu , teda je sprvne psa (, ). Z praktickch dvodov budem vinou asov argument, i dokonca oba argumenty,
vynechva. V svislosti s tmto si pripomeme skrtan oznaenia pre asov a sradnicov derivcie

= ,

v prpade druhch derivci napeme dve iarky, resp. dve bodky.


Nakreslime si prehadn obrzok, aby sme sa mali od oho odrazi.
143 V prpade pozdnych vn by malo vznam sradnice danho bodu na nedeformovanom telese a jeho posunutie z tejto polohy do okamitej polohy.

139

FX E3

Struna

Budeme uvaova len mal uhly , aby sme mohli pouva aproximciu
sin tg . V tom prpade zrejme meme psa () = ().
Teraz pome riei dynamiku lohy. Ak sily psobia na vybran innitezimlny ksok struny? Struna je napnan nejakou silou . T je urite
kontantn pozd struny, ak struna nie je deformovan. Pri vemi malch
deformciach bude zrejme aj zmena pozd struny mal a preto ju zanedbme. Sily ahajce konce vybranho ksku s teda rovnako vek a odliuj
sa len smerom, akm psobia. Vslednica sl vo vertiklnom smere je preto
d

=
=
=

sin( + d) sin
( + d) ()
()d.

Z Newtonovho pohybovho zkona mme d = d , kde d je hmotnos ksku struny d = d a kde =


je jeho zrchlenie vo zvislom
smere. Dosadenm za dostvame
(,
)d =
2 (, )
=
2

(, )d
2 (, )
.
2

To je hadan vlnov rovnica. Ale preo sa tak nazva? Vlnov rovnica


m popisova vlnu, tj. prol pohybujci sa uritou rchlosou . V tomto
prpade m teda vchylka zvisie od asu a sradnice cez argument (
140

FX E3

Struna

).144 Derivovanm ( ) ahko odvodme145

2
2

2
2
=
.
2
=

To znamen, e rovnica, ktorej rieenm je pohybujci sa prol (vlna),


mus spa prve odvoden vzah. Preto sa uveden rovnica vol vlnov,146
hoci m aj mnostvo inch rieen.147
Porovnanm dostvame

=
=
,

kde = / tandardne oznauje mechanick naptie v pascaloch.


as (b)
Na to, aby sa na strune mohla ri vlna, mus by struna napnut. Pnutie
svis s predenm poda Hookovho zkona
= =

kde je zadan Youngov modul prunosti v ahu.


Teraz prejdime k samotnej vlne. Pri zkladnej frekvencii plat = 2.
V prpade kadej vlny s (fzov) rchlos, frekvencia a vlnov dka viazan
vzahom = .148 Dosadenm dostvame

0 =
=
.
2
2

144 Znamienko uruje smer pohybu vlny. Premyslite si, preo voba mnus znamen pohyb
v kladnom smere osi !
145 Mono to derivova naprklad ako zloen funkciu, tj.

( )

( )

=
=

d( ) ( )

d( )

d( ) ( )

.
d( )

146 Takto

pekn vysvetlenie ma nauil docent Martin Moji, za o som mu van.


si mete overi princp superpozcie, tj. e ak 1 () a 2 () s rieenm
istej vlnovej rovnice, tak aj 1 1 ( ) + 2 2 ( ) je rieenm tej istej vlnovej rovnice
pre hocijak 1 , 2 .
148 Predstavme si snusov vlnu. Nech v istom bode struny sa v ist okamih nachdza
vrch kopeka. Vrch alieho kopeka sa sem dostane za as 1/ . Za ten ist as sa prv
kopek stihol posun prve o vlnov dku . Jeho rchlos je teda = / = .
147 ahko

141

FX E3

Struna

Aby sme boli v obraze, njdime predenie potrebn k vyldeniu komornho a, ktor m frekvenciu 0 = 440 Hz. Dostvame
=

2
(20 ) 0, 018.

To znamen, e nami vybrat konkrtna struna je na naladenej gitare natiahnut o 1,8% oproti svojej pvodnej dke.
as (c)
Zanedbvame rozanos gitary, take vlnov dka zkladnej frekvencie
ostva rovnak. Rchlos vlny sa zmen, pretoe sa zmen naptie . Ochladenm sa zrejme zv, preto nov naptie ozname ako + . Ako sa
tm zmen zkladn frekvencia? Postupne upravujeme

1
+
0 + =
2

=
1+
2

)
(
1

1+
2
2

,
= 0
2
kde sme v jednom kroku vyuili, e pre mal plat (1 + ) 1 + .149
V zskanom vzahu by sme radi videli hmatatenejie veliiny, ne aksi
mechanick naptie a jeho zmenu. Mechanick naptie si vyjadrm pomocou
0 . Plat
2
= (20 ) .
A ako sa vlastne pri ochladen zmen mechanick naptie? Aby sme urili
, treba si uvedomi, ako to s tmi predeniami funguje.
Majme strunu, ktor ma pri istej teplote a pri nulovom mechanickom
napt dku 0 . Ak ju schladme150 o , skrti sa na (1 )0 . Ak

)0 . Ak predpokladme, e
ju naahujeme naptm , predli sa na (1 +
koecienty a s pre mal natiahnutia a mal zmeny teploty kontantn,
149 Priamoiarej

postup cez derivcie je




0
=
=
.
0
2

150 Schladme.

142

FX E3

Struna

tak meme psa


=

(
(1 ) 1 +
(1 +

)0 ,

teda pri oboch efektoch sasne plat pre predenie


=

Gitara, ktor predstavuje aksi neroztiahnuten mdium, ponech celkov dku struny po ochladen rovnak, ako pred ochladenm. Predenie
sa teda ochladenm nezmen. Ak ma zaujma zmena mechanickho naptia,
dostvam rovnos

= .
=

Po dosaden a do vzahu pre dostvame po prave

=
.

80 2
V slade so sksenosou gitaristov dostvame vsledok, e najviac sa zmen
frekvencia pri malch hodnotch 0 , tzn. pri hlbokch tnoch.
Poznmka 1: Rchlos renia vlny mono vdy vyjadri v zvislosti od
loklnych parametrov (v prpade prienych vn struny to bolo mechanick
naptie a hustota). Loklnych parametrov vinou nie je vea, preto mono
rchlos renia vlny a na seln kontantu vemi rchlo odvodi z rozmerovej analzy. seln kontanta je v drvivej vine prpadov rovn 1.
Vnimkou je renie zvuku v plynoch, kde sa rovn odmocnine z Poissonovej
kontanty .
Poznmka 2: Je vemi zaujmav, e dva sasne znejce tny znej
isto, ak pomer ich frekvenci je zlomok malch celch sel. Ladenie nstrojov v naich koninch je zaloen
na tom, e pomer frekvenci poltnu, tj.
12
dvoch najblich
klves,
je
1
:
2. Rozdiel poltnov zodpoved pomeru
( )
frekvenci 1 : 12 2 . Jedna oktva (12 poltnov) zodpoved pomeru frekvenci 1 : 2. Kvarte (5 poltnov) a kvitne (7 poltnov) zodpovedaj pribline
pomery frekvenci 3 : 4, resp. 3 : 2. To mono bra ako vysvetlenie, preo
znej istejie ne ostatn intervaly.

Zostame vak pri sle 12 2. S jeho pomocou vieme zisti, o ak poet


poltnov sa vlastne gitare pri ochladen rozladila. Ak predpokladme mal
143

FX E3

Struna

zmenu frekvencie, tak mme

(
)
0 +
log 122
0
(
)

ln 1 + 0

ln 12 2
12
0 ln 2
3
.
202 2 ln 2

Ak si vezmeme frekvenciu najniej (82 Hz) a najvyej (330 Hz) struny na


gitare, ahko dopotame prslun rozladenie 0,15, resp. 0,0094 poltnu na

C. V literatre151 mono njs, e citlivos udskho ucha je asi 0,05 poltnu. Chce to ale dobre nastraen ui. Najhrubia struna sa rozlad o cel
poltn pri ochladen o 7 C.

151 Naprklad

http://en.wikipedia.org/wiki/Pitch_(music).

144

FX E4 ab
Baa si na zhrade postavila dlh ab na odvod vody. Jeho rka je a
sklon . Ke sa raz rozpralo, v abe tiekla voda prdom o vke (meranej
kolmo na dno abu). Ak bol prietok vody abom, ak jej prdenie povaujeme
za laminrne? Mete predpoklada .
Zdroj: lohu vymyslel Tom

l Vzork Marcelka l
Najprv si povieme nieo o viskozite. Ak kvapalina nem vo vetkch
miestach rovnak rchlos, vznikaj v nej trecie sily kvapalina sa brni
pohybu. Zo sksenosti vieme, e niektor kvapaliny sa brnia pohybu viac,
in menej: naprklad med teie vemi pomaly, ale voda teie rchlo. Viskozita je vlastnos kvapaliny, ktor hovor, ak vek s vntorn trecie sily
v pohybujcej sa kvapaline. Preto veta med teie pomaly sa d nahradi
slovami med m vek viskozitu .
Predstavme si nasledujci experiment: zoberieme si dosku s plochou ,
polome ju na vodu s hbkou a zaneme ju aha tak, aby doska mala
rchlos . Meme si odmera silu , ktor je potrebn na takto ahanie.
Je oakvaten, e m vou rchlosou chceme dosku aha, tm viu
silu na to budeme potrebova. Naproti tomu, m tenia je vrstvika vody,
tm to pjde aie (aha nejak platu v bazne je ahie ako aha ju po
milimetrovej vrstvike vody). Potrebn sila tie zvis od plochy dosky m
via doska, tm via sila.

v
F
h
Zoberme ako experimentlny fakt, e sila zvis od danch velin ako

= ,

o shlas s intuciou. V tomto vzahu sa okrem spomnanch velin objavilo


psmenko . To je kontanta, ktor volme dynamick viskozita kvapaliny.152
152 Pozor, existuje aj kontanta nazvan kinematick viskozita. Medzi oboma viskozitami
je jednoduch vzah, ale netreba si ich poplies, lebo to me spsobi rdovo zl vsledky.

145

FX E4

ab

Kad kvapalina ju m in a d sa njs v tabukch. (Aby to nebolo a tak


jednoduch, nie vetky kvapaliny sa sprvaj takto. S aj tak, kde viskozita nie je kontanta, ale funkcia rchlosti kvapaliny. Voda vak, naastie,
uveden vzah spa.)
Niekedy je rozumnejie hovori o sile na plochu, teda o akomsi napt.
Toto naptie sa vol mykov naptie a oznauje sa . Plat pre vzah
=

= .

Predstavme si teraz, e ns zaujma rchlos kvapaliny na rznych miestach v kvapaline. (Stle sa rozprvame o ahan dosky, nie o naom abe.
Berme to ako jednoduch motivan prklad, ktor nm pome zoznmi
sa s viskozitou a lepie si ohmata, ako to funguje .) Predpokladajme, e
doska je ovea via ako vka vody, a e prdenie je ustlen. Zo symetrie153 problmu vyplva, e vetka kvapalina bude ma rchlos rovnoben
s rchlosou dosky a e rchlos kvapaliny zvis len od z-ovej sradnice
(zvislej, kolmej na dosku). Zaujma ns teda funkcia ().
Zamerajme sa na nejak vrstvu vody vo vke . Rchlos tejto vrstvy
ozname (). o keby na mieste tejto vrstvy bola tenk plata ahan
rchlosou ()? Ni, voda by si to nevimla a sprvala by sa rovnako ako
predtm. V vahch teda meme nahradi vrstvu vody platou.

F
h-z
F

F
z

F
Zoberme si teraz len mal valek vody s podstavou . Na jeho horn
podstavu psob sila = (), no a kee je prdenie ustlen, celkov
sila psobiaca na valek (aj vo vodorovnom smere) bude nulov. Preto na
jeho spodn podstavu psob sila . Ak si vak valek vo vke rozdelme
na dve asti (naprklad tenkou platou), meme pre obe asti zopakova
rovnak vahu. Vslednica sl psobiacich na doln as valeka mus by
153 O symetrich a ich vyuit, naprklad i o Ntherovej vete sa mete dozvedie viac
naprklad v Prruke mladch fyzikov.

146

FX E4

ab

nulov, ie na horn as (resp. plata) psob silou , a podobne doln


as psob na horn opanou silou .
Toto ale znamen, e voda v spodnom valeku sa sprva tak, akoby sme
vo vke ahali platu silou . Z naich vah o viskozite potom ale vyplva,
e () = = (), ie () = () a teda

()
()
=
.

Inmi slovami, funkcia () je linerna, konkrtne


() =

()
.

Sksme ete nahliadnu, ako vyzer diferencilny tvar rovnice = /.


Predstavme si dve vrstvy vody, jednu vo vke , druh vo vke + d.
Rchlos vrchnej vrstvy vzhadom na spodn je ( +d)(). Predstavme
si, e vrchn vrstva je plata pohybujca sa rchlosou ( +d). Sila, ktorou
by sme takto platu museli aha, je
( + d) ()
,
d
o v limite d 0 dva derivciu rchlosti poda vky. Odtia mme vzah
pre mykov naptie vo vke ;
=

d
.
d
Dostali sme vzah pre mykov naptie v najjednoduchom monom prpade:
ke je prdenie ustlen a rchlos vody rovnoben vo vetkch miestach.
Ke u poznme diferencilny tvar rovnice pre mykov naptie, mme
sa ete inm spsobom pozrie na rchlos kvapaliny, na ktorej je poloen
doska ahan rchlosou (). Op si pozrime nejak vrstvu vo vke ,
ktor m hrbku d. Na vrstvu psobia vo vodorovnom smere len mykov
sily jedna smerom dopredu (od vrchnch vrstiev), jedna smerom dozadu.
Kee predpokladme ustlen stav, tieto dve sily musia ma rovnak vekos. To znamen, e () = ( +d), ie mykov naptie je vade rovnak
a nezvisl od zvislej sradnice. o z toho vyplva? Derivcia rchlosti poda
vky je kontanta:
d
= ,
d
take funkcia () mus by tvaru () = + . Navye vieme, e voda
v nulovej vke m nulov rchlos (z oho = 0) a e voda vo vke platne
m rchlos platne: () = + . Z toho dostvame u znmy vzah
=

() =

()
.

147

FX E4

ab

Po obohaten sa o poznatky o viskozite sa meme pusti do rieenia


samotnho prkladu. V zadan je napsan, e prdenie je laminrne. To
znamen, e je hladk jednotliv vrstvy vody maj rovnoben rchlos
a nepremieavaj sa. Mme si teda dovoli predpoklada, e rchlos vody je
rovnoben s rovinou abu. Naviac, op je zo symetrie zrejm, e rchlos
vody bude ma len jeden smer (len nadol a nie do bokov). Tie meme
predpoklada, e na podstatnej asti abu bude prdenie ustlen, a kee
<< , meme zanedba javy na okrajoch abu. Op ns teda zaujma
len zvislos (), priom je v tomto prpade sradnica kolm na rovinu
abu.

F(z+dz)
FG

F(z)

dz
z

Zamerajme sa znova na tenuk vrstvu vody vo vke , ktor m plochu


a hrbku d. Vo vke bude mykov naptie
() =

d
()
d

a vo vke + d bude
( + d) =

d
( + d).
d

Voda nad vrstvikou bude strhva vrstviku smerom dopredu, voda pod
vrstvikou zase smerom dozadu. (Je intuitvne jasn, e bliie k podloke
m voda meniu rchlos.) Vslednica mykovch sl psobiacich na vrstvu
teda bude teda
(
)
d
d
= ( ( + d) ()) =
( + d)
() .
d
d
Vimnime si, e ak poleme d 0, potom
d
d (

+ d)
d

d
d ()

148

d2
(),
d 2

FX E4

ab

o je druh derivcia rchlosti poda . Odtia mme vzah pre vslednicu


mykovch sl psobiacich na vrstvu vody vo vke :
= d

d2
().
d 2

Zrove na vrstviku psob okrem mykovch sl zloka gravitanej sily


rovnoben so abom. Tto m vekos = d sin . Kee prdenie
je ustlen, vslednica vetkch sl psobiacich na vrstviku mus by nula,
ie (uvaujc len sily v smere rovnobenom so abom)
+ = d sin + d

d2
() = 0.
d 2

Po jednoduchej prave dostvame diferencilnu rovnicu


d2
sin
.
=
d 2

Rieenm tejto rovnice je funkcia, ktorej druh derivcia je kontanta. Tto


podmienku spa polynm druhho stupa, preto
() = 2 + + ,
kde
=

sin
.
2

U len potrebujeme zisti vekos kontnt a . Na to pouijeme okrajov podmienky: vieme, e voda vo vke = 0 m nulov rchlos a mykov
trenie vo vke = je nulov (vzduch vodu nebrzd). Kee
(0) =

() =

d
() = (2 + ),
d

dostvame = 0 a = 2.
Zadanie sa pta, ak je prietok vody abom. Ke u poznme funkciu
(), je to jednoduch:

() d =

[
]
3
( 2 2) d =
2
3
0

2
sin 3
3 = 3 =
.
3
3
3
3

149

FX E5 Rutherford
Pn Rutherford znova vytiahol zo pajze zlat fliu a delo astc. Vystrelil asticu rchlosou smerom na jadro zlata, a ona sa mu odchlila
o uhol (od pvodnho smeru letu). Zistite, o koko pn Rutherford netral toto jadro zlata! (t.j. ako aleko od pvodnej priamky letu sa toto jadro
nachdzalo)
Zdroj: tandardn uebnice fyziky

l Vzork Kubus l
Takto prklad meme zrta bu nejakou ntou a osvietenm nadhadom, alebo pomocou hutnho apartu. No, preo nie? Pome si ho vybudova.154
Polrne sradnice
Centrlny pohyb155 sa vemi dobre bude popisova polrnymi sradnicami. Ak sme doteraz polohu (polohov vektor) nejakej astice zaznamenvali ako =
+
,156 teraz si budeme radej psa jej vzdialenos od
stredu a uhol , ktor zviera vektor od nejakej referennej priamky, napr.
od osi . Kladn uhol budeme poda zvyklosti pota proti smeru hodinovch ruiiek. Nae jednotkov vektory bud teraz a . Ukazuj v smere
narastajceho resp. , ako to znzoruje obrzok:

154 Upozornenie

155 Tj.

hne na zaiatok: vektory budeme znai tunm psmom, tj. namiesto

pohyb s centrlnou silou, alebo vlastne hocio, o vyzer dostatone centrlne


symboly a nie s nim inm ako jednotkovmi vektormi v smere os a

156 Divn

150

Rutherford

FX E5

Vidme, e pri tomto znaen =


. o vak s a , ktor potrebujeme
na popsanie pohybu? Pri kartzskych sradniciach sme boli zvyknut, e
nae jednotkov vektory a boli kontantn, preto bolo rovn
+ .
Teraz sa nm vak menia aj nae a , konkrtne sa spolu s otaj
okolo poiatku. Vyjadrime si ich teda pomocou a :

cos + sin
= sin + cos

(41)
(42)

Ak toto poderivujeme poda asu, dostvame


d
(
cos + sin )
d
= ( sin ) + (cos )

(43)

( cos ) + ( sin )
= .

(44)

a analogicky

aa, ak pekn vsledok.157 Teraz sa meme pusti do zisovania, o


bude vlastne a , pouvajc (43) a (44).
=

=
+ =
+

(45)

To je intuitvny vsledok rchlos je nielen zmena vzdialenosti od


Deristredu, ale m aj kolm komponent v smere , obvodov rchlos .
vujme vak alej.
)
d (

+
d
d ( )
= + +
+
d
+
+
2
= +
(
)
(
)
= 2 + 2 +
(46)
Ak to ndhern vraz pre druh derivciu polohovho vektora, krsne
rozporciovan do vrazov a ! Vimnime si hne vraz druh derivciu vzdialenosti a 2 dostrediv zrchlenie pri . Takisto si meme
d
158
vimn, e vraz pri je rovn 1 d
(2 ).
celkom intuitvny, ak si otajce sa vektory a nakreslme.
Ak moment hybnosti? Kontantn? Nulov zrchlenie v radilnom smere?

157 Navye
158 o?

151

Rutherford

FX E5

Apart vybudovan, meme sa pusti do rieenia. Kee jadro zlata je


ak a pomerne dobre zait v mrieke, meme predpoklada, e sa nebude pohybova. Ak si do teraz polome stred naej sradnicovej sstavy,
bude sila psobiaca na alfa asticu rovn = /2 , kde je nejak kontanta. No a kee druh Newtonov zkon nm hovor =
, dostvame
pohybov rovnicu:

(
)

2
=

2
(

d ( 2 )

d

2
=
=

(47)
(48)

kde je nejak kontanta.159


Takto sme sa dostali ku pohybovm rovniciam pre a v (prefkanej)
zvislosti od asu. Riei tieto diferencilne rovnice nie je najjednoduchie,
a navye, my sme chceli vedie iba tvar trajektrie. Ak teda odstrnime as
z tchto rovnc, budeme ma u len zvislos a . Nebude to ani tak vemi
bolie, pretoe v = /2 mme uiton vzbu. Take meme psa:
d
d d
d
=
=
d
d d
d 2
Toto nie je vnimone pekn vraz, tak si dovome ete jednu ntu
ak urobme substitciu = 1/ (ktor by sme asi museli urobi neskr pri
rieen nejakej karedej diferencilnej rovnice), uhlad sa nm to na
=

=
a potom

d 1
d
d
=

=
d 2
d
d

d
d
=
=
=
d
d

d2
2
d

2 2d
=

2
d2

(49)

(50)

Bsnik by povedal, pome spolu lieta: (inmi slovami, napme si (47)


a (48) a aplikujeme (50))
(
)
2
4 = /2

2
d

2 2 2 2 3 = 2
d

d2

+ =
=
(51)
2
d
2
159 Ako

to bolo s tm zkonom zachovania momentu hybnosti?

152

Rutherford

FX E5

Mil diferencilna rovnica, sce nehomognna, ale bez kontanty na pravej


strane je to stle len rovnica pre harmonick kmity. Jednm z PV160 rieen
je = a preto veobecnm rieenm bude () = cos + sin + .
Kontanty a urme z poiatonch podmienok.
Nastavme si sradnicov sstavu tak, e smer ku Rutherfordovmu delu
bude = 0, take (0) = , teda (0) = 0. Z toho po dosaden do rovnice
pre () dostvame = .
Takisto, po chvke zamyslenia = 2 = ,161 kde sme ako oznaili
hadan vzdialenos. Vidme tie, e (0)

= . Z rovnice (49) mme d


d =
/,

take
d
(0)

1
(0) =
=
= .
d

Z toho po dosaden do d
(0)
dostaneme

=
1/.
d
Tak sa napokon dostvame k rovnici drhy alfa astice:
() = (1 cos ) +

sin

(52)

a hadme druh rieenie () = 0 (rzne od = 0)162


sin
(1 cos ) +

(
)

= 1 cos2 + sin2
+ 2 sin cos
2
2
2
2

= 2 sin + 2 cos ,
2
2

0 =

teda

1
=
.
2

Ak toto je , pre ktor 0, teda , potom odklon od pvodnho


smeru letu bude = , teda /2 = /2/2 a preto tg(/2) = cotg(/2).
Take

tg = =
=
=
(53)
2
2
2 2
2
U si len spomeme, e bola kontanta mernosti v odpudivej sile.
1 2
2
= 4

2 , preto = ( )/(20 ) (kde je atmov slo zlata, t.j.


0
79) a teda konene

2
tg =
(54)
2
20 2
tg

160 Pozriem

Vidm.
sa zachovva a v kadom ase je teda rovnak ako na zaiatku
162 V prvej prave vyuvame vzahy cos(2) = cos2 sin2 a sin(2) = 2 cos sin ,
v druhej e hadme rieenie, pre ktor sin = 0.
161

153

Rutherford

FX E5
alebo
=

cotg
2
20
2

(55)

Tak. A mme vsledok. Tmto som nechcel nikomu nahovori, e na vypotanie tohto prkladu potrebujeme takto kladivo. S ctou si vak vimnime, e podobne vieme odvodi i tvar drhy alfa astice, tvar trajektri
plant (a s trochou dodatonej snahy vieme vtlai nasp aj as), a mme
pevn zklady pre hocio radilne. Prklad sa vak neptal na vetky tieto
detaily, preto sme si mohli dovoli viacer skratky a nty.
Osvieten nadhad
Polrne sradnice s pri takchto aplikcich celkom sympatick, pookrejme pri nich ete chvku. Nebudeme si vak vetko matematicky budova od zaiatku, my predsa vieme, e plat zkon zachovania momentu
hybnosti!163 Takisto meme vyui i dobr vlastnosti z naej kartzskej sstavy. Ak si nastavme sstavu tak, e alfa astica prichdza z uhla = ,
najbliie bude ke = 0 a (zo symetrie) odde do nekonena nechvajc
, bude sa nm hne lepie pota!

Pozrime sa teraz, ako sa men -ov zloka rchlosti.

163

d2

= 2 cos
2
d

d2

=
cos
d2
2

= = 2 = kont.

154

(56)

Rutherford

FX E5
Aha, kontantn moment hybnosti v menovateli!
kojne integrova poda asu:

d2

d
=
cos d
d2

=
sin
d

164

Teraz meme spo-

(57)

Vyuili sme, e integran kontanta je nula. Sta si vimn situciu


= 0.) Tto rovnica by nm bola pomerne nani, ale majme na pamti,
o o sa vlastne sname. My chceme zisti len hranin uhol , resp. jeho
vzah s . Pozrime sa na rovnicu (57), ke = . Kee vieme, e moment
hybnosti = , tak:

sin

=
tg
2

cos =

(58)

Teraz si u len uvedomme, e vlastne = 2 a teda tg() =


cotg(/2), a kee = (2 )/(20 ):
=

cotg
20 2
2

No a o ak nechceme robi velijak mgiu s polrnymi sradnicami a tri


vnoren nty? Mme ete jednu monos spomenieme si, e drha takejto
astice by mala by kueoseka, a to konkrtne hyperbola, pohraba sa trochu v jej geometrickch vlastnostiach a potom sa utopi v mori psmeniek,
rovnc a vzorcov.
Geometria hyperboly
Vieme, e hyperbola je mnoina bodov, ktor maj rovnak rozdiel vzdialenost od dvoch pevnch ohnsk, a jadro zlata bude jednm z nich. Z tohoto
vieme odvodi uiton vlastnosti.
164 No, nie je to tak prevratn zistenie, to sme predsa spravili naschvl, aby sa nm
vyhubilo 2 .

155

Rutherford

FX E5

Naprklad, tieto body spaj rovnicu 2 /2 2 / 2 = 1 pre nejak


a . Vzdialenos vrcholov mus by 2.165 Uhol, ktor zvieraj asymptoty
s osou je = arctg(/).166 Vzdialenos ohnsk od asymptot .167 alej poda pravho obrzku 2/2 = tg().168 Porovnanm ostatnch dvoch
rovnc dostvame, e = .
Ak ete ozname ako vzdialenos poda pravho obrzka, dostvame
z naznaenho trojuholnka Pytagorovou vetou ( + )2 = 2 + 2 . Suma
sumrum mme dve rovnice:

( + )2

tg

(59)

= 2 + 2

(60)

Stle vak potrebujeme ete jednu rovnicu, aby sme spojili tieto geometrick vahy s fyzikou, o sa tam naozaj deje.169 Vimnime si, e bod najbliieho priblenia alfa astice ku jadru bude vo vrchole hyperboly, tto
minimlnu vzdialenos ozname . Mme teda = + 2. Vzdialenos
hada rieenie hyperboly pre = 0.
asymptot dostaneme, ak v rovnici hyperboly nahradme jednotku na pravej
strane nulou.
167 Poda zadania. Jadro prichdzajce z nekonena prichdza akoby po asymptote.
168 Ke sa alfa astica z nekonena pribliuje k jadru zlata, vid obe ohnisk prakticky
v tom istom smere. Vieme, e rozdiel ich vzdialenosti od astice je 2. Preto je dka 2
odvesnou naznaenho pravouhlho trojuholnka.
169 Navye mme na dve rovnice akosi privea neznmych.
165 Sta

166 Rovnice

156

Rutherford

FX E5

vak vieme vyjadri zo zkonov zachovania momentu hybnosti a energie!

1
2
2
Dosadme z (61) do (62):

=
1

=
2 + .
2

(61)
(62)

1
1
2

2 = 2 2 +
2
2

2 2 2 2 2 = 0

2 + 4 2 + 42 4 2
=
2
2
2
2 + = = + + 2

(63)

kde = / . Roznsobenm (60) a odtanm dostvame


2

2 + 2 = 2 .
Vimnime si, e av strana tejto rovnice je presne -nsobkom avej
strany rovnice (63). Ich vzjomnm vydelenm dostvame
=

.
+ 2 + 2

Potom znova z rovnice (63) dostvame:

2 = + 2 + 2
(
)

=
+ 2 + 2
+ 2 + 2

(
)2 (
)
(
)
+ 2 + 2
2 + 2 2 2 + 2

(
)(
)
=
+ 2 + 2 2 + 2

(
)
(
)
2 + 2 + 2 2 2 + 2
=
2
= 2.
Toto dosadme do (59), spomenieme si na tg() = cotg(/2) a potom u
iba rozbaujeme psmenk, dostvajc sa k vytenmu vsledku.
= tg
= cotg(/2)

=
cotg(/2)
2
2

=
cotg .
2
20
2
157

FX E6 Pinkaka
Jano chce porazi Jura v squashi a tak poctivo trnuje. Minule si naprklad zohnal loptiku s hmotnosou a katuu tvaru kvdra s hmotnosou
. Potom kopol do katule tak, aby sa mkala po zemi rchlosou
smerom kolmo na stenu, a do jej drhy poloil vo vzdialenosti od steny
nehybn loptiku. Vypotajte do akej najmenej vzdialenosti od steny sa
krabica dostane. Trenie katule aj loptiky o zem povaujte za nulov, vetky
zrky za dokonale prun a predpokladajte, e krabica sa neota (cel pohyb loptiky sa deje na jednej priamke kolmej na stenu). Odpove sta do

.
prvho rdu v
Zdroj: BAUPC

l Vzork Kubus l
Ozname si rchlos krabice po -tej zrke s loptikou ako a rchlos
loptiky po tejto zrke ako , priom kladn smer rchlosti volme smerom
ku stene. Medzi zrkami sa rchlos krabice nemen, rchlos loptiky sa
prunm odrazom od steny zmen na opan. Pred ( + 1)-vou zrkou sa
teda krabica a loptika pohybuj rchlosami a . Logicky si teda
ozname rchlos krabice pred prvou zrkou ako 0 = a rchlos loptiky
ako 0 = 0 = 0, aby sme mohli s prvou zrkou pota symbolicky rovnako
ako s ostatnmi.
Pozrime sa na teraz na ( + 1)-v zrku. Je to jednoduch prun jednorozmern zrka dvoch telies so veobecnmi hmotnosami a a veobecnmi rchlosami a . Mohli by sme si napsa zkon zachovania
hybnosti a zkon zachovania energie a vytrias z nich rchlosti +1 a +1 ,
my vak pouijeme mal ntu na zjednoduenie potania. Pozrime sa na
cel zrku v sstave spojenej so spolonm aiskom krabice a loptiky.
Pretoe na krabicu a loptiku poas zrky nepsobia iadne vonkajie sily,
ich aisko nebude zrchova, ie sstava spojen s nm mus by inercilna. Take ZZH a ZZE v nej stle musia plati. Navye celkov hybnos
je v tejto sstave nulov, o by nm rieenie rovnc trochu zjednoduilo
ale najlepie je na tom to, e my ich vbec riei nemusme. Vieme toti, e
rovnice pre ZZH a ZZE maj po zrke jednoznan rieenie,170 sta teda,
ke jedno tak rieenie tipneme. A to vbec nie je tak ak. Ak v takejto
aiskovej sstave zmenia obe teles rchlos na opan, celkov hybnos sa
170 Nie je to plne pravda, rieenia pre rchlosti loptiiek vyjd dve. Jedno z nich bud
pvodn rchlosti, druh bud rchlosti po zrke.

158

FX E6

Pinkaka

tie zmen na opan, ale kee je nulov, tak sa vlastne vbec nezmen.
Kinetick energia sa tie zrejme nezmen, kee zvis iba od druhch mocnn rchlost. To znamen, e takto jednoduch odraz v aiskovej sstave
spa ZZH aj ZZE, a teda sprvne popisuje zrku dvoch telies.
Predtm ako sa bezhlavo vrhneme do psania rovnc a vyjadrovania rchlost po zrke, zabudnime na chvu na aiskov sstavy a pome sa pozrie
na to, o to vlastne potrebujeme. Chceli sme porta vzdialenos krabice od
steny, ke k sebe bud najbliie. Tento okamih zrejme nastane pri nejakej zrke,171 pozrime sa teda na vzdialenos krabice od steny pri -tej
zrke s loptikou. Po predolej, ( 1)-vej, zrke m krabica rchlos 1
a mus prejs vzdialenos 1 ku miestu -tej zrky. Loptika m
rchlos vekosti 1 a mus prejs vzdialenos 1 + , kee sa medzitm ete odraz od steny. Obom to mus samozrejme trva rovnak as,
dostvame teda rovnicu
1 +
1
=
,
1
1
ktorej rieenm je
= 1

1 1
.
1 + 1

Ak pouijeme tento vzah viackrt, dostvame


)(
) (
)(
)
(
1 1
2 2
2 2
1 1
...
.
= 1
1 + 1
2 + 2
2 + 2
1 + 1
A teraz pozor! o je to 1 +1 ? Je to rchlos pribliovania sa loptiky ku
krabici tesne pred druhou zrkou, lebo krabica m po prvej zrke rchlos
1 smerom ku stene a loptika po prvej zrke a odraze od steny rchlos
1 smerom od steny. A o je 2 2 ? No predsa rchlos vzaovania sa
loptiky od krabice tesne po druhej zrke. A o je na tom tak skvel?
No predsa to, e v aiskovej sstave krabica aj loptika zmenia rchlosti
na opan, take ich vzjomn rchlos sa tie zmen na opan. Rchlos
pribliovania sa pred zrkou je rovnak ako rchlos vzaovania sa po
zrke, ie 1 + 1 = 2 2 .
Podobnou vahou pre -t zrku dostaneme rovnos 1 + 1 =
, pre ubovon . Teraz sa u len pozerajme, ako sa zo susednch
zlomkov v naej rovnici pre vymltia menovatele s itatemi, dostvajc
= 1

1 1
0 + 0
=
.
1 + 1

171 Inak sa krabica prve pohybuje ku stene alebo od steny, a v takom okamihu neme
by najbliie. Alebo stoj, ale vtedy je od steny rovnako aleko ako pri najbliej zrke.

159

FX E6

Pinkaka

Dosadenm poiatonch rchlost 0 = a 0 = 0 dostaneme


=

Intuitvne vidme, e pri takejto pinkake bude krabica pri zrkach


postupne spomaova a loptika zrchova. Ak s poiaton podmienky
sprvne nastaven, krabica pri jednej zo zrok zastane a loptika bude nies
cel kinetick energiu sstavy, ie budeme ma = 0 a 21 2 = 21 2 ,

odkia = /. Potom by sme dostali

a to by bola aj najbliia vzdialenos krabice od steny, kee krabica by


medzi -tou a + 1-vou zrkou stla.
Ak krabica po iadnej zo zrok nezastane, nemusme zfa. Ak je loptika vemi ahk ( ), pri kadej zo zrok odovzd krabici len mlo
hybnosti a rchlos krabice po zrke najbliej ku stene (tj. tej, ke rchlos
krabice zmen znamienko) bude len vemi mal.
Meme urobi aj nejak konkrtnejie odhady.
Z energetickch dvodov
bude rchlos loptiky ohranien hodnotou

/, pri kadej zrke sa

teda jej hybnos zmen najviac o 2 . Najviac o toko sa pri kadej


zrke zmen
aj hybnos krabice, o pri jej hmotnosti znamen zmenu rchlosti o 2 / . Ak sa rchlos krabice men nanajv takmito malmi
krokmi, a ak je pred najbliou (-tou)
zrkou jej rchlos kladn a po nej
zporn, mus
nutne
plati

2
/ . Asymptoticky meme napsa

= ( / ) ke .172 Zo ZZE potom mme


2
1
2

+ 12 2 = 12 2 ,

ie
=

2 2 =
1

(
)
=

1 + ( ) .

172 Na znaenie asymptotickch odhadov vekosti funkci sa pouva -notcia. V rznych


textoch sa pouva i zneuva vemi vone, jej presn dencie si njdite v literatre alebo
na internete. V tomto texte bude ( ()) znamena nejak funkcia , ktor pre dostatone mal nepresahuje nejak kontantn nsobok () . Menej formlne, naprklad

pod (
) si mete predstavi dao, o je najviac rdovo
.

160

FX E6

Pinkaka

V naom vzahu pre to znamen

= (


1 + ( ) ( / )

1
=

1 + ( ) (
)

(
)

1 + (
=
) .

Vidme teda, e naa odpove / je presn v dominantnom rde,

chyba je rdovo a
-krt menia ako skuton odpove. Takto presn
odhad chyby som od vs v rieen neiadal, sasti aj preto, e v zadan sa
o presnosti do prvho rdu hovorilo trochu zahmlene.
To by ale nebol vzork FX, keby skonil takto narchlo. Pome si sprvanie loptiky spota trochu detailnejie. Vrme sa nasp k potaniu
( + 1)-vej zrky a zapme rozprvky o aiskovej sstave poriadne do rovnc. Rchlos spolonho aiska loptiky a krabice je
=


,
+

rchlosti krabice a loptiky v aiskovej sstave bud pred zrkou


=

a po zrke len zmenia znamienko na


=

= + .

Rchlosti po zrke v sstave spojenej so zemou bud teda


+1 = + = 2

+1 = + = 2 + .

Po dosaden za dostvame
+1 =
+1 =


+
2
+

2
+ ,

+ .

(Vimnite si, e rovnice s symetrick, tak, ako by sme mohli oakva. Hoci
to tak na prv pohad nevyzer, vymenenm s a s sa ni
nezmen.)
Vyjadrili sme si teda +1 a +1 ako linernu kombinciu a .
Takto vzah sa d jednoducho a prehadne zapsa pomocou vektorov a
matc:
(
) (
)( )
2
+1

+
+
=
.

2
+1

+
+
161

FX E6

Pinkaka

o je vak na tomto zpise najlepie, ahko vieme napsa veobecn vzah


pre rchlosti a , a to konkrtne ako
( ) (
) ( )
2

+
+
=
.
2

0
+
+
Ak si nau maticu a vektor ozname ako
)
(
2

+
+
=
a
2

+

( )
0
=
,
0

sta nm u len vypota . Toto je celkom veobecn problm z linernej algebry, a takmer vdy sa d riei naprklad aj takto. o ak by sme
by bol len -nsobkom pre nejak slo ?
nali tak mil vektor , e
Potom by sme celkom jednoducho dostali aj = , pretoe nsobenie
slom () meme vya spred nsobenia maticou. Ak by sme takchto milch vektorov nali viac, hoc by mal aj kad svoju vlastn , tak ich
linerne kombincie by sme tie vedeli jednoducho nsobi maticou :

) = .
( ) = ( ) = (
Sta nm teda napsa nae ako kombinciu takch -iek a sme hotov.
V naom konkrtnom prpade sa ukazuje, e to naozaj ide. Njdime
=
,
najprv nejak vhodn -k. Ak si napeme (vektorov) rovnos
dostvame dve rovnice o troch neznmych (dve komponenty a slo ).
Jedna mon dvojica rieen je naprklad
(
)


2
1,2 =
a

1,2
+
+ .

Skste si vyriei sami, alebo jednoducho len skontrolova, e tieto a


= . No a kee
naozaj spaj
( ) ( )
0

1 + 2 ),
=
=
= 2 (
0
0
dostvame
ie

= 2 ( 1 + 2 ) = 2 (1 1 + 2 2 ),
(

=
2

1 + 2

(1 2 )

Aby sme vedeli


krajie vyjadri mocniny , vimnime si, e plat rovnos
( )2 + (2 )2 = ( + )2 , take ak si v pravouhlom trojuholnku
162

FX E6

Pinkaka

so stranami ( ), 2 a ( + ) ozname uhol pri odvesne dky


( ) ako , potom
1,2 =

2
+

= cos sin = .

Odtia
1 + 2 = + = 2 cos()
1 2 = = 2 sin(),
o meme dosadi do vzahu pre rchlosti a :
= cos()

= sin()
.

Toto s vzahy pre rchlosti krabice a loptiky po -tej zrke. V neakane peknej a elegantnej forme. Pre rieenie pvodnej lohy ns zaujma
tak , ke je prvkrt zporn (resp. nekladn). To bude vtedy, ke
bude argument kosnusu prvkrt v ne 2 , konkrtne pre

.
2
4
2 arcsin 2
+

Z prvej asti vzorku vieme, e vzdialenos -tho odrazu


od steny bude

.
Vtedy
sme
odvodili,
e
je
to
pribline

/
, teraz pre u
=

mme po dosaden za a presn vzah


=

,
sin()

cos()

a = arccos
. Pre bude 1, preto bude
kde = 2
+

blzko /2, ie naozaj / .


Tento presn vsledok nie je ktovieako pekn a uiton, ale postup, ktorm sme sa k nemu dostali, obsahuje vea hodnotnch mylienok. Ak mte
ete chu, skste si nakresli, ako sa menia rchlosti a poda vyieuvedench vzahov. Ak sa vm ako trvili vetky tie algebraick manipulcie
s maticami a vektormi, skste si vetky tie vahy zopakova s konkrtnymi
objektami, ktor dan matice a vektory reprezentuj. (Vektory len schovvaj dvojicu rchlost loptiiek, matica je len opertor, ktor vezme
rchlosti pred zrkou a vypuje rchlosti po zrke.) Napokon, ak sa vm
to cel zd nejak podozriv, tie sa mete zamyslie nad tm, kde vade sa
nm pokazia nae rovnice po tom, ako sa loptika od krabice odraz poslednkrt. (Vetky tieto problmy sme vo vzorku zamlali, lebo ns zaujmala
len zrka najbliia ku stene, a dovtedy sa to naozaj nepokaz.)

163

FX E7 Via ferrata

Mme oceov lano dky = 20 m, prierezu = 1 cm2 , s medzou pevnosti = 1 GPa, hustotou = 8000 kg.m3 a modulom prunosti =
220 GPa. M sli na zaistenej ceste v horch (t.j. na via ferrate) pre odvneho feratistu na prelez ponad roklinu. Aby mohol by takto most uznan
bezpenm, tak naptie v lanch nesmie prekroi 10-tinu medze pevnosti
lana, ak je feratista s hmotnosou = 80 kg v strede lana. Urte, ak najiriu roklinu vieme pomocou tohoto lana prekona za dodrania predpisov!
Straty hmotnosti feratistu v dsledku fyziologickch prejavov strachu mete
zanedba.

Zdroj: lohu vymyslel Jakub

l Vzork Jakub l

Nasledovn text obsahuje dva prstupy k rieeniu tejto lohy. Prv sa


d nazva analytick a druh m pre FX nezvyajn experimentlnu povahu. V oboch prpadoch vyuijem oznaenie pre dkov hustotu lana =
= 0, 8 kg.m1 , pre hmotnos lana samotnho = = 16 kg a ozna
enie max = 10
= 10 kN pre maximlne prpustn naptie v lane. Taktie
v oboch prstupoch budem v prvom priblen povaova lano za nenaahovaten, kee relatvne natiahnutie pri maximlnej prpustnej zai je iba

10 0, 05 %.
1. Analytick rieenie
Zanem odvodenm rovnice pre idelne nenaahovaten, dokonale ohybn
lano umiestnen v homognnom tiaovom poli v statickom prpade. Zavediem si do problmu sradnice a , priom bude vo vodorovnom smere
a bude vo zvislom smere. Vka lana nad konkrtnym miestom bude dan
funkciou (). Vyberiem si ksok lana, ktorho priemet do vodorovnho
smeru je d, vi obrzok.
164

Via ferrata

FX E7

Na tento ksok psobia dokopy 3 sily, ktor maj by v statickej situcii


v rovnovhe:

tiaov sila o vekosti d = (d)2 + (d )2 = 1 + 2 () d


psob v zvislom smere nadol173 ,
ahov sila lana zava psob na lano zvislou zlokou ,1 nadol a
vodorovnou zlokou doava,
ahov sila lana sprava psob na lano zvislou zlokou ,2 nahor a
vodorovnou zlokou doprava174 .
Kee nae lano je dokonale ohybn, tak smer naptia v lane mus by
v kadom mieste dotynicou k lanu samotnmu. Vaka tomu dostvame
vzahy
,1
= tan = (),

,2
= tan = ( + d) = () + [ ()] d = () + () d.

Ke teraz zapeme podmienku pre rovnovhu nho vybranho ksku ohadom zvislch zloiek psobiacich sl, tak dostaneme hadan rovnicu

1 + 2 () d = ,2 ,1 ,

1 + 2 () = () .

(64)

oznaenie pre derivciu iarkou, teda () = d .


ksok lana m by v rovnovhe, preto musia by v rovnovhe aj vodorovn
zloky psobiacich sl, preto s obe vodorovn zloky naptia v lane rovnak. To je aj
dvod, preo je vodorovn zloka naptia rovnak pozd celho lana.
d ()

173 Pouvame

174 Sledovan

165

Via ferrata

FX E7

Tto rovnica je nelinerna diferencilna rovnica prvho rdu pre neznmu


ozn.
funkciu () = (). Meme ju riei separciou, ie oddelenm lenov
zvislch od a od ,

d
d =

1 + 2
a nsledovnou integrciou. Ide o technikalitu. ahko sa vak over175 , e
0
rovnicu spa rieenie () = cosh
+ 0 , ak plat = = .176

Mme teda rieenie


[
]

cosh
( 0 ) + 0 ,
() =

[
]

() = sinh
( 0 )

[
]

() =
cosh
( 0 ) .

[
]

2
1 + () = cosh
( 0 ) ,

Je na ase urobi si nrt situcie, aby sme mohli uri okrajov podmienky pre nae veobecn rieenie. Zrejme bude situcia symetrick ohadom bodu v strede lana. Sta nm preto njs rieenie pre vybran polovicu
lana177 . Zo symetrie je zrejm, e aj zaaenie bude symetrick a teda kad
polovica lana nesie polovicu feratistu.

175 Napodiv, po jednom dorieen tejto lohy sa ten vsledok ahko aj zapamt tak to
bolo aj v mojom prpade. ie, homognne idelne lan visia vdy ako nejako posunut
a ponaahovan cosh (hyperbolick kosnus).
176 Mal exkurzia do sveta hyperbolickch a k nim inverznm funkcim

e + e
,
2

e e
sinh =
,
2
sinh
tanh =
,
cosh

cosh =

d cosh
= sinh ,
d
d sinh
= cosh ,
d
d tanh
1
=
,
d
cosh2

(
)

ozn.
cosh1 = arccosh = ln + 2 1 ,
(
)

ozn.
sinh1 = arcsinh = ln + 2 + 1 ,
tanh1 = arctanh =
ozn.

1+
1
ln
2
1

a pridm ete univerzlnu identitu pre hyperbolick funkcie cosh2 sinh2 = 1.


177 Zvolili sme si prav as, vi obrzok.

166

Via ferrata

FX E7

Prv podmienka na lano bude, aby v strede v mieste zavesenia odvlivca


bola zvisl zloka naptia vo vybranej asti rovn polovici tiae visiaceho,
o zapeme
0

(0) = sinh
=
.
(65)

2
Druh podmienka na lano bude, aby zvisl zloka naptia v bode uchytenia
bola rovn polovici celkovej zavesenej tiae, teda
() = sinh

( 0 )
( + )
=
.

(66)

aliu podmienku nm diktuje zadanie, toti, e maximlne naptie v lane


max
nesmie prekroi hodnotu 10
. Kee vodorovn zloka naptia v lane je
vo vetkch miestach rovnak a zvisl je v mieste rovn polovici tiai lana
visiaceho v intervale (, ) a feratistu178 , tak je oividn, e najvyie zaaenie bude v lane pri chytoch do stien rokliny. Taktie je intuitvne jasn,
e vyia hodnota znamen vie naponovanie lana a tm men previs
lana voi chytom a s tm aj viu mon rku roklinu, ktor preklenie.
V zmysle urenia maximlnej prpustnej rky rokliny teda budeme iada

max
1 + 2 () =
.
(67)
10
Vyuitm podmienky (66) vieme z (67) zska vzah

2
max
( + )2 2
=

.
2
10
4
Pouitm 65) vieme vypota179
0 =

arcsinh
.

178 To sa d vidie z toho, ke lovek zoberie lano v intervale (, ) aj s feratistom,


poaduje symetriu voi = 0 a iada statick situciu.
179 Vyjadrenie pomocou zadanch velin nechm na D..

167

Via ferrata

FX E7
Nakoniec potom u vieme aj vyjadri aj
=

( + )
arcsinh
+ 0 .

Pre hodnoty zo zadania dostvame 9, 9889 kN; 0 49, 987 m;


9, 991 m. Teraz si meme vimn, e naptie v lane je prakticky po cemax
= 10 kN a teda v druhom priblen je lano dky =
lej dke zhruba 10
1

(1+ 10 ) 20, 009 m, potom m dkov hustotu =


0, 7996 kg.m
a po dopotan vyjde maximlna rka rokliny, ktor to me preklen
2 19, 990 m.
2. experimentlne rieenie
Hne na vod by som napsal, e toto rieenie nebude priamo experimentlne mera vsledok, le pjdeme na to trochu okukou. Najm by sme
boli radi, keby sme cel experiment mohli nejako zmeni a urobi pohodlne
doma, a pokia mono nie s oceovm lanom poadovanho prierezu, lebo
tak sa bene v domcnosti nevyskytuj. Rieenie bude zaloen na postupe
zvanom klovanie.
Zaneme s priblenm nenaahovatenho lana. Premyslite si, e vodorovn zloka naptia v lane mus by pozd celho lana rovnak!

Analyzujme sily psobiace na kus lana s visiacim ujom v intervale (, ).


Vyuijeme symetriu lohy voi stredu lana a zapeme rovnicu, ktor popisuje lano v statickej situcii a je vemi intuitvna. Vyuijeme, e zvisl zloky
sl psobiacich na lezca a vybran kus lana musia by v rovnovhe
() = tan =
() =

+ ()

168

+ ()
,

pre [0, ],

(68)

FX E7

kde () =

Via ferrata

1 + 2 () d

je dka lana v intervale (0,).

Tto rovnica je integrlna, lebo do nej vstupuje cez () integrl obsahujci


neznmu funkciu (). Rovnica (68) je ekvivalentn rovnici (64) s okrajovmi podmienkami (65) a (66). Rozdiel je len v tom, e tto rovnica m
jasn interpretciu, lebo je zapsan pre predstavitene vek teleso, kdeto
rovnica (64) popisuje innitenzimlny ksok lana. Hrajme sa vak, e sme
as 1 netali180 a rieenie nepoznme.
Tto rovnicu mus nutne spa aj oceov lano s feratistom preklenujce roklinu pri maximlnom povolenom zaaen. Maximlne zaaenie je
jednoznane uren podmienkou pre uhol pri chyte
()
sin =
=
2 () + 2 ()

2
max
10

=5

+
.
max

(69)

Ozname rieenie rovnice (68) s podmienkou pre maximlne zaaenie (69)


ako ().
Teraz vyuijeme ideu klovania. Pome zisti, i nm samotn existencia
rieenia () rovnice (68) s podmienkou (69) nezabezpeuje existenciu inch
podobnch rieen. Sksme hada skutone geometricky podobn rieenia
konkrtne -krt zven, t.j. rieenie v tvare
()
() =
( )

() =

= , =

( )
=
()

180 A ak sme ju nhodou tali, tak sa tvrme, e sme ju bu nepochopili alebo sme ju
razom stihli zabudn.

169

Via ferrata

FX E7
pre nejako preklovan parametre

=
=

=
=

=
= .

Pri vbere = a = bude funkcia () spa rovnicu (68), lebo


plat
( )
( )

(68)

() =
=
+

()

=
+
pre [0, ].
2

Preklovan rieenie istee spa podmienku (69), kee ide o geometricky


podobn rieenie. klovacia podmienka = nm xuje pomer medzi

hmotnosou visiaceho a hmotnosou lana, je ekvivalentn podmienke


=

.
Druh
klovacia
podmienka

vypoved
o
tom,
e
sily
sa
musia

klova rovnako ako zavesen hmotnosti, o je pri geometricky podobnom


rieen evidentn.
= .181 Hmotnos
Zoberme lanko nejakej dky, povedzme nech to je
182
= .
alej si zoberiem zvaie o hmotnosti
=
lanka oznam
a dm ho do stredu lanka. alej natiahnem lanko tak, aby chyty boli
v rovnakej vke a aby uhol, ktor zviera dotynica lanka s horizontlnym
smerom, bol rovn poda podmienky (69). Potom urite existuje rieenie
nho problmu zo zadaniu, len prslune (-nsobne) zmenen.183 Potom
mi sta zmera vzdialenos chytov a predeli ho -kom a mm odpove.
Tu by sa patrilo ete overi, e rieenie je jedin a teda sme nali to
sprvne. Sksim to aspo fyziklne odargumentova. Poksme sa urobi nasledovn mylienkov experiment: budeme hada rieenie tvaru lana vychdzajc od zvaia v strede. Vieme, e situcia bude symetrick. Zvome si
nejak vekos horizontlneho naptia v lane. Potom pridvajme na obe
strany od zvaia postupne malik ksky lana. Ich smer u je vekosou
dan, lebo sila psobiaca na u uloen asti mus by v smere pridanho
ksku a vekos zvislej zloky naptia je xovan u uloenmi objektami.
Takto viem skontruova virtulne cel rieenie tvaru lana. Je jasn, e pre
sme vybrali a zaxovali koecient podobnosti .
sme dourili vetky von parametre klovania.
183 Nae bude slo menie ako 1, nenechajte sa zmias.
181 Tm

182 Tmto

170

Via ferrata

FX E7

vie bude uhol, ktor zviera lano s horizontlou, v kadom mieste vo


vzdialenosti () pozd lana od zvaia men ako pre menie . Preto
bude dka, ktor lano preklenuje dlhia ako pre menie . Toto nm jasne
ukazuje, e rka rokliny, ktor lano preklenie rastie s rastcim , o je aj
intuitvne. Teda existuje prve jedno rieenie naej lohy pre xovan uhol
, cez ktor je xovan naptie a teda aj rka rokliny 2.
pre domce meranie.
Realizcia: Zoberiem si pagt vhodnej dky
Aby som nemusel vi ahk pagt nepresnmi kuchynskmi vhami, tak

si zvaie urobm zo pagtu, ktorho dka je
a tento prevesm cez stred.
Potom si urobm dva chyty v rovnakej vke tak, aby sa aspo s jednm
dalo hba k/od druhho. Ete si( zhotovm) nejak slune vek trojuholnk
s poadovanm uhlom = arcsin 5 +
max a snam sa nastavi vzdialenos
chytov tak, aby dotynica pri jednom z chytov bola prve . Ke sa mi
a
to podar, tak si odmeriam vzdialenos medzi chytmi a dku lanka

mm odpove na otzku zo zadania: = . Teda, aspo teoreticky. Toto
meranie bude zaaen chybou a t by bolo vhodn odhadn. Pokus som
nerealizoval.

171

FX E8 James Bond
James Bond sa chyst na aliu akciu, kde sa mus vyplha na strechu
vysokho domu. Zaobstaral si kotvu o hmotnosti = 2 kg, ktor doke
vystreli rchlosou 0 = 25 m/s. Kpil si tie horolezeck lano, ktorho jeden meter vi = 100 g. Do akej najvej vky doke vystreli kotvu
(s upevnenm lanom, samozrejme)? Odpor vzduchu neuvaujte.
Zdroj: lohu vymyslel Kubus

l Vzork Bzduo & Kubus l


Najprv si ukeme ako sa tto loha nem riei. Ozname hadan vku
ako . Dme do rovnosti kinetick energiu na zaiatku s potencilnou na
konci. Nezabdajc na potencilnu energiu lana, dostvame
1

02 = + .
2
2
Bezhlavo vyrieime kvadratick rovnicu a po drobnej prave hne mme

02

1+
1 20,92 m.
=

Toto rieenie je zl! Energia sa nezachovva. Predstavte si, e Bondovo lano je v skutonosti reaz zloen z malch obrok. Vdy ke sa kotva
o ksok posunie, pohybujca reaz naberie jednu stojacu obrku. To je neprun zrka, pri ktorej sa nleite strca energia! V prpade skutonho
lana nemme obrky ale mnostvo na zemi leiacich nehybnch kskov lana
s hmotnosami d. K zrkam teda dochdza spojite. A tak sa nm bude
spojite as energie premiea na teplo.
Vidme, e kotvu nebrzd len tia, ale aj namotvajce sa lano. Aby
sme sme videli, odkia presne toto brzdenie pochdza, napme si pohybov
rovnicu. Druh Newtonov zkon hovor
d
d
d
d
=
+
d
d
=

(70)
(71)

Zmenu hybnosti sme rozvinuli poda pravidla pre derivciu sinu. Vidme, e zmena hybnosti sa sklad jednak zo zrchovania (spomaovania)
172

FX E8

James Bond

kotvy a u odvinutho lana (t.j. d


d ), ale tie prve odvinut kus lana zmenil svoju rchlos z nuly na (len d
d ). Kee aktulna hmotnos kotvy
a odvinutho lana je = + 184 , meme psa
d( + )
d
+ ( + )
d
d
( )2
2
d
d
( + ) =
+ ( + ) 2
d
d
( + ) =

alebo

(72)

Ak to krsna diferencilna rovnica! Tto rovnicu nevieme nijak jednoducho riei, my vak mme konkrtny problm: (0) = 0, (0) = 0 ,
a chceme zisti najvyiu vku, do akej sa kotva poas pohybu dostane.
Numericky meme zisti aspo priblin hodnotu vsledku. Vedie k nej samozrejme vea ciest (run kalkulcia, tabukov editor, vlastn simulcia),
tu je jedna v programovacom jazyku C++:
d o u b l e m, l , h , a , g , v , t , dt ;
m = 2;
g = 9.81;
l = 0.1;
dt = 1 e 4;
t = 0;
h = 0;
v = 25;
while ( v > 0)
{
a = g ( l v v ) / (m + l h ) ;
t += dt ;
h += dt v ;
v += dt a ;
}
p r i n t f ( " t=%l f :

h=%l f \n " , t , h ) ;

Priblin vsledok bez odporu vzduchu je 15,9 metra, plus nejak ten
meter a pol Bondovej vky navye, ak vystreovadlo kue dr v rukch.
To je rchle a ikovn rieenie lohy. Otzkou zostva, i sa d riei aj pre
veobecn hodnoty zadanch velin, t.j. i m loha aj analytick rieenie.
Ukazuje sa, e m. Cesta k nemu je vak vemi zloit. alia as vzorku
je teda celkom nepovinn, ale zujemci sa dozvedia nejak tipy na rieenie
184 Vku budeme trochu nezvykle oznaova ako , nulov hodnotu m na rovni Bondovej kue.

173

FX E8

James Bond

podobnho typu diferencilnych rovnc.


Analytick rieenie
Preo je naa diferencilna rovnica zloit?
Je druhho rdu, obsahuje aj premenn a jej prv a druh derivciu.
Je nelinerna. Prv derivcia premennej je umocnen na druh.
Druh z tchto skutonost asto spsobuje, e diferencilna rovnica
nem explicitn rieenie v elementrnych funkcich. V tomto prpade vak
mme astie. Ukazuje sa e, v lohch z mechaniky, v ktorch z nejakch
prin dochdza k podobnm spojite neprunm zrkam, sa oplat prejs
od hadania funkcie () k hadaniu 2 (). pecilne si vimnime, e:
( )
( )
( )2
d2
d d
d d d
1 d d
1 d 2
(73)
=
=
=
=
d2
d d
d d d
2 d d
2 d
Ozname 2 () = () a pozrime sa, ako sa zmen naa rovnica, ak
prejdeme od psmeniek a k a :185
( + ) = +

1
d
( + )
2
d

(74)

asn! Zrazu mme linernu diferencilnu rovnicu prvho rdu. S tm


by sa u malo da nejako pohn. ia, rovnica nie je separovaten,186 a
treba njs nejak ntu.
Pri rieen niektorch DR si vimneme, e rieenie bude nejak exponencila, alebo nejak snusoida. Dosadenm do rovnice sme potom schopn
vemi rchlo njs sprvne rieenie. Otzkou ostva, i si aj v tomto prpade
nevieme nejako tipn, ako by asi rieenie mohlo vyzera. Ani po uprenom
pohade nm vak iadny dobr tip nenapad.187
Dobr tip vak meme zska, ak zanedbme niektor len rovnice a
vyrieime jednoduchiu rovnicu. len s derivciou zrejme vynecha nememe, pretoe by sme u nerieili DR. Ukazuje sa, e zanedbanie lena na
avej strane rovnice je schodn cesta.188 Aby sme nali vhodn tip, vyrieime
diferencilnu rovnicu
0 = +

1
d
( + )
.
2
d

(75)

185 Psmeno sce ostva, ale treba si uvedomi e u nie je hadanou funkciou, ale jej
argumentom.
186 Ned sa upravi na tvar ()d = ()d, ktor sa d priamo integrova.
187 Komu no, mal by sa nad sebou vne zamyslie a prihlsi sa u najbliieho psychiatra.
188 Tm sme vlastne vypli gravitciu.

174

FX E8

James Bond

To je jednoduch separovaten rovnica


1 d
d

=
2
+

1
d
1
d

+
(
)
1

ln = ln
+ +
2

1
2 2
=
2
( + )

() =
2,
( + )

(76)

kde je integran kontanta, ktor by sme urili z poiatonch podmienok. N tip na rieenie pvodnej DR spova v poven na funkciu
(), tzn. predpokladme rieenie v tvare:
() =

()
2

( + )

(77)

Tento n tip dosadme do diferencilnej rovnice a poksime sa zisti


nieo o (). Budeme pri tom potrebova, e
d
1
2
d
=

2
3.
d
( + ) d
( + )

(78)

Dosadenm do (74) dostvame:


1
d

2
2 ( + ) d
( + )
( + )
1
d
( + ) =
2 ( + ) d

( + ) =

(79)

Vzjomn vykrtenie dvoch lenov naznauje, e lo o dobr tip. Rovnako


aj fakt, e tto rovnica je u separovaten a () meme njs jednoduchm integrovanm. Zaveme substitciu = +
. Kee d = d, hne
meme prepsa a upravova alej:
1 d
2 d
22 2 d = d
2
= 2 3 +
3
2
3
= ( + ) +
3
=

175

(80)

FX E8

James Bond

Tu je naozaj kontantou a urme ju z poiatonch podmienok. Ne


tak spravme, napme konene vyjadrenie ():

() =
()

()
=
2
( + )

=
( + ) +
(81)
2.
3
( + )
Vieme, e na zaiatku, ke = 0 je vraz pod odmocninou rovn 02 .
Z toho dostvame, e
2 3
= 02 2 +
.
(82)
3
Tak dostvame konene zvislos rchlosti kotvy od vky:

( 2
)
3 + 02 2
2
.
(83)
= ( + ) + 3
2
3
( + )
Vidme, e keby ns zaujmala vka, v ktorej bude ma kotva nejak
rchlos, rtala by sa nm vo veobecnosti kubick rovnica. My mme astie,
pretoe pre maximlnu vku sa rieenie znova zjednodu. Pre maximlnu
vku mus plati:
2
2 3
3
( + ) =
+ 02 2
3
3

3 2 2
+ = 3 3 +
0
2

3
302
=
1+
1 15, 89 m

(84)

Radostne meme skontatova, e simulcia i matematick analza nm


dali rovnak vsledky.
Poznmka 1: Po pretan tohoto vzorku vm mono rieenie diferencilnych rovnc pripomna alchmiu. Tento postup v skutonosti nebol
ikovn nhodou, ale plne cielene. Bohuia, poriadne pochopenie vetkch
mylienok si vyaduje pr semestrov matematickej analzy a nie je mon
ich vysvetli v tak krtkom vzorku.

176

FX E8

James Bond

Poznmka 2: Prechod od () k () nm zafungoval a zafungoval by


v hocijakej inej lohe o odvjajcom sa lane. Tto substitcia ikovne zlikviduje druh derivciu, ako i druh mocninu prvej derivcie v naej DR. Nebola
by vak pouiten, ak by v DR vystupovala aj prv derivcia v prvej mocnine, o sa nestva pri lohch s retiazkou. Tak DR s zva nerieiten
v elementrnych funkcich.
Poznmka 3: Preli sme si dlh postup, pri ktorom sme porieili nelinernu DR druhho rdu a napokon takmer rieili kubick rovnicu. Napriek
vetkmu sme zskali explicitn rieenie. Otzkou je, i vbec m zmysel hada analytick rieenie takouto zloitou cestou, ak stle uvauje len vemi idealizovan prpad (dokonale ohybn lano, nulov odpor vzduchu). Numerick
rieenie je nielen omnoho rchlejie, ale jednoduchie obohatiten o odpor
vzduchu, o by analytick prstup denitvne zloilo na kolen. Sce sme sa
teda nauili riei pecilny typ diferencilnych rovnc, avak vidme, e za
mlo muziky pta privea peaz.

177

FX E9 Pruinky
Samo naiel rovnakch teliesok s hmotnosou , + 1 pruiniek s tuhosou , a jednu priamku. Pruinky teda pospjal za seba na priamku a
medzi kad dve nasledujce upevnil jedno teliesko. Zaiatok prvej pruinky
a koniec poslednej pevne zaxoval, ale telieska nechal vone pohybova po
danej priamke. Na obrzku vidte nkres situcie pre = 2:

(a) Urte peridu vetkch harmonickch pohybov, ktor me sstava vykonva, pre = 2.
(b) Urte peridu vetkch harmonickch pohybov, ktor me sstava vykonva, pre = 3.
(c) Kvalitatvne popte, o sa bude dia pre vie hodnoty .
Zdroj: tandardn uebnice fyziky

l Vzork Bzduo l
as (a)
Pre zaiatok si v pamti oivme tandardn konvenciu, teda e asov derivcie oznaujeme dvoma bodkami nad derivovanou veliinou, tj. naprklad
=
.
alej sa zamyslme, ak harmonick pohyby by mohli nasta. Ke trochu
popremame, hne nm napadne jeden: Mohlo by to cel kmita stredovo
symetricky. Potom sa d pekuova alej. Ako ale postupova, aby sme si
boli ist, e mme vetky rieenia? Treba si da pozor, pretoe zvaia mu
ma vo veobecnosti rzne amplitdy a rzny fzov posun.189
Zvolme teda in taktiku: Pouitm Newtonovho zkona vydolujeme pohybov rovnice a spravme ansatz190 , tj. do pohybovch rovnc dosadme
rovnice pre harmonick pohyb. To je taktika. Pre zaiatok sa dohodnime
na sradniciach. Tie sa potaj vzhadom na rovnovnu polohu poda obrzka:
189 Alebo
190 Z

nie? Zamyslite sa!


nemeckho der Ansatz.

178

Pruinky

FX E9

Ak sily psobia na teles, pri vchylkch 1 , 2 ? ahko sa presvedte,


e191
1
2

= 1 + (2 1 )
= 2 (2 1 ),

kde teda 1 a 2 je vsledn sila psobiaca na av a prav teleso. Po vydelen


hmotnosti a vyuit substitcie / = 02 dostvame pohybov rovnice

= 202 1 + 02 2
= 202 2 + 02 1 .

aa, rovnice spriahnutch osciltorov. U vieme, ako vyzeraj. :-)


Potiato iaden problm (sn tie dve bodky nepsobia odstraujco).
Teraz prv netrivilny npad, spomnan ansatz. Vieme, e hadan harmonick pohyb mus by popsaten rovnicami
1 () = 1 cos
2 () = 2 cos ( + ) ,
kde = 2/ je uhlov frekvencia harmonickho pohybu, 1 a 2 s
amplitdy pohybov zva a je fzov posun kmitov prvho telesa oproti
druhmu. Vimnite si, e pre druh derivcie automaticky plat, e
=
2 . Po dosaden za druh derivciu dostaneme
2 1

= 202 1 + 02 2

2 2

= 202 2 + 02 1 .

o vlastne chceme spota? Peridu. To znamen, e uhlov frekvencia


nm ako rieenie celkom sta. Vimnime si, e z tejto sstavy vieme elegantne vyli premenn . Sta si vyjadri 2 pomocou 1 z druhej rovnice a
dosadi do prvej z nich. To znamen
2 =

02
1
2

202

191 Tak ist rovnice by nm vyli dokonca i v prpade, ke s v zkladnom stave pruiny
natiahnut. Premyslite si to!

179

Pruinky

FX E9
a odtia192
2 1 = 202 1 +

04
1 .
2

202

Rovnicu mem smelo vydeli193 premennou 1 a po drobnej prave dostvame bikvadratick rovnicu
4 402 2 + 304 = 0.
Odtia ahko nahliadneme
(kee mus by kladn), e existuj dve
rieenia 1 = 0 a 2 = 30 . Z toho vieme ahko dopota aj peridy
harmonickch pohybov.
Pristavme sa vak ete na chvu pri problme, ako z tchto uhlovch
frekvenci uri, ako bude njden harmonick pohyb vlastne vyzera. Kov je, ako inak, rovnica, ktor dva do svisu 1 a 2 . Z nej vidno, e pre
jednotliv vypotan bude pre vchylky
v kadom ase plati 1 = 2

(pre = 0 ), resp. 1 = 2 (pre = 30 ). A problm je vyrieen.


Skr ne sa touto metdou pustme aj do druhej asti lohy, zamyslime
sa nad tm, o by nm asi vylo. Budeme ma tri od asu zvisl vchylky
1 , 2 a 3 a je zrejm, e budeme ma o kus viacej roboty s vyluovanm
dvoch z nich z rovnc. Napokon by sme dostali rovnicu tretieho stupa pre
2 a t by sme u nejako vyrieili.
as (a) inak194
Ukeme si ete in postup, ktor sa ukazuje by uiton najm pre
vie poty zva. Ide o to, e pohybov rovnice mem prepsa (pomocou
maticovho nsobenia) do tvaru
(

(
=

202
02

02
202

)(

1
2

)
.

Ten stpec -ov nie je ni in, ne stpec sel (zvislch od asu), tzn.
mono ho interpretova ako vektor (zvisl od asu), ozname ho . Maticu,
ktorou je vektor nsoben meme oznai ako . Pohyb zvai je potom
popsan rovnicou
.
=
vimnite si, e z tohoto u vyplva = 0 alebo .
by mohol nasta problm, pokia sa 1 prve rovn nule. Je to funkcia od asu
a v niektorch asoch bude aj nulov. Korektn argumentcia je, e sin funkcie 1 a
nejakho kontantnho vrazu je rovn nule. Kee hadme rieenie, ke 1 je v nejakch
asoch rzne od nuly, kontantn vraz za 1 mus by rovn nule.
194 Poriadnejie vs tento postup nau na matfyze docent Fecko na teoretickej mechanike.
192 Mimochodom,
193 Tu

180

Pruinky

FX E9

Toto sa a na formlne znamienko dos podob na rovnicu linerneho


harmonickho osciltora (LHO), v rovnici je vak skrytch ovea viacej sel,
ne je napsanch.195
Teraz aplikujeme ten ist ansatz, ako v prvom spsobe. To znamen, e

= 2 1
= 2 2 ,

o sa d zpisa ekvivalentne maticovo


(
)
(
)
(
)(
)

1
1
1 0
1
= 2
= 2

2
2
0 1
2
alebo ete inak
= 2 2 ,
kde magick znak 2 je jednotkov matica rozmeru 2 2.
, avak nie
Pointa je, e pohybov rovnica m v skutonosti tvar =
pre kad sa d prepsa na prve odvoden tvar, tj. na tvar popisujci
harmonick pohyb. Ns ale zaujmaj prve tie, ktor sa daj! Pre tie mus
plati rovnos
(

= 2 2

)
+ 2 2 = 0

Slovami: Matica krt vektor je nulov vektor. Tu mi ako fakt muste zobra vedomos, e ak nenulov vektor nsoben maticou dva nulov vektor,
tak determinant tejto matice mus by nulov.196
195 Pokia by ste teraz hdali rieenie rovnice ako = sin (), tak sce ikovne

hadte analgiu, ale vo vsledku mte snus matice. Sami uznte, e to vyzer akosi
obskrne. Analgia s LHO samozrejme existuje, ale budeme ju hada inak.
196 Tvrdenie m viacero dkazov, vetky sa vak opieraj o pojmy linernej algebry, ktor
sa vina z vs nau a na vysokej kole. Napriek tomu jeden uvediem:
Dencia determinantu je hrzostran, jeho vlastnosti s ale pvabn. Ak sa obmedzme
na tvorcov matice , , tak plat

det ( ) = det ( ) = det det


1

Inverzn matica
k matici je tak, e sin 1 = 1 = . Na zklade
vlastnost determinantu na sine to ale znamen, e
det det 1 = det = 1,
kde posledn rovnos, tj e determinant jednotkovej matice je 1, zatia vezmime ako fakt.
Determinant je slo, ktor vznik ako vybran kombincia sinov prvkov matice, teda
pre relne matice je to relne slo. Ak m vak matica determinant rovn nule, tak
jej inverzn matica by mala ma nekonene vek determinant. Skutonos je tak, e
inverzn matica k nej ani neexistuje, tj. neexistuje matica tak, e = .

181

Pruinky

FX E9

A sta u len spota. Determinant matice budeme oznaova zvislmi


ztvorkami. Dostvame 197
0

+ 2 2
202 + 2
02
=
2
0
202 + 2
(
)2
= 2 202 04 .

Tto rovnicu sme u raz rieili. Rieenia s 1 = 0 a 2 = 30 . To


nm ako rieenie sta. Keby sme chceli njs vektory vchyliek
prislchaj)
(
cich uhlovej frekvencii , tak sta si rozpsa rovnicu + 2 2 = 0 do
komponent a tak njs vzah medzi 1 a 2 .198
as (b)
Kto chce, me si to znova prepota prvm uvedenm postupom. My
ostatn si vyskame novonauen postup. Vchylky zva si ozname plne
analogicky psmenami 1 , 2 , 3 . ahko sa presvedme, e

= 202 1
= + 02 1
=

+ 02 2
202 2
+ 02 2

+ 02 3
202 3

(
)
Vezmime si rovnicu + 2 2 = 0 a nsobme zava obe rovnice maticou inverznou
0 = 0 , e
k oztvorkovanej matici. Tak v avej asti rovnice dostvame 2 = , v pravej 0
= 0 . My ale chceme nenulov rieenia. Tie zrejme mu existova, len pokia posledn
pravu nemono urobi, teda pokia neexistuje inverzn matica k , teda pokia jej
determinant je nulov. 2
197 Pripomeme si, e determinant matice v zvislosti od jej rozmeru sa pota ako
22

...

33

...

..
.

...

11
21

12
= 11 22 12 21
22

11 12 13
+ 11 22 33
21 22 23 =
31 22 13
31 32 33
..
.

+ det ,
det =

=1 (1)

12 23 31
32 23 11

13 21 32
33 21 12

kde oznauje ubovone zvolen riadok a kde v poslednom riadku oznauje maticu
rozmeru ( 1) ( 1), ktor vznikne z vynechanm -teho riadku a -teho stpca.
Posledn rekurentn urenie determinantu sa vo vych kruhoch nazva Laplaceov rozvoj
determinantu (poda -teho riadku).
198 Gratulujem! Prve ste sa nauili hada takzvan vlastn sla a vlastn vektory matice
. Koho by to zaujmalo viacej, me hada na internete. Anglick preklad eigenvalue
a eigenvector tie pochdza z neminy.

182

Pruinky

FX E9
To je v maticovom zpise ekvivalnentn s

1
202
02
0
1

2 = 02
202
02 2 .
2

3
0
0
202
3

Ozname maticu ako , stpec vchyliek ako . Hadme linerne harmonick pohyby, ansatz nm preto dva
= 2 . Pri druhej derivcii
teda treba kad zloku vektora vynsobi slom 2 a preto = 2 .
Ak toto vetko zhrnieme do jednej rovnice, dostvame
.
2 =
av stranu rovnice mono prepsa na 2 3 . Popresvanm lenov
rovnice dostvam
(
)
+ 2 3 = 0.
Povedali sme si, e nenulov rieenia tejto rovnice existuj len pokia
je determinant matice, ktorou sa nsob, nulov. Teda
(
)
det + 2 3 = 0.
Teraz si ukeme, ako mono poui pod iarou spomnan Laplaceov
rozvoj determinantu. Pre matice 3 3 a pri vbere199 = 1 dva napsan
rovnica vzah
11
21
31

12
22
32

13

23 = 11 22
32
33

23

12 21
33
31

23

+ 13 21
33
31

22
,
32

priom spota determinant matice 2 2 nie je iadna vek veda. Potanie


sa ete znane zjednodu vaka niektorm nulm v matici. Po nevekom
trpen dostaneme z podmienky nulovho determinantu rovnicu
( 2
)3
(
)
202 = 204 2 202 .
Rieenia njdeme rchlo. Prv nastva, ak sa ztvorky vavo i vpravo rovnaj nule. alie dve rieenia dostaneme vydelenm rovnice touto ztvorkou
a odmocnenm. To znamen:
12
22
32

= 202
(
)
=
2 + 2 02
(
)
=
2 2 02

199 Tzn., e rozvoj robme poda prvho riadku. Mono si, samozrejme, vybra hociktor
riadok a vsledok bude ten ist.

183

Pruinky

FX E9

Peridu jednotlivch harmonickch pohybov dortame jednoducho ako


= 2/ . Ukeme
( si ete,)ako vyzeraj njden pohyby. Zaneme s 1 .
M plati rovnica + 12 3 = 0 . Tak dostvame (po vydelen oboch
strn) rovnice 02 :


0 1 0
1
0
1 0 1 2 = 0 ,
0 1 0
3
0
o po rozpsan do komponent vedie na rovnice
2
1 + 3

=
=

0
0

Z toho, e jedna rovnica sa nm tam zopakovala dvakrt, si netreba robi


ak hlavu. Pekne z rovnc vidme, e stredn zvaie stoj a av s pravm
kmitaj stredovo symetricky. Vektor vchyliek mono preto zapsa ako
= (1, 0, 1) ,
kde je zvolen amplitda.
alej pre 2 . Tm istm spsobom prdeme k

2 1
0
1
0
1
2 1 2 = 0
3
0
0
1
2
To sa d znova rozpsa do komponent a riei ako sstavu troch rovnc
o troch neznmych.200 Vyjde, e vektor vchyliek je (zapsan ako riadok)
(
)
= 1, 2, 1 .
Napokon, o to rob pre 3 : plne analogicky prdeme k rovnici

2
1
0
0
1

1
2
1 2 = 0 ,

0
3
0
1
2
200 Komu nieo vrav Gaussova eliminan metda, sta riei sstavu s rozrenou maticou ( 0 ), tzn. v tomto prpade


2 1
0
0
1
2 1
0 .
0
1
2 0
.

184

Pruinky

FX E9
o vedie na vchylky

( )
= 1, 2, 1 .

as (c)
Ak sa obzrieme na doterajie rieenie, napadne nm, e pre vek by
malo existava rznych harmonickch pohybov (tzv. mdov ). Skutone je
to tak, dokazova to vak nebudeme.201 Me sa sta, e niekoko rznych
mdov bude ma rovnak uhlov frekvenciu. Ete by som vak rd poukzal
na dve zaujmavosti. Tak sksme zapoa veobecn rieenie.
Ak zaneme hada pohybov rovnice, zistme, e zrchlenie nejakho
konrtneho zvaia v danom momente zvis len od jeho okamitej polohy
a od okamitej polohy jeho susedov (len pruiny medzi nimi na psobia
nejakou silou). ahko sa presvedme, e vsledn pohybov rovnice mono
prepsa do tvaru:

1
2

3
..
.

202

02

02

202

0
..
.

02
..
.

02
..
.

...

..

.
0

...
..
.
..
.

0
..
.

202
02

02
202

1
2
3
..
.

Maticu ozname ako . Ak sa pustme do hadanie linernych( harmonic- )


kch pohybov (vlastnch sel matice!), budeme riei rovnicu det + 2 =
0. Ozname ete + 2 = .
Prv zaujmavos je toto: Ak si spravme Laplaceov rozvoj determinantu
poda prvho riadku, dostaneme len dva leny (pretoe v prvom riadku s
len dva nenulov leny). Avak o dostaneme, ke v krtnem prv riadok
a prv stpec? Dostaneme 1 . Zana to vyzera fajn.
al nenulov len je v druhom stpci, pri vpote lena pri om treba
v krtn prv riadok a druh stpec. To u nie je tak pekn matica,
ale ak si vimneme, e v jej prvom stpci je len jeden nenulov prvok, tak
meme urobi rozvoj aj tohoto determinantu poda prvho stpca. A uduj
sa svete, zrazu tam vyjde matica 2 . Skste si to napsa na papieri. Malo
by vm vyjs, e
(
)
det = 2 202 det 1 04 det 2 .
201 Bolo by potrebn sa odvolva na vetu o hlavnch osiach, ktor je dos obscnne
vedie u na strednej kole.

185

Pruinky

FX E9

To znamen asi toko, e pokia mm vyrieen problm pre = 2 a


= 1, lacno ho vyrieim aj pre = . Skste si teraz ikovne odvodi,
e uhlov frekvencie pre = 4 s
)
(
(
)
2
1
1,2,3,4 = 2 2 3 5
02 ,
o sa mimochodom d ikovne upravi na
(
)
2
1,2
= 12 3 5
(
)
2
3,4
= 12 5 5 .
V upravenom tvare sa vak ned dobre pozorova skutonos,202 e ak nejak
2 je rieenm problmu, tak aj 402 2 je jeho rieenm (tj. e rieenia s
symetrick okolo 202 .). To nie je nhoda
( a mete
) si to dokza ako cvienie
z algebry. Treba vyjs z rovnice det + 2 = 0. Potom vezmeme ten
ist vraz kde zamenme 2 (402 2 ). Po pr pravch dostaneme, e
potom aj tento vraz m nulov determinant, a teda e aj 4 2 2 je naozaj
rieenm problmu.
alou zaujmavosou me by, o rob tento systm pre .
Vtedy m lep zmysel skma poet d mdov prislchajcich na interval (; + d), teda aksi hustotu mdov () = d/d. To o by sme
takto zskali by boli predpovede klasickej fyziky o sprvan sa relneho jednorozmernho krytlu.203
Ak sa niekedy budete zaobera tuhmi ltkami, budete schopn si odvodi, e v zadanej limite plat vsledok:

(
(0 , 20 )

)
2

1 2
0
() =
0

/ (0 , 20 )
Ako sa dopracova k tomuto vsledku, o tom
u naozaj niekedy inokedy
a niekde inde. Mimochodom, vimnite si, e ()d = , tj. e celkov
poet mdov sa aj v limite rovn potu zva.

ktor sme si mali vimn aj pre = 2 a = 3.


dostali by sme len do uritej miery sprvny vsledok, pretoe svet sa riadi zkonmi
kvantovej fyziky. Pre "atmy na pruinkch", ktor s dos mal, m kvantovanos vrazn
vplyv.
202 Skutonos,

203 A

186

F
Termodynamika

Software is like entropy. It is dicult to grasp, weighs nothing, and obeys


the second law of thermodynamics; i.e. it always increases.
Norman Ralph Augustine

187

FX F1 Puding

Lenka navarila dva hrnce pudingu:


Vanilkov puding m tepeln kapacitu
teplotu 1 , okoldov m kapacitu 2
vo forme makroskopickej prce z nich
Potajte rchlo, km nevychladne!)

jeden vanilkov a jeden okoldov.


1 (nie mern, t.j. u v J.K1 ) a
a teplotu 2 . Koko najviac energie
vie vyai? (Puding pri tom nezje.

Zdroj: lohu vymyslel Bzduo

l Vzork Jakub l

Ahojte, ukeme si dve rieenia. To prv bude sprvne, jednoduch a


inn - avak nepraktick. To druh bude sprvne, komplikovanejie, menej inn - no o mnoho praktickejie. Pre jednoduchos (avak bez ujmy
na veobecnosti) budeme predpoklada, e okoldov puding nem vyiu
teplotu ako vanilkov (1 2 ).
Rieenie 1.
Zoberme si na pomoc idelny plyn s molekulami a teplotou < 2 .
Tento plyn zavrime do izolovanej rry uzatvorenej idelnym pohyblivm
piestom204 . Dme ho do tepelnho kontaktu s naimi pudingami, avak tak,
aby teplo mohlo prestupova na chladnej pracovn plyn iba pomaly, presnejie povedan tak, aby procesy boli kvzistatick205 . alej pomocou znakovej elektroniky budem kontrolova pohyb piestu tak, aby sa teplota plynu
nemenila. Jedno z monch vyuit je aplikcia ako vahu.
204 Idelny pohybliv piest sa pohybuje bez trenia a nem iadnu hmotnos. Ako sa
obvykle pri kvalitnom tovare stva, vo vine obchodnch reazcov je tento artikel u
dvno vypredan.
205 Kvzistatick procesy s tak, pri ktorch sa zastnen teles nachdzaj stle v rovnovhe. To sa d zabezpei pomalosou zmien vzhadom na relaxan dobu - dobu potrebn na ustanovenie rovnovhy. pecilne potom pre idelny plyn pri takomto deji plat
stavov rovnica v ubovonom okamihu.

188

Puding

FX F1

V takomto usporiadan zjavne plynu dodm celkov teplo = 1 (1 ) +


2 (2 ) a toto vetko som oividne206 vyuil na vyzdvihnutie kvdrikov207
v tiaovom poli.
m niia bude teplota pracovnho plynu, tm viac prce vah vykon.
Limitne pre 0 viem ochladenm pudingov vykona mechanick prcu
o vekosti = 1 1 + 2 2 .208 Nepraktinos tohoto vahu je v tom, e
pracuje neperiodicky ie ho po pouit nie je hne mon znovupoui.
Rieenie 2.
Tentoraz sa obmedzm na periodicky pracujci stroj. Pomem si Carnotovm strojom. Carnotov stroj je cyklicky pracujci stroj.
206 Trpaslk Tlai db o to, aby dej plynu bol izotermick. V slade s tm vyhadzuje na
jednotlivch poschodiach kvdriky, aby bol tlak v plyne pod piestom vdy = . Pri
izotermickom deji sa vntorn energia plynu nemen, take vetko prijat teplo sa vyuije
na konanie mechanickej prce.
207 aisko plynu stpa tie. To vak nevad, lebo zdvhanie plynu je mechanick prca.
Ak sa nm to napriek tomu nepi, tak sa to d oetri naprklad tak, e piest dme do
vodorovnej polohy a pomocou dmyselnho nehmotnho systmu ln a kladiek budeme
zdvha ploinu s trpaslkom.
208 V skutonosti to tak dobre nepjde, lebo pri nzkych teplotch sa tepeln kapacita
kadho telesa bli k nule, lim = 0, o je dsledok 3. vety termodynamickej. Plat to
0

aj pre model idelneho plynu, u ktorho je znme, e tepeln kapacita je kontantn. Pri
nzkych teplotch treba toti poui model idelneho kvantovho plynu.

189

Puding

FX F1

Idelne pracuje s idelnym plynom, ktorho pracovn schma pozostva zo


tyroch kvzistatickch fz:
1. Plyn v polohe A pri teplote 1 rozpnam. Plyn prijme celkov teplo
1 .
2. Piest presuniem do polohy B a adiabaticky (tepelne izolovane) ho rozpnam a km nenadobudne teplotu 2 .
3. Piest presuniem do polohy C a pri teplote 2 ho stlam a po objem
. Plyn pritom odovzd celkov teplo 2 .
4. Piest presuniem do polohy B a adiabaticky ho stlam a km nenadobudne teplotu 1 . Potom ho presuniem do polohy A.
Objem je stanoven tak, aby bol cyklus plynu uzavret, t.j. tak, aby po
dokonen cyklu bol plyn v rovnakom stave ako na zaiatku. Nie je ak
def
2 209
ukza, e innos takhoto stroja je =
= 1 21 = 1
1 .
1
Teda ak prijme teplo , tak vykon prcu = a chladiu dod teplo
= 21 .
Carnotov stroj vak me pracova aj pospiatky, v obrtenom reime.
Funguje to vaka tomu, e kad fzu tvor kvzistatick dej, ktor me
bea oboma smermi, poda toho na ktor stranu prikeme trpaslkovi Tlaiovi posva (pomaliky) piest. V tomto usporiadan sa Carnotovmu stroju
zvykne hovori aj Carnotova chladnika. Nasleduje popis prce tohto stroja:

1. Plyn v polohe A pri teplote 1 stlam. Plyn odovzd celkov teplo


1 .
209 Ak

to itate ete nikdy nepotal, tak je najvy as!

190

Puding

FX F1

2. Piest presuniem do polohy B a adiabaticky (tepelne izolovane) ho rozpnam a km nenadobudne teplotu 2 .


3. Piest presuniem do polohy C a pri teplote 2 ho rozpnam a po objem
. Plyn pritom prijme celkov teplo 2 .
4. Piest presuniem do polohy B a adiabaticky ho rozpnam a km nenadobudne teplotu 1 . Potom ho presuniem do polohy A.
Objem op stanoven tak, aby bol cyklus plynu uzavret. V tomto usporiadan stroj pohame. Ak teplo, ktor stroj odoberie chladnejiemu telesu
je , tak poas jednho cyklu musme na om vykona mechanick prcu
o vekosti = ( 21 1). Teplejie teleso prijme teplo = 12 . Na tomto
princpe funguj okrem chladiacich zariaden aj tzv. tepeln pumpy, ktor
teplom vykuruj budovy.
Skutone dleit je vak poznatok, e innos ubovonho cyklicky
pracujceho stroja nie je vyia ako innos Carnotovho stroja. Toti, pripusme existenciu cyklicky pracujceho tepelnho stroja, ktor pracuje pri
teplotch 1 a 2 a m uinnos vyiu ako Carnotov stroj. Budem ho vola
BlackBox na znak toho, e pre ma me predstavova iernu skrinku, o ktorej neviem viac ako to, e to je cyklicky pracujci tepeln stroj (zaloen na
nejakom mne utajenom princpe). Ak takto stroj pri jednom cykle odoberie
teplejiemu telesu teplo , tak vykon prcu = a chladiu odovzd
teplo = (1 ).210 Spriahnime teraz tento stroj dokopy s Carnotovou
chladnikou, ktor nastavme tak, aby chladnejiemu telesu odoberala pri
1 cykle akurt teplo . Poda predpokladu o innosti bude pri takomto
nastaven odovzdva teplejiemu telesu teplo menie ako a bude na
pohon potrebova mechanick prcu meniu ako je .

Takto kombinovan stroj bude vo vsledku kona mechanick prcu iba


ochladzujc teplejie teleso.211 Naa sksenos vak je, e takto stroj by
210 Tento vzah vychdza zo zkona zachovania energie. Kee BlackBox je cyklicky
pracujci stroj, tak jeho stav na konci cyklu je zhodn so stavom na zaiatku cyklu a
teda jeho energia sa nezmenila. Preto teplo, ktor poas cyklu prijal mus by rovn stu
mechanickej prce, ktor vykonal, a teplu, ktor odovzdal chladiu.
211 Chladnejie teleso me by znane mal, lebo sli len na to, aby sa od neho poas
kadho cyklu doasne nejak teplo zobralo a nsledne vrtilo.

191

Puding

FX F1

sme sce vemi radi postavili, ale akosi to nejde. Existenciu takhoto stroja
popiera druh veta termodynamick: Ned sa zostroji perpetuum mobile
druhho druhu t.j. cyklicky pracujci stroj, ktor by teplo menil priamo
na prcu. Ak jej uverme, tak sme prve doli k sporu. Toti, ak by existoval
BlackBox, tak viem skontruova perpetuum mobile druhho druhu a neplat druh veta termodynamick. Jedna z ekvivalentnch formulci druhej
vety termodynamickej je vak aj t, e teplo spontnne neprechdza zo studenieho telesa na teplejie. A to sa, uznte, skutone zd by pravda. Nu,
potom treba akceptova aj to, e BlackBox poadovanch kvalt neexistuje
a teda najvyia mon innos tepelnho stroja pracujceho pri teplotch
1 a 2 je prve innos Carnotovho stroja.
Pome konene k samotnmu rieeniu lohy. Mme dva pudingy. Uvaujeme iba cyklicky pracujce stroje. Zrejme by sme sa mali obmedzi iba
na stroje, ktor po ukonen cyklu vbec nemenia teploty inch telies ako
prve pudingov. Nu, kad stroj vak mem urobi ubovone mal a povaova teploty pudingov poas jednho cyklu za kvzikontantn. No a ja
viem, e najvyiu innos m z takchto strojov pre kad jeden cyklus
prve Carnotov stroj. Carnot poas jednho cyklu s pudingami o teplotch
1 , resp. 2 odoberie teplejiemu telesu teplo 1 = 1 d1 a chladnejiemu
dod teplo 2 = 21 1 = 21 1 d1 = 2 d2 . Odtia mm diferencilnu
rovnicu v separovanom tvare
2 d2
d1
=
,
1
1 2
odkia vieme integrciou zska vzah medzi teplotou 1 teplejieho telesa
a teplotou 2 chladnejieho telesa. Pre jednoduchos si zavediem oznaenie
2
=
1 .

2
d1
d2
=
1
2
2
[
] 1
1

2 =
.
1 2

Z tohoto vzahu vieme uri aj teplotu , ktor bud ma oba pudingy


vtedy, ke sa ich teploty vyrovnaj,
[

1
=

] 1

192

= [2 1 ] +1 .

Puding

FX F1
Potom je u urenie celkovej prce pln lahoda

1 2
1 d1
1

[1
]
1
1
1
= 1 (1 ) + (1 2 ) (1 ) .
Do tohoto vzahu ete treba dosadi vrazy za , resp. . Konen vzah m
celkom aleko od estetina, tu je

1
) +
(
) (12+ )
(
2
2
1
1
2
2
1
2
+ 2 2 1 12 1 21
.
= 1 1 1 21

193

FX F2 Kompost
Judita sa rozhodla vyui teplo kompostoviska vo svojej zhradke na dobroinn ely. S tm zmerom si postavila dva vratn stroje periodicky pracujce poda dejov a . Pracovnou ltkou je vzduch, pretoe je
lacn. Ak je pomer innost tchto dejov?

Zdroj: lohu vymyslel Bzduo

l Vzork Jakub l
Zdravm pekne vetkch nhodnch itateov. innos cyklickho deja
s plynom je denov ako podiel vykonanej mechanickej prce a dodanho
teplo. Tto dencia m hlavu a ptu, ale je dobr ma na zreteli, e je to
len otzka dohody. V naom prpade je to zrejme to najrozumnejie, o sa
d vymyslie.
Vzduch budem pre nae potrske ely povaova za idelny plyn s potom stupov vonosti = 5, lebo je dominantne zloen z dvojatmovch
molekl 2 , resp. 2 .212 Teda pri ohriat o vntorn energia plynu
stpne o 2 , kde je Boltzmannova kontanta a je poet molekl.
Pre idelny plyn plat stavov rovnica = . Deje budem povaova za
vratn, resp. kvzistacionrne, t.j. tak, pre ktor plat, e intenzvne veliiny
(tlak, teplota) s vo vetkch miestach plynu a v kadom ase rovnak ako
v stacionrnom prpade.213
Oividne, prca, ktor oba stroje vykonaj pri jednom cykle, je pre oba
stroje rovnak, konkrtne 020 . Na urenie pomeru innost nm ostva u
iba zisti mnostvo dodanho tepla pri jednom a druhom cykle.
212 Toto
213 Inak

skutone nie je reklama na mobilnho opertora.


ani nem zmysel uvaova stavov rovnicu.

194

Kompost

FX F2

Stroj s cyklom ABD: Pri deji plyn jednoznane teplo prijma, jeho
termodynamick teplota stpa poda stavovej rovnice na dvojnsobok. Ak
ozname jeho teplotu v bode ako 1, a poet jeho molekl ako 1 , tak
vieme poveda, e vntorn energia plynu stpla o 2 1 1, , o pomocou
stavovej rovnice mem vyjadri ako 2 0 0 . Pri deji teplota po
stred uhloprieky najprv stpa, potom kles. Pritom plyn stle kon prcu.
Ned sa jednoznane prostm nahliadnutm uri, i plyn pozd celej iary
prijma teplo.
Parametrizujme si teda dej napr. pomocou premennej , ktor
sa bude meni od hodnoty 0 v bode po 1 v bode , priom tlak pre
konkrtne bude (2 ) 0 a objem (1 + ) 0 . Skmajme, i plyn pre nejak
konkrtne pri innitezimlnom nraste parametra o d teplo prijma
alebo odovzdva. Prijat teplo vypotame ako set prrastku vntornej
energie d a plynom vykonanej mechanickej prce d . Vntorn energia
pri nraste parametra o d narastie o (pouijeme stavov rovnicu)214

[1 ( + d ) 1 ] ,
2

= [( + d)( + d ) ] ,
2
[
]

= 0 0 (2 )d (1 + )d + (d)2 ,
2

= 0 0 (1 2) d.
2

d =

Mechanick prca je jednoducho


d = d = (2 )0 0 d.
Teda teplo je rovn
[
= 0 0

]
+4
( + 1) d,
2

+4
odkia poahky vidme, e plyn teplo prijma pre < 2(+1)
= 34 . Ozname
teda ako bod bod prislchajci hodnote parametra 1 = 34 .

214 Pri

pravch vyuijeme, e pre innitezimlne d je (d)2 exaktn nula.

195

Kompost

FX F2

Meme poveda, e plyn prijma teplo na seku . Konkrtne prijme


na tomto seku
+ = 0 0

2 + (2 1 )

1 + [(1 + 1 )(2 1 ) 2] 0 0 ,
2
2

kde som prcu vyjadril ako plochu lichobenka v -diagrame. Pri deji
plyn teplo odovzdva.
Pri deji plyn strca vntorn energiu a zrove je na om konan
prca, teda nutne teplo pri tomto deji odovzdva. Preto celkov dodan teplo
pri deji naim strojom je rovn
1 =

]
0 0 [
(4 1 )1 + (1 + 1 21 ) .
2

Stroj s cyklom BCD: Pri deji plyn teplo jednoznane prijma,


konkrtne prijme teplo +2
2 (20 0 ). Pri deji zasa pre zmenu jednoznane teplo odovzdva. Dej si op parametrizujem pomocou
, rovnako ako pri analze predolho stroja. Musme si vak uvedomi, e
dej prebieha v opanom smere. Teda parameter od 1 kles ku 0 a prrastok d je zporn. Vyuitm predolch vpotov teda vieme, e na seku
plyn teplo prijma. Konkrtne prijme
+ = 0 0

1 + (2 1 )

(11 )+ [(1 + 1 )(2 1 ) 2] 0 0 .


2
2

Dokopy teda stroj prijme teplo


2 =

]
0 0 [
(1 + 41 21 ) + (2 + 1 21 ) .
2

Pomer innost strojov teda je


115
2
(1 + 41 21 ) + (2 + 1 21 )
1
=
=
=
.
2
1
(4 1 )1 + (1 + 1 21 )
67

196

G
Teria relativity a kozmolgia

Astrophysicists are often in error, but never in doubt.


Lev Landau

197

FX G1 Supernova
Kdesi aleko vybuchla supernova a obrovskou rchlosou vyvrhla materil na vetky strany. Peo tieto jej pozostatky u ist as pozoruje so svojm alekohadom. Pozn vzdialenos supernovy , zmeral aj uhlov rchlos
rozpnania jej oblky (vzaovania pozostatkov od hviezdy) , a tak si jednoduchm vpotom ( = ) vypotal, e sa oblka rozpna rchlosou 4/3
krt vou, ako je rchlos svetla. Akou rchlosou sa oblka rozpna naozaj?
(Oblka supernovy sa rozpna rovnomerne.)
Zdroj: lohu vymyslel Peo Matk

l Vzork Bzduo & Kubus l


Uvedomme si najprv, v om spova problm nadsvetelnej rchlosti. Najelementrnej prpad je, ke sa k nm po priamke pribliuje teleso (naprklad Vek pes215 ) vekou (samozrejme podsvetelnou) rchlosou .

V nejakom ase k nm vyle fotn.216 O d neskr sa k nm nachdza


bliie o d a vyle al fotn. Vimnime si, e dva fotny nesce informciu
o zmene polohy o d sa nachdzaj vo vzdialenosti ( ) d od seba. Do
nho oka prdu v intervale
d =

( ) d
.

V priebehu tohto asovho intervalu spozorujeme posunutie o d, teda


pozorovan rchlos je
d

=
=
.

215 Po

latinsky Canis Major.


viacero fotnov, ktorch zaostrenm sme zchopn zisti, ako aleko sa od ns
teleso nachdza.
216 Resp.

198

Supernova

FX G1

ahko si mete overi, e pri rchlostiach vch ako /2 pozorujeme pohyb


nadsvetelnou rchlosou. Pre = 0,99 pozorujeme Vekho psa pribliova
sa rchlosou 99. To sa nm ale iba zd! V skutonosti ide iba podsvetelnou
rchlosou.
Nieo podobn sa deje aj s oblakom supernovy. Kvli pribliovaniu oblaku
v prednej asti supernovy vidme pohyb zrchlen. To, o pozorujeme ako
zvovanie jej polomeru vak neme svisie s asou oblaku, ktor sa
rozpna po priamke ku nm. T predsa nem iadnu kolm zloku rchlosti
a nememe ani iadny kolm pohyb spozorova. Za nadsveteln pohyb tie
neme as oblaku rozpnajca sa kolmo na spojnicu supernova-Zem. Tam
toti prichdzaj fotny stle z rovnakej vzdialenosti a d = d. To, o mi
pozorujeme ako rozpnanie bude spsobova nejak smer medzi.
Ukeme si dva spsoby rieenia problmu. Hne naprklad vidme, e
v nejakom smere rozpnania budeme pozorova najviu zdanliv rchlos.
Extrmy funkci hadme predsa pomocou derivci. Ale ete pred tm si
ukeme rieenie vyuvajce analytick geometriu, pretoe sa pri om dozvieme celkom zaujmav skutonosti. A teraz u rchlo do rieenia, lebo ns
dostihne a zje rchlo sa enci Vek pes.
tipka analytickej geometrie
Zaveme si sradnicov sstavu (posta dvojrozmern) so stredom v supernove a osou smerujcou smerom k Peovi. Kee vzdialenos supernovy
je rdovo via ako jej rozmery, meme predpoklada, e le, ktor Peo
vid, s rovnoben s osou . Nech sa oblka supernovy rozpna rchlosou .
Ak Peo vid nejak jej kus v mieste [, ] v ase po vbuchu supernovy,
mus plati 1 + 2 = , kde 1 je as potrebn na to, aby sa do danho
miesta dostal materil z miesta vbuchu, a 2 as potrebn na to, aby sa
svetlo z tohoto miesta dostalo ku Peovi. Ukazuje sa ako uiton zavies
si as meran od okamihu, kedy Peo vid vybuchn supernovu, t.j. as
odsaden od o hodnotu /.217
1 + 2 =

2 + 2

+
= 0 +

2 + 2

(85)

(86)

Dostali sme teda rovnicu vetkch monch miest, kde Peo vid nejak
ksok oblky v jeho ase . Upravujme alej:218
217 Niet
218

divu, pre takto zaveden as musme pre = 0 dosta pekn rieenie: Jedin bod.
1
je relativistick Lorentzov faktor.
2

1 2

199

Supernova

FX G1
2 + 2
2
(
)
2
2


+ 2
2 1 2 2

2
2

2
+ 2
2

(
)2

+ 2

= 2 +

2 2
+ 2

= 2 2
= 2 2
= 2 2 +

2 4 2
.
2

(87)

Zrazu nm z neba spadla zaujmav skutonos, e Peo vid v kadom


okamihu oblku supernovy ako elipsu. T sa s asom rozpna poda odvodenej rovnice.219 Ns vak zaujma iba jej zdanliv rka. Kee supernova je
aleko, bude ou teda maximlna sradnica prislchajca asu . Sradnica je najvia, ke bude ztvorka s lenom nulov, take
2 4 2
2

2 2
2
2
= 1 + 2 = 1 + 2
=

2
2 2
=

= =

2 = 2 2 +

(88)

Zdanliv rchlos rozpnania supernovy bude teda krt via ako t


skuton, pri vzdialenom objekte akm je supernova bude to ist plati
i o uhlovch rchlostiach. Z tejto rovnice jednoducho vyjadrme skuton
rchlos poda zdanlivej:
=

2 2 2 2 = 2 2

=
=
2
2
+
1+

2
2

(89)

Po dosaden hodnoty = 43 dostvame spoahlivo podsveteln rchlos


rozpnania oblky = 0,8 . Kadopdne, mali sme astie, e nm pozorovan mnoina bodov vyla ako elipsa. Rovnako tak sa mohlo sta, e by
219 Uvedomte si fakt, e body, ktor Peo pozoruje sasne, nie s sasn! Odohrali sa
v rznych asoch! Kvli rznej vzdialenosti k nm vak svetlo z nich prilo naraz.

200

Supernova

FX G1

vsledkom bolo nejak vajce a hadanie najvieho by nebolo vbec tak


jednoduch.
Bim, teda derivujem
Uvaujme, e na zaiatku sa cel oblka nachdzala v jednom bode
v strede supernovy. Bez ohadu, ktorm smerom sa nejak as oblaku rozpna, vetky budeme pozorova ods z tohoto bodu naraz. Pome teraz
skma, v ktorom smere pozorujeme ak axilny pohyb. Tento smer by mal
vyjs nezvisl od asu.220 To znamen, e oblak v danom smere budeme
neustle pozorova ako najrchlejie sa rozpnajci a od samho vbuchu sa
nm bude javi ako okraj supernovy.
Zjednodume si cel vpoet nasledovne: Predpokladajme, e v strede
supernovy je teleso, ktor sa pohybuje rchlosou a pod uhlom k spojnici
so Zemou. Ak kolm zloku rchlosti pozorujeme?

Uvaujme dva fotny vyslan v asovom odstupe d. Z obrzku vidno,


e nes informciu o posunut v kolmom smere o sin d. Do oka prdu
v asovom intervale221
d =

( cos ) d
= (1 cos ) d.

(90)

Pre pozorovan kolm zloku rchlosti plat

1 sin d
sin
= =
=
.

d
1 cos

(91)

220 Ak pohyb danou rchlosou a v danom smere pozorujeme ako pohyb uritou inou
rchlosou, neme to zvisie na ase, v ktorom pozorovanie robme.
221 = je alia substitcia hojne pouvan v relativite.

201

Supernova

FX G1

Zskali sme krsnu a jednoduch zvislos. Ostva nm len zisti, kde na


intervale 0; /2 sa nachdza extrm a njs jeho hodnotu:222

d
cos (1 cos ) sin sin
=
=0
2
d =
(1 cos )

(92)

cos

cos
2

sin = 0
2

cos 2 = 0
cos =

(93)

Pritom sme vyuili fakt, e = 0. Zskali sme tak smer, v ktorom


vidme najrchlej zdanliv kolm
pohyb. Teraz

sta dosadi, do rovnice


(91), vyuijc fakt, e sin = 1 cos2 = 1 2 . Tak dostvame
=

1 2

=
.
1 2
1 2

(94)

Bystr oko si iste vimlo, e tto rovnice je vlastne len (88) prepsan do
inch psmen. alej je teda postup rovnako jednoduch a vsledok ten ist,
t.j.

.
=
2
1 2

Nadhad cez geometrick optiku


Ak nemte radi derivcie, urite ocente npad Petra Pereniho, ktor
lohu vyrieil ikovnou vahou. Vimol si, e rovnica (86) popisuje tandardn situciu, kde sa osi pohybuje nejakou rchlosou v jednom prostred a inou rchlosou v druhom prostred priom lohou je njs optimlnu cestu, ktorej prislcha minimlny as. Rieenie vak vieme z geometrickej optiky: Snellov zkon!
Pre xovan as existuje aksi oblka,223 vntri ktorej sa osi pohybuje rchlosou rozpnania supernovy a rchlosou svetla mimo nej.
222 V samom vode lohy sme si vysvetlili, preo mus existova maximum na tomto
intervale. Ak zistme, e sa v tomto intervale nachdza jedin extrm, tak netreba pota
druh derivciu a meme si by ist, e ide o maximum nami hadan.
223 Touto oblkou je v skutonosti elipsa njden v prvom rieen lohy. Presn tvar ns
vak teraz nezaujma.

202

Supernova

FX G1

Zo Sneellovho zkona mme


sin( )

= .
sin

Ns zaujma uhlov rka, tj. prpad = 90 . Odtia hne dostvame, e


najvie rozpnanie pozorujeme pre uhol cos = = , o je plne identick rovnici (93). To spolu s rovnicou (91) vedie po tret raz na ten ist
vsledok.

203

FX G2 Vesmr
Peo vyniesol do vesmru svoju obben druicu. Druica sa dlho tlala
hlbokm vesmrom a dostala sa a do vzdialenosti od Zeme, ke jej zaalo
by smutno a zaala vysiela rdiov signl s frekvenciou . Kubo si chcel
naladi rdio na Peovu druicu, a tak sa zaal zama nad nasledujcimi
problmami.
(a) Ak frekvenciu signlu druice nameria Kubo, ak sa druica pohybuje
(hoc i vekou) rchlosou smerom od Zeme?
(b) Ak frekvenciu signlu druice nameria Kubo, ak druica stoj, ale vesmr sa rovnomerne rozpna tak, e vzdialenos medzi druicou a Zemou
sa v ase vysielania zvuje rchlosou ? Rovnomern rozpnanie vesmru v tejto lohe znamen, e objektvna vzdialenos medzi kadmi
dvoma bodmi sa zvuje kontantnou rchlosou, priom v kadom momente je rchlos vzaovania sa dvojice bodov priamo mern vzdialenosti tchto bodov.
Zdroj: lohu vymyslel Jakub

l Vzork Jakub l
Pod rdiovm signlom obyajne rozumieme elektromagnetick vlnu, obvykle smerovan vysielaom prevane do nejakho elanho priestorovho
uhla, s vlnovmi dkami od pribline 1 do 1000 metrov. Samotn informcia sa kduje pomocou modulcie (superpozcia viacerch monofrekvennch
vn a vznik tzv. rzov). Technicky sa jedn o netrivilne zleitosti. Rozobra
takto situciu by zrejme bolo nad nae sily a preto sa uchlime k zjednoduenmu mylienkovmu experimentu (Gedankenexperiment) komunikcia
bude prebieha pomocou elektromagnetickho iarenia vysielanm fotnov224
z druice v smere priamo k Zemi po dvkach s frekvenciou . Informciu budem kdova pomocou potu vyslanch fotnov (ie pomocou amplitdy,
ak sa bavme v termnoch vn).
as (a).
Spomnan rchlos druice me by poda zadania vek. Tm sme
chceli poveda, e me by porovnaten s rchlosou svetla a e teda
tto as treba rozobra relativisticky. Zrejme ste u pouli o dilatcii asu
224 Malo by by znme, e elektromagnetick javy maj dulny charakter a d sa uvaova
o elektromagnetickch vlnch ako aj o prde astc fotnov.

204

FX G2

Vesmr

a kontrakcii dky v pecilnej terii relativity. Je to dsledok pecilnych


Lorentzovch transformci
= ( )
=
=

(
)
= 2 ,

kde
1
=
1

2
2

Tieto rovnice nm hovoria, ako sa transformuj sradnice polohy a asu pri


prechode z inercilneho systmu s kartzskou sstavou sradnc , a
a asom do inercilneho systmu s kartzskymi sradnicami , ,
a asom , ktorho poiatok sa vzhadom na hbe rchlosou v smere
kladnej -ovej polosi. Sradnin osi a s rovnoben s osami a , osi
a sa prekrvaj, a v ase = = 0 sa poiatky oboch sstav prekrvaj.

Tieto vzahy uruj, kde ( = ( , , )) a kedy ( ) budem v systme


pozorova udalos, ktor nastala v systme v mieste = (, , ) a v ase
.
V naom prpade meme uvaova systm spojen s druicou a
bude spojen so Zemou225 . Nech sa teda Zem hbe od druice rchlosou .
Druica vyle v ase v mieste = (, , ) = 0 prv signl. V ase + ,
kde = 1 , vyle op v mieste = (, , ) = 0 druh signl.
225 Tu prve robme znan zjednoduenie problmu, lebo Zem sa nepohybuje rovnomerne! V skutonosti by sme mali riei problm tak, e by sme spotali, kedy k nm
signl prilet, a rchlos druice vzhadom na Slnko by sme mali relativisticky sta s rchlosou Zeme voi Slnku v danej dobe prletu signlu. V takomto prpade u vak ani smer
renia signlu nemus by rovnoben so smerom vzjomnho pohybu druice vzhadom
na Zem v dobe prjmu...

205

FX G2

Vesmr

Poda vyie napsanch transformanch vzahov urme, kedy a kde


tieto dve udalosti (ppnutia) nastan v sstave . Ppnutie 1 nastane v ase
1 = v mieste 1 = (1 , 1 , 1 ) = (, 0, 0). Ppnutie 2 je v sstave
udalos, ktor nastane v ase 2 = ( + ) v mieste 2 = (2 , 2 , 2 ) =
(( + ), 0, 0). Teraz nm sta vyui zkladn princp pecilnej terie
relativity toti, e vetky inercilne vzan sstavy s si rovnocenn a
teda nutne rchlos svetla (teda i rchlos renia nho signlu) je v kadej
sstave rovnak. Spotame as 1 , kedy dolet prv signl k Zemi:
1 = 1 +

(
1
)
= 1+
.

Obdobne spotame as prletu druhho signlu 2 a ich rozdiel ur peridu


prijmanho signlu

(
)
1 +

= 2 1 = 1 +
=
=
.

Potom prijman frekvencia je


1
= =

.
+

as (b)
Najprv si obrazne vyjasnme, o to vlastne rozpnanie vesmru je. Pod rozpnanm vesmru sa mysl, e samotn priestor sa nafukuje . Ak chceme takto nafukovanie pozorova, tak zrejme potrebujeme nieo, o svoju dku
v takomto podivnom svete nemen. Naprklad pevn ty. Take ak mme dva
rzne body226 a v priestore, tak meme uri ich vzjomn vzdialenos
226 Ak sa itateovi body nepia, tak si do nich me umiestni nejak teles. Tieto
teles vak musia by jedno vzhadom na druh v pokoji, nehybn.

206

FX G2

Vesmr

pomocou nejakho potu ty, ktor poukladme do rovnho radu227 jednu


veda druhej. Po istom ase sa n priestor nafkne a to v kadom mieste
rovnako. Teda v kadej skulinke medzi atmami pribudne ksok novho
priestoru. Avak oakvame, e fyzika bude fungova rovnako ako v sasnosti, take atmy sa posnaia, aby sa ich vzdialenosti nemenili. Potom sa
ale nebud meni ani dky ty. Zkonite ale teda vetok pribudl priestor
objavme medzi tyami, ktor sa postupom asu bud rozchdza. Ke ich
znovu dme do radu tak, aby zanali v bode/telese , tak nm zrejme bude
chba niekoko ty pri . Doplnenm meme zisti, o koko sa vzdialenos
zvila za uplynul as. Vieme tak uri rchlos pribdania priestoru medzi bodmi a , ktor v zadan oznaujeme . Tu by som ale
zdraznil, e nejde o rchlos v pravom slova zmysle. Teles v bodoch a
sa navzjom nehbu228 . Len priestor medzi nimi pribda! Kvli pribdaniu
priestoru sa pojem rchlosti mimo poiatku systmu komplikuje, pribda
aksi drift spsoben rozpnanm. V takomto prpade bude rovnocennos
inercilnych sstav iba loklna teda rchlos svetla bude iba v poiatku
danej sstavy229 .
Prenos signlu meme teda uvaova tak, e letiaci fotn sa bude vdy
vzhadom na miesto, kde sa nachdza, pohybova rchlosou , a sasne sa
priestor pred a za nm bude rozpna. Ke signl dolet na Zem, u nemusme
poui transformcie sradnc a asu znme zo pecilnej terie relativity,
tak, ako sme ich pouili v predolej asti pri prechode z jednej vzanej
sstavy do druhej230 .
Meranie vzdialenosti pomocou pevnch ty budem povaova za spsob
urenia tej v zadan spomenutej objektvnej vzdialenosti. Spsob, ako ich pouklada do rovnho radu, vyrieim elegantne tak, e bud dut a ich stredom
227 Vznam slova rovno me by v krivch priestoroch odlin od toho, o povaujeme za
rovno my. Vo veobecnej terii relativity sa za rovno povauje smer, akm sa ri svetlo.
No, ale v blzkosti masvnych hmotnch objektov sa smer renia svetla zakrivuje, ako
keby padalo na ne. Ide o tzv. gravitational lensing, ie aksi gravitan oovkovanie,
lebo masvny objekt funguje pre svetlo fakticky ako spojn oovka.
228 Vo svojej podstate ide o veobecno-relativistick problm, ktor sa riei zavedenm
tzv. comoving sradnc. Tieto sa rozpnaj presne poda klovacieho parametra () a
objekty iba unan priestorom s v nich nehybn.
229 Ak by mal niekto problm s tm, e potom sa svetlo vzhadom na zdroj me asom
hba rchlejie ako , tak toho mem utei , e vo veobecnej terii relativity to nie
je na skok z mosta...
230 Sksim laicky nartn, ako vnmam ja, preo netreba transformova sradnice, ale je
to len moja naivn predstava neznalca veobecnej terie relativity. V prpade pecilnej
terie relativity s neobvykl transformcie sradnc a asu dsledkom toho, e svetlo
sa hbe vzhadom na ubovonho pozorovatea rchlosou , a to aj vtedy, ke s tto
pozorovatelia vo vzjomnom pohybe. V tomto prpade iaden takto klasick nezmysel
nenastva, lebo uvaujeme fotn, ktor sa hbe vzhadom na miesto, kde sa nachdza,
rchlosou .

207

FX G2

Vesmr

pjde laserov l z do . Kee uvaujeme tzv. ploch priestor231 , tak


sa svetlo bude ri rovno v zmysle nm bene znmom. Nae meranie je sce
ako prevediten, ale nm ide len o fyziklne realizovaten kontrukciu.
Vaka tomu sme nmu pojmu objektvnej vzdialenosti dali soldny zklad.
Teraz spracujeme informciu zo zadania o rovnomernosti rozpnania,
ktor je kvalitne zaifrovan. Tvrdme, e vzdialenos kadch dvoch bodov sa zvuje kontantnou rchlosou (kontantnou v ase!), a e rchlos
vzaovania je (v kadom danom momente) priamo mern vzdialenosti.
Tu, ia, chba informcia, e kontanta mernosti je univerzlna pre cel
vesmr, o sa vak sn dalo domyslie z prvlastku rovnomern. Konverzia
tohoto slovnho labyrintu do rei vzorcov je na poudovanie vemi praliv
() = (0 ) + 0 ( 0 )(0 ) = ()(0 ).
Sami sa mete presvedi, e tento vzah dva naozaj v ase kontant
rchlos vzaovania sa, a tie priamu meru rchlosti od vzdialenosti pre
ubovon zvolen as. Veliina () = 1 + 0 ( 0 ) sa nazva klovac
parameter. Udva, kokokrt sa natiahla vzdialenos medzi asmi 0 a .
V naom jednoduchom modeli je to linerna funkcia asu . Kontanta oznaen symbolom 0 m doln index 0, lebo ide o prv derivciu () poda
asu vyjadren v ase 0 .232 Psmenko H si zasli vaka jej objaviteovi, je
to toti Hubblova kontanta. Udva rchlos, s akou rastie vzdialenos, na
jednotku dky v ase 0 .233
V naom prpade teda plat = 0 a asov vvoj objektvnej vzdialenosti druice od Zeme je dan rovnicou () = + , v ktorej za as = 0
povaujem okamih, ktor je v zadan popsan ako as zaatia vysielania.
Meme napsa rovnicu, ktor bude popisova, ako sa ri signl rchlosou
svetla od druice k Zemi.234 Nech () je objektvna vzdialenos signlu (vypustenho druicou v ase = 0 smerom k Zemi) od druice. Potom plat
diferencilna rovnica

()
=+
,
(i)
+
231 To

znamen, e nie je zakriven, o je ekvivalentn tomu, e plat d2 = d2 + d 2 +


o je Pytagorova veta pre diferencily v 3-dimenzionlnom priestore.
232 V naom prpade ide o prv asov derivciu v ubovonom ase, ale ak uvaujeme
komplivanejiu funkciu (), tak potom to u jedno nemus by. V reli je funkcia ()
naozaj komplikovanejia.
233 Pre predstavu uvediem hodnotu Hubblovej kontanty v nami pozorovanom vesmre
km/s
v sasnosti: 0 = (71 3,5) Mpc , kde Mpc = 106 pc 3,26.106 ly. [pc je znaka pre
d 2 ,

jednotku dky parsek, o je skrtenina od slovnho spojenia paralaxa 1 oblkovej sekundy


vi Wikipdia a ly je znaka pre sveteln rok, light year.]
234 Mimochodom, formlne presne rovnako vyzer prklad z kuchyne pna ubomra
Muchu o muche, ktor kra rchlosou po idelnej gume. Zana liez od steny, o ktor
je gumika prichyten. Muche to vak nleite same tm, e druh koniec gumy ahme
kontantnou rchlosou pre od steny. Otzka je, za akch podmienok prde mucha na
kraj gumy, ktor ahme, a koko jej to bude trva.

208

FX G2

Vesmr

ktor hovor, e rchlos narastania objektvnej vzdialenosti signlu od druice je set rchlosti svetla a rchlosti narastania objektvnej vzdialenosti
medzi druicou a signlom. Podmienka vyjadrujca kedy signl doraz na
Zem je zrejme
() = + .
Nu, popravde, zapsan v tchto sradniciach to vskutku vyzer nevbne. Ve len tak bez rieenia ani nevieme poveda, i signl niekedy
k Zemi vbec doraz. Hop? Ale to by sme mohli vedie! Uvaujme srad()
nicu () = +
, ktor vyjadruje, v akej asti cesty od druice k Zemi
sa nachdza signl v ase . Vaka rovnomernosti rozpnania priestoru sa
zrejme as za druicou nafkne merne svojej objektvnej dke a as pred
druicou tie teda samotn rozpnanie priestoru nespsobuje zmenu ().
V premennej () dostaneme rovnicu
d =

d
+

()

,
+

(ii)

ktor hovor, e jedin asov zmena v sradnici () je spsoben vlastnm


pohybom signlu rchlosou . Podmienka vyjadrujca kedy signl doraz na
Zem je jednoducho
() = 1.
Prechod od rovnice (i) k rovnici (ii) sa nazva transformcia premennch
v diferencilnej rovnici a veobecne sa te obrovskej popularite v druhom
semestri analzy :-).235
Kadopdne, naa rovnica (ii) je separovaten, o znamen, e rieenie
vieme ahko njs jej integrovanm cez as od = 0 po , o bude as prletu
signlu na Zem

d =
d.

0
0
Integrl vavo je rovn 1, lebo signl za as letu k Zemi prejde prve cel
cestu od druice k Zemi. Integrl vpravo sa d riei substitciou = +
a je rovn ln +
. Z toho mme vyjadrenie pre as letu
]
) [ 1 ( )2
( /
=
1 =
+
+ ...


2
[
]

1
=
1+
+ ... .

2
235 N postup prechodu k novej sradnici bol intuitvny, lebo sme chpali vznam naich
sradnc. Ak vak lovek stoj zoi-voi nejakej diferencilnej rovnici a m v nej previes
transformciu premennch (matematick cvienia nie s ovea npaditejie ako toto...),
tak je to nieo celkom in. Potom u prde na invariantnos tvaru prvch diferencilov...

209

FX G2

Vesmr

Ke u vieme, ak as potrebuje signl na prlet na Zem, tak sa meme


konene zaa zaobera zmenou frekvencie vyslanch pulzov. V ase = 0
sme vypustili jeden signl a ten na Zem dorazil v ase 1 . Druica vysiela
s frekvenciou , take nasledovn signl vyle v ase = 1 . Ten doraz
poda predolch vpotov236 na Zem v ase
)
( /
2 =
1 + / .

Na Zem priletia tieto dva signly v asovom rozostupe


= 2 1 = / .
ahko sa mono presvedi, e -ty pulz vyslan druicou v ase =
doraz na Zem skutone o as neskr po nultom signli. Prijman frekvencia na Zemi preto bude
[
]
1 ( )2 1 ( )3
= / = 1 +

+ ... .

2
6
N vsledok vychdza v zhode s vlnovou predstavou. Ak sme druicou
vyslali fotn s vlnovou dkou v ase a tento prilet na Zem v ase , tak
)
bude ma vlnov dku = (
() . To odpoved jednoducho preklovaniu
vlnovej dky fotnu podobne ako sa kluj objektvne vzdialenosti. Pre
1+0
frekvencie dostaneme vzah237 = 1+
, o pre nae = a k nim
0
prslun vypotan hodnoty vypuje zhodn vzah.
Mal zhrnutie a porovnanie posunov frekvenci (nielen svisiacich s tmto
prkladom):
Nerelativistick Dopplerov efekt
Funguje pre vlnenia v prostred s rchlosou renia vzruchu 0 ovea
menou ako rchlos svetla . Ak je rchlos detektora vzhadom
na prostredie v smere od zdroja a je rchlos zdroja vzhadom na
prostredie v smere od detektora, tak plat
=

1+
[
]
+ ( + ) ( + ) 2
= 1

+
.
.
.
.
+
0
20
30

nastane jedine v hraniciach integrovania 0 7 , resp. 7 2 .


efekt sa zvykne oznaova ako kozmologick erven posun. Prvlastok erven m historick korene. Ide o to, e znme okom viditen spektrlne iary sa pri pozorovan vzaujceho sa zdroja elektromagnetickho iarenia posvaj k ervenej farbe,
ktor m spomedzi iarenia, ktor vidme, najniiu frekvenciu. Podobne to funguje, ak
medzi zdrojom a nami pribda priestor.
236 Zmena

237 Prslun

210

FX G2

Vesmr

Relativistick Dopplerov efekt


Pre vlny riace sa rchlosou svetla a zdroj vzaujci sa od ns (detektor) rchlosou plat

=
+
[
]
1 ( )2 1 ( )3
= 1 +

+ ... .

2
2
Kozmologick posun
Pre iarenie emitovan v ase a prijman v ase plat vo veobecnosti posun (vi vlnov predstava na rozpnanie vlnovej dky o osi
vyie)
()
.
=
( )
Pre linearizovan klovac parameter () = 1 + 0 ( 0 ) sme vypotali, e prijman frekvencia signlu vyslanho v ubovonom ase
z miesta, ktor malo v ase 0 objektvnu vzdialenos , bude
= 0 /
[
]
1 ( )2 1 ( )3
= 1 +

+ ... .

2
6
Gravitan erven posun
Pre iarenie emitovan z povrchu hmotnho sfricky symetrickho objektu o hmotnosti a polomere plat (d sa na prostonaivne prs
zo zkona zachovania energie pre fotn, ktormu prisdim hmotnos
= /2 = /2 )
[
]
1

= 1+ 2
.

Na zver by som povedal nieo o naich predstavch o vesmre. Poda
tandardnho modelu sa vesmr rozpna (priom aktulnu hodnotu Hubblovej kontanty som uviedol u vyie) a toto svoje rozpnie zrchuje (teda
funkcia () m aj vyie ako linerne leny). Tento prekvapiv fakt m ma
na svedom tzv. tmav energia 238 , ktor m zporn tlak. alej sa predpoklad, e rchlosti vzjomnho pohybu s vo vesmre nevek (rdovo na
238 Okrem toho, e vieme, ak hustotu by pribline mala ma, o nej nevieme skoro ni.
Pvodne lo o dodaton len v Einsteinovej rovnici vo veobecnej terii relativity. Dnes
ho udia zapracuvvaj do celkovej hustoty energie a hovoria o tmavej energii.

211

FX G2

Vesmr

rovni 1000 km/s) a preto na vekch vzdialenostiach239 je erven posun


dominantnm efektom nafukovania priestoru ie kozmologickho charakteru poprpade kombinovan s gravitanm posunom (ten je vznamn
u hviezd s vekou hustotou, napr. bielych trpaslkov).

239 O

kozmologickch vzdialenostiach hovorme, ak maj vekos rdovo aspo 100 Mpc.

212

FX G3 Zbavn fotny

Halucinka nala na povale zopr fotnov vo vemi dobrej nlade. Jej obben elektrn je u niekoko dn smutn a nehybne stoj na mieste, rozhodla
sa ho teda obveseli: poslala k nemu usmievav fotn s vlnovou dkou .
S poteenm zistila, e po tomto stretnut sa elektrn predsa len zaal pohybova. Vyslan fotn sa vak pri stretnut odchlil od pvodnho smeru o uhol
a zmenil svoju vlnov dku. Ako zvis nov vlnov dka fotnu a kinetick energia elektrnu od uhla ?
Do rozbehnutho elektrnu po ase narazil druh fotn. Po tejto zrke
zostal elektrn znova smutne st a fotn sa odchlil do smeru, v ktorom sa
pvodne pohyboval prv fotn. Ak je vlnov dka odlietajceho fotnu?
Nezabudnite, e rchlos elektrnu medzi zrkami me by relativistick.

Zdroj: IPhO

l Vzork Bzduo l

Mil deti. Dnes som si tak sadol k obedu a poriadne si ho vychutnal.


Ve bol dobr. Ke som tak dojedol, prila na ma nava a zadriemal som.
Snvalo sa mi, e je jar a prechdzam sa rozkvitnutou zelenou lkou. Stebl
trvy sa hompali vo vnku. Vhad na svet naokolo vo mne vzbudzoval
pocit opiieho mua... a zrazu ma zobudila mama. Kntri kury motyka!
lohu zrejme vyrieime pomocou zkonov zachovania. Zachovva sa nm
najm hybnos (vo vetkch zlokch), energia, no tie elektrick nboj,
mnostvo domcich loh a cena tovarov.240 Naorientujme si cel prbeh do
roviny os a tak, nech sa Halucinkou vypusten fotn na zaiatku pohyboval v kladnom smere osi . Spomeme si, e hybnos fotnu s vlnovou
dkou je = /, kde je Planckova kotanta. Jeho energia je -krt
via.
240 Menme

menu, nie cenu!

213

Zbavn fotny

FX G3

Po zrke sa fotn odchli o nadol, priom zmen vlnov dku na .


Aby platil zkon zachovania vo zvislom smere, elektrn mus is nejako ikmo
nahor. Nech sa pohybuje pod uhlom , jeho energia je poda zadania . Ak
si pozrieme vzahy pre relativistick energiu a hybnos
= 2

= ,

vidme jednoduch prepoet = /2 .241


Zkony zachovania (postupne pre , a ) nm dvaj rovnice

sin

+ 2

=
=
=

sin
2

cos + 2 cos

+ .

Tieto rovnice s relativistick, take ns popis je pln. Ni nechba.


Vimnime si ale, e v troch rovniciach mme a tyri neznme: , , a .
Prv dve z nich s vak zviazan rovnicou pre relativistick energiu, take
vetko je v poriadku.
Efektvna cesta k vsledku je naprklad takto: V druhej rovnici nechme
na pravej strane len len obsahujci cos . Porovnme s rovnicou pre .
Vidme raz snus, raz kosnus. Iba raz spakruky zatome, umocnme na
241 Mono stoj za zmienku, e v celom vzorku je pokojov hmotnos a e pre vetky
astice (hmotn i nehmotn) plat relativistick Pytagorova veta

2 = 2 2 + 2 4 ,
ktor ochvu aj odvodme. Pre nehmotn astice mono dosadi = 0. Pre hmotn
astice, ktor sa nepohybuj sta dosadi = 0 a dostvame znmy Einsteinov vzorec
= 2 .

214

Zbavn fotny

FX G3
druh a stame. Dostvame rovnicu
(
)
1
2
1
2 2
2

cos

+
= 4 .
2

Teraz to chce zasa nejak vnuknutie


rovnice nm napadne vyjadri si podiel
energiu:
2
=
=
2
1 2

zhora. Po kratom zadvan sa na


2 /2 zo vzahu pre relativistick
2
2 4
=
1

2
2

Do naej rovnice potrebujeme (vzah pre mme zo zkona zachovania


energie) pozna
2 2
4

2
2
2
(
)2

=
+ 2 2

(
)
2

1
22
2
1
=

+ 2 + 2
.
2

Po dosaden dostvame rovnicu


)
(
)
(
2
1
2
22
2
1
1
1

cos

+
=

+
+
2

.
2
2

2
2

2

Tu sa nm vea lenov vykrti. Konkrtne, ani nehnem brvou a mm
(
)

1
1
(1 cos ) =


vihnem biom242 (inmi slovami vynsobm a preusporiadam leny)
a zrazu

= +
(1 cos ) .

Prve sme odvodil vzah pre tzv. Comptonov rozptyl. Energiu elektrnu
u dopotame jednoduchm dosadenm. Po pr ikovnch pravch dostvam
2 (1 cos )
=
+ 2 .
( + (1 cos ))
Zostva vyriei druh ast pohybu. Znova platia vetky spomnan zkony zachovania. Avak, km sa ponorme do tajuplnho (nie prli pohostinnho) sveta rovnc, sksme trochu poprema. Predstavme si, e cel
proces zachytm na kameru a pustm odzadu.
242 Aj

vy tak zboujete kone?

215

Zbavn fotny

FX G3

Vidme fotn s hadanou vlnovou dkou ako naraz na stojaci elektrn.


Ten sa odchli o rovnak uhol a zska rovnak energiu ako to bolo v predolom prpade (prv zrka a normlny beh asu). Na takto pustenej pske
ale stle platia zkony zachovania, preto mus ma odletujci fotn rovnak
vek ako odletujci elektrn (teda aj ako odletujci fotn pri prvej zrke
a normlnom behu asu.) Tieto vahy ns napokon privdzaj a k tomu,
e rozdiel energi prilietajceho a odletujceho fotnu je rovnak ako pri
prvej zrke. To ist by sme zistili aj o zlokch hybnosti. Ak sa nad tm
poriadne zamyslme, tak jedin mon rieenie je, e druh zrka, pusten
odzadu, je identick s prvou, pustenou odpredu. Vlnov dka odletujceho
fotnu je preto .
Avak, s to dos plan rei a chcelo by to aj nejako rigorzne matematicky odvodni. To chce ale znova zostavi podobn hbu rovnc, ako
pred chvou a bi do nich znova. Naume sa vak nieo nov, aby sme
podobn lohy vedeli riei efektvnejie. Zoznmime sa s takzvanmi tvorvektormi.243
tvorvektory
Vieme, e v benom (trojrozmernom) priestore existuj veliiny charakterizovan trojicou sel. Tie pri zmene sradnicovch os (naprklad otoenie)
menia svoje zloky rovnako ako polohov vektor. To vezmeme ako denciu
vektora. Aby sme sa chpali, typickm prkladom zmeny sradnicovch os
je otoenie okolo osi o uhol v kladnom smere. Overte si, e plat prepoet


cos sin 0

= sin cos 0 .
0
0
1

Ak je nejak veliina vektorov, tak sa jej nov zloky spotaj nsobenm tou istou maticou.244 Ve intucia nm sama vrav, e vektorov veliina
je pka, rovnako tak, ako je aj polohov vektor pka.
V relativite sa vetky veci dej v tvorrozmernom, tzv. Minkowskho
priestore. Slovo dej je tu namieste. Prvkami tohoto priestoru s (z fyziklneho hadiska) udalosti charakterizovan tvoricou sel (, , , ), tzn.
asom a miestom.245 V celom alom texte ich budem oznaova vekmi
243 Neakajte sa. Terminolgia je tu podobne jednoduch a rovnako intuitvna, ako aj
v inch vednch odvetviach. Uvte naprklad slov tvorhlas (hudba), tvora (medicna),
tvorbob (port), tvorlstok (botanika), tvorhlav pes (genetick zoolgia), potvornoky
(teria umvania dlky), i tvorhrbolie (teria pravy cestnej komunikcie).
244 Pre priaznivcov matematickej analzy, matica sa nazva Jacobiho a jej zloky sa
zrtaj ako parcilne derivcie starch sradnc poda novch.
245 Rchlos svetla pred asom je tam len z rozmerovch dvodov. V relativite sa navye
asto stotouje jeden meter a jedna sekunda. Prehlsi sa, e as jeden meter je as, za
ak svetlo prejde jeden meter. Potom je = 1 a v rovniciach ho mono plne vynecha.

216

Zbavn fotny

FX G3

psmenami a bud ma za sebou ete doln (alebo horn) index nejak


grcke psmeno. No a aby to nestailo, skalrny sin je tu zadenovan
inak, ako sme zvyknut. Ak mme tvorvektory = (0 , 1 , 2 , 3 ) a
= (0 , 1 , 2 , 3 ), tak ich skalrny sin je (z dencie v danom priestore!)
= = 0 0 1 1 2 2 2 3 .
Hornmi a dolnmi indexami sa netreba necha zmias. Je konvencia e
pri skalrnom sine sa jeden z indexov nape hore. Jeho zmysel sa poriadne vyjasn a na prednkach z terie relativity. Vekosou tvorvektora
budeme rozumie
= 20 21 22 23
a vidno e me by kladn, zporn i nulov (a to dokonca aj pre tvorvektor
s nenulovmi zlokami).
Pri prechode k inak natoenm osiam sa mus vekos tvorvektora zachova. My z relativity vieme, e prkladom takejto dobrej zmeny sradnc
s naprklad pecilne Lorentzove transformcie

0 0

0 0

=

0
0
1 0

0
0
0 1

To je najjednoduh prpad. Vo veobecnosti sa transformuje nejakou


maticou . Ak ozname polohov tvorvektor ako , tak transformcie
sradnc mono zapsa ako
= .
Z dencie bude tvorvektorom kad tak tvorica sel, ktor sa bude
transformova rovnako ako polohov tvorvektor.
tvorvektor rchlosti
Predstavte si, e pohyb nejakho astronauta je zadan parametricky
mnoinou tvorc sel
( ) = (( ), ( ), ( ), ( )),
kde je jeho vlastn as, to jest ast ktor by on nameral na svojich hodinch.246 Derivujme tto svetoiaru 247 poda vlastnho asu a nazvime ju
246 as tu m zmysel asu na hodinch pevne spojench so zvolenou inercilnou vzanou sstavou, v ktorej pohyb pozorujeme.
247 Tak sa naozaj vol t trajektria v tyroch rozmeroch.

217

Zbavn fotny

FX G3
tvorrchlosou . To jest
=

d
=
d

(
)
d d d d
,
,
,
.
d d d d

Poksme sa jej zloky spota explicitnejie. Ak budeme funkcie ( ) (a


ostatn) chpa ako funkcie (( )), prejde derivovanie priestorovch sradnc na
d d
d
=
.
d
d d
d
Ak si ete spomenieme, e d
= ( )248 , tak dostvame
= (, , , ) .
Vimnite si, e priestorov zloky s ben rchlos nsoben ( ).249
Take ten nzov tam asi nebude plne nadarmo. A ako si overme, e sa pri
zmene sradnc transformuje tvorrchlos rovnakou maticou ako tvorvektor polohy? Jednoducho derivujme iarkovan tvorvektor a uvidme:
=

d
d
( ) =
=
d
d

Vetko je v poriadku, tvorrchlos sa transformuje, ako m. Pri zmene


sradnc nezmen svoju vekos.
tvorvektor energie a hybnosti
Denujme tvorhybnos (celkom intuitvne) ako
= ,
kde je (pokojov) hmotnos astice, ktorej pohyb opisujeme. Kee ide
o nsobenie slom (skalrny nsobok), dostali sme znova tvorvektor. Jeho
zloky s
= (, , , ) = (/, , , ).
aa. Energia a hybnos spolu tvoria tvorvektor! A nielen to! My navye
vieme, e vetky 4 veci v poslednom okienku s zachovvajce sa veliiny.
To znamen, e aj tvorvektor energie a hybnosti nejakej sstavy je zachovvajca sa veliina.250 Ke u sme a tu, spotajme si vekos tohoto
tvorvektora. Poda vyie denovanch pravidiel mus plati
= 2 /2 2 .
248 Ak

si nespomenieme, tak si treba natudova nejak lnok o dilatcii asu.


si vimnite, e vekos tvorrchlosti je vdy 2 .
250 Rados v tvri, smev v oiach. Rovnice treba riei s entuziazmom. :-)
249 Tie

218

Zbavn fotny

FX G3

Vekos tvorvektora ale nezvis od zvolenej vzanej sstavy. Naprklad


vo vzanej sstave, kde teleso stoj, m tvorhybnos zloky (, 0, 0, 0) a
vekos = 2 2 . A tak prichdzame k dleitej rovnici
2 /2 = 2 + 2 2 .
Vekos kadho tvorvektora hybnosti a energie je teda 2 2 . pecilne
si vimnite, e pre astice s nulovou pokojovou hmotnosou, napr. fotny, je
vekos tvorhynbosti nulov.
S novm apartom sp k lohe
Do prvej zrky vchdzaj astice s nejakmi tvorhybnosami a vystupuj s nejakmi inmi tvorhybnosami. Zaveme oznaenia
1 (resp. 1
1 (resp. 1
kou,

2 (resp. 2
po zrke a

) = ( , , 0, 0) pre tvorhybnos fotnu pred zrkou,

) = (, 0, 0, 0) pre tvorhybnos elektrnu pred zr-

) = ( , cos , sin , 0) pre tvorhybnos fotnu

2 (resp. 2 ) = ( ,
2 cos , 2 sin , 0) pre tvorhybnos elektrnu po zrke.

To nevyzer moc pekne, ale a na konvenn by boli oznaenia rovnako kared aj v klasickej mechanike. Zkon zachovania tvorhybnosti sa
d zapsa ako
(
)
(
)

= 2 + 2
1 + 1

Vimnite si, e v 2 vystupuje vemi vea neznmych velin (, a


). Ale my u vieme z predolho odstavca, e vekos tejto tvorhybnosti
je 2 2 . Ak ponechme tto tvorhybnos na avej strane rovnice sam a
umocnme na druh, vetky tieto neznme nm vypadn! Postupne dostvame:
( )
(
)

=
2
1 + 1 2

(
) (
)
( ) ( )

1 + 1 2
1 + 1 2
=
2
2

Vyzer to hrozivo? Neme, je to len mechanick narbanie s vrazmi.


Roznsobme poda pravidla maximlnej promiskuity.251 Na pravej strane
251 Kad

s kadm.

219

Zbavn fotny

FX G3

ostane 2 2 . av strnu upravme na dvakrt, pamtajc na podivn skalrny sin:


1 1

21 1

0
2

1 1

21 2
2
2

2
(1 cos )

2 2

21 2

2 2

0
2

+
=

2 2

Sce tam bolo vea lenov, ale ete viacej nl v zlokch tvorhybnost.
Cel sa to teda zvrhlo na jednoduch nsobenie. Teraz mme v rovnici jedin
neznmu . Po drobnch pravch mme prakticky bez nmahy 252
= +

(1 cos ) .

Ako bez nmahy overme, ak bude vlnov dlka odletujceho fotnu


v druhej zrke? Vchdzajci fotn so sebou nesie vea neznmeho - svoju
vlnov dku a uhol, z ktorho prichdza. Veci, ktor ns vbec nezaujmaj!
Ale vieme, e vekos tohoto tvorvektora je nulov.253 Aby sme mohli zapsa rovncie, zaveme ete oznaenia
2 (resp. 2 ) = ( 1 , 1 cos , 1 sin , 0) pre tvorhybnos fotnu vchdzajceho do druhej zrky,
3 (resp. 3
zrke a

) = (, 0, 0, 0) pre tvorhybnos elektrnu po druhej

3 (resp. 3 ) = ( 2 , 2 , 0, 0) pre tvorhybnos fotnu vychdzajceho z druhej zrky.


252 Cel

nmaha sa tu zvrhva na pochopenie tvorvektorov. Akonhle si s nimi potrasieme rukou a pochopme rei ich kmea, meme ich pouva plne zadarmo so vetkmi
vhodami, ktor prinaj. Tch je irok spektrum! Informujte sa u najbliieho dodvatea!
253 astice s nulovou pokojovou hmotnosou maj
=0

220

2 2 = 0.

Zbavn fotny

FX G3

Zkon zachovnia tvorhybnosti dva:


(
)
(
)

2 + 2
=
3 + 3

(
)
( )

2 =
3 + 3 2

(
) (
)

0 =
3 + 3 +2
3 + 3 2

1 cos
0 = 2 2 + 0 + 2 2 + 2 2 2
2
2

Tm istm spsobom sa meme zbavi odletujceho fotnu v prvej


zrke a prdeme k rovnici
)

0 = 2 2 + 0 + 2 2 + 2 2 2
1 cos .

Zskan dve rovnice maj rovnak av a teda aj prav strany. Po vykrten rovnakch lenov v oboch rovniciach a pr pravch dostvame
(
(
))
))
1
(

1
(

1
1

cos

=
1

cos

2
2

= 2 ,
o sme chceli ukza.

221

FX G4 Enterprise
Cez vianon przdniny sa Marcel kadorone vracia z interntu domov. Kee doma nem internet, vinu asu trvi hranm potaovch
hier a pozeranm serilov. Naposledy naprklad dal za jeden de toko dielov
Star Trek-u, e mal sen, v ktorom bol na palube vesmrnej lode Enterprise.
T sa prve nachdzala v asti vesmru, odkia bolo vidno hviezdy na oblohe rozmiestnen rovnomerne, t.j. tak, e poet hviezd na priestorov uhol
d/d = bol pribline kontantn.
(a) Ak rozdelenie hviezd by na lodi pozoroval, keby sa vzhadom na predol sstavu pohyboval dopredu rchlosou ?
Predstavte si, e by sme namiesto hviezd mali na oblohe monochromatick
zdroje svetla s frekvenciou s celkovm vkonom .
(b) Ako sa zmen frekvencia svetla v pohybujcej sa sstave v zvislosti od
smeru, ktorm sa Marcel pozer?
(c) o vieme na zklade ast () a () poveda o zmene hustoty iarivho
vkonu = d/d hviezdneho pozadia?

Zdroj: Teria relativity (skript FMFI UK)

l Vzork Bzduo l
Kadodenn sksenos z komernch sfr v oblasti science-cton ns presvieda, e bez plazmovej televzie a akho-takho poznania vybranch kapitol zo pecilnej terie relativity sme v modernej spolonosti plne straten.
Cieom tejto lohy je rozri si obzory a dosta sa tak o krok bliie k tomuto poznaniu.254 Na vod si vezmime jednoduch pozorovanie zo serilu
Star Trek. Lo Enterprise mus pred prechodom do hyperpriestoru zrchova na rchlos blzku rchlosti svetla. Autori serilu ns presviedaj, e
z paluby lode pri tom uvidme asi toto:
http://www.fks.sk/fx/zbierka/hyperspace_jump.mp4
254 Zujemcovia

sa mu o krok pribli aj k plazmovej televzii, o tie roziruje obzory.

:-)

222

Enterprise

FX G4

(Komentr: Polohy vetkch hviezd sa vzdialia od bodu, ku ktormu sa pohybujeme.)


Bdanie vak odhauje zdrvujcu skutonos Rapdne odlin od masovej produkcie lmovho neba! To, o by sme videli naozaj je krsne zobrazen
na tomto videu:255
http://www.fks.sk/fx/zbierka/special_relativity.mp4
(Komentr: Polohy vetkch predmetov sa posun do bodu, ku ktormu sa
pohybujeme. Okrem toho sa v zkom priestore pred nami prejav modr
Dopplerov posun a vrazne sa zvi intenzita dopadajceho svetla. Pri rchlostiach blzkych rchlosti svetla sa cel n obraz sveta zrti do malej a
vemi iarivej oblasti pred nami. V ostatnch smeroch svet upadne do tmy.)
Ak vs tto skutonos prekvapila, znamen to, e ste lacn konzumenti
lmovej produkcie. Ukeme si, e prihliadajuc na fakt, e rchlos svetla
je rovnak vo vetkch vzanch sstavch, s deje na druhom video plne
intuitvne. Tak o? Dostaton motivcia ta alej?
as (a)
Na mechanike ns nauili, ako vyzer pohyb telies z pohadu rznych
vzanch sstav. Ak sa sstava S vzauje rchlosou od sstavy S a
v sstave S sa pohybuje teleso T rchlosou , tak v sstave S sa pohybuje
teleso T rchlosou + . Vektory si predstavme ako pky a nov rchlos
zskame ako set dvoch pok.
Ak uvime relativistick efekty, mylienka skladania rchlosti zostva.
Pravidlo, ako sa to rob, tu vak vyzer inak. Mus to by inak. Predstavte si,
= . Ak prejdeme do inej sstavy a pripotame
e teleso T je fotn, teda
nejak rchlos ,256 tak vsledn rchlos mus ma op vekos . Fotn
sa predsa pohybuje rchlosou vo vetkch vzanch sstavch a prve
pohyb fotnov je to, o ns v tejto lohe zaujma.
Ben skladanie pok tu evidentne nefunguje.257 Aby sme odhalili sprvne
pravidlo, zrekapitulujme si pr dleitch znalost zo pecilnej terie relativity.
Cel relativita sa zjednodu, ak prijmeme dve bizarn mylienky:
255 Toto

video mi poslal Luk Tomek, za o som mu van.


pre fyziklne prpustn situcie mus by < .
257 Mono ste si niekedy predstavovali, e dosiahnu nadsveteln rchlos nie je vbec
ak. Sta vysla zo Zeme raketu rchlosou /2 a vntri rakety vrhn v smere jej
pohybu nejak teleso op rchlostou /2. o by sme vak pozorovali zo Zeme nie je
pohyb telesa rchlosou svetla, ale len rchlosou 4/5 < . V relativite teda 12 + 21 = 45 .
Kov pojem: Dilatcia asu. (o? Ak skracovanie asu? Pozorujem deje v rakete
spomalene? )
256 Samozrejme,

223

Enterprise

FX G4

1. Existuje aksi tvorrozmern priestor (tzv. priestoroas258 ). Jeho body


s udalosti, tj. tvorice sel (, , , ) popisujce kedy a kde sa nieo
udialo.
2. tyri rozmery okrem inho znamenaj aj tvorkomponentn vektory
(tzv. tvorvektory). Pri prechode z jednej vzanej sstavy sa transformuj rovnako ako polohov tvorvektor (, , , ).259 Skalrny sin
dvoch tvorvektorov a je tu denovan(!) ako
= = 0 0 1 1 2 2 3 3 .
Ak prechdzam zo vzanej sstavy S do sstavy S, tak nejako menm
smer priestorovch a asovch os260 a polohu poiatku, bodu (0, 0, 0, 0).
Vetky sradnice sa pritom menia na nov jednotnm vzahom

00 01 02 03
0

10 11 12 13 1

= 20 21 22 23 + 2 ,

30 31 32 33

3
kde stpek vpravo vyjadruje posunutie poiatku a prvky matice261 s
kontanty.262
My budeme skma najjednoduch prpad, ke maj sstavy S a S
spolon poiatok, rovnako natoen osi a ke sa sstava S pohybuje voi
sstave S rchlosou v kladnom smere osi . Vtedy je stpek vpravo nulov
258 V literatre njdete aj nzov asopriestor. Ide samozrejme o t ist vec a sprvne
nzvy s oba. Vznikli vak prekladom z dvoch rznych slov: Z anglickho spacetime, resp.
z nemeckho die Raumzeit.
259 A to z toho istho dvodu, preo sa v naom trojrozmernom priestore vetky vektory
transformuj ako polohov vektor.
260 Pokia sa S a S voi sebe nepohybuj, ide me s o ben rotcie priestorovch os,
ktor ponechvaj rovnak asov os. Pokia sa voi sebe sstavy pohybuj, ide o tzv.
hyperbolick rotcie (anglicky boost) pri ktorch sa nejako men aj smer asovej osi.
261 Pre neznalcov: Matica je len vhodn zpis sstavy linernych rovnc. Cel t vec sa
d prepsa ako

=
=
=
=

00
10
20
30

+
+
+
+

01
11
21
31

+
+
+
+

02
12
22
32

+
+
+
+

03
13
23
33

+
+
+
+

0
1
2
3 .

Koecienty v rovniciach musia spa ist poiadavky, ktor sa lepie formuluj v rei
matc.
262 Pre znalcov: Matica me by takmer ubovon. Mus spa len podmienky
det = 1, 00 1 a = , kde je matica 4 4, ktor m na diagonle sla
(1, 1, 1, 1) a zko svis s tvarom skalrneho sinu.

224

Enterprise

FX G4
a matica m jednoduch tvar263

=
0

0
0

0
0
1
0


0

0
1

, kde =

1
1

( )2 .

Aby sme objavili zkon skladania rchlost, uvaujme raketu FX (pozri


obrzok), ktor sa v S pohybuje rchlosou v kladnom smere osi .

Rchlos rakety v sstave S je denovan ako


=

d
,
d

o sa v tomto prpade redukuje na = d


d .
Je dleit si uvedomi, e as v menovateli je as v sstave S. Ke
chceme pozna rchlos w v sstave S, musme derivova neiarkovan sradnice poda neiarkovanho asu, teda
=

d
,
d

264
o sa v tomto prpade zjednodu na = d
Ale ve toto nie je vbec
d .
ak spota. Sta sa pozrie na transforman zkon (no, t matica) a
263 Tu si treba da pozor na to, e je zporn, ak sa sstava S pohybuje proti smeru osi
. To docielime aj zmenou SS. To znamen, e pri prechode od iarkovanch sradnc
k neiarkovanm vyzer matica rovnako, akurt zmen znamieno na opan.
264 Rozmyslite si, preo vznikne zloenm dvoch rovnobench pohybov vsledn pohyb
v rovnakom smere. Nie, tu nejde o relativitu, ale o skutonos, e vo vesmre neexistuje
iadny preferovan smer.

225

Enterprise

FX G4
meme psa
=

d
d

=
=
=

( d ) + d
(
)
1

( d ) + d

d + d
d + 2 d
+
.
1 +
2

To je zloen rchlos. Vimnite si, e pre mal rchlosti dva naozaj


= + . Tie si vimnite, e ak jedna z rchlost je rovn , zloen
rchlos je tie rovn . Presne, ako sme chceli!
Ke sme u prili a sem, je jednoduch njs zoveobecnen vzah pre
transformciu veobecnho vektora rchlosti = ( , , ).

Po aplikovan transformanho zkona dostvame

d
+
=

d
1 +
2
d

)
= (

d
1 +
2
d

).
= (

d
1 +
2

Polome si ete opan otzku: Ako sa transformuj rchlosti pri prechode z S do S ? Ak si uvedomme, e v transformanej matici sta spravi
zmenu , tak t ist zmenu treba spravi aj vo vsledku. Opan
226

Enterprise

FX G4
transfomrcie teda vyzeraj

d

=

d
1
2
d

)
= (

d
1
2

d
).
= (

d
1
2

Vrme sa konene k vesmrnej lodi Enterprise. Umiestnime ju do vzanej sstavy S. Vieme, e v sstave S prichdzaj fotny z nejakej vzdialenej
hviezdy pod uhlom . Ich rchlos je teda po zlokch
= cos

= sin

= 0

Uhol, pod ktorm uvidme t ist hviezdu v sstave lode Enterprise,


meme ur ako
cos

+ cos


=
.
=


+ cos

Nm sa, ako asom uvidme, hod inverzn vzah. Po pr pravch dostaneme


cos
cos =
.
cos
U vieme, ako sa posun polohy vetkch hviezd na oblohe. Ako sa zmen
hustota hviezd? Vetky hviezdy nachdzajce sa v rozmedz uhlov (, +
d) sa posun do (, + d). Ak si zrtame prslun priestorov uhly a
prensobme prslunmi hustotami hviezd na oblohe, dostvame rovnicu
2 sin d = 2() sin d,
227

Enterprise

FX G4

kde sme zaviedli oznaenie () pre hadan zvislos novej hustoty hviezd
od uhlu ku smeru pohybu.

pravou ostatnej rovnice dostvame


() =

sin d
d(cos )
=
,
sin d
d(cos )

kde zapsan derivciu vieme zrta vaka vyjadreniu cos od cos vyie.
Plat
() =

d(cos )
d(cos )

=
=

( cos ) + ( cos )
2

( cos )
2
2
2

( cos )
1
= (
)2 .
2
1 cos
Vsledok. Mete si sami vyska, o to rob pre = 0 a uvidte, e
vpredu sa hustota hviezd skutone zhusuje. Tak ako ukazovalo vodn video.
Integrciou cez cel priestorov uhol si tie mete overi, e celkov poet
hviezd ostal nezmenen.
as (b)
Ako sa zmen frekvencia pozorovanho svetla v zvislosti od smeru? Odpove pre = 0 a = poznme. V tch prpadoch je to dobre znmy
Dopplerov jav. My ho mme njs pre veobecn uhol. Napriek tomu sa d
loha poriei priam prekvapivo jednoducho, ak vyuijeme druh zo spomnanch bizarnch mylienok, z ktorch vyplvaj vetky zkony relativity:
tvorvektory.
228

Enterprise

FX G4

So tvorvektormi sme mali monos sa bliie zoznmi vo vzorku lohy


FX G3 Zbavn fotny. Tam sme predstavili tvorvektor polohy, tvorvektor rchlosti a tvorvektor energie a hybnosti. Rovnako ako tam, aj tu sa
uke vemi uiton prve posledne spomnan. Hadan frekvencia svetla
toti zko svis s energiou jeho fotnov, a t vieme spota Lorentzovou
transformciou! Tzv. tvorhybnos budeme oznaova . Jej zloky s265
= (/, , , ) .
Ak si ete spomenieme, e pre fotn = a = /, tak meme
napsa rovnos medzi tvorhybnosou fotnu v sstave S a tvorhybnosou
fotnu v S. Plat Lorentzova transformcia

0 0

cos 0 0
cos

sin 0
0 1 0 sin

0
0 0 1
0
0
pecilne si vimnime rovnos

cos ,


odkia jednoduchou pravou dostaneme hadan zvislos
() = (
1

cos

).

as (c)
Ako sa zmen hustota iarivho vkonu ? Vieme, ako sa vo vybranom
smere zmen poet zdrojov svetla (ozname tento nsobok ako ()) a frekvencia, a teda i energia prichdzajcich fotnov (ozname ako ()). ahko
by ns mohlo napadn, e iariv vkon v tomto smere sa zmen (()())nsobne. To je vak omyl! Zabudli sme na skutonos, e sa zmen aj poet
dopadajcich fotnov.
Aby sme do problmu lepie nahliadli, predstavme si, e ist zdroj svetla
vyaruje fotny v rovinnch vlnch vzdialench od seba jednu vlnov dku
.266 V inej vzanej sstave s tieto roviny vzdialen o . Fotny sa vak
265 Ak ste pri tan tohoto odstavca poctili zvrate a neistotu, skste si znova prelistova
spomnan vzork lohy FX G3.
266 Druh sada fotnov je vyslan tak, aby sa nachdzala napr. v minime vlny predchdzajcich fotnov. To je znova rovina a jej body maj skvel vlastnos nulovos a .
Ak je EM pole v istom bode a ase jednej vzanej sstavy nulov, tak v tomto bode a
ase je EM pole nulov aj vo vetkch inch vzanch sstavch. Takto body s v drhe
fotnu vzdialen prve o vlnov dku fotnu v danej vzanej sstave. A to s vzdialenosti
rovn, ktor uvaujeme. Tmto sme vzjomn polohy fotnov denovali objektvne.

229

Enterprise

FX G4

stle pohybuj rchlosou svetla, a preto ich budeme registrova tokokrt


viac, kokokrt sa zmenila ich vlnov dka. To je to vak op (), pretoe
vlnov dka zvis len od prevrtenej hodnoty frekvencie.
Takto sa vahou dostvame k sprvnemu vsledku
() = ()2 () =

(
[ 1

) .

4
cos ]

Dosadenm napr. pre = /2 meme zisti, e priamo pred nami sa


iariv vkon zv na 9-nsobok, za nami sa zmen na jednu devtinu.
Na kapitnskom mostku lode Enterprise je teda poriadne horco a urite
by sme si na nemali zabudn vzia slnen okuliare. S tmto nertali ani
tvorcovia Hviezdnych vojen!

230

H
Matematick lohy

In mathematics, you dont understand things. You just get used to them.
John von Neumann

231

FX H1 Romantika
Bea nedvno tala Malho Princa a rozhodla sa, e aj jej sa vemi pia
zpady slnka. Aby jej vak nezovedneli a aby si ich patrine vychutnala,
odsahovala sa na severn pl. Tam je toti zpad slnka len raz za rok, a
navye trv sakramentsky dlho. Ozaj, ako dlho?
Zdroj: lohu vymyslel Kubus

l Vzork Jakub l
Tento prklad vyrieim za vdatnej pomoci vektorov a ikovnch operci
s nimi.
Oznam si jednotkov vektor v smere zemskej osi. Tento vektor je
kolm na povrch Zeme na ploch. Teda je to normlov vektor dotykovej
roviny severnho plu. alej oznam polohov vektor Zeme voi Slnku.
Vektor predelen svojou dkou bude jednotkov vektor v smere zo Slnka do
= cos() =
stredu Zeme, oznam ho . Potom plat, e skalrny sin
cos , kde uhol je uhol medzi zvislicou na severnom ple a spojnicou
Zem-Slnko. Naou lohou je njs as, za ktor sa tento zmen z hodnoty
90 (vtedy sa slnen kot dotkne obzoru) na 90 + (vtedy sa prve
cel slnen kot dostal pod obzor), kde 16 je zdanliv polomer Slnka
zo Zeme.

Ostva nm vhodne vybra sradnicov sstavu (t chceme pravouhl,


aby skalrny sin jednotkovch vektorov udval skutone kosnus uhla nimi
zovretho!), uri vektory a , z nich skalrny sin a sme skoro hotov.
Poiatok mojej sstavy umiestnim do stredu Slnka. Smery sradnicovch
os zvolm tak, ako je to znzornen na obr.2, ktor dokumentuje situciu
na jesenn rovnodennos (23. september) kedy na severnom ple zapad
slnko: rovina je teda rovina ekliptiky (v nej le trajektria obehu Zeme
okolo Slnka), Zem v ase jesennej rovnodennosti le na kladnej poloosi .
232

FX H1

Romantika

Na rovnodennos s vektory a na seba kolm ( = 0), preto vektor


mus ma -ov zloku = 0. Jeho zvyn komponenty urm ahko,
lebo viem, e s normlou ekliptiky (d sa zapsa pomocou vektora (0; 0; 1))
zviera uhol 23 30 . Os je kolm na a jej orientciu som zvolil tak,
aby priemet vektora do -ovho smeru bol kladn ( aj rchlos Zeme na
rovnodennos bude ma kladn -ov zloku). Os je kolm na ekliptiku a
kladn polos smeruje nahor.

V nami zavedenej bze vyjadrm vektory pri aproximcii obenej drhy


Zeme okolo Slnka krunicou a zanedban pohybov zemskej osi:
= ( ; ; ) = (0; ; cos ) vektor zemskej osi, resp. normla
dotykovej roviny na ple (kontanta v ase),
2
= ( ; ; ) = (cos 2
; sin ; 0) vektor od stredu Slnka do stredu
Zeme, kde T je perida obehu Zeme (= 1 rok) a je as uplynul od niektorej
jesennej rovnodennosti. Potom
= + + = sin sin

2
.

Ak 1 je as, kedy sa slnen kot dotkne horizontu, a 2 as, kedy sa


slnen kot akurt ponra pod horizont, tak samotn doba zpadu slnka
je = 2 1 . Pre asy 1 a 2 mm podmienky
sin sin

21
= cos(90 ) = sin ,

22
= cos(90 + ) = sin .

Samozrejme, e rieenia 1 resp. 2 ns nezaujmaj vetky (sledujeme


iba zpad v jednom roku, v alom by sme u boli premrznut na kos :-)),
zjavne iba tie z rozsahu ( /2; /2) a preto dostvame jedin rieenie
sin sin

= 2 1 =

sin
arcsin sin

sin
arcsin( sin
)

2
233

sin
arcsin sin

32, 5h.

FX H1

Romantika

Pri vyslovan si treba da pozor na to, kedy potame s radinmi a kedy so


stupami.
Zanedbvam: ohyb svetla pri prechode atmosfrou (prve pri pozorovan objektov nzko nad obzorom je najmarkantnej, chyba me dosiahnu
a rd hodn!), excentricitu (eliptinos) obehu Zeme okolo Slnka, rozmery
Zeme voi vzdialenosti Zem-Slnko, precesiu a nutciu zemskej osi, obeh Zeme
okolo spolonho aiska Zem-Mesiac, slapov javy, zakrivenie povrchu Zeme
v mieste pozorovania (teda predpokladm, e pozorovate je nzko nad povrchom) a pravdepodobne ete mnoho inho. Napriek tomu si trfam tvrdi,
e sme dostali dos presn vsledok. Nothing else matters...
In sprvny prstup k rieeniu tohoto problmu spova v tom, e ak zanedbme pohyby Zemskej osi (precesia, nutcia), tak dotykov rovina na ple
sa voi vzanej sstave spojenej so stredom Slnka nenaklpa, ale len posva.
A zpad slnka trv v takomto modeli prve od vtedy, ke sa dotykov rovina
dotkne jednho okraja Slnka, a pokm sa nedotkne toho druhho. Nakoko
vieme, e uhol medzi normlou dotykovej roviny a normlou ekliptiky je ,
tak vieme poveda, o koko sa Zem musela pohn (v smere urenom priemetom zemskej osi do ekliptiky), aby dotykov rovina plu prerezala cel
Slnko (vzdialenos rovn jeho priemeru ) = / sin . No a tto vzdialenos je tetivou oblka (ak aproximujeme drhu Zeme krunicou), po ktorom
sa Zem poas zpadu slnka na ple pohybovala. Dostvame as

2 arcsin 2
sin

32, 5h.
sin

Ete jedna poznmka: Hoci presn rovnice (v slade s horeuvedenmi


aproximciami) nedvaj pekn rieenie pre vku slnka na poludnie poas
roka, rieenie tohoto problmu v tvare = /2 sin 2
pre miesto
severne od obratnka Raka (junejie sa vo vraze menia znamienka) na
zemepisnej rke dosahuje pre pozemsk parametre presnos lepiu ako
18 . Pouitm tohto priblenia sa dalo dospie k slunmu vsledku =
arcsin

2 2
32h. Treba poznamena, e toto priblenie plat len
preto, e pre nae plat relatvne presne sin . Vzah = /2
arcsin(sin sin 2
) je presn pre n zjednoduen model. To vidno pre pl
, ktor vak je mlo nzorn.
z nho starho znmeho skalrneho sinu
Preto pridvam ete jeden obrzok, kde sa na situciu pozerme z dotykovej
roviny severnho plu.
234

FX H1

Romantika

Trajektria Slnka je krunica leiaca v rovine zvierajcej s naou rovinou


pozorovatea uhol . Vka Slnka nad obzorom je = arcsin
. Vku si
vak viem jednoducho vyjadri z tch pravouhlch trojuholnkov. Vyznaen
trojuholnk je v rovine kolmej na priesenicu dotykovej roviny s rovinou
obehu Slnka okolo severnho plu (teda je v rovine kolmej na obe spomenut
roviny), preto tam vystupuje prve uhol, ktor zvieraj tieto dve roviny, ie
. Ak ete dodm zvislos = 2
pre rovnomern pohyb Slnka po krunici,
tak dostanem star znmy vraz
= arcsin(sin

2
sin ).

A len tak mimochodom, ke u vieme vku Slnka na ple, v ubovonej


zemepisnej rke bude na poludnie vka Slnka len posunut o /2 . (Na
rozdiel od plu, toto naozaj plat len na poludnie, t.j. ke s vetky iary
na obrzku v jednej rovine.)267

267 Rozsiahleji text o tejto problematike mete njs na adrese


http://www.fks.sk/fx/zbierka/slnecny_voz.pdf

235

FX H2 Starcraft
Ferko a Tom s zarat do svojej obbenej potaovej hry, klikaj ostoes, stavaj armdy, schyuje sa k vekej bitke. Tom posiela do boja jednotiek, Ferko rovnakch jednotiek. Njdite asov priebeh potu jednotiek
na oboch stranch, as trvania bitky i konen stav.
Mete predpoklada, e obaja hri s sksen jednotky po sebe strieaj fotnovmi delami optimlnou stratgiou, a (km sa nevykynoia), je
ich vemi vea. Jedna jednotka stlym strieanm zabije druh (nebrniacu
sa) za as .
Zdroj: lohu vymyslel Kubus

l Vzork Kubus l
Najprv si musme rozmyslie, ako bud vlastne jednotky po sebe striea.
Najvhodnejie je, ke bud vetci striea na t ist sperovu jednotku
takto sa zrejme minimalizuj kody, ktor me nepriate spravi nm. Inmi
slovami, minimalizujeme as, ktor m nepriate jednotky k dispozcii.
V istom ase m Tom jednotiek a Ferko jednotiek.268 Ak zanedbme
Ferkove straty za as, km zabije jednu Tomovu jednotku, bude tento as
dlh /. Podobne, Ferkove jednotky bud ubda po jednej kad as /.
Predpokladajme, e jednotiek je vemi, ba priam a spojite vea. Potom m
eme poveda, e rchlos ubdania Tomovch jednotiek je d
d = 1/ =

. Podobne to bude s Ferkom. Pre jednoduchos si ozname = 1/,


potom si meme napsa diferencilne rovnice pre vvoj stavu bitky:
d
=
d
d
=
d

(95)
(96)

Teraz u len sta tto sstavu rovnc vyriei, o sa d robi viacermi


spsobmi: Od trpezlivho rotenia cez vysokovkonn metdy a po znebaspadnuvie ahko nahliadneme, e . Vimnime si najprv, akho peknho
tvaru je tto sstava. Rchlos v kadom bode (stave bitky, meme si ho
reprezentova i bodom v rovine) je pevne uren iba tmto bodom (jeho
sradnicami , ). V kadom bode si vieme nakresli, ktorm smerom sa
bude bitka vyvja a potom ju iba chladnokrvne odsimulova. Ke u ni
in, dostaneme aspo predstavu o tom, o sa tu deje.
268 Hne

vidme zaveden konvenciu = (0) a = (0)

236

Starcraft

FX H2

Na chvu sa nezaujmajme o asov vvoj situcie, ale len o aksi relatvny vvoj a . Vieme, e
d
=
d

d
d
d
d

= .

Rieme tto rovnicu tak, ako kad in separovaten:


d

=
d

d = d
()
()
d =
d

2 2 = 2 2
2 2 = 2 2 = kont. = 2

(97)

Vidme, e rozdiel tvorcov potov jednotiek oboch armd zostva kontantn.269 To znamen, e v fzovom priestore sa bude stav bitky pohybova po hyperbole. Takisto hne vidme, ak bude konen stav, zrejme ke
= 0, tak = . Tm sme u vyrieili jednu as lohy.
Ak bude asov priebeh bitky? Dosame si do rovnice (95) za poda
(97):

d
= = 2 2
d
269 Zavedenm substitcie 2 2 = 2 predpokladme, bez ohadu na veobecnsti, e
>.

237

Starcraft

FX H2

Toto meme znova riei ako obyajn separovaten diferencilnu rovnicu:

()

1
d
=
2
d

d

=
d
2 2
0

(98)

o so karedm integrlom naavo? Bu sme makai a ten hyperbolick


snus v tom vidme, alebo . . . nad
tm chvu dumme, a raz ho tam uvidme.
V kole ns uili, e integrl 1/ 1 2 je arcsin . Ako (sme) na to prili? Ak
si urobme substitciu = sin , menovate sa nm kvli sin2 + cos2 = 1
vyhub s pozostatkom po substitcii a u integrujeme iba d. Inpirujme sa
touto ntou, spomeme si, e cosh2 sinh2 = 1, dajme si substitciu
= cosh() a upravujme:

d
=
2 2

sinh
2

cosh2 2

1d

=+
= arccosh(/) +
(
)
( )

2
= ln
+
1) +

Posledn prava nebola potrebn, ale je zaujmav ju spomen a derivovanm si mete odvodi jej pravdivos.270 Vrme sa ale nasp ku naej
270 Odvodi sa d vemi jednoducho priamo z dencie hyperbolickho kosnusu. Nech
(
)
= cosh = 12 + . Potom plat inverzn relcia = arccosh . Upravujme vak
prv rovnicu. Prensobenm a preusporiadan lenov mme:

1 2
1
+ = 0
2
2

2 1

Sptnm dosadenm a zdravmi vahami sa d prs na to, e sprvne je znamienko plus.


Potom obyajnm zlogaritmovanm oboch strn rovnice dostvame
)
(

= ln + 2 1 .

238

Starcraft

FX H2
rovnici. Teraz meme rovnicu (98) previes na tvar
arccosh

()

( )
()

arccosh
= arccosh

(
( )
)

= cosh arccosh
.

arccosh

Fj, ak kared rovnica. Ale aspo mme explicitn vyjadrenie potu


Tomovch jednotiek od asu. Z rovnice (97) vieme vytc aj Ferkov stav i
as trvania bitky, ale je dos neprjemn to z tohto stavu upravi na prjemne
vyzerajci vsledok.
ikovn nta
Sme predsa nejak fyzici, musme njs nejak kraj spsob rieenia271
zamyslime sa nad tm, ako jednotky vlastne ubdaj. V kadom momente
nejak jednotka kyno nejak in. Ak teda zabudneme na farby, celkov
poet jednotiek ubda merne sm sebe. Podobne si meme rozmyslie, e
(neformlne povedan) Ferkove jednotky ubdaj o toko rchlejie, koko
m Tom naviac jednotiek, teda rozdiel potov jednotiek za zvyuje merne
sm sebe. Alebo jednoducho, napme si set a rozdiel rovnc (95) a (96) a
sledujme tie divy:272
d( + )
= ( + )
d
d( )
= ( )
d
Ak sa na chvu budeme tvri, e nae premenn s + a ,
vyrieime dve naozaj jednoduch difky a dostvame
( + ) = ( + )
( ) = ( ) .
271 Pekn spsob Pea Pereniho, je vyjadri si z (95) a (96) druh derivcie a .
Dostaneme dve jednoduch odseparovan diferencilne rovnice druhho rdu.
272 Pre tie silnejie povahy vina takchto linernych sstav diferencilnych rovnc
prvho stupa o premennch sa d riei takmto spsobom. Zavedieme si nov premenn, ktor s linernou kombinciou starch tak, aby sme zo zadanch rovnc vedeli
vyrobi rovnice obsahujce iba po jednej novej premennej. Tie potom vieme jednoducho
riei a posklada nasp star premenn.

239

Starcraft

FX H2
Stame, odtame, ako v klke:
)
1(
( + ) + ( )
2
)
1(
=
( + ) + ( )
2
)
1(
=
( + ) ( )
2
)
1(
=
( ) + ( + )
2

alebo ete krajie,


= cosh() sinh()
= cosh() sinh().
Vylo nm to pekne krsne symetricky (akoby nie!), kto chce, me si
overi, ako z toho vypadne konen stav, poduma nad svislosami s hyperbolami, my u len dodme, e stav, ke bude = 0, teda koniec bitky,
nastane v ase273
sinh() = cosh()

tanh () =

( )
1

= arctanh

(
)

+
alebo
= ln
.
2

273 Ako

cvienie si skste sami dokza, e


arctanh =

1
ln
2

240

)
1+
.
1

FX H3 Brka

Mat prve sedel na lke a potal fyziku, ke sa zrazu prihnala vek


brka. Poas brky do zeme nhodne udieraj blesky, priom na kilometer
tvorcov za cel brku priemerne udrie bleskov. Ak je priemern hodnota
vzdialenosti Mata a miesta, kde udrel k nemu najbli blesk?
Zdroj: lohu vymyslel Mat

l Vzork Kubus l

Rieenie 1.
Pravdepodobnos, e najbli blesk udrel vo vzdialenosti odo ma, je
sinom pravdepodobnosti, e vo vzdialenosti nejak blesk udrel, a pravdepodobnosti, e bliie ako iaden neudrel. Samozrejme, pravdepodobnos
e blesk udrie presne do vzdialenosti je nulov, ale my budeme v konenom
dsledku integrova, a teda hra sa s pravdepodobnosami e blesk udrel vo
vzdialenosti z intervalu [, + d).
Ozname si pravdepodobnos, e v nejakom pevne urenom zem s plochou poas celej brky neudrel blesk, ako (). (Nhodnos udierania bleskov nm zaru, e tto pravdepodobnos je rovnak pre ubovon zemie
s plochou .) Potom pravdepodobnos, e vzdialenos najbliieho blesku je
v intervale [, + d), bude rovn
[1 (( + d)2 2 )] (2 ).
(Ak nevie odkia sa tento vzah vzal, pozri si znova prv vetu vzorku.)
Samozrejme, pre mal d meme napsa
( + d)2 2 = (2 + 2 d + d2 2 ) 2 d,
a teda
(( + d)2 2 ) (2 d) (0) + (0)2 d = 1 + (0)2 d.
241

FX H3

Brka

(Zrejme pravdepodobnos (0) e do przdnej plochy neudrie blesk je 1.)


Priemern hodnota vzdialenosti, v ktorej udrel najbli blesk, bude teda
(
= lim

=
0
=
0
=

)
[vzdialenos najbliieho blesku [, + )]

=0

[vzdialenos najbliieho blesku [, + d)]


[1 (( + d)2 2 )] (2 )
2 (0)2 (2 ) d.

(Prv resp. druh vzah meme bra ako denciu priemernej vzdialenosti.
Ak vm nie je po chuti, skste si priemern vzdialenos denova sami.
Ak to nejde, skste si priemern hocio denova na konenejom priestore
udalost.)
Teraz nm u len sta zisti funkciu , priom vieme e na jeden kilometer tvorcov udrie priemerne bleskov. Zistme ju naprklad takouto
sedliackou vahou . Vezmime si nejak zemie s plochou + a rozdeme
ho na zemia s plochami a . Potom mus plati
( + ) = [do zemia + neudrel blesk]
= [do zemia neudrel blesk ani do zemia neudrel blesk]
= [do zemia neudrel blesk] [do zemia neudrel blesk]
= ()().
Takto vzah mus plati pre ubovon , > 0. Avak jedin funkcie, ktor
tento vzah spaj, s exponencilne funkcie tvaru () = . (Preo?
Skste si naprklad zderivova rovnicu ( + ) = ()() poda .) Ete
stle vak musme njs kontantu , zrejme bude zvisie od . Zoberme si
vemi mal plochu d. Priemern poet bleskov, ktor udrie na tto plochu
poas brky, bude z dencie jednoducho d, ale tie ho meme napsa
ako
d =

[na plochu d udrie bleskov].

=0

242

FX H3

Brka

Lene, ke bude plocha d vemi mal, meme pravdepodobnos e na u


udrie viac ako jeden blesk oproti ostatnm zanedba, m dostvame
d [na plochu d udrie 1 blesk]
[na plochu d udrie aspo 1 blesk]
= 1 (d) = 1 d
1 (1 + d) = d,
ie = a teda pravdepodobnos, e na plochu s obsahom poas brky
neudrie iaden blesk, bude
() = .
Dosadenm do pvodnho vzahu pre dostvame

22 d,
2

o nm u len zostva zintegrova. Nechajme si to a na koniec a pozrime sa


na trochu in metdu rieenia.
Rieenie 2.
Zoberme si nejak plochu a rozdeme ju na malch kskov. Ak bud
tieto ksky dostatone mal, meme v prvom priblen predpoklada, e
na kad z nich dopadne nanajv jeden blesk. Pozrime sa na takto jeden
ksok. Bu na dopadne jeden blesk alebo nula bleskov. Jeho plocha je
/ , take z dencie na dopane priemerne / bleskov. No ale to
znamen, e jeden blesk tam bude s pravdepodobnosou / a nula bleskov
s doplnkovou pravdepodobnosou 1 / . (Mimochodom, rovnak vahu
sme robili v prvom rieen.)
Celkov pravdepodobnos, e na cel plochu dopadne bleskov, bude
potom v naom priblen
( )(
) (
)

1
.

(Lebo na niektorch z plok dopadne jeden blesk a na zvyn nula


bleskov.) No a zrejme m vyie bude , tm menie bud jednotliv ksky
a tm presnejie bude nae priblenie, e na kad z nich dopadne najviac
jeden blesk. A d sa aj ukza, e ak poleme do nekonena, n vzorec
243

FX H3

Brka

bude presn. Teda pravdepodobnos, e na plochu udrie bleskov, bude


( )(
) (
)

(
) (
)

!
()

1
= lim
1
( )!!

(
) (
)

!
()
= lim
1

( )!
!

()
=1
1.
!

[, ] = lim

Sami si premyslite, preo m kad zo siniteov uveden limitu. V uren



limity tretieho z nich sme pouili znmy vzah (1 +
) .
Kto chce vedie viac, nech si naprklad na internete vyhad Poissonovo
(a na oivenie pamti aj Binomick) rozdelenie, kto nie, nech si aspo over,
e tento n vzah dva sprvne vsledky pre pravdepodobnos e na plochu
udrie aspo nula bleskov (mala by by samozrejme rovn 1), pre oakvan
poet bleskov (malo by by , z oho sme vychdzali), aj pre pravdepodobnos, e na plochu neudrie iaden blesk (mala by by rovnako () ako
sme zrtali predtm).
My tento vzah u len pouijeme na spotanie prkladu. Podobne ako
minule napeme pre priemern hodnotu vzdialenosti najbliieho blesku,
len u nebudeme pouva pvodn ale n vzah.
(
= lim

)
[vzdialenos najbliieho blesku [, + )]

=0

[vzdialenos najbliieho blesku [, + d)]


[bliie ako udrelo 0 bleskov]

[vo vzdialenosti [, + d) (v tom medzikru) udrel 1 blesk]

(2 )0 2 (2 d)1 2 d

0!
1!
0

2
2
1
2
=

1 d =
22 d,
1
1
0
0
=

ako v predolom rieen.


(Mimochodom, vimnite si, e tieto dve rieenia obsahovali vea rovnakch vah, lili sa vlastne len v tom, ako presne sme vyhtali vzorec
244

FX H3

Brka

s exponencilnou funkciou. Tie si vimnite, e druh rieenie je o osi veobecnejie.)


Integrl
Nakoniec len portajme integrl, ktor vyiel v oboch rieeniach. Meme sa s nm popasova per partes (skste si!), alebo nm pome nejak
matematick softvr, alebo nasledujca drobn nta. (Nie a tak potrebn,
kee to ide per partes, ale celkom zaujmav a uiton, tak si ju ukeme.)

2
2
Vieme, e 0 d = 21 . Ak si ozname () = 0 d = 12 ,
derivovanm vetkch troch strn rovnosti poda dostvame

2 2
() =

d =
.
4 3
0
(Mdre vety nm hovoria, e za istch podmienok meme vymeni poradie
2
2
d
derivcie a integrlu, ie d
(
) d, odkia mme

d = 0
0
prostredn vzah.) Vynsobenm 2 dostvame


1
2 2
2
=
,
2
0
no a dosadenm za dostvame hadan hodnotu integrlu

2
1
=
22 d = .
2

245

FX H4 Mesiac
Kokonsobne silnejie iari Mesiac v splne ne v prvej tvrti? Uvaujte
Lambertovsk model rozptylu svetla na mesanom povrchu.
Zdroj: Kvant

l Vzork Jakub l

Pred samotnm rieenm lohy si dohodneme vyjadrovac apart. Budem


pouva fyziklnu veliinu hustota iarivho toku, ktor vyjadruje vkon
iarivej energie prechdzajcej cez jednotkov plochu kolm na smer toku.
Budeme ju znai . Jej zkladnou jednotkou je W.m2 . alej budem pouva oznaenie M-Z pre vzdialenos Zem-Mesiac, M pre polomer Mesiaca.
lohu vyrieim s nasledovnmi aproximciami: mesan povrch je sfra
o polomere M , Mesiac obieha okolo Zeme po krunici s polomerom M-Z
v rovine ekliptiky, hustota toku iarenia zo Slnka pri Mesiaci je nezvisl
od fzy274 . Zanedbvam svit hviezd. Samotn odraz uvaujem lambertovsk s rovnakm koecientom odrazivosti na celom povrchu.275 Zanedbm
akkovek pohlcovanie iarenia v priestore medzi Zemou a Mesiacom.
Strune osvetlm ete motivciu lambertovskho rozptylu. Ide o model,
poda ktorho je hustota iarivho toku do vybranho smeru proporcionlna
kosnusu uhla medzi normlou vyarujcej plochy a vybranm smerom. To
zabezpeuje, e osvetlen plku budem vnma zo vetkch smerov rovnako
jasn, lebo hustota iarivho toku dopadajca do mjho optickho senzora
bude proporcionlna priestorovmu uhlu, pod ktorm tto plku vidm. Je
to dobr aproximcia pre relatvne drsn povrchy, avak vemi nedobr pre
zrkadliace plochy.
Nart situcie: Chcem zisti pomer hustt iarivho toku pre tvrfzu
( = 2 ) a spln ( = ). V tomto vzorku spotame hustotu Mesiacom
odrazenho toku iarenia v zvislosti od uhla .
274 Nakoko relatvna zmena vzdialenos Slnko-Mesiac je pri obehu Mesiaca okolo Zeme
v rde tiscin, tak ide o rozumn zjednoduenie.
275 V reli je odrazivos funkcia zvisl od konkrtneho miesta, od smeru, odkia je toto
miesto osvetlen, od frekvencie dopadajceho iarenia a taktie od smeru, v ktorom uvaujem odraz.

246

FX H4

Mesiac

Zoberme si mal plku povrchu Mesiaca s obsahom d a jednotkovou


normlou276 . Ak je tto plocha osvieten Slnkom, tak vkon dopadajci
na u je rovn sinu hustoty iarivho toku Slnka Slnko vo vzdialenosti
1 AU a plochy priemetu naej plochy do smeru toku iarenia.

Plocha priemetu plochy do smeru toku iarenia je rovn sinu plochy d


a kosnusu uhla , ktor zvieraj jednotkov vektor toku iarenia Slnko =
(1, 0, 0) a normla plochy . Teda iariv vkon dopadajci na plochu je
ddopadajci = Slnko cos d.
Pre lambertovsk povrch potom plat, e v smere odchlenom o uhol
od normly plochy vo vzdialenosti M-Z bude hustota Mesiacom odrazenho
iarivho toku
cos
dmesan =
ddopadajci ,
2
M-Z
kde je albedo ie kontanta vyjadrujca podiel odrazenho iarivho vkonu ku dopadajcemu vkonu277 . Kosnus uhla viem op ikovne vyjadri
276 Normla plochy je vektor kolm na dan plochu. Ke hovorm o jednotkovej normle,
tak tm myslm, e jej dka je jednotkov. Kad dostatone mal plocha je prakticky
rovinn a teda m normlu.
277 Pre Mesiac je hodnota albeda v priemere pribline 7, 2 %.

247

FX H4

Mesiac

pomocou skalrneho sinu jednotkovho vektora normly plochy s jednotkovm vektorom v smere k pozorovateovi Zem = (cos , sin , 0).
Zoberme si plku na povrchu Mesiaca na mieste s polrnym uhlom278
a zemepisnou dkou o uhlovch rozmeroch d, resp. d, vi obrzok.

2
Tto plka bude ma obsah d = M
sin d d. Jej jednotkov normla
bude = ( sin cos , sin sin , cos ).
Vyjadrime si kosnusy uhla , resp. . Plat

Slnko = sin cos ,


cos =
Zem = sin cos ( + ).
cos =
Ostva nm u iba positova hustotu odrazenho iarivho toku od vetkch osvetlench plch Mesiaca. Osvetlen

bud zjavne plochy, ktorch je


z intervalu 0, a z intervalu 2 , 2 . Zo Zeme
279 s viditen plochy s

3
z intervalu 0, a z intervalu 2 , 2 .
Teraz u vieme,
e vidi
ten a osvetlen plochy maj z intervalu 0, a z intervalu 2 , 2 ,
kde sa predpoklad, e 0, .
Nasleduje vpoet hustoty toku iarenia odrazenho Mesiacom videnho
pozemskm pozorovateom:

cos cos d
mesan () =
2
osvetlen a viditen plochy

2
= 2 M Slnko
sin3 cos cos ( + ) d d

M-Z
0,, 2 ,
2

278 Polrny
279 Plat

uhol je obdoba zemepisnej rky, ale meria sa od severnho plu.


iba v priblen M M-Z , v skutonosti vidme o mlo menej.

248

FX H4

Mesiac

Takto dvojn
integrl

meme pota tak, e si integran priestor


0, , 2 , 2 rozdelme na psiky rovnoben s osou pre . Potom
potame vlastne 2 integrly v sebe. D sa vak vidie, e pre ubovon
potrebujem pota rovnak integrl ohadom premennej . Preto sa d
tento dvojn integrl napsa ako sin integrlov 1 a 2 , priom

1 =
sin3 d,
0

2 =

cos cos ( + ) d.

Vpoet integrlu 1 sa d urobi nasledovne. Vo vzniknutom integrli


pouijeme substitciu = cos .

sin3 d
1 =
0

sin (1 cos2 ) d
=
0


=
sin d
sin cos2 d
0

= [ cos ]0
[

3
=2
3

]1

0
1
1

2 d

= 4/3.
1

Vpoet druhho integrlu 2 je prakticky priamoiary.


2
1 =
cos cos ( + ) d

[ 2
]
cos cos cos sin sin d

2
1 + cos 2
sin 2
d sin
d
= cos

2
2
2
2
[
]
[
]
2 + sin 2 2
cos 2 2
= cos
sin
4
4

1 cos 2
cos sin
2 sin 2
cos
sin =
.
=
4
4
2
Dostvame vsledok
mesan () =

2
2M
(sin cos ) Slnko .
2
3M-Z

249

FX H4

Mesiac

Odtia hne vidme, e lambertovsk Mesiac by v splne odral -nsobne


viac slnenho iarenia ako vo tvrfze. Tie vieme takto uri, e osvit
Mesiacom v splne by bol zhruba milinkrt slab ako slnen osvit.
Ako sa to dalo riei jednoduchie pre nae pekn fzov uhly = 2 , resp.
= ? Nu, treba si vimn, e hustota toku iarenia prichdzajceho od
reektujceho lambertovskho povrchu zvis od dopadajcej hustoty toku
a priemetu plochy, ktor vidm odra. Ako druh si treba vimn, ako vyzer mnoina bodov na povrchu Mesiaca s rovnakou dopadajcou hustotou
toku slnenho iarenia. Ako tretie treba nahliadnu ako to vidme v prslunej fze zo Zeme. V splne dostvame pekne rotane symetrick obraz.

Zoberme si tak sstreden medzikruie s vntornm polomerom a vonkajm + d. Toto je osvetlen Slnkom
rovnako, dopad na
hustota slnenho
2 2

2 2

M
iarivho toku dopadajca = Slnko MM , kde vraz
nie je ni in
M
ako kosnus uhla medzi normlou ubovonej plky medzikruia a smerom
ku Slnku. Teraz vyuijeme, e povrch Mesiaca je lambertovsk, teda nech
sa pozerm z ubovonho smeru, tak plochu vidm rovnako jasn a hustota
odrazenho iarivho toku je priamo mern ploche, ktor vidm. Plocha
medzikruia, t.j. priemet rovnako osvetlench miest na povrch Mesiaca do
smeru pozorovatea na Zemi, je d = 2 d. Teda hustota toku odrazenho
touto plochou v miestach Zeme je dmesan = 2 dopadajca d. RozpM-Z
san na drobn a spotan dokopy dostvame celkov hustotu mesanho
iarivho toku pre spln

spln

2Slnko
= 2
M-Z M

M
0

2 2 d.
M

Tento integrl sa d vypota substitciou, no uke sa, e pota ho vbec


nie je potrebn. Toti, v tvrfze dostvame tie pekn obrzok, tentoraz
vak rovnako osvetlen vidme zvisl sliky namiesto sstredench innitenzimlnych medzikru.
250

FX H4

Mesiac

Dopadajca intenzita je tentoraz dopadajca = Slnko M , kde vraz M je


op kosnus uhla normlyplochy a smeru ku Slnku t.j. -ovho smeru.
2 2 d. Teda celkov hustota odrazenho
Plocha slikov je d = 2 M
toku je
tvr =

2Slnko
2
M-Z
M

M
0

2 2 d.
M

Hne vidme, e plat spln = tvr .


Poznmka: Mesiac vak nie je dobrm prkladom telesa s lambertovskm rozptylom. Prejavuje sa na om relatvne siln opozin efekt, ktor
spsobuje, e odrazen iarenie bude preferova smer, odkia prichdza.
Inmi slovami povedan, ak je objekt nasvieten spoza pozorovatea, tak
bude vrazne jasnej ako by sa dalo usudzova z lambertovskho modelu.
Vysvetuje sa to tm, e na povrchu Mesiaca (a Marsu...) sa nachdza porzny materil, ktor prakticky vdy vrh tie na as viditenho povrchu
prve okrem prpadu, ke je zdroj iarenia priamo za nami. V prpade Zeme
je tu ete t komplikcia, e dokonal spln nie je zo Zeme pozorovaten,
lebo v takom prpade nastva zatmenie Mesiaca. Poda pozorovan posdok
programu Apollo je osvit pri plnom splne a o 30 % silnej ako pri najlepie pozorovatenom splne zo Zeme. Treba poznamena, e Mesiac vstupuje
do polotiea, ak uhol nadobda uhol zhruba 178, 78 . Pri lambertovskom
modeli by bol relatvny nrast osvitu pre = 180 iba necel tvr promile.
V skutonosti sa pozoruje asi 10-nsobn nrast osvitu v splne voi tvrfze. Pomer osvitu Mesiacom v splne ku osvitu Slnkom je pribline 1/450000.
Ete spomeniem, preo je Slnkom neosvetlen as Mesiaca na hviezdnej oblohe relatvne svetl. Hviezdy vo veci nebud, ale me za to prve Zem...
Prikladm ete vygenerovan obrzky, ako by vyzeral Mesiac, keby bol lambertovsk v porovnan so skutonosou.
251

FX H4

Mesiac

252

FX H5 Elektrika
Ak vonku pr, schodky na nstup do elektriky s pinav od blata. Ke
sa vak otvraj a zatvraj dvere, as tchto schodkov pri svojom pohybe
poumvaj kefami pripevnenmi na ich spodku. Vypotajte tvar a plochu
tchto poumvanch ast.280 Vimnite si tie horn upevnenie druhej dosky dver. V novch modeloch elektriiek tu nie je xovan jej horn roh
v rovine zatvorench dver, ale bod na upevnenom vstupku, pomerne aleko
od samotnch dver. uduj sa svete, vodiaca lita pre tento vstupok je tie
rovn. Ak polohu (vzhadom na druh dosku dver) me ma vstupok vo
vodiacej lite, aby tto mohla ma tvar seky?
Zdroj: lohu vymyslel Kubus

l Vzork Kubus l
Tvar plochy
Vmajme si iba jedno krdlo dver, ktor nech m pre jednoduchos celkov dku 1. Jednotliv dosky bud ma teda rku 1/2. Vynechvajc zvisl
rozmer dver, umiestnime ich pevn bod (otav ty) do poiatku sradnicovho systmu a nechajme opan koniec dver pohybova sa po osi .
Dvere bud zatvoren, ke sa tento koniec bude nachdza v bode [0, 1] a
otvoren, ke sa bude nachdza v [0, 0]. Kb spojenia dosiek dver ozname
.

Bod sa zrejme bude pohybova po tvrkrunici s polomerom 1/2 a


stredom v . Poas jeho pobytu v hornej polovici tvrkrunice281 bude uhol
280 Pri rien uvaujte naprklad elektriky Tatra T3, kde sa jedno krdlo dver sklad
z dvoch rovnakch zvislch obdnikovch dosiek navzjom spojench kbom na zvislej
hrane. Von hrana jednej z nich je upevnen na zvislej otavej tyi, hrana druhej m
pohyb obmedzen v rovine prechdzajcej touto tyou.
281 Osminokrunica?

253

FX H5

Elektrika

zvieran dvoma doskami dver () ostr, preto druh doska dver ()


nebude zasahova von z tohto kruhovho vseku. Tto osminka krunice
bude teda hranicou poumvanej asti.

alej bude hra lohu aj druh doska. Jednm zo spsobov ako njs
oblas, kam vade bude poas svojho pohybu zasahova, je njs najvy
bod (s najvou sradnicou ) na zvislej priamke (pre kontantn ), kam
sa kedy nejak jej ksok dostane. Ozname si uhol = . Zrejme aj
= .

Po chvke umenia na obrzok vidme, e priamka m rovnicu282


= sin tg
Extrmne hodnoty pre xovan a rzne uhly njdeme derivovanm
poda :
d

= cos
=0
d
cos2
= cos3
282 Dka paliky je 1/2. Hne vidme, e na obrzku je = sin . Pri posune o doava
sa zmen o tg .

254

FX H5

Elektrika

Dosadenm nasp do rovnice priamky dostvame = sin . Tieto body


bud teda tvori horn hranicu poumvanej asti schodov, samozrejme, iba
pre uhly 0, /4. Doln hranica je zrejme os a av hranica je os
. Patrilo by sa ete zapsa njden
krivku hornej hranice neparametricky.

Niekto by mohol napsa = arccos 3 , take = sin3 arccos 3 . Toto je


dos kared tvar, o nieo prjemnejie sa nm bude pota, ak si vimneme,
e
cos2 + sin2 = 2/3 + 2/3 = 1
3

teda
= (1 2/3 )3/2
Tto krivka sa vol astroida.
Pred tm, ako sa pustme do potania obsahu poumvanej plochy, ukme
si ete jeden spsob na njdenie jej hraninej krivky. Len pre sprehadnenie
situcie si ozname priesenk priamky s osou ako a vimnime si, e
dka bude rovn 1 bez ohadu na hodnotu uhla . seka sa bude
sprva ako tuh palika o dke 1, ktorej konce maj pohyb obmedzen na
osi a . Ukeme, e plocha, ktor poumva seka , bude prve
pln astroida.

Uvaujme nejak bod na hranici poumvanej plochy. Urite le na


nejakej seke (pre konkrtny uhol ). Ak teraz zvme o malik
ksok, seka sa jemne pohne (na ) a znova bude nejak jej bod
lea na hranici poumvanej plochy. Tento bod nebude aleko od bodu ,
no a priesenk a bude pribline niekde medzi a . Ak budeme
takto po vemi malch kskoch posva seku , bud sa priesenky
255

FX H5

Elektrika

jej susednch polh pohybova vemi blzko hraninch bodov. Ak budeme


napokon zmenova posunutia seky do nuly, mnoina priesenkov jej
susednch polh splynie s hranicou poumvanej asti. uvakovo povedan,
ale tvrdo pravdiv.
Po takejto vahe u hranin krivku njdeme jednoducho. Majme situciu tak, e = [, 0] a = [0, ], zrejme = cos a = sin .
Zvme uhol o mal d, m sa zv283 o d a o d . Ak si ete
ozname = a = (kde je teraz u priesenk a ),
potom z obrzku vidme, e plat
d = d sin
d = d cos
resp.
d

d sin
sin2
2
d sin
=
= d
=
=

d cos
cos
cos2
2
d

Kee vak 2 + 2 = 1 = + , dostvame = 2 a = 2 . Ak si


ozname sradnice bodu ako [, ], z podobnosti trojuholnkov dostvame,
e E le na astroide.

= 3 = cos3

= 3 = sin3

Obsah plochy
Vidme, e poumvan plocha rozdelen priamkou = sa sklad z osminy kruhu s polomerom 1/2 a osminy plnej astroidy. Z tabuliek mme
1
1 3
5
obsah astroidy 3
8 , take celkov poumvan plocha je 8 4 + 8 8 = 64 , ak
sa drme toho, e jedno zatvoren krdlo dver m jednotkov dku. Pre
cel dvere to bude dvojnsobok.
Ako vak zrtame obsah astroidy bez tabuliek? V jednom kvadrante (t.j.
tvrtina obsahu) to je naprklad

(1 2/3 )3/2 d =
0

sin3 arccos

d,

ale tieto integrly s oriekom pre nejak mazan softvr284 a nie pre ns,
smrtenkov. Pozrime sa ale na druh postup pri potan tvaru poumvanej
283 V

skutonosti sa zmen, pretoe d je zporn.


http://integrals.wolfram.com

284 alebo

256

FX H5

Elektrika

asti. Ak budeme pri posvan seky so zvujcim sa uhlom vyfarbova trojuholnky medzi bodmi a ( ), postupne vyfarbme
cel tvrtinu astroidy. Tento trojuholnek aproximujeme rovnoramennm,
s uhlom pri vrchole o vekosti d a dvoma stranami s dkou . Jeho obsah
bude potom
1
1
1
d = 2 d = 4 d = sin4 d
2
2
2
Obsah tvrtiny astroidy potom meme vypota:

/2

d =
0

1
2

/2

sin4 d

Tento integrl nevyzer nijako prjemne, ale existuje hne niekoko spsobov, ako si s nm rchlo poradi. My si ukeme dva z nich. Najprv budeme
pouva vzahy pre snus a kosnus polovinho uhla:
1 cos 2
2

sin2 =

cos2 =

resp.

1 + cos 2
2

Pomocou tchto vzahov meme prepsa:


(
sin
4

)2
1 cos 2
2
(
)
1
1 2 cos 2 + cos2 2
4(
)
1
1 + cos 4
1 2 cos 2 +
4
2
1
(3 4 cos 2 + cos 4)
8

=
=
=
=

To je pekn. Teraz sa nm bude primitvna funkcia hada omnoho jednoduchie. asn vak je, e ju nemusme hada vbec, pretoe zo symetrie
/2
/2
hne vieme, e 0 cos 2 d = 0 cos 4 d = 0. Preto meme napsa.
1
=
2

/2

1
sin d =
2

/2

3
1 3
3
d =
=
8
2 8 2
32

In ikovn cesta je metda per partes. Pripomeme, e poda nej

d = []
257

d.

FX H5

Elektrika

Nech naalej oznauje hadan obsah tvrtiny astroidy. Nech v naej rovnici je = sin a = sin3 . Tak dostvame:

]/2 1 /2
1[
cos sin3 0 +
3 cos2 sin2 d
2
2 0

3 /2
=0+
cos2 (1 cos2 ) d
2 0

3 /2
=
cos2 d 3
2 0

1 3 /2
3
3
=
cos2 d =
=
42 0
84
32

Obohaten o dva zaujmav vpoty zskanho uritho integrlu sa posmelen pustme do alej asti lohy.
Vodiaca lita
Zostaneme pri u zavedenej sradnicovej sstave, musme si ete nejako
popsa polohu vstupku vzhadom na druh dosku dver, na ktor je xovan. Nech je jeho (orientovan) vzdialenos od tejto dosky a nech je
rovnoben vzdialenos od konca dver .285

Teraz si meme napsa [, ] sradnicu dver v zvislosti od ich otvorenia (uhla ):


= cos cos + sin
= sin + cos
My hadme tak polohu vstupku vzhadom na druh dosku dver (t.j.
hodnoty a ), pre ktor sa s meniacim uhlom bude vstupok pohybova
285 Pri

zatvorench dverch splva so sradnicou a = 1 .

258

FX H5

Elektrika

po priamke. To znamen, e sklon krivky pohybu vstupku


kontantn. Tento sklon vieme vypota naprklad ako
d
=
d

d
d
d
d

d
d

mus by

sin + sin + cos


cos sin

Aby sa sklon nemenil s meniacim sa uhlom , mus plati


( )
d d
d
=0
d

(
)
d sin + sin + cos
0=
d
cos sin
( cos + cos sin )( cos sin )
0=
( cos sin )2
( sin + sin + cos )( sin + cos )
+
( cos sin )2
0 = cos ( cos sin ) + ( cos sin )2
sin ( sin + cos ) + ( sin + cos )2
0 = 2 cos2 + 2 sin2 + 2 cos2 + 2 sin2
cos2 sin2
0 = 2 + 2
(1/2)2 = 2 + (2 1/2)2
Inmi slovami, vstupok xovan k druhej doske dver mus lea na
krunici so stredom v kbe a polomerom 1/2 (o je dka jednej dosky).
V takom prpade sa bude pri otvran dver pohybova po priamke a vodiaca
lita me by rovn.
Nadenci si mu spota jeho drhu v prpade, e na spomnanej krunici nele.:-)

259

FX H6 Skmavka
Evka sa na biolgii rada hr so skmavkami, minule naprklad do skmavky tvaru valca s polomerom 1 cm naliala vodu a pozorovala, ak tvar
bude ma jej povrch. Njdite kontaktn uhol medzi povrchom vody a sklom,
tie njdite celkov prevenie hladiny vody na kraji a v strede skmavky,
a nakreslite tvar jej povrchu. Povrchov energia rozhrania voda-vzduch je
70 mJ.m2 , energia rozhrania voda-sklo 40 mJ.m2 a energia rozhrania sklovzduch 100 mJ.m2 . seln vsledky plne staia.
Zdroj: lohu vymyslel Tom

l Vzork Tom l
Aby som citoval pna veobecne uznvanho fyzika Filipa K.: Nakdim
si to v pascale . Zska analytick rieenie v tomto prpade toti nie je jednoduch. Najprv si lohu trocha sformalizujeme. e situcia bude stredovo
symetrick je nad Slnko jasnejie. Nech teda () vyjadruje vku vody vo
vzdialenosti od stredu, priom nulu kladieme na rove vody, ktor by
v skmavke bola neby vypuklosti, i skr vpuklosti vody. Tento stav (rovn
voda) budeme povaova za referenn aj pri rtan energi. Kad hne
vid, e v porovnan s tmto stavom m sstava energiu:

=2 ()( ) +

2 ()
0

()
d
2

2 1 + ()2 d,

kde je povrchov energia rozhrania medzi a (-voda, -sklo, -air),


je polomer skmavky a hustota vody.
Tento vrcholne nepedagogick krok (grcn vzorec nm len tak padol
z neba) sksim trochu zjemni miernym dovysvetlenm jednotlivch lenov.
Prv len vyjadruje zmenu energie vntornho povrchu skmavky (pri zdvihnut vody sa as vzduchu nahrad vodou). Druh len predstavuje zmenu
potencilnej energie ktor dostaneme ako integrl cez vemi uzuk medzivalia (polomer takhoto medzivalia je , rka vka () a jeho aisko
sa nachdza v polovici jeho vky, ie ()/2. Posledn len rta povrchov
energiu voda-vzduch, idea rozsekan na medzivalia je stle iv, akurt sa
lepie pozrieme na ich horn podstavu, ktor nie je rovn ale ikm. Tangens naikmenia je akurt derivcia funkcie a trocha goniometrie nm
260

FX H6

Skmavka

umouje zrta kosnus tohto uhla. Skuton povrch hornej podstavy medzivalia bude potom 2 vynsoben / cos . Na zver podotknime, e aby
bola skutone rozdielom energi oproti referennmu stavu, mali by sme
ete od nej odta energiu voda-vzduch v referennom stave. Toto je vak
kontanta a teda na minimalizciu (za chvu sa k tomu dostaneme)
nem zmysel.
Naou lohou je teda njs tak , pre ktor vyjde minimlne. Na
toto existuje pardna vec, ktor sa vol tum varian poet a na fyzike
na matfyze vs ju nau v druhom ronku Fecko. Na FKS na seminri pre
drskov vs ju mono nau aj Kulich. Problm s touto metdou je ten, e
sa jedn vlastne o problmy dva. Menovite
Prv problm: Vina z vs nem absolvovanho Fecka.
Druh problm: Nm nesta hocijak funcia, hadme tak funkciu
, ktor spa

2 () d = 0.
0

Tmto chcel bsnik (akoe ja) poveda, e voda v skmavke sa nekot


a jej mnostvo je rovnak ako v referennom prpade.
Preto nastupuje na rad numerika. Aby sme nali aspo priblin seln
rieenie, interval [0, ] rozsekme na hrozne prav ksky o vekosti d a
v nich si budeme pamta hodnoty funkcie . Integrovanie nahradme sumciou a derivciu rozdielom dvoch susednch hodnt predelenm d. K danej
u teda poahky vieme njs hodnotu ! Ako vak njs funkciu s minimlnou ?
Prvoplnovm a pre tento problm pouitenm rieenm je zobra nejak, hociak , a ska na u aplikova drobn zmeny. Ak sa zni,
zmenu zapamtme, ak nie, zrume ju. Takto skame postupne vetky
zmeny. V okamihu, ke u iadna zmena nevedie k zmeneniu , prehlsime, e sme nali optimum. A oe s to presne tie nae zmeny? Prvoplnov
(zl) rieenie je zobra () a zmeni jeho hodnotu o , o bude nejak nami
zvolen mal kontanta. Toto je vak cesta do pekla nae zmeny nm menia objem vody v skmavke. Preto to spravme inak. Okrem toho, e ()
zvme (zmenme) o , zmenme (zvme) vetky hodnoty funkcie
o presne tak hodnotu , e (( + )2 2 ) = 2 . Takmto
trikom iba prelejeme vodu z jednho miesta v skmavke na in. A je to.
Kokoe to touto skvelou metdou vylo? Kee som prli leniv to kdi,
zacitujem pnov v.u.f. Kubinu a Maza: rozdiel vok 2.8 mm, kontaktn
uhol 59 .286
286 Zamyslite sa nad tm, ako tento kontaktn uhol priamo vypota. Konkrtne si skste
si rozmyslie, ak sily psobia na molekulu vody priamo na trojrozhran voda-sklo-vzduch.

261

FX H6

Skmavka

Aby som vynahradil svoju lenivos problm vlastnorune naprogramova,


prikladm na zver aspo zopr lozockch otzok, nad ktormi je dobr sa
zamyslie: Ako zvoli , aby sme dostali o najlep vsledok za o najkrat
as? Ako by mohla vyzera optimlna ako funkcia kongurcie (tj. asu,
vky vody, zlepenia za posledn iterciu, . . . )? S veliiny ako ,
priamo zistiten nejakm meranm? o je pre vsledok naozaj dleit?
V tomto konkrtnom prklade sme mali trochu astia podarilo sa nm
navrhn tak sadu drobnch zmien, ktor zachovvali nau vzbu, t.j. mnostvo vody v skmavke. Viete si predstavi systm, kde by navrhn takto
zmeny nebolo jednoduch? Ako by ste postupovali potom?

262

placeholder

263

Sie [slvy]
Zoznam rieiteov, ktor v prslunom ronku dosiahli aspo v jednej
lohe aspo polovin poet bodov.

Ronk 2005/2006
10 loh
A3 A5 A7 B3 D3
E5 E8 G1 H2 H5
1. Tom Bzduek, G Pierra de Coubertina, Pieany
(9 A3 A5 B3 D3 E5 E8 G1 H2 H5)
2. Marcela Hrd, G Jura Hronca, Bratislava
(5 A5 A7 D3 E5 G1)
3. Peter Pereni, G Jozefa Gregora Tajovskho, Bansk Bystrica
(4 A5 B3 H2 G1)
4. Jakub Imrika, G Jura Hronca, Bratislava
(1 B3)

Ronk 2006/2007
9 loh
(nie je v zbierke, lohy vak mete njs na www.fks.sk/fx/fx2.php)
1. Tom Bzduek, G Pierra de Coubertina, Pieany, (6)
2. Filip Kubina, G Pavla Orszgha Hviezdoslava, Doln Kubn (3)

Ronk 2007/2008
18 loh
A1 A2 B2 B4 C2 C3
C4 C5 C6 D4 E1 E2
E4 E6 G2 H1 H3 H6
1. Filip Kubina, G Pavla Orszgha Hviezdoslava, Doln Kubn
(14 A1 A2 B2 B4 C2 C3 C4 C5 D4 E1 E2 E4 G2 H6)
2. Jan Hermann, G esk Krumlov
(11 A2 B4 C2 C3 D4 E1 E2 E4 E6 H1 H3)
3. Peter Vanya, G Jura Hronca, Bratislava
(6 C2 C3 D4 E1 E4 E6)
4. Luk Konen, G Pierra de Coubertina, Pieany
(4 A1 C2 E2 H1)
5. Michal Spiiak, G Grsslingova 18, Bratislava
(4 A1 B2 C3 E1)
6. Dalimil Maz, G Jna Keplera, Praha
(3 C6 E6 H6)
7. Mria Kieferov, G Sv. Frantika z Assissi, ilina
(3 C3 D4 E2)
8. Samuel Hapk, G Sv. Grsslingova 18, Bratislava
(2 B2 H3)
9. Lucia Simanov, G Sv. Grsslingova 18, Bratislava
(1 A1)
10. Dvid Vendel, G Potov, Koice
(1 A1)

Ronk 2008/2009
15 loh
A4 A6 A8 B1 C1
D1 D2 E3 E7 E9
F1 F2 G3 G4 H4
1. Eugen Hruka, G Hlohovec
(12 A4 A8 B1 D1 D2 E3 E7 E9 F2 G3 G4 H4)
2. Mria Kieferov, G Sv. Frantika z Assissi, ilina
(7 A6 B1 D1 E3 E9 F2 G3)
3. Jn Bogr, G udovta tra, Trenn
(4 A4 B1 D2 G3)
4. Prabhat Rao Pinnaka, G Hyderabad, India
(3 D1 E3 E9)
5. Kateina Honzkov, G Jna Keplera, Praha
(2 D1 D2)
6. Martin Polako, G Alejov, Koice
(2 B1 E9)
7. Andrea Ploekov, G Pierra de Coubertina, Pieany
(2 A6 F2)
8. Peter Vanya, G Jura Hronca, Bratislava
(1 D2)

Autori [zbierky]
Tom Bzduek (Bzduo)
FMFI UK, Bratislava
bzduso@fks.sk
http://www.fks.sk/bzduso

Jakub Imrika (Jakub)


FMFI UK, Bratislava
jakub@fks.sk
http://www.fks.sk/jakub

Jakub Zvodn (Kubus)


University of Oxford
kubus@fks.sk
http://www.fks.sk/kubus

... a kolektv ...


Marcela Hrd (Marcelka), aklecram@gmail.com
Filip Kubina (Filip), fkubina@gmail.com
Tom Kulich (Tom), tomas.kulich@gmail.com

Zdroje loh:
Boston Area Undergraduate Physics Competition (BAUPC),
http://www.physics.harvard.edu/academics/undergrad/problems.html
KMaL - Mathematical and Physical Journal for Secondary Schools,
http://www.komal.hu/info/bemutatkozas.e.shtml
Fiziko-matematieskij urnal Kvant,
http://kvant.mirror1.mccme.ru/
International Physics Olympiad,
http://www.jyu.fi/kastdk/olympiads/
Archv Fyziklneho korepondennho seminra,
http://www.fks.sk/archiv/archiv.php
Richard Feynman, Robert Leighton, Matthew Sands:
Feynmans Lectures on Physics
Peter Gndig, Gyula Honyek, Ken Riley:
200 Puzzling Physics Problems
Milan Noga:
Teria relativity (skript FMFI UK)
almi nvrhmi prispeli Mat Medo, Peter Matk a autori zbierky.
Nzov: Zbierka FX
Autori: Tom Bzduek, Jakub Imrika, Jakub Zvodn
a kolektv Marcela Hrd, Filip Kubina, Tom Kulich.
Sadzbu programom -TEX a LATEX 2 pripravil: Tom Bzduek
Grack rieenie oblky: Jakub Imrika
Neprelo jazykovou pravou.
Vydal: Trojsten (www.trojsten.sk), Bratislava, 2009
Tla: Edis, vydavatestvo ilinskej univerzity
Nklad: 120 ks
Rozsah: 288 strn
ISBN: 978-80-970297-0-8
Vydan s nannou podporou Agentry na podporu vskumu a vvoja, projekt
LPP-0103-09.

You might also like